Sie sind auf Seite 1von 848

1

2013
H1 CHEMISTRY
1 ANGLO - CHINESE 2013
2 ANDERSON 2013
3 DUNMAN 2013
4 HWA CHONG INSTITUTION 2013
5 INNOVA 2013
6 JURONG 2013
7 MILLENIA INSTITUTE 2013
8 MERIDIAN 2013
9 NATIONAL 2013
10 NANYANG 2013
11 PIONEER 2013
12 RAFFLES INSTITUTION 2013
13 RIVER VALLEY 2013
14 SERANGOON 2013
15 TAMPINES 2013
16 VICTORIA 2013
17 YISHUN 2013

www.gradtutors.com
2
2

Section A
For each question there are four possible answers, A, B, C, and D. Choose the one you consider
to be correct.

1 How many electrons are needed to be removed to completely ionise


1.0 x 10–6 mol of K atoms to K+ ions?

A 6.02 x 1023
B 1.0 x 10–6
C 1.0 x 10–6 x 6.02 x 1023
D 1.0 × 10 −6
6.02 × 1023

2 0.200 mol of a hydrocarbon undergoes complete combustion to give 26.4 g of


carbon dioxide and 10.8 g of water as the only products.
What is the molecular formula of the hydrocarbon?
A C2H4
B C2H6
C C3H6
D C6H12

3 A 50.0 cm3 sample of an iron (II) solution required 34.60 cm3 of 0.100 mol dm–3
potassium dichromate (VI) solution for complete reaction.
Calculate the mass of iron (II) ions in 1 dm3 of the iron (II) solution.
A 0.42 g
B 1.16 g
C 3.86 g
D 23.2 g

4 Elements J and Q have the following successive ionisation energies in kJ mol-1.


1st 2nd 3rd 4th 5th 6th 7th
J 587 1817 2745 11577 14842 18379 23326
Q 870 1790 2698 3610 5668 6820 13200

What is the likely formula of the compound that is formed when J reacts with
Q?
A JQ3 B J2Q3 C J3Q2 D J2Q

ACJC 2013 8872/02/Sept/13 [Turn over


www.gradtutors.com
3
3

5 X, Y and Z are elements in Period 3 of the Periodic Table.


 X has the highest melting point among all the Period 3 elements.
 The chloride of Y has the second highest melting point among all the Period
3 chlorides.
 The oxide of Z is amphoteric.

Which of the following statements is incorrect?


A The atomic radius decreases in the following order: Y > Z > X.
B The electronegativity decreases in the following order: X > Z > Y.
C The electrical conductivity decreases in the following order: Y > Z > X.
D The first ionisation energy decreases in the following order: X > Y > Z.

6 One mole of the chloride of each of the following elements was mixed
separately with 1 dm3 of water. Which one of the following graph shows the pH
of the solutions produced?
A B

C D

ACJC 2013 8872/02/Sept/13 Preliminary Examination


www.gradtutors.com
4
4

7 In which of the following sequences are the species quoted in order of


decreasing boiling points?
A Ti, K, Ca
B I2, HF, HCl
C SCl2, AlCl3, MgCl2
D CCl4, CH4, CH2Cl2

8 The shapes of three species K, L and M are trigonal pyramidal, bent and
square planar respectively. Which of the following can be K, L and M?

K L M
A SO32- F2O XeF4
B BCl3 SnCl2 ICl4+
C PF3 CS2 BH4-
D ICl3 ICl2- BrF4-

9 The standard enthalpy change of formation of ethane is -84 kJ mol-1.


Given that the standard enthalpy change of combustion for carbon and ethane
are -393 kJ mol-1 and -1560 kJ mol-1 respectively, what is the standard enthalpy
change of combustion of hydrogen?
A -286 kJ mol-1
B -417 kJ mol-1
C +230 kJ mol-1
D +361 kJ mol-1

10 For which of the following is the lattice energy likely to have the greatest
numerical value?

A lithium fluoride
B lithium iodide
C rubidium chloride
D sodium chloride

ACJC 2013 8872/02/Sept/13 [Turn over


www.gradtutors.com
5
5

11 An experiment was carried out to investigate the initial rate of reaction between
ammonia peroxodisulfate, (NH4)2S2O8 and potassium iodide, KI.
S2O82-(aq) + 2I-(aq) → 2SO42-(aq) + I2(aq)
The initial concentrations of the (NH4)2S2O8 and KI solutions in the mixture,
together with the time taken for the mixture to darken for the various
experimental runs are given below:

Initial concentration Initial concentration Time taken to


Expt
of (NH4)2S2O8 / mol dm -3
of KI / mol dm -3 darken / s

1 0.10 0.20 35
2 0.05 0.20 70
3 0.10 0.067 105
4 0.02 0.75 ?

Which of the following statements about the reaction is correct?


A The rate constant has units of mol-1 dm-3 s-1.
B The reaction does not involve the formation of an intermediate.
C The time taken for the mixture to darken in Experiment 4 is 80 s.
D The slow step involves the reaction between 1 mole of (NH4)2S2O8 and
1 mole of KI.

12 Which suggested mechanism is consistent with the experimentally obtained


rate equation?

rate equation suggested mechanism


A rate = kA[H2][I2] (slow) H2 (g) → 2H (g)
(fast) 2H (g) + I2 (l) → 2HI (g)
B rate = kB[HBr][O2] (slow) 2HBr (g) + O2 (g) → 2HBrO (g)
(fast) HBrO (g) + HBr (g) → H2O (g) + Br2 (g)
2
C rate = kC[NO] [H2] (slow) 2NO (g) + H2 (g) → N2O (g) + H2O (g)
(fast) N2O (g) + H2 (g) → N2 (g) + H2O (g)
D rate = kD[NOBr2][NO] (fast) NO + Br2 NOBr2
(slow) NOBr2 + NO → 2NOBr

ACJC 2013 8872/02/Sept/13 Preliminary Examination


www.gradtutors.com
6
6

13 The graph shows the results of an investigation of the initial rate of


decomposition of hydrogen peroxide, H2O2 by the enzyme, catalase. The
concentration of catalase was constant in all experiments.

Which one of the following statements is false about the above reaction?
A When [H2O2] is low, the rate is first order with respect to [H2O2].
B Increasing the amount of catalase will increase the rate of the reaction.
C An increase in the system pressure will decrease the rate of
decomposition of H2O2.
D At temperatures above 40 oC, rate of reaction will decrease.

14 Two moles of compound P were placed in a vessel. The compound P was


partly decomposed by heating. A dynamic equilibrium between chemicals P, Q
and R was established. At equilibrium, x mol of R were present and the total
number of moles present was (2 + x). What is the equation for this equilibrium?
A P 2Q + R
B 2P 2Q + R
C 2P Q+R
D 2P Q + 2R

15 Methanol is manufactured by reacting carbon dioxide and hydrogen.


CO2(g) + 3H2(g) CH3OH(g) + H2O(g) ∆H = –49 kJ mol–1
What would increase the equilibrium yield of methanol in this process?
A adding a catalyst
B adding an excess of steam
C increasing the pressure
D increasing the temperature

ACJC 2013 8872/02/Sept/13 [Turn over


www.gradtutors.com
7
7

16 Water dissociates as follows: H2O(l) H+(aq) + OH-(aq). At a certain


temperature above 298 K, the ionic product of water is 1.0 x 10-13 mol2 dm-6
Which of the following deduction can be made from these information?
A The total concentration of ions in the equilibrium mixture is
6.32 x 10-7 mol dm-3
B The water is acidic at this temperature.
C The ionic product of water is defined as [H+][OH-] / [H2O]
D The dissociation of water is exothermic

17 Which of the following pairs of species cannot act as a buffer?


A NaHCO3 and H2CO3
B NaH2PO4 and Na2HPO4
C NaNO3 and HNO3
D CH3COONa and CH3COOH

18 Which formulae show propanone and propanal as different compounds?


A Empirical, molecular, structural and displayed formulae
B Molecular, structural and displayed formulae only
C Structural and displayed formulae only
D Displayed formulae only

19 Acrylic acid is produced from propene, a gaseous product of oil refineries.

Which statement about acrylic acid is not correct?


A Both bond angles x and y are approximately 120°.
B It decolourises aqueous bromine.
C It gives an orange precipitate with 2,4-dinitrophenylhydrazine reagent.
D It reacts with an alcohol to give an ester under appropriate conditions.

ACJC 2013 8872/02/Sept/13 Preliminary Examination


www.gradtutors.com
8
8

20 Which reaction would not give ethanoic acid as a product?


A Heating ethanenitrile, CH3CN, under reflux with dilute sodium hydroxide
B Heating ethanenitrile, CH3CN, under reflux with dilute sulfuric acid
C Heating ethanal under reflux with acidified sodium dichromate(VI)
D Heating ethanol under reflux with acidified sodium dichromate(VI)

21 The diagram shows the structure of the naturally-occurring molecule


cholesterol.

H3C
CH3
H
CH3 H

H H
HO
cholesterol
Student X claimed that the seventeen carbon atoms in the four rings all lie in
the same plane.
Student Y claimed that this molecule displays cis-trans isomerism at the C=C
double bond.
Which of the students are correct?
A both X and Y
B neither X nor Y
C X only
D Y only

ACJC 2013 8872/02/Sept/13 [Turn over


www.gradtutors.com
9
9

22 When 1-bromopropane is treated in succession with two reagents, X and Y, it


produces propanoic acid. What are reagents X and Y?
X Y
A NaOH(aq) H / Cr2O72–(aq)
+

B NaOH(aq) CO2
C KCN in ethanol HCl(aq)
D KCN in ethanol NaOH(aq)

23 An organic compound J reacts with sodium to produce an organic ion with a


charge of –3.
J reacts with NaOH(aq) to produce an organic ion with a charge of –1.
What could be the structural formula of J?
A HO2CCH(OH)CH2CO2H
B HO2CCH(OH)CH2CHO
C HOCH2CH(OH)CH2CO2H
D HOCH2COCH2CHO

24 How many isomeric esters have the molecular formula C4H8O2?


A 2 B 3 C 4 D 5

25 CH3COCH2CH2OH CH3COCH2CHO CH3CH(OH)CH2CH3 CH3CH2CH2CHO


W X Y Z
Which of these compounds can be oxidised by acidified dichromate(VI) solution
and also gives a positive response to Tollens’ reagent?
A W and X only
B W and Y only
C X and Z only
D Y and Z only

ACJC 2013 8872/02/Sept/13 Preliminary Examination


www.gradtutors.com
10
10

Section B
For each of the questions in this section, one or more of the three numbered statements 1 to 3 may
be correct.

Decide whether each of the statements is or is not correct (you may find it helpful to put a tick
against the statements that you consider to be correct).

The responses A to D should be selected on the basis of

A B C D

1, 2 and 3 1 and 2 2 and 3 1 only


are only are only are is
correct correct correct correct

No other combination of statements is used as a correct response.

26 A stream which contains a mixture of gaseous particles was passed through an


electric field. One of the particles, 1H+ was deflected as follows. Which of the
following particle(s) is/are correctly represented by the different line(s)?

1 Line (1): 4He2+


0
2 Line (2): −1 e
3 Line (3): 16O2-

27 A refractory material is one that retains its strength at high temperature. These
materials have chemical and physical properties which make them applicable
for structures which are exposed to environments with temperatures higher
than 2000oC.
Which of the following compound(s) can be used as a refractory material?
1 Silicon carbide
2 Magnesium oxide
3 Gallium trichloride

ACJC 2013 8872/02/Sept/13 [Turn over


www.gradtutors.com
11
11

28 The graph below shows the variation in the second ionisation energies for the
consecutive elements Q to Z in the Periodic Table, all with proton number
below 20.

8000

6000
-1
2 IE / kJ mol

4000

2000
nd

0
Q R S T U V W X Y Z
Elements
Elements

What can be deduced from the above?


1 Q has a smaller ionic radius than R.
2 Effervescence is observed when a magnesium strip is dipped into the
aqueous solution containing the chloride of element V.
3 The oxide of T dissolves in water to form a alkaline solution.

29 Which statements about the photochemical chlorination of ethane are correct?


1 Hydrogen gas is one of the products.
2 A propagation step in the mechanism is C2H6 + Cl • → C2H5• + HCl.
3 The initiation step is the homolytic fission of chlorine.

ACJC 2013 8872/02/Sept/13 Preliminary Examination


www.gradtutors.com
12
12

30 The compound cis-hex-3-enal is responsible for the characteristic smell of cut


grass. The human nose is particularly sensitive to this compound, being able to
detect 0.25 parts per billion in air.

cis-hex-3-enal

Which reagents will react with cis-hex-3-enal?


1 sodium
2 sodium borohydride
3 Fehling’s reagent

END OF PAPER

ACJC 2013 8872/02/Sept/13 [Turn over


www.gradtutors.com
2 13

Section A

Answer all questions in this section in the spaces provided

1 Drink driving is a common problem observed during festive seasons. To detect the For
Examiner’s
drivers who have violated the law, the traffic police would carry with them a portable use
breathalyser to test for alcohol content from the suspect’s breath. One of the first
chemical breathalysers was based on the oxidation process of ethanol using
acidified potassium dichromate(VI).
(a) Given the half equation for the oxidation of ethanol, using relevant [1]
information from the Data Booklet, write a balanced equation for oxidation of
ethanol using acidified potassium dichromate(VI).

CH3CH2OH + H2O → CH3CO2H + 4 H+ + 4 e–

…………………………………………………………………………………...........

(b) What colour change would you expect for a positive test? [1]

…………………………………………………………………………………...........

The ethanol content in a can of beer with a volume of 330 cm3 can be determined
in the laboratory by titrating a sample of beer with potassium dichromate(VI)
acidified by dilute sulfuric acid.

A 25.0 cm3 sample of beer was pipetted from a brand new can to a 250 cm3
volumetric flask. Deionised water was added to the flask until the 250 cm3 mark.
25.0 cm3 of the diluted beer was then pipetted out and it required 17.15 cm3 of
0.050 mol dm-3 of acidified potassium dichromate(VI) for complete titration.

(c) Calculate the number of moles of ethanol titrated. [2]

ACJC 2013 8872/02/Sept/13 [Turn over


www.gradtutors.com
3 14

(d) Calculate the number of moles of ethanol present in one can of beer. [2] For
Examiner’s
use

(e) Hence calculate the concentration of ethanol in the can of beer. [1]

[Total: 7]

2 1-bromobutane may be made by reacting together butan-1-ol, sodium bromide and


concentrated sulfuric acid. In one such synthesis,

• 35 g of sodium bromide was mixed with 25 cm3 of butan-1-ol and 30 cm3 of


water in a flask.
• 25 cm3 of concentrated sulfuric acid was then added dropwise into the flask
containing the reagents.
• As the acid was added the contents were well shaken and cooled occasionally
in an ice bath.
• Once all the acid has been added, the mixture was gently boiled for some time
shaking the flask gently every now and then.
• The apparatus was then rearranged to distil off the 1-bromobutane

The reaction equations are as follows:

NaBr + H2SO4 → NaHSO4 + HBr


CH3CH2CH2CH2OH + HBr → CH3CH2CH2CH2Br + H2O

Substance Boiling point / oC Density / g cm-3 Mr


1-bromobutane 102 1.35 137
Butan-1-ol 118 0.81 74
NaBr 1390 3.20 103
Concentrated H2SO4 330 1.84 98

ACJC 2013 8872/02/Sept/13 Preliminary Examination


www.gradtutors.com
4 15

(a) Despite the larger Mr, 1-bromobutane has a lower boiling point than [2] For
Examiner’s
butan-1-ol. Why? use

…………………………………………………………………………………

…………………………………………………………………………………

…………………………………………………………………………………

…………………………………………………………………………………

…………………………………………………………………………………

(b) Based on the amounts used in the synthesis above, one of the reagents, [2]
butan-1-ol or sodium bromide will be present in an excess. Use the data
above to determine which reagent is in excess.

(c) After distillation and purification, 20 cm3 of 1-bromobutane was collected. [2]
Calculate the percentage yield in the above preparation.

ACJC 2013 8872/02/Sept/13 [Turn over


www.gradtutors.com
5 16

(d) In the light of your answer to (c) suggest why is the percentage yield not [1] For
Examiner’s
100%? use

………………………………………………………………………………….

………………………………………………………………………………….

(e) During the dropwise addition of concentrated sulfuric acid to the reaction [2]
mixture, occasional cooling was necessary. What does this suggest about
the nature of reaction taking place and why is cooling necessary?

………………………………………………………………………………….

………………………………………………………………………………….

………………………………………………………………………………….

………………………………………………………………………………….

(f) What type of reaction took place between butan-1-ol and the HBr generated [1]
in the first step?

…………………………………………………………………………………

(g) In the light of your answer to (f), why was it necessary for the mixture to be [1]
boiled for some time?

………………………………………………………………………………….

………………………………………………………………………………….

(h) By means of a simple chemical test, how will you distinguish between [1]
1-bromobutane and butan-1-ol.

[Total: 12]

ACJC 2013 8872/02/Sept/13 Preliminary Examination


www.gradtutors.com
6 17

3 Silicon is a tetravalent metalloid, which is less reactive than carbon, but more For
Examiner’s
reactive than germanium. The element has a large impact on the modern world use
economy, as it is used in chemical and semiconductor industries. Silicon is also an
essential element in biology and is of much importance to the metabolism of plants.

(a) (i) The atomic radii of silicon and aluminium are given as follows: [3]

Element Silicon Aluminium

Atomic radius/ nm 0.117 0.143

Account for the difference between the atomic radius of silicon and
aluminium.

……………………………………………………………………………………

……………………………………………………………………………………

……………………………………………………………………………………

……………………………………………………………………………………

……………………………………………………………………………………

(ii) The melting points of silicon and phosphorus are given as follows:

Element Silicon Phosphorus


Melting point/ oC 1414 44.2

Account for the difference in the melting points of the 2 solids.

…………………………………………………………………………………..

………………………………………………………………………………......

…………………………………………………………………………………..

…………………………………………………………………………………...

(b) Silicon tetrachloride is used as an intermediate in the manufacture of high [3]


purity silicon. Silicon derived from silicon tetrachloride is used in the
semiconductor industry and also the production of photovoltaic cells.

(i) Illustrate the formation of silicon tetrachloride from its elements with an
equation, including state symbols.

……………………………………………………………………………………

ACJC 2013 8872/02/Sept/13 [Turn over


www.gradtutors.com
7 18

(ii) Describe, with the aid of a suitable equation, how and why silicon For
Examiner’s
tetrachloride is able to react with water. use

……………………………………………………………………………………

……………………………………………………………………………………

……………………………………………………………………………………

(c) The following graph shows a plot of the successive ionisation energies (IE) of [2]
silicon:
40000

35000

30000

Energy / 25000
kJ mol-1
20000

15000

10000

5000

0
1st 2nd 3rd 4th 5th 6th 7th 8th 9th 10th

(i) Account for the general increase in successive ionisation energies.

……………………………………………………………………………………

……………………………………………………………………………………

……………………………………………………………………………………

……………………………………………………………………………………

(ii) Account for the large increase between 4th and 5th ionisation energies.

……………………………………………………………………………………

……………………………………………………………………………………

……………………………………………………………………………………

……………………………………………………………………………………

[Total: 8]

ACJC 2013 8872/02/Sept/13 Preliminary Examination


www.gradtutors.com
8 19

4 (a) 1-iodopropane can be hydrolysed by NaOH(aq) to form propan-1-ol. Results [5] For
Examiner’s
of an investigation into the kinetics of this reactions are given below. use

Experiment Concentration / mol dm-3 Relative


number 1-iodopropane OH- initial rate
1 0.10 0.20 1.00
2 0.20 0.40 4.00
3 0.30 0.20 3.00
4 0.40 0.40 8.00

(i) Determine the order of reaction with respect to 1-iodopropane and OH-.

(ii) Hence give the rate equation for the hydrolysis of 1-iodopropane.

………………………………………………………………………………….

(iii) Draw a fully labeled energy profile diagram for this hydrolysis.

ACJC 2013 8872/02/Sept/13 [Turn over


www.gradtutors.com
9 20

(b) 15 drops of 1-chloropropane, 1-bromopropane and 1-iodopropane was put in [2] For
Examiner’s
3 separate test tubes. Dilute silver nitrate solution was added to each test use
tube and all 3 test tubes were heated in a water bath. The following
observations were obtained.

Test tube Colour of ppt Time taken


1 Cream After 8 minutes
2 White After 20 minutes
3 Yellow After 1 minute

(i) Identify the halogen derivative in each test tube.

Test tube 1: ………………………………………………………………….

Test tube 2: ………………………………………………………………….

Test tube 3: ………………………………………………………………….

(ii) Explain why the time taken in all 3 test tubes is different.

…………………………………………………………………………………..

…………………………………………………………………………………..

…………………………………………………………………………………..

(c) A student attempted to determine the enthalpy change of combustion of [6]


propan-1-ol in the laboratory. She set up the experiment according to the
diagram given below:

thermometer

can containing
300 g of water

burner containing
propan-1-ol

The following data were recorded:


Volume of water in beaker = 300 cm3
Initial temperature of water = 27 oC
Final temperature of water = 37 oC
Initial mass of spirit lamp with propan-1-ol = 158.65 g
Final mass of spirit lamp with propan-1-ol = 158.25 g

ACJC 2013 8872/02/Sept/13 Preliminary Examination


www.gradtutors.com
10 21

(i) Using the information given above, calculate the enthalpy change of For
Examiner’s
combustion of propan-1-ol. use

(Specific heat capacity of water = 4.18 J g−1 K−1)


(Mr of propan-1-ol = 60.0)

(ii) The student compared the enthalpy change of combustion that she
obtained to the value found in the textbook. The value given in the
textbook is -2020 kJ mol-1. Suggest a reason for the difference.

…………………………………………………………………………………..

(iii) Use the bond energies given in the Data Booklet to calculate another
value for the standard enthalpy change of combustion of propan-1-ol.

(iv) Suggest a reason for the discrepancy between this value and that
quoted in (c)(ii).

…………………………………………………………………………………..

[Total: 13]

ACJC 2013 8872/02/Sept/13 [Turn over


www.gradtutors.com
11 22

Section B

Answer two questions from this section on separate answer paper

5 (a) It was discovered that butylpropanoate is a commonly used ester in food flavouring. [9]
This ester can be made in the laboratory by heating butan-1-ol and propanoic acid
with trace amount of concentrated sulfuric acid according to the equation given.
O

CH3 CH 2 CH 2CH 2OH (l ) + CH3 CH 2 COOH ( l ) CH3 CH 2 C OCH 2CH 2CH2 CH 3 ( l) + H2 O (l )

(i) Concentrated sulfuric acid plays 2 roles in the above reaction. What are the 2
roles?
(ii) Write the Kc expression.
(iii) The synthesis of butylpropanoate was done using the method suggested
above. At equilibrium, the concentration of butan-1-ol and propanoic acid was
found to be 0.480 mol dm-3 and butylpropanoate and water was found to be
1.32 mol dm-3. Calculate the value of Kc.
(iv) State and explain what would happen to the position of equilibrium and Kc value
if more butan-1-ol is added to the reaction mixture.
(v) The structure of butylpropanoate is given below. State the hybrid state of Ca
and Cb, and shape and angle around Ca and Cb.
O

Ca
H3 CH 2 Cb OCH 2 CH 2CH 2CH3

(b) Butan-2-ol can be formed from butan-1-ol via this pathway in the laboratory: [4]
Step I Step II
CH 3 CH 2CH2 CH2 OH A CH 3 CH 2CH(OH)CH3

(i) Give the reagents and conditions needed for Step I and II.
(ii) Draw the structure of A.
(iii) State and explain whether A can exhibit geometrical isomerism.

(c) B and C are isomers with molecular formula C4H10O. B and C react with sodium to [7]
give a gas which extinguishes a lighted splint with a “pop” sound. When heated with
acidified manganate(VII), B and C decolourise manganate(VII) to give D and E
respectively. D does not react with NaHCO3 but gives an orange precipitate upon
adding 2,4-dinitrophenylhydrazine and yellow precipitate with aqueous alkaline iodine.
E reacts with NaHCO3 giving a colourless and odourless gas. C reacts with hot
concentrated H2SO4 to give an alkene. The alkene decolorizes hot acidified
manganate(VII) to give a neutral organic compound with the formula C3H6O.
Deduce the structures of B, C, D and E with reasoning.
[Total: 20]

ACJC 2013 8872/02/Sept/13 Preliminary Examination


www.gradtutors.com
12 23

6 Aluminium is the third most abundant element and most abundant metal in the Earth’s crust.
Aluminium is highly sought after and has many commercial uses in the aerospace and
construction industries, due to its low density and its ability to resist corrosion.
(a) Aluminium oxide is produced or extracted from minerals on a large scale. It is a [2]
common catalyst for many industrial processes and also widely used as an abrasive
due to its hardness.
Explain, with the aid of equations, the amphoteric properties of aluminium oxide.

(b) Some of the common compounds of aluminium are given as follows: [4]

AlCl3 AlF3 Al2O3 AlBr3


(i) Arrange the compounds of aluminium in increasing order of melting point.
(ii) Explain your answer in (b)(i), in relation to the structure and bonding of the
aluminium compounds.

(c) Aluminium chloride is used in petroleum refining and in the production of synthetic [5]
rubber and polymers. Its anhydrous form is a powerful Lewis acid, which is an
electron-pair acceptor, and reacts vigorously with bases, hence suitable precautions
must be taken when handling this compound.
(i) Draw a dot-and-cross diagram showing the arrangement of outer shell electrons
in AlCl3. Suggest the shape of the molecule and the bond angle about the central
atom.
(ii) Draw the structure of AlCl3 upon dimerization. Hence, suggest if all of the
chlorine atoms lie on the same plane.

(d) Friedel-Crafts alkylation reactions are developed in 1877 to attach alkyl-substituents to [9]
an aromatic ring. These reactions are usually catalysed by anhydrous aluminium
chloride.
The following equation illustrates a Friedel-Craft alkylation reaction:

AlCl 3
+ R-Cl R + HCl

(i) Benzene reacts with compound F, in the presence of anhydrous AlCl3 to form
compound G, C8H10.
Deduce the structure and physical state of compound F.
(ii) (1) Oxidation of compound G in the presence of hot MnO2(s) produces
compound H, C7H6O, which forms an orange precipitate with 2,4-
dinitrophenylhydrazine.
Compound J is formed when compound G is reacted with chlorine gas in
the presence of UV light. When J is heated under reflux with NaOH(aq),
compound K is formed. When K is heated at high temperatures and
pressure with Al2O3(s), compound L is formed.
When K is heated with acidified K2Cr2O7(aq), compound M is formed, which

ACJC 2013 8872/02/Sept/13 [Turn over


www.gradtutors.com
13 24

also forms an orange precipitate with 2,4-dinitrophenylhydrazine.


Deduce the structures of compounds H, J, K, L and M.

(2) Construct a balanced equation to illustrate the reaction between compound


M and 2,4-dinitrophenylhydrazine.
(3) By means of a simple chemical test, suggest how you could distinguish
between compounds H and M.
[Total: 20]

7 (a) 3-chloropropanoic acid is a drug used in scientific research due to its structural [4]
resemblance to 4-hydroxybutanoic acid, a naturally occurring substance found in the
central nervous system.
O

Cl OH
3-chloropropanoic acid
(i) The pKa of propanoic acid is 4.87. Predict, with reasons, how the pKa of
3-chloropropanoic acid would differ from propanoic acid.
(ii) A positional isomer of 3-chloropropanoic acid is 2-chloropropanoic acid. Compare
the acid strength of 2-chloropropanoic acid with 3-chloropropanoic acid. Explain
your answer.

(b) 0.020 mol of 3-chloropropanoic acid was dissolved in 50.0 cm3 of water to produce [9]
FA1 which was then titrated with FA2, a 0.50 mol dm–3 solution of NaOH. After
20.0 cm3 of NaOH was added, the pH of the solution was found to be 3.98.
(i) Determine the pKa of 3-chloropropanoic acid.
(ii) The resultant mixture from (b) acts as a buffer mixture. By means of balanced
equations illustrate how this mixture acts as a buffer.
(iii) Calculate the pH of the solution FA1.
(iv) What will be the pH of the solution mixture at the equivalent point if FA1 was
neutralized completely with FA2.

(c) Outline how you would synthesise 2-chloropropanoic acid starting from ethanal, stating [5]
reagents and conditions for each step of the conversion.

(d) By means of a simple chemical test, suggest how you could distinguish between [2]
3-chloropropanoic acid and propanoic acid.

[Total: 20]

ACJC 2013 8872/02/Sept/13 Preliminary Examination


www.gradtutors.com
25
H1 Prelims 2013 Suggested Solutions

Paper 1

1 C 11 D 21 B
2 C 12 C 22 A
3 D 13 C 23 C
4 B 14 B 24 C
5 C 15 C 25 C
6 A 16 A 26 D
7 B 17 C 27 B
8 A 18 C 28 C
9 A 19 C 29 C
10 A 20 A 30 C

Paper 2 Section A

1 (a) (i) 2Cr2O72- + 16H+ + 3 CH3CH2OH → 4Cr3+ + 3CH3CO2H + 11H2O

(ii) Orange to green

(iii) No. of moles of potassium dichromate

= 0.01715 x 0.05

= 8.58 x 10-4

2Cr2O72- : 3CH3CH2OH

No. of moles of ethanol in 25 cm3

= 8.58 x 10-4/2 x 3 = 1.287 x 10-3

(iv) No. of moles of ethanol in 250 cm3

= 1.287 x 10-3 x 10 = 0.01287

No. of moles of ethanol in 25 cm3 sample = 0.01287

No. of moles of ethanol in a can

= 0.01287 x 330/25

= 0.1698

= 0.170 (3 s.f.)

(v) [ethanol] = 0.170 / 0.330 = 0.515 mol dm-3

2 (a) • Butan-1-ol forms H-bonds as intermolecular forces


• H-bonds stronger than the VDW between bromobutane molecules
(b) • n(butan-1-ol) = (25 x 0.81)/74 = 0.274 mol
• nNaBr = 35 / 103 = 0.340 mol
• NaBr in excess
(c) • Theoretical Yield = 0.274 x 137 = 37.5g
• Experimental Yield = 20 x 1.35 = 27.0g
• % yield = 27 x 100/37.5 = 72.0%

www.gradtutors.com
26
(d) • loss of alcohol due to evaporation
• formation of by-products
(e) • Reaction is exothermic
• To prevent further oxidation of HBr or dehydration of alcohol
(f) • Substitution

(g) • Substitution involves breaking of strong covalent bonds in the reactants and hence
heat energy is required.
(h) • Na metal
• Brisk effervescence of hydrogen with butan-1-ol but not with 1-bromobutane.

3 (a) (i) Across the period, effective nuclear charge increases, hence the electrons in the
outermost principal quantum shell experience stronger electrostatic forces of
attraction to the nucleus. Atomic radius of Si is smaller than that of Al.

(ii) Silicon is a giant covalent molecule, whereby the silicon atoms are held by strong
covalent bonds. Phosphorus is a simple covalent non-polar molecule, whereby the
molecules are held together by weak induced dipole-induced dipole interactions.
Hence more energy is required to overcome the strong covalent bonds in Si, hence
Si has a higher melting point.

(b) (i) Si (s) + 2Cl2 (g) → SiCl4 (l)

(ii) SiCl4 (l) + 4H2O (l) → SiO2.2H2O (s) + 4HCl (aq)

Si has low-lying, vacant 3d-orbitals available to form dative covalent bonds with
water molecules, hence SiCl4 will hydrolyse completely in water.

(c) (i) With the removal of each electron, the proton-to-electron ratio increases, hence
remaining electrons are more strongly attracted to the nucleus, hence more energy
is required to remove the subsequent electrons.

(ii) 4th electron is removed from 3s-subshell, whereas 5th electron is removed from the
inner 2p-subshell, which is of lower energy as compared to the previous. Hence,
more energy will be required to remove the 5th electron.

4 (a) (i) Using expt 1 and 3, where [OH-] is kept constant, when [1-iodopropane] is tripled,
rate is tripled. Hence first order wrt 1-iodopropane.

Using expt 1,

Rate = k[1-iodopropane][OH-]

1 = k (0.10) (0.20)n

Using expt 2,

4 = k (0.20) (0.40)n

1 0.1(0.2)n
=
4 0.2(0.4)n

n = 1 Hence it is first order wrt to OH-

www.gradtutors.com
27
(ii) rate = k [1-iodopropane][OH-]

(iii)

energy

Ea

CH3CH2CH2Br ∆H CH3CH2CH2OH
r + OH- + Br-

Reaction
pathway

(b) (i) Test tube 1: 1-bromopropane


Test tube 2: 1-chloropropane
Test tube 3: 1-iodopropane
(ii) C-I bond is weakest and most readily broken hence the yellow ppt was obtained
very quickly.
OR
C-Cl bond is the strongest and least readily broken hence the white ppt was
obtained the slowest.
(c) (i) Heat evolved = mc∆T = 300 x 4.18 x 10 = 12540 J

Amount of ethanol used = 0.4 / 60 = 6.67 x 10-3

∆Hc = - Q / n = - 12540 / 6.67 x 10-3 = 1881 kJ mol-1

(ii) Heat loss to the surroundings.

(iii) Bond breaking = 2(350)+7(410)+360+460+9/2(496) = +6622 kJ mol-1

Bond forming = 6(740)+8(460) = -8120 kJ mol-1

∆Hc = +6622 – 8120 = -1498 kJ mol-1 = -1500 kJ mol-1

(iv) Heat of vaporization was not taken into account as the values of bond energies are
meant for gaseous particles.

Paper 2 Section B

5 (a) (ii) Conc H2SO is a catalyst.

Conc H2SO4 is a dehydrating agent. It removes H2O and POE shifts right
generating more ester, thus increasing the yield.

(iii) [CH3CH2COOCH2CH2CH2CH3 ][H2O]


Kc =
[CH3CH2CH2CH2OH ][CH3CH2COOH ]

(iv) Kc = (1.32)2 / (0.48)2 = 7.56 (dimensionless)

www.gradtutors.com
28
(v) By LCP, increasing [butan-1-ol] causes equilibrium to shift to the right.

Kc value remains unchanged.

(vi) Ca: sp2, trigonal planar, 120o


Cb: sp3, tetrahedral, 109.5o
(b) (i) Step I: conc H2SO4, 170oC

Step II: conc H2SO4 0 oC followed by boiling with H2O

(ii) H H

C C

H3CH 2C H

(iii) No. There are 2 H atoms on the same sp2 C.

(c) Evidence Deduction


B and C has a formula C4H10O which B and C are alcohol
reacts with Na to give a gas Gas produced is H2
B and C reacts with manganate(VII) to B and C is either a primary or
give D and E secondary alcohol
D does not react with NaHCO3 D is not an acid
D reacts with 2,4-DNPH D is a ketone
D reacts with aq alkaline I2 D has the structure CH3C=O
E reacts with NaHCO3 E is an acid
C reacts with hot conc H2SO4 Elimination reaction
+
Alkene with hot KMnO4/H gives a Oxidation reaction
neutral compound C3H6O Propanone is formed.

H3 C C CH 2 CH 3

B: OH

CH 3

H3 C C CH 2 OH

C: H

D: H3 C C CH 2 CH 3

www.gradtutors.com
29
CH 3

H3 C C COOH

E: H

6 (a) Al2O3 (s) + 6H+ (aq) → 2Al3+ (aq) + 3H2O (l)

Al2O3 (s) + 2OH- (aq) + 3H2O (l) → 2[Al(OH)4]- (aq)

(b) (i) AlCl3 < AlBr3 < AlF3 < Al2O3

(ii) AlF3 and Al2O3 are ionic compounds, while AlCl3 and AlBr3 are simple covalent
molecules.

The magnitude of lattice energy is a measure of the strength of an ionic bond and
q+ q-
given as: LE α r .
+ + r-

Although ionic radius of F- is smaller than ionic radius of O2-, but O2- carries a
charge of -2, while F- carries a charge of -1. Hence the ionic bond strength in Al2O3
is stronger and hence it has a higher melting point.

AlCl3 has a smaller electron cloud size than that of AlBr3, hence is less polarisable,
thus the induced dipole-induced dipole interactions are weaker in AlCl3 and it has a
lower melting point.

(c) (i) Dot-and-cross diagram of AlCl3

Shape of AlCl3: Trigonal planar

Bond angle: 120o

(ii)

No. The chlorine atoms do not lie on the same plane.

(d) (i) H H

H C C H

Structure of F: H Cl

Physical state of F: Liquid

(ii) (1) H
C
Structure of H: O

www.gradtutors.com
30

H H
C C H

Structure of J: Cl H

H H
C C H

Structure of K: OH H

H
C
C H
Structure of L: H

H
C C H

Structure of M: O H

(2) H
O2N

C C H + H N N NO2
O H
H H

H
H
C
H
C + H2O
O 2N
N

N NO2

(3) • NH3(aq), AgNO3(aq), warm (Tollen’s Reagent)


• Silver mirror will be formed for C but no silver mirror is formed for G

• NaOH(aq), I2(aq), warm (to 60oC)


• Yellow crystals of CHI3 are formed for G but not for C

www.gradtutors.com
31
7 (a) (i) • pKa of 3-chloropropanoic acid would lower than that of propanoic acid
• electron withdrawing chlorine helps to disperse the negative charge on the anion hence
reducing its basicity
(ii) • 2-chloropropanoic acid is stronger than 3-chloropropanoic acid
• Proximity of electron withdrawing chlorine to the carboxylate increases the electron-
withdrawing effect
(b) (i) At half-neutralisation, the pH = pKa = 3.98

(ii) • ClCH2CH2COOH + OH- → ClCH2CH2COO- + H2O


• ClCH2CH2COO- + H+ → ClCH2CH2COOH
(iii) • [H+] = (10-3.98 x 0.4)1/2 = 6.47 x 10-3
• pH = 2.19
(iv) • At neutralization, total volume = 90.0 cm3
• [ClCH2CH2COO-] = 0.02 / 0.09 = 0.222 mol dm -3
• Kb (ClCH2CH2COO- ) = 10-14/10-3.98 = 9.55 x 10-11 mol dm-3
• [OH-] = (9.55 x 10-11 x 0.222)1/2 = 4.60 x 10-6 mol dm-3
• pH = 8.66
(c) • Step 1 HCN, CN- 10 – 20 oC
• Step 2 PCl5 , RT
• Step 3 dilute H+, heat under reflux
(d) • NaOH (aq), boil and acidify with HNO3(aq), followed by AgNO3(aq)
• White ppt with 3-chloropropanoic acid but not with propanoic acid

www.gradtutors.com
32

ANDE
ERSON
N JUNIO
OR COLL
LEGE
2013 JC2
2 PRELIM
MINARY
Y EXAMIN
NATIONS
S

CHEMIISTRY 8872/01
8
Higher 1 233 Septemb
ber 2013
Paper 1 Multiple Choice
C 50 minutes

Additiona
al Materials: Multiple
M Cho
oice Answer Sheet
Data
D Bookle
et

HESE INST
READ TH TRUCTIONS
S FIRST

Write in ssoft pencil.


Do not usse staples, paper clips, highlighte rs, glue or correction
c fluid.

There arre thirty qu a questions. For eachh question there are


uestions on this paperr. Answer all
four posssible answe
ers A, B, C and
a D.
Choose the one yo ou considerr correct an
nd record your
y choice in soft peencil on the
e Multiple
Choice AAnswer Sheet.

Each corrrect answer will score one mark. A mark will not be deducted for a wrong answ
wer.
Any roug
gh working should
s be done in this booklet.

Mu
ultiple Cho
oice Answe
er Sheet

W
Write your na
ame, PDG and
a NRIC / FIN numbe
er, including
g the refereence letter.

Sh
hade the NR
RIC / FIN nu
umber.

Exxam Title: JC2 Prelim

Exxam Detailss: H1 Chem


mistry / Pape
er 1

Da
ate: 23/09/2013

This document
d cconsists of 17
1 printed pages.
p

AJC JC2 P
PRELIM 2013 8872/01/H1 ver
[Turn ov
www.gradtutors.com
33
2

Section A

For each question there are four possible answers A, B, C and D. Choose the one you consider to be
correct.

1 Use of the Data Booklet is relevant to this question.

Which of the following statements is incorrect?

A 35.5 g of chlorine gas contains 6.0 ×1023 chlorine atoms.

B 24 dm3 of hydrogen gas at 25 °C and 1 atm contains 1.2 ×1024 hydrogen atoms.

C 500 cm3 of 1 mol dm–3 aqueous magnesium nitrate(V) contains 3.0 × 1023 nitrate(V)
ions.

D 4 g of helium gas contains 6.0 ×1023 helium atoms.

2 Which mass of gas would occupy a volume of 3 dm3 at 25 oC and 1 atmospheric pressure?
[1 mol of gas occupies 24 dm3 at 25 oC and 1 atmospheric pressure]

A 3.2 g O2 gas

B 5.6 g N2 gas

C 8.0 g SO2 gas

D 11.0 g CO2 gas

3 The table below contains incomplete information about the three isoelectronic ions X, Y and
Z. The atoms of Y and Z are isotopes.

mass atomic number of number of


ion charge
number number neutrons electrons
X 55 25 b

Y 56 26

Z a 31 +3

What are the values of a and b?

a b

A 56 +2

B 56 +3

C 57 +2

D 57 +3

AJC JC2 PRELIM 2013


www.gradtutors.com
8872/01/H1
34
3

4 Use of the Data


a Booklet is relevant to this questio
on.

243
94 P ergo a natural radioacttive decay, where one of its electrrons enters the nucleus
Pu can unde
to ch
hange a pro
oton into a neutron,
n to fform a new element M.

Wheen M is putt in an ionis


sation cham mits a high energy -pparticle (which is a 4He
mber, it em
nucle
eus).

electron
1

2
3

at is the iden
Wha ntity of the element
e Ma mitted -partticle in an electric
and the patth of the em e field?
?

Chemicaal symbol Path


h of
off M (–partticles)
3
243
A 3
93 M 2

3
243
B 5
95 M 1

4
244
C 3
93 M 2

4
244
D 5
95 M 3

5 Amo
oxicillin is on
ne of the mo
ost common
n antibiotics
s prescribed
d for childreen.

Amoxic
cillin

w many lone
How ectrons are present in this molecu
e pairs of ele ule?

A 13 B 15
1 C 17
1 D 119

[Turn ov
ver
www.gradtutors.com
AJC JC2 PRE
ELIM 2013 8872/01/H1
35
4

6 The African weaver ant defends its territory by spraying an intruder with a mixture of
compounds. The ease by which these compounds are detected by other ants depends upon
the volatility, which decreases as the strength of the intermolecular forces in the compound
increases.

Which compound in the mixture would be the most volatile?

A CH3CH2CH2CH2CH2CH3

B CH3CH2CH2CH2CH2CHO

C CH3CH2CH2CH2CH2COOH

D CH3CH2CH2CH2CH2OH

7 Trimethoprim (TMP) is used for the treatment and prevention of urinary tract infection,
traveller’s diarrhea, respiratory and middle ear infections. It has the following structure.

OCH3
OCH3 NH2
H w H N
 
H C O C N H
x y z
H H N H

In which sequence is the bond angles quoted in decreasing order?

A w=y>x>z

B x>y>w>z

C x>y>z>w

D w>x>y>z

AJC JC2 PRELIM 2013


www.gradtutors.com
8872/01/H1
36
5

8 The conversion of diamond into graphite is an exothermic reaction. Diamond does not readily
change into graphite.
Which reaction pathway correctly represents this conversion?

A B

C D

9 The table shows the enthalpy change of neutralisation, H, for the various acids and bases
listed.

acid base H / kJ mol–1


sulfuric acid sodium hydroxide –57.0
P sodium hydroxide Less exothermic than –57.0
sulfuric acid Q Less exothermic than –57.0
R potassium hydroxide –57.0

What are the likely identities of P, Q and R?

P Q R

A hydrochloric acid lithium hydroxide nitric acid

B phosphoric acid ammonia ethanoic acid

C nitric acid lithium hydroxide hydrochloric acid

D hydrogen cyanide ammonia nitric acid

AJC JC2 PRELIM 2013 8872/01/H1 [Turn over


www.gradtutors.com
37
6

10 Ammonia is manufactured by the Haber process, in an exothermic reaction.

Assuming that the amount of catalyst remains constant, which change will not bring about an
increase in the rate of the forward reaction?

A decreasing the size of the catalyst pieces

B increasing the pressure

C increasing the temperature

D removing the ammonia as it is formed

11 The value of the equilibrium constant, Kc, for the reaction to form ethyl ethanoate from
ethanol and ethanoic acid is 4.0 at 60 oC.

C2H5OH + CH3CO2H CH3CO2C2H5 + H2O

What is the number of moles of ethyl ethanoate formed when 1.0 mol of ethanol and 1.0 mol
of ethanoic acid are allowed to reach equilibrium at 60 oC?

1 2 1 3
A B C D
3 3 4 4

12 Bromocresol green is an acid–base indicator with a pH range of 3.8 to 5.4. The acidic colour
of the indicator is yellow and the alkaline colour is blue.

Two drops of the indicator are added to each of the four aqueous solutions listed below.

Which solution has its colour correctly stated?

Solution Colour

A Blood plasma Green

B Aqueous solution of MgCl2 Yellow

C Dilute HCl of concentration 3.0 x 10–5 mol dm–3 Yellow

D Aluminium oxide added to water Blue

13 Which of the following equations does not show a Brønsted acid−base reaction?

A Na+NH2− + NH4+Cl− Na+Cl− + 2NH3

B 2NCl3 + 6NaOH N2 + 3NaCl + 3NaOCl + 3H2O

C OCl− + H2O OH− + HOCl

D H2O + SO32− OH− + HSO3−

AJC JC2 PRELIM 2013


www.gradtutors.com
8872/01/H1
38
7

14 Below shows the Maxwell–Boltzmann distribution curve for a chemical reaction in living
systems.

fraction of molecules
with a given energy

fraction of molecules with


energy > Ea at a particular
temperature

kinetic
0 Ea energy
(activation energy)

Which of the following deductions is correct?

A The total area under the graph will increases at high temperature.

B The shaded area under the graph will decrease when enzymes are denatured.

C An increase in the surface area of the reactants decreases the Ea.

D The order of reaction with respect to the chemical reaction is first order when the kinetic
energy is low.

15 Why is the ionic radius of a chloride ion larger than the ionic radius of a sodium ion?

A A chloride ion has one more occupied electron shell than a sodium ion.

B Chlorine has a higher proton number than sodium.

C Ionic radius increases regularly across the third period.

D Sodium is a metal whilst chlorine is a non−metal.

AJC JC2 PRELIM 2013 8872/01/H1 [Turn over


www.gradtutors.com
39
8

16 The diagram represents the first ionisation energy of four consecutive elements in the third
period of the Periodic Table, with element R showing two oxidation states in its chlorides.

1st IE

R S
Q

proton number

The sketches below represent another two properties of these elements.

Property M Property N
Q

P Q P

R
S
R S

proton number proton number

What are properties M and N?

property M property N

A pH of oxides in water melting point

B ionic charge 5th ionisation energy

C ionic radius melting point

D electrical conductivity 5th ionisation energy

AJC JC2 PRELIM 2013


www.gradtutors.com
8872/01/H1
40
9

17 Elements X and Y are both in Period 3 of the Periodic Table.

When the chloride of X is added to water, it reacts and a solution of pH 1 is produced.

When the chloride of Y is added to water, it dissolves and a solution of pH 7 is produced.

Which statement explains these observations?

A Both chloride hydrolyse in water.

B X is phosphorus and Y is aluminium.

C X is silicon and Y is sodium.

D X is sodium and Y is phosphorus.

18 How many possible products (including stereoisomers) can be formed when the following
compound is reacted with excess concentrated sulfuric acid at 180 C?

OH
CH3CH2 C CH(CH3)2
CH3

A 3 B 4 C 5 D 6

AJC JC2 PRELIM 2013 8872/01/H1 [Turn over


www.gradtutors.com
41
10

19 A hydrocarbon, on heating with an excess of hot concentrated acidified KMnO4(aq), produces


CH3COCH2COOH as the only organic product.

Which of the following is not a possible structure of the hydrocarbon?

AJC JC2 PRELIM 2013


www.gradtutors.com
8872/01/H1
42
11

20 Limonene is a constituent of lemon oil and is a useful starting material for the manufacture of
perfumes.

limonene

How many σ and  bonds are there in a molecule of limonene?

 

A 8 4

B 10 2

C 24 4

D 26 2

21 Compound Z, which has an aromatic ring structure, is subjected to oxidative degradation


under suitable conditions.

What are the most likely organic products from this reaction?

O OH O
A B
O O

OH OH

O O O

HO HO OH
C D
HO HO

O O

AJC JC2 PRELIM 2013 8872/01/H1 [Turn over


www.gradtutors.com
43
12

22 The following compound is treated with hot ethanolic potassium hydroxide.

Br

Br Br
Which of the following is not a possible product?

Br

Br

C
Br Br

D
Br

AJC JC2 PRELIM 2013


www.gradtutors.com
8872/01/H1
44
13

23 Which of the following will not give tri–iodomethane on warming with alkaline aqueous
iodine?

A N
H

OH
I OH
B
I O

O
I
C

O
Cl
D
Cl
Cl

24 Which of the following involves a pair of addition reactions?

Reaction 1 Reaction 2

A But–1–ene  2–chlorobutane Butanone + 2,4–dinitrophenylhydrazine

B Butanal  2–hydroxypentanoic acid But–1–ene + PCl5

C But–1–ene  butanone But–1–ene + HCN

D But–1–ene  butan–2–ol Butanal + HCN

AJC JC2 PRELIM 2013 8872/01/H1 [Turn over


www.gradtutors.com
45
14

25 Potassium sodium tartrate, also known as Rochelle salt, is used medicinally as a laxative and
has the following structure.

Which of the following could be part of the reaction sequence to synthesise Rochelle salt?

alcoholic KCN KOH(aq) + NaOH


A CH2BrCH2Br intermediate Rochelle salt
reflux reflux

alcoholic KCN KMnO4 , KOH


B HOCH2(CH2)2Cl intermediate Rochelle salt
reflux reflux

HCN(aq) + NaCN KOH(aq) + NaOH


C CHOCHO intermediate Rochelle salt
10 C - 20 C reflux

HCN(aq) + NaCN KMnO4 , NaOH


D CH3COCH3 intermediate Rochelle salt
10 C - 20 C reflux

AJC JC2 PRELIM 2013


www.gradtutors.com
8872/01/H1
46
15

Section B

For each of the question in this section, one or more of the three numbered statements 1 to 3 may be
correct.

Decide whether each of the statements is or is not correct (you may find it helpful to put a tick against
the statements that you consider to be correct.)

The responses A to D should be selected on the basis of

A B C D
1, 2 and 3 1 and 2 2 and 3 1 only
are correct only are correct only are correct is correct

No other combination of statements is used as a correct response.

26 The kinetics of a first order reaction X Y was investigated under different


conditions.

The table shows pairs of quantities that were plotted as graphs.

Which pairs gave the following graph?

y–axis

0
0 x–axis

y–axis x–axis

1 Rate [X]

2 [Y] time

3 [X] [Y]

AJC JC2 PRELIM 2013 8872/01/H1 [Turn over


www.gradtutors.com
47
16

The responses A to D should be selected on the basis of

A B C D
1, 2 and 3 1 and 2 2 and 3 1 only
are correct only are correct only are correct is correct

No other combination of statements is used as a correct response.

27 P reacts with Q according to the equation below.

P(g) + Q(g) R(g) ∆H < 0

Which statement about the reaction is correct?

1 When the activation energy of the forward reaction is decreased, the rate constant of the
reverse reaction increases.

2 Increasing temperature will lower the activation energy, resulting in a greater fraction of R
molecules with energy greater than activation energy.

3 The activation energy of the forward reaction is larger than the activation energy of the
backward reaction.

28 Which of the following correctly describes the compounds formed when Period 3 elements
from Na to Al react with oxygen?

1 covalent character increases

2 solubility in water decreases

3 pH of solution when dissolved in water decreases

29 In the European Union, petrol is often blended with ethanol. Which reagent (s) could be used
to detect the presence of ethanol in the petrol which also consist of a mixture of alkanes and
alkenes?

1 Na

2 KMnO4

3 2,4−dinitrophenylhydrazine

AJC JC2 PRELIM 2013


www.gradtutors.com
8872/01/H1
48
17

nses A to D should be selected on


The respon n the basis of
o

A B C D
1, 2 an
nd 3 1 and 2 2 and 3 1 only
are correct only are co
orrect on
nly are corre
ect iss correct

ombination of statemen
No other co nts is used as a correc
ct response.

30 Allyl acetate is an
a organic compound
c with following skeletal formula.
w

Whicch of the folllowing state e true about allyl acetatte?


ements are

1 It gives an
n orange pre
ecipitate wh
hen heated with
w Brady’s reagent.

2 e sp2 carbon atoms an d two sp3 carbon


It has three c atom
ms.

3 It can be formed
f from
m reacting e
ethanoic acid and propa
an–1,3–dio l, HOCH2CH2CH2OH in
concentratted sulfuric acid and he
eating unde
er reflux.

AJC JC2 PRE


ELIM 2013
www.gradtutors.com
8872/01/H1
49
18

H1 Chemistry 8872 AJC 2013 JC2 PRELIM Paper 1 – 30 marks

Question Question
Key Key
Number Number
1 C 16 A
2 C 17 C
3 C 18 B
4 A 19 D
5 B 20 D

6 A 21 C
7 C 22 B
8 D 23 A
9 D 24 D
10 D 25 C

11 B 26 D
12 D 27 D
13 B 28 A
14 B 29 D
15 A 30 C

AJC JC2 PRELIM 2013 8872/01/H1


www.gradtutors.com
50

ANDE
ERSON
N JUNIO
OR COLL
LEGE
2
2013 JC 2 PRELI MINARY
Y EXAMIN
NATIONS
S

NAME::_________________________________ PDG:___
___ /12 R
Register No: ______

CHEMIISTRY 8872/0
02
Higher 1 16 Sep
ptember 20
013
Paper 2 Structure
ed Questio
ons 2 hou
urs

Candidattes answer Section A on


o the Quesstion Paper
Additiona
al Materials: Answ
wer Paper
Graph Paper
Data Booklet

HESE INST
READ TH TRUCTIONS
S FIRST
Write youur name, PDDG and register numbe er.
Write in d
dark blue orr black pen..
You mayy use a penccil for any diagrams, grraphs or rou ugh workingg.
Do not usse staples, paper clips, highlighte rs, glue or correction
c fluid.

Section A
Answer a
all the quesstions.

Section B
Answer ttwo questio
ons on sepa
arate answe
er paper.

At the en
nd of the exa
amination, fasten
f all yo
our work seecurely toge
ether.
The num e of each question orr part questtion.
mber of markks is given in brackets [ ] at the end

For Exa
aminer’s Usse
Pap
per 1 / 30
0
Q1
Totaal %
Q2
Q3
Q4
Q5
Q6 Gradee
Q7
Pap
per 2 / 80
0

This document
d cconsists of 15
1 printed pages.
p

AJC JC2 P
PRELIM 2013 8872/02/H1 [Turn ove
er
www.gradtutors.com
51
2

1 The Pollutant Standards Index (PSI) is an air quality indicator. It is based on five pollutants:
particulate matter (PM10), sulfur dioxide, carbon monoxide, ozone and nitrogen dioxide.

To calculate the overall PSI, the PSI value is first calculated for each of the five pollutants, using
the following table.

Concentration, Ci
PSI value,
i PM10 SO2 CO Ozone NO2
Pi
(g m–3) (g m–3) (mg m–3) (g m–3) (g m–3)
1 50 50 80 5.0 118 −
2 100 150 365 10.0 157 −
3 200 350 800 17.0 235 1130
4 300 420 1600 34.0 785 2260
5 400 500 2100 46.0 980 3000
6 500 600 2620 57.5 1180 3750

[1 g = 10−6 g; 1 mg = 10−3 g]

Given the concentration of a pollutant (with units as stated in the table above),
Pi1  Pi
PSI of pollutant = (concentration of pollutant  C i )  100 ,
C i1  C i
where Ci+1 > concentration of pollutant > Ci

The overall PSI is the maximum value out of the five calculated PSI pollutant values.

In Singapore, the 24−hr PSI is used by the National Environment Agency (NEA) to provide health
advisory.

Persons with
24−hr Elderly, Pregnant
Healthy Persons chronic lung disease,
PSI women, Children
heart disease, stroke
< 100 Normal activities

Minimise prolonged
Minimise prolonged
101–200 or strenuous outdoor
outdoor activity.
physical exertion.

Avoid prolonged or strenuous Avoid all


outdoor physical exertion. Avoid all outdoor activities.
201–300
If outdoor activity is unavoidable, outdoor activities. If outdoor activity
wear N95 mask. is unavoidable,
If outdoor activity is wear N95 mask.
Minimise all unavoidable, wear
outdoor exposure. N95 mask (for
>300 adults).
If outdoor activity is unavoidable,
wear N95 mask.

AJC JC2 PRELIM 2013


www.gradtutors.com
8872/02/H1
52
3

(a) (i) In a 1 m3 sample of air, the mass of PM10 and carbon monoxide were found to be
320 g and 20 mg respectively. Calculate the PSI values for each of the above
pollutants, leaving your answers to 3 significant figures.

PM10: CO:

(ii) What is the maximum concentration of carbon monoxide, in mg m–3, allowed in a


sample of air in order for its PSI value to not exceed 150?

CO: mg m–3

(iii) Given that the PSI value of sulfur dioxide, ozone and nitrogen dioxide are 150, 112 and
133 respectively, use your answer to (a)(i) to determine the overall PSI.

PSI:

(iv) Assuming that the current overall PSI level is your answer in (a)(iii), what advice would
you give to a 7–year–old who wants to play at the outdoor playground?

[5]

AJC JC2 PRELIM 2013 8872/02/H1 [Turn over


www.gradtutors.com
53
4

(b) The amount of sulfur dioxide in a sample of air can be determined by first reacting it with
sodium iodate, NaIO3, and then titrating the sulfuric acid formed with sodium hydroxide.
Iodine is also formed together with sulfuric acid.

(i) Write an ionic equation for the reaction between sulfur dioxide and sodium iodate.

(ii) When a 1 m3 sample of air was bubbled through a solution of sodium iodate, the
resulting solution was neutralised by 10.0 cm3 of 0.005 mol dm−3 sodium hydroxide
solution.

Calculate the concentration of sulfur dioxide, in g m−3, in the sample of air.

Concentration of SO2 : g m−3


[3]

(c) Some countries have set limits for particulates in the air. For example, the European Union
has a daily average (24−hour) limit of 180 g m−3 for PM10. Cities that violate this daily limit
face a hefty financial penalty that is calculated with consideration of many factors such as the
severity the violation has on the ecosystem, duration of the non–compliance and country’s
GDP etc.

(i) A collected sample of air from the German city of Leipzig contains 0.00002% by mass
of PM10. Given that the density of air is 1 kg m−3, calculate the concentration of PM10
in the sample collected.

(ii) Hence, deduce whether the German city of Leipzig will be faced with any financial
penalty.

[2]
[Total: 10]

AJC JC2 PRELIM 2013


www.gradtutors.com
8872/02/H1
54
5

2 The burning of fossil fuels, which results in the production of CO2, have been a concern to the
environment. It was known that ocean water has become 30% more acidic (in terms of H+
concentration) over the last 150 years. When carbon dioxide reacts with water, carbonic acid,
H2CO3 is formed and this causes acidification of water in oceans and affects marine life.

The following equilibrium in ocean water is established.

CO2 + H2O + CO32 2 HCO3

HCO3 and its conjugate base are the main components in the ocean that regulates its pH.

(a) (i) A 100 g sample of ocean water saturated with pure carbon dioxide at atmospheric
pressure contains 0.15 g of dissolved CO2. Calculate the concentration of dissolved
CO2, in mol dm–3, present in this sample.
[Assume that ocean water contains only CO2 and water with a density of 1 g cm–3]

[CO2(aq)] = mol dm–3

(ii) If the pH of ocean water 150 years ago was 8.25, calculate its current pH.

(iii) With the aid of two balanced equations, show how the pH of the ocean is regulated.

[5]

AJC JC2 PRELIM 2013 8872/02/H1 [Turn over


www.gradtutors.com
55
6

(b) In an experiment, 25 cm3 of 0.100 mol dm3 H2CO3 is titrated with aqueous potassium
hydroxide. The titration curve is shown below. The acid dissociation of H2CO3 is as shown in
the following equations.

H2CO3 HCO3 + H+

HCO3 CO32 + H+

pH

10.33

6.35

3.68

0
9.4 18.8 28.2 37.6 Volume of KOH / cm3

(i) Justify that carbonic acid, H2CO3, is a weak acid with relevant calculations.

(ii) Calculate the concentration of KOH, in mol dm–3, used in the titration.

(iii) Suggest, with a reason, a suitable indicator for the first end point of this titration.

[5]
[Total: 10]

AJC JC2 PRELIM 2013


www.gradtutors.com
8872/02/H1
56
7

3 (a) In an analyytical chemistry labora


atory, the plasma torch
h is often uused to ion
nise a varie
ety of
ssamples.

In one seriees of analy


ysis, sample
es of unkno
own elements were paassed through the pla asma
ttorch. The second
s ion
nisation en
nergies of ten
t success sive elemennts Q to Z in both Period 2
aand 3 of thhe Periodic
c Table aree recorded and plotte
ed as showwn below, with
w one of the
eelements beeing phosph
horus.

Z
U
X Y
W
T V

((i) Which one of the


e ten eleme
ents represe
ents phosph
horous?

((ii) Element Q and Y are in the


e same group. Using the electronnic configura ations of Q+ and
+
Y , exxplain why the
t second ionisation energy
e of Y is found too be lower th
han that of Q.
Q

Electrronic config
guration

Q+ Y+

((iii) Suggest the form


mula of a co
ompound that element V will form with chlorin
ne.

Formula of comp
pound:
[5]

ver
[Turn ov
www.gradtutors.com
AJC JC2 PRE
ELIM 2013 8872/02/H1
57
8

(b) Liquid phosphorous tribromide is prepared by the reaction of bromine with red phosphorus,
P4. An energy level diagram which starts from P4(s) and Br2(g) is shown below.

energy /
kJ mol–1
P(g) + 3Br(g)

3/2 x ∆H1

P(g) + 3/2Br2(g) 3 x ∆H2

∆H3

1/4P4(s) + 3/2Br2(g)
0

PBr3(g)
∆H4
∆H5
PBr3(l)

(i) Define the term standard enthalpy change of formation.

(ii) Given that the enthalpy changes of the processes involved in the energy level diagram
above are

 ∆H f (PBr3),

 ∆H vap (PBr3),
 bond energy (P–Br) and
 bond energy (Br–Br),

use them to assign to the enthalpy changes represented as

∆H1

∆H2

∆H4

∆H5

AJC JC2 PRELIM 2013


www.gradtutors.com
8872/02/H1
58
9

(iii) Hence, use the information given below and relevant data from the Data Booklet to
calculate the bond energy of P–Br bond.

∆H3 +314.6 kJ mol–1


∆H4 –184.5 kJ mol–1
∆H5 +38.8 kJ mol–1

bond energy (P–Br) = kJ mol–1


[5]
[Total: 10]

AJC JC2 PRELIM 2013 8872/02/H1 [Turn over


www.gradtutors.com
59
10

4 Organic chemistry is the chemistry of carbon compounds. The types of organic reactions that you
have studied are listed below.

addition substitution elimination


oxidation reduction hydrolysis

The natures of addition and substitution reactions are listed below and further described as follows,

electrophilic nucleophilic free radical


(substitution only)

Electrophiles are electron–poor species that can form a covalent bond by accepting a pair of
electrons from the reactive carbon. They can be either neutral or positively charged.

The chlorination of benzene is an example of an electrophilic substitution reaction. Cl+ acts as an


electrophile by accepting a pair of electrons from the reactive carbon to form the C–Cl bond in this
substitution reaction.

+ Cl+ Cl + H+

Conversely, nucleophiles are electron–rich species that can form a covalent bond by donating a
pair of electrons to the reactive carbon. All nucleophiles possess a lone pair of electrons and they
can be either neutral or negatively charged. An addition or substitution reaction involving a
nucleophile is described as nucleophilic addition or nucleophilic substitution respectively.

A free radical contains an atom with an odd number of electrons in its valence shell and it is highly
reactive. Examples of radicals include ·CH3 and ·H.

(a) Complete the table below by stating only the types of reaction using the lists given above.
Use both lists given above where appropriate.

organic reaction type of reaction


(i)

CH2CH3 COOH

(ii)

CH3CH2CHBrCH3 CH3CH2CH(OH)CH3

(iii)

Br
(iv) Br

+ Br2
Br
[6]

AJC JC2 PRELIM 2013


www.gradtutors.com
8872/02/H1
60
11

(b) The reactant from (a)(i), ethylbenzene, can react with chlorine to undergo electrophilic
substitution or free radical substitution depending on the reacting conditions.

State the two different conditions and draw two possible mono–chlorinated products that can
be formed under each condition.

(i) Electrophilic Substitution

Condition:

(ii) Free Radical Substitution

Condition:

[4]
[Total: 10]

AJC JC2 PRELIM 2013 8872/02/H1 [Turn over


www.gradtutors.com
61
12

Section B

Answer two questions from this section on separate answer paper.

5 (a) (i) State and briefly explain the general trend of first ionisation energy from sodium to
argon.

(ii) With reference to relevant data from the Data Booklet, discuss the anomaly in the trend
of first ionisation energy between phosphorous and sulfur.
[4]

(b) Allotropy is the property of an element to exist in two or more different forms, known as
allotropes e.g. diamond and graphite are allotropes of carbon.

One of the most common allotrope of phosphorous is white phosphorous, P4. It is produced
industrially by heating phosphorous–containing rocks, fluorapatite, Ca5(PO4)3F, to 1500 oC
with silicon dioxide and coke (carbon). The by–products of this process include carbon
monoxide, calcium silicate (CaSiO3) and calcium fluoride. The mole ratio of fluorapatite to
white phosphorous produced is 4:3.

(i) Write a balanced chemical equation for the process.

(ii) Each atom of white phosphorous molecule has a valency of 3. Suggest by means of a
diagram how the phosphorous atoms might be joined together.
[2]

(c) PCl3 can be made in the laboratory by heating phosphorous with chlorine gas. In the
presence of excess chlorine gas, PCl5 will be produced.

Draw the structure of PCl3 and PCl5 and suggest the shape and bond angles of each
molecule.
[4]

(d) 10 g of PCl5 vapour was heated in a closed container of 5.00 dm3 flask at 500 K.

PCl5(g) PCl3(g) + Cl2(g) ∆H > 0

When equilibrium has been established, 45% of PCl5 was dissociated.

(i) Write an expression for Kc for this equilibrium, stating the units.

(ii) Calculate the concentration, in mol dm–3, of PCl5, PCl3 and Cl2 present at equilibrium.

(iii) Calculate a value for Kc for the equilibrium.

(iv) For each of the case below, deduce what will happen to the position of equilibrium
when

I volume of the container is halved;

II temperature inside the flask is raised from 500 K to 600 K.


[10]
[Total: 20]

AJC JC2 PRELIM 2013


www.gradtutors.com
8872/02/H1
62
13

6 Compounds of sulfur have many uses in everyday life, e.g. disinfectants, matches and in making of
paper.

(a) Phosphorus sulfide, P4S3, is used in small amounts in the tip of a match stick. On striking a
match stick, this compound burns to form sulfur dioxide and a solid oxide.

(i) Construct an equation for this reaction.

(ii) The melting points of the two oxides formed in (a)(i) differ significantly from that of
silicon(IV) oxide (SiO2).
Briefly relate this observation to the structure of, and bonding in, each of the oxides.

(iii) Both oxides formed in (a)(i) dissolve in water to give acidic solutions. Construct an
equation for the reaction of each oxide with water and give an approximate pH for each
of the solution formed.
[6]

(b) Concentrated sulfuric acid can be used under two different sets of reaction conditions to
synthesise but–2–ene from but–1–ene via a 2–step reaction scheme as shown below.

I II
CH3CH2CH=CH2 CH3CH2CH(OH)CH3 CH3CH=CHCH3

(i) Describe the condition necessary for reaction II.

(ii) But–2–ene occurs in two isomeric forms but cyclobutene occurs in only one form.

Draw the displayed formulae of the two isomers of but–2–ene and explain why such
isomerism is not possible with cyclobutene.

But–2–ene, on treatment with cold concentrated H2SO4 followed by the addition of hot water,
gives an addition product J, C4H10O. J produces a yellow precipitate on warming with alkaline
aqueous iodine.

J turns orange hot acidified potassium dichromate green. The product, K, formed is then
reacted with HCN, in the presence of NaCN to give compound L, C5H9NO.

Treatment of L with hot, dilute sulfuric acid gives compound M. 0.590 g of M reacts with
excess sodium metal to give 120 cm3 of hydrogen gas (measured at room temperature and
pressure).

On heating in the absence of air, M loses water to give a single compound N (C10H16O4). N no
longer reacts with sodium metal.

(iii) Deduce the structures of compounds J, K, L, M and N and use them to explain the
chemistry of the reactions described.
[There is no need to comment on the chemistry of the formation of J from but–2–ene.]

(iv) An isomer of but–2–ene gives only one monobrominated product with Br2 under uv light.

Suggest the structural formula of this isomer.


[14]
[Total: 20]

AJC JC2 PRELIM 2013 8872/02/H1 [Turn over


www.gradtutors.com
63
14

7 (a) Chlorine is found in the third period of the Periodic Table (sodium to argon). Some elements
in the third period are solids, whilst some are gases at room temperature. This is attributed to
the variation in melting points and boiling points of the elements across the third period.

(i) State and explain the general trend in melting points of the elements in the third period
of the Periodic Table (from sodium to argon) in terms of their structures and bonding.

(ii) Sulfur, S8, exists as a yellow solid whilst chlorine, Cl2 exists as a yellowish–green gas at
room temperature and pressure. Suggest a reason for this difference.
[5]

(b) Both bromine and chlorine can behave as oxidising agents.

Aqueous bromine oxidises methanoic acid, according the following equation.

Br2 + a HCOOH b Br– + c H+ + CO2

(i) Determine the values of a, b and c in the chemical equation.

(ii) This reaction is catalysed by an acid. State and explain the effect of a catalyst on the
rate constant for this reaction.
[3]

To investigate how the rate of reaction depends on the concentration of aqueous bromine and
methanoic acid, two experiments were carried out.

Experiment 1: 20 cm3 of 0.25 mol dm–3 of aqueous bromine reacted with 20 cm3 of 5 mol dm–3
methanoic acid.

Experiment 2: 20 cm3 of 0.25 mol dm–3 of aqueous bromine reacted with 20 cm3 of
2.5 mol dm–3 methanoic acid.

The rate of the reaction was followed by measuring the volume of carbon dioxide gas
produced at various times in the reaction. The results are tabulated below. All volumes are
measured at room temperature and pressure (r.t.p).

Volume of CO2 / cm3


Time / s
Experiment 1 Experiment 2
0 0 0
20 8.0 4.1
40 15.5 8.0
100 35.1 19.1
160 51.1 29.1
240 67.8 40.8
320 80.4 51.1
400 90.0 60.0
480 97.3 67.8
560 102.8 74.5

(iii) Given that 1 mole of CO2 occupies a volume of 24 000 cm3 at r.t.p, show that the
maximum volume of CO2 that can be produced from Experiment 1 is 120 cm3.

AJC JC2 PRELIM 2013


www.gradtutors.com
8872/02/H1
64
15

(iv) Using the same axes, plot graphs of volume of CO2 against time for Experiment 1 and
2. Label your graphs clearly.

(v) Use your graph to determine the order of reaction with respect to Br2 and HCOOH,
showing your working clearly. Hence, give the rate equation for the reaction.

(vi) In both experiments, the concentrations of HCOOH were far more concentrated than
that of Br2. Explain the significance of doing this.

(vii) Sketch a graph to show how the concentration of Br2 in experiment 1 will vary with
time.

Experiment 3 was carried out using 0.5 mol dm–3 Br2(aq) instead of 0.25 mol dm–3 while
keeping the concentration of HCOOH at 5 mol dm–3.

On the same axes, sketch another graph to illustrate how rate will vary in experiment 3.
Label your graphs clearly.
[12]
[Total: 20]

AJC JC2 PRELIM 2013


www.gradtutors.com
8872/02/H1
65
Preliminary Exam 2013
H1 Chemistry Paper 2
Suggested Answers and Comments

1 (a) (i) 200  100


PSI of PM10 = (320  150)  100 = 185
350  150
300  200
PSI of CO = (20  17)  100 = 118
34  17 [2]

(ii) Let maximum concentration of CO be x mg/m3


200  100
(x  10)  100 = 150
17  10
x = 13.5
maximum concentration of CO is 13.5 mg m-3 [1]

(iii) overall PSI is the maximum value out of 185, 118, 112, 133 and 150.
Hence overall PSI is 185. [1]

(iv) I would advise her not to play for too long. [1]

(b) (i) SO2 + 2H2O  SO42− + 4H+ + 2e− (x 5)


2IO3− + 12H+ + 10e−  I2 + 6H2O

overall: 2H+ + 5SO2 + 4H2O + 2IO3−  5H2SO4 + I2

or 5SO2 + 4H2O + 2IO3−  5SO42− + 8H+ + I2 [1]

(ii) H2SO4 + 2NaOH  Na2SO4 + 2H2O


n(NaOH) reacted = 0.01  0.005 = 5  10−5 mol
n(H2SO4) reacted with NaOH = 5  10−5  2 = 2.5  10−5 mol
= n(H2SO4) produced from SO2
[1]
n(SO2) in 1 m3 sample of air = 2.5  10−5
mass of SO2 in 1 m3 sample of air = 2.5  10−5  64.0
= 0.0016 g = 1600 g
concentration of SO2 = 1600 g m−3 [1]

(d) (i) In 1 m3,


mass of air is 1 kg
mass of PM10 is 0.00002/100  1 = 2  10−7 kg = 0.0002 g = 200 g
Hence concentration of PM10 is 200 g/m3 [1]

(ii) Since concentration calculated in d(i) is more than 180, the sample of air has
exceeded the limit.

The German city of Leipzig will be fined. [1]

www.gradtutors.com
66
2 (a) (i) Mass of CO2 = 0.15g
No. of moles of CO2 = 0.15 /44 = 0.003409 mol

Mass of H2O = 99.85g


Volume of water = 99.85cm3 [1]

Concentration of CO2 in 1dm3 water = 0.003409/ 99.85 x 1000


= 0.0341mol/ dm3 [1]

(ii) pH = 8.25 => [H+] = 10-8.25

Current [H+] after 150 years = 0.30(10-8.25) + 10-8.25 = 7.3 x 10-9 mol dm-3
Current pH = -lg (0.30(10-8.25) + 10-8.25) = 8.14 [1]

(iii) Upon addition of small amount of H+,


H+ + CO32  HCO3 [1]

Upon addition of small amount of OH,


OH + HCO3  H2O + CO32 [1]

(b) (i) Initial [H+(aq)] = 10–3.68 = 2.09  10–4 mol dm-3 < 0.100 mol dm-3 [1]

H2CO3 is partially/ weakly dissociated. Hence, H2CO3 is a weak acid. [1]

(ii) [KOH] = 25/18.8 x 0.100 = 0.133 mol dm-3 [1]

(iii) Phenolphthalein. [1] as the first equivalence point lies within the working range
of the indicator of pH 8-10. [1]

3 (a) (i) Element: W [1]

(ii) Electronic configuration of Q+: 1s22s22p4


Electronic configuration of Y+: 1s22s22p63s23p4 [1]

The election removed from Q+ is from 2p orbital, whereas the electron removed
from Y+ is from 3p orbital which is at a higher energy level [1], much lesser
energy is required to remove electron from Y+ than from Q+ and hence a
decrease in second ionisation energy [1].

Or

The election removed from Q+ is from 2p orbital, whereas the electron removed
from Y+ is from 3p orbital. Since Y+ has a larger ionic radius than Q+, the
valence e- in Y+ experience a smaller effective nuclear charge effect and hence
less attracted towards nucleus [1]. Much lesser energy is required to remove
electron from Y+ than from Q+ and hence a decrease in second ionisation
energy [1].

(iii) VCl4 [1]

(b) (i) Standard enthalpy change of formation is the amount of heat absorbed or
evolved when one mole of a substance is formed from its constituent elements,
all in their standard states at 298 K and 1 atm. [1]

www.gradtutors.com
67
(ii)
energy /

kJ mol–1
P(g) + 3Br(g)

3/2BEBr–Br

P(g) + 3/2Br2(g)
3BEPBr

1/4P4(s) + 3/2Br2(g)
0

PBr3 (g)
∆Hf (PBr3)

∆Hvap (PBr3)
PBr3 (l)

[2] All correct labellings


[1] 2 or 3 correct labellings

Coefficient is not required

(iii) By Hess’ Law,

+314.6 + 3/2BE(Br-Br) = -184.5 + (+38.8) + 3BE(P-Br) [1]


+314.6 + 3/2(+193) = -184.5 + (+38.8) + 3BE(P-Br)
3B.E.(P-Br) = + 749.8

Hence B.E. (P-Br) = +250 kJ mol-1 [1]

4 (a) (i) oxidation [1]

(ii) nucleophilic [1] substitution [1]

(iii) elimination [1]

(iv) electrophilic [1] addition [1]

(b) (i) condition: anhydrous AlCl3 / FeCl3 / Fe filings [1]

Cl CH2CH3 CH2CH3
[1]
Cl

(ii) condition: uv light / heat [1]

CHClCH3 CH2CH2Cl [1]

www.gradtutors.com
68
5 (a) (i) First ionisation energy generally increases from sodium to argon. [1]
Each electron experiences a greater effective nuclear charge due to
increased number of protons in the nucleus. [1]

(ii) 1st IE of P = 1060 kJ mol-1 > 1st IE of S = 1000 kJ mol-1


The 1st IE of sulfur is lower than expected. [1]

P: 1s2 2s2 2p6 3s2 3p3 S: 1s2 2s2 2p6 3s2 3p4
The first electron of sulfur to be removed experiences inter-electronic
repulsion. [1]
Less energy is needed to remove the electron.

(b) (i) 4 Ca5(PO4)3F + 18 SiO2 + 30 C → 3 P4 + 30 CO + 18 CaSiO3 + 2 CaF2 [1]

(ii)
[1]

(c) Cl
Cl
P Cl
P Cl
Cl Cl Cl
Cl [1]
[1] trigonal bipyramidal;
120o within plane
Trigonal pyramidal; 107o [1] 90o between plane and axis [1]

(d) (i) [PCl 3 ][Cl 2 ]


Kc  [1] Units: mol dm-3 [1]
[PCl 5 ]

(ii) PCl5 PCl3 Cl2 [1]


Initial / mol 10 0 0
 0.0480
208.5
Change / mol -0.45 (0.0480) +0.45 (0.0480) +0.45 (0.0480) [1]
= -0.0216 = +0.0216 = +0.0216
Eqm / mol 0.0480 – 0.0216 0.0216 0.0216
= 0.0264 [1]
Conc / mol dm-3 0.0264 0.0216
 4.32x10 -3
0.0216
 4.32x10 -3
 5.28x10 -3
5 5 5
[1]
(iii) (4.32 x 10 ) -3 2
Kc   3.53 x 10 3 mol dm 3
5.28 x 10 -3 [1]

(iv) Total pressure will double. [1]


Position of equilibrium shifts left to reduce the pressure. [1]

(Total concentration will increase.


Position of equilibrium shifts left to reduce the total concentration of gases.)

Forward endothermic reaction is favoured. [1]


Position of equilibrium will shift right to remove the heat energy. [1]

www.gradtutors.com
69
6 (a) (i) P4S3 + 8O2  P4O10 + 3SO2 [2]

(ignore state symbol; do not accept P2O5 and P2O3) [ecf for (ii) & (iii)]

[1]: correct oxide


[1]: balanced equation

(ii) Both SO2 and P4O10 have simple molecular structure with [1]
weak van der Waals forces between the molecules
SiO2 has a giant molecular structure with [1]
strong, extensive covalent bonds between Si and O atoms

hence SO2 and P4O10 have m.p. that are similar but much lower than that of
SiO2.

(iii) SO2 + H2O  H2SO3 (pH < 2) [2]


P4O10 + 6H2O  4H3PO4 (pH < 2)

[1] each: balanced equation, include correct pH

(b) (i) II : excess concentrated H2SO4, 170 / 180 oC [1]

(ii) H H [3]
H H H H C H
C C H C C H
H C C H H C
H H H H
cis-but-2-ene trans-but-2-ene

There will be bond strain (or angle strain) for the trans cyclobutene.

[1] each: correctly labelled geometrical isomers


[1]: correct explanation

(iii) OH OH [9]
O
CH3 CH2 C CH3 CH3 CH2 C CH3
CH3 CH2 C CH3
H CN

J K L
O
OH O
CH3 CH2 C CH3
CO2H O
O
M N

www.gradtutors.com
70

Type of
Observation Deductions
reaction
Compound J produce a oxidation J contains
yellow precipitate on (triiodomethane OH O
warming with alkaline / iodoform test) CH3 C OR CH3 C
aqueous iodine.
H
J turned orange hot oxidation J is an alcohol / (aldehyde)
acidified potassium
dichromate green. K is a ketone / (aldehyde /
carboxylic acid)

The product formed is addition J is a 2o alcohol as the


then reacted with HCN, product formed from its
in the presence of NaCN oxidation (K) is a ketone
to give compound L,
C5H9NO. L is a cyanohydrin

Treatment of L with dilute hydrolysis M contains a carboxylic acid


sulfuric acid gives group
compound M.

0.590 g of M reacts with redox 120


excess Na to give n(H2) = = 5 x 10-3 mol
24000
120 cm3 of H2(g) 0.590
n(M) = = 5 x 10-3 mol
118.0

Since n(M) = n(H2) evolved,


there is presence of 2 –OH
group / an alcohol and a
carboxylic acid in M

On heating M in absence (self) N contains (cyclic) ester


of air, compound N esterification /
(C10H16O4) is formed condensation

N no longer reacts with Absence of –OH group due to


Na. formation of ester in the
previous step.

[1] each: structures J, K, L, M and N


[4]: 10 – 13 pts for explanation
[3]: 7 – 9 pts
[2]: 4 – 6 pts
[1]: 1 – 3 pts

(iv) [1]
cyclobutane

www.gradtutors.com
71
7 (a) (i) The melting generally increases across the period. [1] [4]

Na, Mg and Al have high melting points because of their giant metallic
structures with strong electrostatic forces of attraction between the metal
cations and ‘sea’ of delocalised valence electrons.

Si has an extremely high melting point because it has a giant molecular


structure with strong and extensive covalent bonds between the Si atoms.

P4, S8, Cl2 and Ar have simple molecular structures where the discrete
molecules are held by weak van der Waals’ forces of attraction between them.

[1] each: correct structure and bonding described for each type of structures.

(ii) The number of electrons and hence the strength of Van der Waals’ forces of [1]
attraction between molecules of S8 is greater than Cl2.
Thus, more energy is needed to break the stronger Van der Waals’ forces of
attraction and melting points and boiling points of S8 is greater than Cl2.

(b) (i) Br2 (aq) + HCOOH (aq)  2 Br- (aq) + 2H+ (aq) + CO2 (aq)

a = 1, b = 2, c = 2 [1]

(ii) A catalyst increases the rate of reaction, and hence the rate constant. [1]
From the rate equation, rate = k[A] where A is the reactant, if rate increases
despite all concentrations remaining the same, then the value of k must also [1]
increase.

(iii) 20 [1]
Volume = 1000 x 0.25 x 24000 = 120 cm3

(iv) 120 [2]

100
Expt 1

80
Volume of CO2/ cm3

Expt 2
60

40

20

0
0 100 200 300 400 500 600
time/s
[1]
correct axes + units
[1] correct shape of curve

www.gradtutors.com
72

(v) To determine order of reaction w.r.t. Br2: [6]

Using Experiment 1,
Half-life is a constant at approximately 200 s.
Thus, reaction is first order with respect to Br2.

[1] show on graph how t1/2 is determined (at least 2 half-lives)


[1] correct conclusion

To determine order of reaction w.r.t. HCOOH:

Initial rate of formation of CO2 in experiment 1 = 80 - 0 = 0.400 cm3/s


200 - 0
Initial rate of formation of CO2 in experiment 2 = 40 - 0 = 0.200 cm3/s
200 - 0

Let the rate equation be: rate = k [Br2]1 [HCOOH]x

0.400 = 2.5 x
( )
0.200 1.25

x = 1. Thus, reaction is first order with respect to HCOOH.

Or

Comparing experiments 1 and 2, when [HCOOH] doubles while keeping [Br2]


constant, the rate of formation of CO2 doubles.
 reaction is first order with respect to HCOOH.

[1] each: correct calculation of initial rate


[1] each: correct determination of order of reaction (either in words or by
calculation, ignore arithmetic error in calculation of initial [HCOOH])
[1] draw tangent to find initial rate

Rate= k [Br2] [HCOOH]


[1] correct rate equation

(vi) This is so that the concentrations of HCOOH will remain effectively constant [1]
throughout each experiment.

(vii) [2]
[Br2]

0.25

0.125

0.0625
Expt 3
Expt 1
0.03125
t½ 2t½ 3t½ time / s

[1] Downward sloping graph + correct initial concentration.


[1] t1/2 remains constant for both graphs with at least 2 half-lives shown. (shown
clearly with corresponding concentration value)

www.gradtutors.com
73
1

Name: Index Number: Class:

DUNMAN HIGH SCHOOL


Preliminary Examination
Year 6

H1 CHEMISTRY 8872/01
Paper 1 Multiple Choice 27 September 2013
50 minutes
Additional Materials: Optical Mark Sheet
Data Booklet

INSTRUCTIONS TO CANDIDATES
1 Write your name, index number and class on this question paper.
2 There are thirty questions in this paper. Answer all questions. For each question there are
four possible answers A, B, C and D.
Choose the one you consider correct and record your choice in soft pencil on the separate
Optical Mark Sheet.
3 Each correct answer will score one mark. A mark will not be deducted for wrong answer.
4 Any rough working should be done in this booklet.
5 You may use a calculator.

© DHS 2013 This question paper consists of 12 printed pages and 0 blank page. [Turn over

www.gradtutors.com
74
2

Section A

For each question, there are four possible answers A, B, C, and D. Choose the one you
consider to be correct.

1 The surface area of a fine powder can be determined by measuring the volume of
nitrogen gas required to form a layer one molecule thick on its surface. It is then
assumed that the area of one nitrogen molecule is Y m2.
There are 6 x 1023 molecules in 1 mol of nitrogen and the molar volume of all gas at
room temperature and pressure is 24000 cm3.
A sample of powdered titanium(IV) oxide requires Z cm3 of nitrogen to form a layer one
molecule thick at room temperature and pressure.
What is the surface area of this sample in m2?

A (24000 / Z) x 6 x 1023 x Y
B (Z / 24000) x 6 x 1023 x Y
C (24000 / Y) x 6 x 1023 x (1/Y)
D Z x 24000 x Y / (6 x 1023)

2 In an experiment, 10 cm3 of an organic compound in the gaseous state was sparked


with an excess of oxygen. 20 cm3 of carbon dioxide and 5 cm3 of nitrogen were
obtained among the products. All gas volumes were measured at the same
temperature and pressure.
Which of the following compounds would fit these data?

A C2H7N B C2H6N2 C C6H5NO2 D C3H5N

3 Acidified potassium dichromate(VI) and acidified potassium manganate(VII) each


liberate iodine from aqueous potassium iodide.

Cr2O72– + 14H+ + 6e → 2Cr3+ + 7H2O


MnO4– + 8H+ + 5e → Mn2+ + 4H2O
2I– → I2 + 2e

25.0 cm3 of acidified Na2Cr2O7 liberates the same quantity of iodine from a solution of
potassium iodide as 25.0 cm3 of 0.030 mol dm–3 KMnO4. What is the concentration of
the Na2Cr2O7 solution in mol dm–3?

A 0.015 B 0.025 C 0.030 D 0.050

4 Which one of the following is not a redox reaction?

A Br2 + SO2 + 2H2O → SO42– + 4H+ + 2Br–


B 6NaOH + 3Br2 → 5NaBr + NaBrO3 + 3H2O
C 2CuSO4 + 4KI → 2CuI + 2K2SO4 + I2
D 2CrO42– + 2H+ → Cr2O72– + H2O

© DHS 2013 8872/01 [Turn over


www.gradtutors.com
75
3

5 The number of neutrons and electrons of the following particles are given below:
Particle No. of neutrons No. of electrons
P 15 11
Q2– 17 11
R+ 15 10
S– 17 12
T+ 16 13
Which atom is an isotope of particle P?

A Q B R C S D T

6 The successive ionisation energies, in kJ mol–1, of an element Z are given below.


1010 2265 3662 4998 7002 8992 28698 32215
Which one of the following would best represent the formula of the ionic compound
formed by Z?

A ZCl
B ZO2
C CaZ2
D Na2Z

7 Which one of the following ions will be deflected to the greatest extent in an electric
field?
6
A Li+ B 27
Al3+ C 16
O2 D 14
N3

8 The following polymers could be used for contact lenses. A contact lens has to absorb
water so that it can fit comfortably in the eye.
Which polymer will absorb water most easily?

A B
CH3 CH3
H3C C CH2 CH3 H3C C CH2 CH3
CH2 C O
OH H3C

C D
CH3 CH3
H3C C CH2 CH3
H3C C CH2 CH3
CH2
O
C H
O C CH3
O

© DHS 2013 8872/01 [Turn over


www.gradtutors.com
76
4

9 The table shows the enthalpy change of neutralisation per mole of water formed, ∆H,
for various acids and bases.

Acid Base ∆H / kJ mol–1


Hydrochloric acid Sodium hydroxide –57.0
P Sodium hydroxide –54.0
Hydrochloric acid Q –52.0
Nitric acid R –57.0

What are P, Q and R?

P Q R
A Ethanoic acid Ammonia Potassium hydroxide
B Ethanoic acid Sodium hydroxide Ammonia
C Sulfuric acid Ammonia Potassium hydroxide
D Sulfuric acid Sodium hydroxide ammonia

10 The radius and ionic charge of each of six ions are shown in the table below.

Ion J+ L+ M2+ X– Y– Z2–


Radius/nm 0.14 0.18 0.15 0.14 0.18 0.15

The ionic solids JX, LY and MZ are of the same lattice type. What is the correct order of
the lattice energies of JX, LY and MZ, placing the one with the highest numerical value
first?

A JX > LY > MZ
B LY > MZ > JX
C MZ > JX > LY
D MZ > LY > JX

11 The dissociation of dinitrogen tetraoxide into nitrogen dioxide is represented by the


equation below.
N2O4(g) 2NO2(g) ∆H = +57 kJ mol–1

Which of the following statement correctly explains how the volume of the mixture will
be affected if the temperature of an equilibrium mixture of the gases is increased at
constant pressure?

A The volume will increase, only because of a shift of equilibrium towards the right.
B The volume will increase, both because of a shift of equilibrium towards the right
and also because of thermal expansion.
C The volume will stay the same, because any thermal expansion could be exactly
counteracted by a shift of equilibrium towards the left.
D The volume will decrease, because of a shift of equilibrium towards the left would
more than counteract any thermal expansion.

© DHS 2013 8872/01 [Turn over


www.gradtutors.com
77
5

12 The equilibrium equation for the dissociation of a weak acid, H2X, may be written as
follows.
H2X (aq) 2H+ (aq) + X2– (aq)

What is the expression for the acid dissociation constant, Ka?

A 2H  X 
 2 C H  X 
 2

H 2 X H2 X
B H  X 
 2 2 D 2H  X 
 2 2

H2 X H2 X

13 An enzyme, found in the stomach, operates at maximum efficiency when in an aqueous


solution buffered at pH 5.

Which combination of substances, when dissolved in 10 dm3 of water, would give the
necessary buffer solution?

A 2 mol of CH3CO2H and 1 mol of NaOH


B 2 mol of NaOH and 1 mol of CH3CO2H
C 1 mol of HCl and 1 mol of CH3CO2Na
D 2 mol of NH3 and 1 mol of CH3CO2NH4

14 A chemist discovers that 5 g of an iron(II) salt, when used in a certain reaction at a


certain temperature, causes the reaction to speed up. When he tries another 5 g of the
same salt in another reaction at the same temperature, the reaction rate is still the
same even after a long period of time. What is the most likely reaction for this?

A The chemist did not stir the reaction mixture the second time.
B The temperature is not high enough for reaction rate to increase.
C The iron(II) salt has a specific nature as a catalyst.
D Too little of the iron(II) salt is used.

15 The half–life of a radioactive isotope T is twice that of another radioactive isotope U. In


a sample of rock, it is found that the number of atoms of T is 4 times that of U. What will
be the ratio of the number of atoms of T to the number of atoms of U in the rock after
two half–lives of T?

1 1 1
A B C D 1
16 4 2

© DHS 2013 8872/01 [Turn over


www.gradtutors.com
78
6

16 The stoichiometry chemical equation for a hypothetical reaction may be represented as


X + Y + Z → products
The reaction was studied in a series of four experiments in which the initial rate of the
reaction was measured as a function of the concentration of each reactant with the
following results:

Experiment [X] / mol dm–3 [Y] / mol dm–3 [Z] / mol dm–3 Initial rate /mol dm–3 s–1
1 0.3 0.1 0.01 6 x 10–6
2 0.3 0.2 0.01 12 x 10–6
3 0.4 0.2 0.01 16 x 10–6
4 0.4 0.1 0.02 8 x 10–6

Which of the following shows the correct rate equation?

A Rate = k[X][Y]
B Rate = k[X][Y][Z]
C Rate = k[Y][Z]
D Rate = k[X]2[Z]

17 The following shows a graph of effective nuclear charge against proton number for
elements in Period 3 of the Periodic Table, from sodium to chlorine.

Effective Nuclear Charge


7

0
Na
1 Mg
2 Al
3 Si4 P5 S
6 7Cl

Which of the following trends cannot be inferred from the graph above?

A Atomic radii decrease across the Period.

B Electron affinity increase across the Period.

C Electronegativity increase across the Period.

D Electrical conductivity decrease across the Period.

© DHS 2013 8872/01 [Turn over


www.gradtutors.com
79
7

18 F, G and H are consecutive elements in Period 3. Element H has an oxide that reacts
with hot concentrated NaOH, and the first ionisation energy of element G is lower than
F.

What is the identity of elements F, G and H?

F G H
A Mg Al Si
B Na Mg Al
C Al Si P
D Si P S

19 Which of the following alcohols produce the largest number of products, including
stereoisomers, when reacted with excess concentrated H2SO4 at 170 oC?

A CH3CH2CH2CH2CH2OH

B CH3CH2CH2CH(OH)CH3

C CH3CH2CH(OH)CH2CH3

D CH3CH(CH3)CH(OH)CH3

20 The Russian composer Borodin was also a research chemist who discovered a reaction
in which two ethanol molecules combine to form a compound as shown in reaction I.
The product forms another compound on heating in reaction II.

I 2CH3CHO → CH3CH(OH)CH2CHO
II CH3CH(OH)CH2CHO → CH3CH=CHCHO + H2O

Which of the following best describes reactions I and II?

I II
A Addition Elimination
B Addition Reduction
C Elimination Reduction
D Substitution Elimination

© DHS 2013 8872/01 [Turn over


www.gradtutors.com
80
8

21 Molecule Z is a derivative of a compound used to make the hard outer covering of golf
balls.

Which of the following is not true about Z?

A It is insoluble in water.

B It can undergo a condensation reaction to give an orange compound.

C It can react with HBr at room temperature and pressure.

It gives a secondary alcohol as a major product when reacting with steam and
D
phosphoric acid.

22 A sample of 1,3–dimethylbenzene (Mr = 109) is oxidised by acidified potassium


manganate(VII) to the corresponding dicarboxylic acid.

What is the mass of the organic product formed from 1.00 g of 1,3–dimethylbenzene?

A 1.15 g
B 1.28 g
C 1.57 g
D 1.60 g

23 Malic acid is found in apples.


OH
O
CHCH2C
C O OH
OH Malic acid

Which substance will react with all three –OH groups present in the malic acid
molecule?

A Phosphorus pentachloride
B Potassium hydroxide
C Ethanol in the presence of concentrated sulfuric acid
D Sodium carbonate

© DHS 2013 8872/01 [Turn over


www.gradtutors.com
81
9

24 The compound C3H7Br undergoes a sequence of reactions as follows:

OH–(aq) Cr2O72–(aq) Tollens’ reagent/H+


C3H7Br X Y Z + silver mirror

What are the compounds X, Y and Z?

X Y Z
A CH3CH(OH)CH3 CH3CH2CO2H CH3CH2CHO
B CH3CH2CH2OH CH3CH2CHO CH3CO2H
C CH3CH2CH2OH CH3CH2CHO CH3CH2CO2H
D CH3CH(OH)CH3 CH3COCH3 CH3CO2H

25 Which reaction yields an organic product that has more hydrogen atoms than the
reactant?

A H H O
I2, NaOH
H C C C CH3
warm

H H

B CH3
conc H2SO4
H3C C OH

CH3

C
LiAlH4

D
H H O
acidified KMnO4
H C C C OH
heat

H OH

© DHS 2013 8872/01 [Turn over


www.gradtutors.com
82
10

Section B

For each question in this section, one or more of the three numbered statements 1 to 3 may
be correct.

Decide whether each of the statements is or is not correct (you may find it helpful to put a
tick against the statements which you consider to be correct).

The responses A to D should be selected on the basis of

A B C D
1, 2 and 3 1 and 2 only 2 and 3 only 1 only
are correct are correct are correct is correct

No other combination of statements is used as a correct response.

26 Carbon exists as a number of allotropes, for example graphite, fullerenes and acetylenic
carbon, which have the following description.

Allotrope Description
Graphite Each carbon atom is covalently bonded to three other carbon
atoms.
Fullerene Each carbon atom has an unhybridised p orbital with one
electron, which can be delocalised over the whole molecule.
Acetylenic carbon Chains of atoms with alternating triple and single bonds.

Which of the following deductions on the shape of the carbon atom and hybridisation of
the allotrope are correct?

Allotrope Shape Hybridisation


1 Fullerene Trigonal planar sp2
2 Acetylenic carbon Linear sp
3 Graphite Trigonal pyramidal sp3

27 Which statements are correct for the sequence of compounds below, considered from
left to right?

NaF MgO AlN SiC

1 The electronegativity difference between the elements in each compound


increases.

2 The formula–units of these compounds are isoelectronic (have the same number of
electrons).

3 The bonding becomes increasingly covalent.

© DHS 2013 8872/01 [Turn over


www.gradtutors.com
83
11

The responses A to D should be selected on the basis of

A B C D
1, 2 and 3 1 and 2 only 2 and 3 only 1 only
are correct are correct are correct is correct

No other combination of statements is used as a correct response.

28 The diagram below illustrates the energy changes for a set of reactions.
∆H1 = –150 kJ mol–1
M N

∆H3 ∆H2 = +100 kJ mol–1

P Q
∆H4 = +75 kJ mol–1

Which statements about the energy changes involved are correct?

1 The value of ∆H1 + ∆H2 is equal to ∆H3 + ∆H4.


2 The enthalpy change of the reaction M → P is –125 kJ mol–1.
3 The enthalpy change of the reaction P → N is +25 kJ mol–1.

29 In which of the following reactions does NH3 react as a Bronsted–Lowry base?

1 HSO4– + NH3 → SO42– + NH4+


2 2NH3 → NH2– + NH4+
3 Ag+ + 2NH3 → [Ag(NH3)2]+

© DHS 2013 8872/01 [Turn over


www.gradtutors.com
84
12

The responses A to D should be selected on the basis of

A B C D
1, 2 and 3 1 and 2 only 2 and 3 only 1 only
are correct are correct are correct is correct

No other combination of statements is used as a correct response.

30

The steroid shown is an intermediate compound obtained during the synthesis of


Formestane which is used in the treatment of breast cancer.

Which statement about this compound is correct?

1 It reacts with hydrogen cyanide in an addition reaction.


2 It can be oxidised by warm acidified potassium dichromate(VI) to a carboxylic acid.
3 It will react with Fehling’s solution.

© DHS 2013 8872/01 [Turn over


www.gradtutors.com
85
13

Answers

1 2 3 4 5 6 7 8 9 10
B A B D C D D D A C
11 12 13 14 15 16 17 18 19 20
B B A C D A D A B A
21 22 23 24 25 26 27 28 29 30
D C A C C B C B B D

© DHS 2013 8872/01


www.gradtutors.com
86

Name: Index Number: Class:

DUNMAN HIGH SCHOOL


Preliminary Examinations
Year 6

H1 CHEMISTRY 8872/02
Paper 2 Section A 24 September 2013
Section B 2 hours

Additional Materials: Data Booklet, Graph Paper, Writing Paper

INSTRUCTIONS TO CANDIDATES
1 Answer all questions in both sections.
2 Write your name and class on this cover page.

Section A
3 Write your answers in the spaces provided on this question paper.
4 You are advised to spend a maximum of 60 minutes on Section A.

Section B
5 Write your name and class on the Cover Sheet provided.
6 Write your answers on the separate writing papers provided.
7 Start each question on a fresh sheet of paper.
*[Marks will be deducted if you fail to do so.]
8 At the end of the examination:
 Staple or fasten all your work securely together with the Cover Sheet on top.
 Hand in the question paper separately.
9 You are advised to spend a maximum of 60 minutes on
Section B. For Examiner’s Use
Question Section A

1
INFORMATION FOR CANDIDATES 11
The number of marks is given in brackets [ ] at the end of each 2
9
question or part question.
3
You are advised to show all workings in calculations. 11
You are reminded of the need for good English and clear 4
9
presentation in your answers.
Total
40

© DHS 2013 This question paper consists of 13 printed pages and 1 blank page. [Turn over

www.gradtutors.com
87
2

Section A

Answer all questions in the spaces provided.

1 (a) (i) Write equations to illustrate the formation of

 phosphorus(V) oxide when phosphorus is heated with oxygen

 sulfur(IV) oxide when sulfur is heated with oxygen

(ii) When NO2 reacts with water, nitrogen undergoes a disproportionation


reaction to give a mixture of two acids as shown:

2NO2 + H2O → HNO2 + HNO3

Use this reaction to explain the term disproportionation.

(iii) In a similar disproportionation reaction, ClO2 reacts with aqueous NaOH to


produce a solution containing two chorine–containing sodium salts. Suggest
the identities of these two chorine–containing sodium salts formed.

and

[4]

(b) The major source of sulfur for the manufacture of sulfuric acid by the Contact
process is the de–sulfurisation of ‘sour’ natural gas. Many natural gas wells
produce a mixture of volatile hydrocarbons (mainly CH4 and C2H6) together with
up to 25% hydrogen sulfide, H2S.

(i) Complete and balance the following equation showing the complete
combustion of a gaseous mixture consisting of 2 mol of CH4, 1 mol of C2H6
and 1 mol of H2S.

2CH4 + C2H6 + H2S + → SO2 + +

(ii) Suggest why it is important to remove the H2S before burning the natural
gas industrially.

© DHS 2013 8872/02 [Turn Over


www.gradtutors.com
88
3

H2S is removed by passing the ‘sour’ natural gas through a solvent containing
ethanolamine, HOCH2CH2NH2. The following reaction takes place.

HOCH2CH2NH2 + H2S → HOCH2CH2NH3+ + SH–

(iii) State the type of reaction that has occured.

(iv) If a sample of natural gas contains 5% by volume of H2S, calculate the mass
of ethanolamine required to remove all the H2S from a 1000 dm3 sample of
gas, measured under room conditions.

The recovered H2S is converted to sulfur by the following two reactions.

I Part of the H2S is burned in air.

H2S(g) + O2(g) → SO2(g) + H2O(g)

II The gas stream resulting from reaction I is then blended with the remaining
H2S and fed into an iron oxide catalyst bed, where sulfur and water are
produced according to the equation.

SO2(g) + 2H2S(g) → 3S(g) + 2H2O(g)

(v) Use the following data to calculate H for the reaction between H2S and SO2.

Compound Hf / kJ mol–1


H2S(g) –21
SO2(g) –297
H2O(g) –242
S(g) +11

[7]

[Total: 11]

© DHS 2013 8872/02 [Turn Over


www.gradtutors.com
89
4

2 Indigo is the dye used in blue jeans. Although originally extracted from plants of the
type indigofera, it is now almost entirely made artificially.

Indigo is insoluble in water but this disadvantage can be overcomed by converting it


into the water–soluble colourless Leuco–indigo. If cloth soaked in a solution of Leuco–
indigo is left to dry in the air, the Leuco–indigo is converted into the insoluble blue
indigo, which is precipitated out onto the fibres of the cloth.

indigo leuco–indigo

(a) On the structures above, circle and name four different functional groups present
in either indigo or leuco–indigo, which are not benzene rings

[3]

(b) Calculate the percentage by mass of carbon in Indigo.

[2]

(c) (i) What type of reaction is the conversion of Indigo into Leuco–indigo?

(ii) Suggest a laboratory reagent for this reaction.

[2]

(d) Suggest the reagents and conditions for a reaction that could be used to
distinguish between Indigo and Leuco–indigo by identifying Indigo positively. In
your test, state the expected observation for each compound.

Reagents and conditions:

Observations:

[2]

[Total:9]

© DHS 2013 8872/02 [Turn Over


www.gradtutors.com
90
5

3 Sodium benzoate is a preservative and is able to prevent the growth of bacteria and
fungi. It is most widely used in acidic foods such as salad dressings, carbonated
drinks, jams and fruit juices.

According to the United States Food and Drug Administration, all food preservatives
are limited to an amount of 0.1% by weight. However, The International Programme on
Chemical Safety found no adverse effects of sodium benzoate in humans at doses of
647–825 mg/kg of body weight per day.

In acidic foods, sodium benzoate is absorbed into the food cells as benzoic acid. This
lowers the intracellular pH of food, creating an environment which fungi and bacteria
cannot thrive. In typical fruit juices which have a pH of 3.6, the concentration of
benzoic acid was found to be 0.001 mol dm–3. In foods with a pH level of 4.2 to 4.5, a
mixture of benzoic acid and sodium benzoate is present. In foods above pH 4.5,
sodium benzoate becomes ineffective as a preservative.

Sodium benzoate is prepared by adding benzoic acid to a hot concentrated solution of


sodium carbonate until effervescence ceases and all reactants are used up.
Appropriate separation techniques can be applied to obtain pure solid sodium
benzoate, which is stable upon heating.

An alternative to sodium benzoate is potassium benzoate, which works in exactly the


same manner.

(a) A typical jar of jam weighs 500 g. Calculate its maximum allowable mass of
sodium benzoate as a preservative.

[1]

(b) Assuming that an average adult weighs 70 kg, calculate the maximum mass of
sodium benzoate a person can intake per day.

[1]

© DHS 2013 8872/02 [Turn Over


www.gradtutors.com
91
6

(c) (i) Calculate the concentration of H+ ions in typical fruit juices.

(ii) Hence, deduce the value of Ka of benzoic acid, assuming benzoic acid is the
only acid present.

[2]

(d) (i) State the type of solution formed by the mixture of benzoic acid and sodium
benzoate in pH 4.2 to 4.5.

(ii) With the aid of a suitable equation, explain what happens when a small
amount of hydrochloric acid was added to this mixture.

[3]

(e) (i) Write the balanced equation for the formation of sodium benzoate from
sodium carbonate.

(ii) Suggest a simple method to isolate solid sodium benzoate from the reaction
in (e)(i).

[2]

(f) Predict, with explanations, if potassium benzoate has a higher or lower boiling
point than sodium benzoate.

[2]

[Total: 11]

© DHS 2013 8872/02 [Turn Over


www.gradtutors.com
92
7

4 (a) KMnO4 and K2Cr2O7 are oxidising agents, usually used in acidic solutions.

(i) State the colour changes you would expect to observe, when each of these
reagents are heated with butan–1–ol.

KMnO4 : to
K2Cr2O7 : to

(ii) When heated with butan–1–ol, each of these reagents give rise to different
products when different conditions are applied. Suggest how the two
products can be obtained by using KMnO4 and K2Cr2O7. Include a balanced
equation for each set of reagents and conditions.

Reagents and conditions :


Equation :

Reagents and conditions :


Equation :

[6]

(b) The following table shows the solubilities of butan–1–ol and but–1–ene in water.

Compound Mass soluble in 1 dm3


Butan–1–ol 73.0
But–1–ene 2.21

Explain the above data.

[2]

(c) The energy profile diagram below represents the hydrogenation of but–1–ene to
butane. On the same diagram, draw the energy profile when nickel is added to
the reaction.
Energy

But–1–ene
Butane
Reaction pathway
[1]

[Total: 9]

© DHS 2013 8872/02 [Turn Over


www.gradtutors.com
93
8

Section B

Answer two questions from this section on separate answer paper.

5 Dinitrogen pentoxide, N2O5, can be produced by the following reaction sequence in


the car engine:

I N2(g) + O2(g)  2NO(g) ∆HӨ = +180 kJ mol–1


II NO(g) + 1/2O2(g)  NO2(g) ∆HӨ = –57.0 kJ mol–1
III 2NO2(g) + 1/2O2(g)  N2O5(g) ∆HӨ = –110 kJ mol–1

(a) (i) By using relevant data in the Data Booklet, calculate the bond energy of the
N=O bond in nitric oxide in reaction I.

(ii) Suggest an explanation why reaction I occurs in car engine.

(iii) Nitrogen dioxide is a pollutant that is often produced in car engine. Explain
how the pollutant can be removed in the car engine.

(iv) By using the given energy cycle and the data above, calculate the standard
enthalpy change of formation of dinitrogen pentoxide.
∆HfӨ
N2(g) + 5/2O2(g) N2O5(g)

2NO(g) + 3/2O2(g)

2NO2(g) + 1/2O2(g)

[6]

© DHS 2013 8872/02 [Turn Over


www.gradtutors.com
94
9

(b) The rate of reaction for reaction II was investigated and the following data was
obtained.

Time/minutes Experiment 1, Experiment 2,


with [NO] = 0.10 mol dm–3 with [NO] = 0.05 mol dm–3
[O2] / mol dm–3 [O2] / mol dm–3
0 0.0050 0.0050
5 0.0031 0.0045
10 0.0019 0.0040
15 0.0011 0.0036
20 0.0007 0.0032
25 0.0005 0.0029
30 0.0004 0.0026

(i) Explain why nitrogen monoxide is used in large excess.

(ii) Using the same axes, plot graphs of [O2] against time for the two
experiments.

(iii) Use your graphs to determine the order of reaction with respect to O2 and
NO, showing your workings clearly.

(iv) State the rate equation of reaction II and hence calculate the rate constant,
including its units.

[10]

(c) The nitrogen dioxide produced in reaction II is able to form dinitrogen tetraoxide
as shown below:

2NO2(g) N2O4(g)
brown colourless

(i) Draw a dot–and–cross diagram to illustrate the bonding in a nitrogen


dioxide molecule and predict its shape.

(ii) Explain whether the enthalpy change of dimerisation is endothermic or


exothermic. Hence, predict the colour change of the reaction mixture when
the temperature is increased.

[4]

[Total: 20]

© DHS 2013 8872/02 [Turn Over


www.gradtutors.com
95
10

6 (a) An experiment is conducted to determine the value of n in the formula of a cation


[Cu(NH3)n]2+. This cation is formed when copper(II) reacts with ammonia.

100 cm3 of 0.100 mol dm–3 of copper(II) sulfate is mixed with 100 cm3 of aqueous
ammonia. The resulting mixture, which contains the cation [Cu(NH3)n]2+ and an excess
of ammonia, is then shaken with trichloromethane and allowed to stand for equilibrium
to be established. The excess ammonia is dissolved in both the aqueous layer and the
trichloromethane layer.

25.0 cm3 of trichloromethane layer is found to neutralise 26.00 cm3 of


0.025 mol dm–3 hydrochloric acid while 25.0 cm3 of aqueous layer required
21.00 cm3 of 1.00 mol dm–3 hydrochloric acid for neutralisation.

The equilibrium constant, Kc, of ammonia between the aqueous layer and the
trichloromethane layer is 25.0.

NH3 (aq) NH3 (CHCl3)

[ammonia] aqueous
Kc =
[ammonia] CHCl3
Calculate:

(i) the concentration of ammonia in the trichloromethane layer.

(ii) the concentration of free ammonia in the aqueous layer, using the equilibrium
constant.

(iii) the total concentration of ammonia (free and in the cation, [Cu(NH3)n]2+) in the
aqueous layer.

(iv) the value of n in the formula of the cation [Cu(NH3)n]2+, stating one assumption
made in the calculation.

[7]

(b) Both beryllium difluoride and boron trifluoride can also react with ammonia. However,
their mole ratios are different.

Beryllium difluoride and ammonia react in a 1:2 ratio while boron trifluoride and
ammonia react in a 1:1 ratio.

(i) Draw diagrams to show the bonding within these molecules and state the type of
bond formed in the products.

(ii) Explain why the mole ratios are different for these two compounds.

[6]

© DHS 2013 8872/02 [Turn Over


www.gradtutors.com
96
11

(c) The graph below shows the variation in first ionisation energy of some elements in
Period 3.

Elements X and Y have proton numbers n and n+1 respectively.

(i) By considering the trend in first ionisation energy, state and explain which Group
element Y belongs to.

(ii) Describe the behaviour of the oxide of X in dilute hydrochloric acid and sodium
hydroxide respectively.

(iii) Sodium carbonate was added to a solution of the chloride of Y and


effervescence was observed. Explain this observation with the aid of relevant
equations.

[7]

[Total: 20]

© DHS 2013 8872/02 [Turn Over


www.gradtutors.com
97
12

7 (a) A tooth is made up of two parts: the crown and the root. Dental crown is the visible
part of the teeth which is made of enamel and dentine.

Enamel
Crown
Dentine

Root

The enamel is made of hydroxyapatite, Ca5(PO4)3OH. In the mouth, mineral


substances such as calcium ions and phosphate ions that are present in saliva
contribute to the formation and decomposition of the hydroxyapatite. These two
processes occur simultaneously until an equilibrium is reached. The formation process
is called mineralisation of the enamel, whereas the decomposition process is called
demineralisation.

Ca5(PO4)3OH(s) 5Ca2+(aq) + 3PO43–(aq) + OH–(aq)

Explain how an acidic medium can affect the demineralisation of teeth.

[2]

(b) An organic compound R (Mr = 113.6) contains by mass 63.2% carbon, 8.7%
hydrogen, and 28.1% oxygen. R undergoes an addition reaction with bromine in a
1:1 mole ratio. It also reacts with sodium hydrogencarbonate, liberating carbon dioxide
gas.

(i) Calculate the empirical formula of R.

(ii) Determine the molecular formula of R.

(iii) Deduce with reasoning, a possible structure for compound R.

[5]

(c) Suggest a suitable chemical test to distinguish between molecules S and T and
describe the expected observations for each molecule.
OH O OH O

CH3 CH2CH3

S T

[2]

© DHS 2013 8872/02 [Turn Over


www.gradtutors.com
98
13

(d) The elements of Group VII, especially chlorine, have played an important role in the
development of chemistry. The following describes a reaction involving a chlorine
containing compound.

An interhalogen compound A, with the formula IxCly, was dissolved in aqueous


potassium iodide, and a brown solution was obtained.

IxCly + I–  I2 + Cl– (not balanced)

The brown solution was titrated with 0.50 mol dm–3 sodium thiosulfate and 34.0 cm3
was required to reach the end–point. Careful addition of aqueous silver nitrate
produced a white precipitate of silver chloride, which weighed 1.83 g when dried.

(i) Calculate the amount of iodine produced in the reaction shown above.

(ii) Calculate the amount of chloride ions produced in the reaction shown above.

(iii) Determine the formula of compound A and hence construct a balanced equation
between compound A and potassium iodide.

[4]

(e) Anhydrous iron(III) chloride is a dark brown moisture sensitive solid


(melting point = 306 oC) which forms an acidic solution in water. It exists as a dimer,
Fe2Cl6, in the solid state. In the gaseous state, the dimer increasingly dissociates into
o
its monomer until dissociation is complete at 400 C.

Fe2Cl6(g) 2FeCl3(g)

0.450 g of Fe2Cl6 was placed in a 0.500 dm3 evacuated flask and heated to 350 oC.
The amount of gaseous mixture at equilibrium was found to be 2.54 x 10–3 mol.

Using the data above, calculate the following:


 the initial amount of Fe2Cl6(g);
 the amount of dissociated Fe2Cl6(g);
 and hence the degree of dissociation of Fe2Cl6 at 350 oC.

Degree of dissociation = Fraction of dissociated Fe2Cl6(g) x 100 %

[4]

(f) Iron(III) chloride is a catalyst used in the synthesis of chlorobenzene from benzene.

Explain the catalytic effect of iron(III) chloride on the rate of reaction in terms of the
Maxwell-Boltzmann distribution.

[3]

[Total: 20]

© DHS 2013 8872/02


www.gradtutors.com
99
14

BLANK PAGE

© DHS 2013 8872/02


www.gradtutors.com
100

Name: Mark Scheme Index Number: Class:

DUNMAN HIGH SCHOOL


Preliminary Examinations
Year 6

H1 CHEMISTRY 8872/02
Paper 2 Section A 24 September 2013
Section B 2 hours

Additional Materials: Data Booklet & Writing Paper

INSTRUCTIONS TO CANDIDATES
1 Answer all questions in both sections.
2 Write your name and class on this cover page.

Section A
3 Write your answers in the spaces provided on this question paper.
4 You are advised to spend a maximum of 60 minutes on Section A.

Section B
5 Write your name and class on the Cover Sheet provided.
6 Write your answers on the separate writing papers provided.
7 Start each question on a fresh sheet of paper.
*[Marks will be deducted if you fail to do so.]
8 At the end of the examination:
 Staple or fasten all your work securely together with the Cover Sheet on top.
 Hand in the question paper separately.
9 You are advised to spend a maximum of 60 minutes on
Section B. For Examiner’s Use
Question Section A

INFORMATION FOR CANDIDATES 1


11
The number of marks is given in brackets [ ] at the end of each
2
question or part question. 9

You are advised to show all workings in calculations. 3


11
You are reminded of the need for good English and clear 4
presentation in your answers. 9
Total
40

www.gradtutors.com
101
2

Section A

Answer all questions in the spaces provided.

1(a) (i) Write equations to illustrate the formation of


 phosphorous(V) oxide when phosphorus is heated with oxgen

4P + 5O2 → P4O10 [1]

 sulfur(IV) oxide when sulfur is heated with oxygen

S + O2 → SO2

(ii) When NO2 reacts with water, nitrogen undergoes a disproportionation reaction to
give a mixture of two acids as shown:

2NO2 + H2O  HNO2 + HNO3

Use this reaction to explain the term disproportionation.

In the reaction, oxidation state of N increases from +4 in NO2 to +5 in HNO3 and


decreases to +3 in HNO2 at the same time.

(iii) In a similar disproportionation reaction, ClO2 reacts with aqueous NaOH to


produce a solution containing two chorine-containing sodium salts. Suggest the
identities of the two chorine-containing sodium salts.
[4]
NaClO2 and NaClO3

(b) The major source of sulfur for the manufacture of sulfuric acid by the Contact process is
the de-sulfurisation of ‘sour’ natural gas. Many natural gas wells produce a mixture of
volatile hydrocarbons (mainly CH4 and C2H6) together with up to 25% hydrogen sulfide,
H2S.

(i) Complete and balance the following equation showing the complete combustion
of a gaseous mixture consisting of 2 mol of CH4, 1 mol of C2H6 and 1 mol of H2S.

2CH4 + C2H6 + H2S + 9O2  SO2 + 4CO2 + 8H2O

(ii) Suggest why it is important to remove the H2S before burning the natural gas
industrially.

SO2 produced causes acid rain.

H2S is removed by passing the ‘sour’ natural gas through a solvent containing
ethanolamine, HOCH2CH2NH2. The following reaction takes place.

HOCH2CH2NH2 + H2S → HOCH2CH2NH3+ + SH–

(iii) State the type of reaction that has occurred.

Acid-base reaction

© DHS 2013 8872/02 [Turn Over


www.gradtutors.com
102
3

(iv) If a sample of natural gas contains 5% by volume of H2S, calculate the mass of
ethanolamine required to remove all the H2S from a 1000 dm3 sample of gas,
measured under room conditions.

1000 dm3 contains 50 dm3 of H2S


Moles of H2S = 50/24 = 2.08 mol
Mr (ethanolamine) = 24 + 7 + 14 +16 =61.0
Mass of ethanolamine needed = 2.08 x 61.0 = 127 g

The recovered H2S is converted to sulfur by the following two reactions.

I Part of the H2S is burned in air.

H2S(g) + O2(g)  SO2(g) + H2O(g)

II The gas stream resulting from reaction I is then blended with the remaining H2S
and fed into an iron oxide catalyst bed, where sulfur and water are produced
according to the equation.

SO2(g) + 2H2S(g)  3S(g) + 2H2O(g)

(v) Use the following date to calculate H for the reaction between H2S and SO2.

Compound Hf / kJ mol-1


H2S(g) -21
SO2(g) -297
H2O(g) -242
S(g) +11

H = [3(11) + 2(-242)] –[2(-21) + (-297)]


= -451 + 339
= -112 kJmol-1

[Total: 11]

© DHS 2013 8872/02 [Turn Over


www.gradtutors.com
103
4

2 Indigo is the dye used in blue jeans. Although originally extracted from plants of the type
indigofera, it is now almost entirely made artificially.

Indigo is insoluble in water but this disadvantage can be overcome by converting it into
the water-soluble colourless Leuco-indigo. If cloth soaked in a solution of Leuco-indigo is
left to dry in the air, the Leuco-indigo is converted into the insoluble blue indigo, which is
precipitated out onto the fibres of the cloth.

(a) On the structures above, circle and name four different functional groups present
in either indigo or leuco–indigo, which are not benzene rings.
[3]
Alkene, carbonyl (ketone), amine and alcohol

(b) Calculate the percentage by mass of carbon in Indigo.


[2]
Mr of Indigo = 262 (C16H10N2O2)

Mass of carbon present = [(16x12)/262] x 100% = 73.3 %

(c) (i) What type of reaction is the conversion of Indigo into Leuco-indigo?

Reduction

(ii) Suggest a laboratory reagent for this reaction.


[2]
LiAlH4 in dry ether or NaBH4 (in methanol)
Reject H2/Ni as an answer.

(d) Suggest the reagents and conditions for a reaction that could be used to
distinguish between Indigo and Leuco-indigo by identifying Indigo positively. In
your test, state the expected observation for each compound.
[2]

Test Indigo Leuco-indigo


2,4-DNPH, Orange ppt No orange ppt
warm

3 Sodium benzoate is a preservative and is able to prevent the growth of bacteria and
fungi. It is most widely used in acidic foods such as salad dressings, carbonated
drinks, jams and fruit juices.

According to the United States Food and Drug Administration, all food preservatives
are limited to an amount of 0.1% by weight. However, The International Programme on
Chemical Safety found no adverse effects of sodium benzoate in humans at doses of

© DHS 2013 8872/02 [Turn Over


www.gradtutors.com
104
5

647–825 mg/kg of body weight per day.

In acidic foods, sodium benzoate is absorbed into the food cells as benzoic acid. This
lowers the intracellular pH of food, creating an environment which fungi and bacteria
cannot thrive. In typical fruit juices which have a pH of 3.6, the concentration of
benzoic acid was found to be 0.001 mol dm–3. In foods with a pH level of 4.2 to 4.5, a
mixture of benzoic acid and sodium benzoate is present. In foods above pH 4.5,
sodium benzoate becomes ineffective as a preservative.

Sodium benzoate is prepared by adding benzoic acid to a hot concentrated solution of


sodium carbonate until effervescence ceases, and all reactants are used up.
Appropriate separation techniques can be applied to obtain pure solid sodium
benzoate, which is stable upon heating.

An alternative to sodium benzoate is potassium benozate, which works in exactly the


same manner.

(a) A typical jar of jam weighs 500 g. Calculate its maximum allowable mass of
sodium benzoate as a preservative.
Maximum mass of sodium benzoate allowed in 500 g of jam
0 .1
=  500
100
= 0.5 g
[1]

(b) Assuming that an average adult weighs 70 kg, calculate the maximum mass of
sodium benzoate a person can intake per day.
Maximum mass of sodium benzoate a person can intake
= 825 x 10–3 x 70
= 57.75 g
= 57.8 g
[1]

(c) (i) Calculate the concentration of H+ ions in typical fruit juices.


pH of fruit juices = 10–3.6 = 2.51 x 10–4 mol dm–3

(ii) Hence, deduce the value of Ka of benzoic acid, assuming benzoic acid is the
only acid present.
[H  ][benzoate]
Ka =
[benzoic acid]
(10 3.6 ) 2
= = 6.31 x 10–5 mol dm–3
0.001
[2]

(d) (i) State the type of solution formed by the mixture of benzoic acid and sodium
benzoate in pH 4.2 to 4.5.
The solution is a buffer solution.

(ii) With the aid of a suitable equation, explain what happens when a small
amount of hydrochloric acid was added to this mixture.
C6H5COO– + H+ → C6H5COOH
When a small amount of hydrochloric acid was added, the H+ was removed
as benzoic acid. The pH remained fairly constant.
[3]

© DHS 2013 8872/02 [Turn Over


www.gradtutors.com
105
6

(e) (i) Write the balanced equation for the formation of sodium benzoate from
sodium carbonate.
2C6H5COOH + Na2CO3 → 2C6H5COO–Na+ + CO2 + H2O

(ii) Suggest a simple method to isolate solid sodium benzoate from the reaction
in (e) (i).
Pure solid sodium benzoate can be obtained by evaporating the mixture to
dryness.
[2]

(f) Predict, with explanations, if potassium benzoate has a higher or lower boiling
point than sodium benzoate.
Charge of cation: K+ = Na+
Size of cation: K+ > Na+
Lattice energy: K+ < Na+
Thus, boiling point: K+ < Na+
[2]

[Total: 11]

4 (a) KMnO4 and K2Cr2O7 are oxidising agents, usually used in acidic solutions.

(i) State the colour changes you would expect to observe, when each of these
reagents are heated with butan–1–ol.

KMnO4 : Purple to Colourless


K2Cr2O7 : Orange to Green

(ii) When heated with butan–1–ol, each of these reagents give rise to different
products when different conditions are applied. Suggest how the two
products can be obtained by using KMnO4 and K2Cr2O7. Include a balanced
equation for each set of reagents and conditions.

Reagents and conditions : KMnO4, H2SO4 (aq), reflux


Equation : CH3CH2CH2CH2OH + 2[O] → CH3CH2CH2COOH + H2O

Reagents and conditions : K2Cr2O7, H2SO4 (aq), heat with distillation


Equation : CH3CH2CH2CH2OH + [O] → CH3CH2CH2CHO + H2O

[6]

(b) The following table shows the solubilities of butan–1–ol and but–1–ene in water.

Compound Mass soluble in 1 dm3


Butan–1–ol 73.0
But–1–ene 2.21

Explain the above data.


Butan–1–ol is soluble (or more soluble) in water as favourable hydrogen
bonds can be formed between butan–1–ol and water molecules.

But–1–ene is not soluble (or has low solubility) in water as the weak Van der

© DHS 2013 8872/02 [Turn Over


www.gradtutors.com
106
7

Waals’ forces between the but–1–ene molecules cannot overcome the


stronger hydrogen bonds between the water molecules.
[2]

(c) The energy profile diagram below represents the hydrogenation of but-1-ene to
butane. On the same diagram, draw the energy profile when nickel is added to
the reaction.
Energy

But–1–ene
Butane
Reaction pathway
[1]

[Total: 9]

© DHS 2013 8872/02 [Turn Over


www.gradtutors.com
107
8

Answer any two questions.

5 Dinitrogen pentoxide, N2O5, can be produced by the following reaction sequence in the
car engine:

I N2(g) + O2(g)  2NO(g) ∆HӨ = +180 kJ mol-1


II NO(g) + 1/2O2(g)  NO2(g) ∆HӨ = –57.0 kJ mol-1
III 2NO2(g) + 1/2O2(g)  N2O5(g) ∆HӨ = –110 kJ mol-1

(a) (i) By using relevant data in the Data Booklet, calculate the bond energy of the
N-O bond in nitric oxide in reaction I.

BE(N2) + BE(O2) – 2BE(NO) = 180


994 + 496 – 2BE(NO) = 180
BE(NO) = +655 kJ mol-1

(ii) Suggest an explanation why reaction I occurs in car engine.

The high temperature in the car engine is able to overcome the strong O=O
bond and N≡N bond.

(iii) Nitrogen dioxide is a pollutant that is often produced in car engine. Explain
how the pollutant can be removed in the car engine.

Pt
2NO2(g)  N2(g) + 2O2(g) [Equation is not essential]

The nitrogen dioxide is converted to nitrogen and oxygen gas in the


catalytic convertor (with Pt as catalyst) in the car engine.

Pt catalyst is not the essential marking point. As long as catalyst/catalytic


convertor is mentioned.

(iv) By using the given energy cycle and the data above, calculate the standard
enthalpy change of formation of dinitrogen pentoxide. [2]
∆HfӨ
N2(g) + 5/2O2(g) N2O5(g)

2NO(g) + 3/2O2(g)

2NO2(g) + 1/2O2(g)

By Hess’ law,
∆HfӨ(N2O5) = +180 +2(–57.0) – 110
= –44.0 kJ mol-1

(b) The rate of reaction for reaction II was investigated and the following data
was obtained.

© DHS 2013 8872/02 [Turn Over


www.gradtutors.com
108
9

Time/minutes Experiment 1, Experiment 2,


with [NO] = 0.10 mol dm-3 with [NO] = 0.05 mol dm-3
[O2] / mol dm-3 [O2] / mol dm-3
0 0.0050 0.0050
5 0.0031 0.0045
10 0.0019 0.0040
15 0.0011 0.0036
20 0.0007 0.0032
25 0.0005 0.0029
30 0.0004 0.0026

(i) Explain why nitrogen monoxide is used in large excess.

This is to ensure that the concentration of nitrogen monoxide remains


approximately constant so that the rate of reaction will be independent of
the concentration of nitrogen monoxide.

(ii) Using the same axes, plot graphs of [O2] against time for the two
experiments.

t1 t2

(iii) Use your graphs to determine the order of reaction with respect to O2 and
NO, showing your workings clearly.

Using the curve for experiment 1, t1 ≈ t2 ≈ 7 min (accept 6.5 – 7 min). The
half-lives of O2 are approximately constant at 7 min, hence the reaction is
first order with respect to O2.
When [NO] = 0.1 mol dm-3, initial rate = 4 x 10-4 mol dm-3 min-1
When [NO] = 0.05 mol dm-3, initial rate = 1 x 10-4 mol dm-3 min-1
When [NO] is doubled, the initial rate is quadrupled. Hence, the reaction is
second order with respect to NO.

(iv) State the rate equation of reaction II and hence calculate the rate constant,
including its units.

© DHS 2013 8872/02 [Turn Over


www.gradtutors.com
109
10

rate = k[O2][NO]2

Using initial rate method:


4 x 10-4 = k(0.005)(0.1)2
k = 8.00 mol-2 dm6 min-1

or using half-life method:


rate = k’[O2] where k’ = k[NO]2
k’ = ln2 / 7 = 0.09902 min-1
k = 0.09902 / (0.1)2 = 9.90 mol-2 dm6 min-1

(c) The nitrogen dioxide produced in reaction II is able to form dinitrogen


tetraoxide as shown below:

2NO2(g) N2O4(g)
brown colourless

(i) Draw a dot-and-cross diagram to illustrate the bonding in a nitrogen dioxide


molecule and predict its shape. [2]

shape about nitrogen atom = bent

(ii) Explain whether the enthalpy change of dimerisation is endothermic or


exothermic. Hence, predict the colour change of the reaction mixture when
the temperature is increased. [2]

The forward reaction is exothermic as N-N bond is formed during


dimerisation. By Le Chatelier’s principle, an increase in temperature
favours the backward reaction to absorb heat (or the position of equlibrium
shifts to the left). Hence, the colour of the reaction mixture becomes darker
brown.

[Total: 20]

6 (a) An experiment is conducted to determine the value of n in the formula of a cation


[Cu(NH3)n]2+. This cation is formed when copper(II) reacts with ammonia.

100 cm3 of 0.100 mol dm–3 of copper(II) sulfate is mixed with 100 cm3 of aqueous
ammonia. The resulting mixture, which contains the cation [Cu(NH3)n]2+ and an excess
of ammonia, is then shaken with trichloromethane and allowed to stand for equilibrium
to be established. The excess ammonia is dissolved in both the aqueous layer and the
trichloromethane layer.

25.0 cm3 of trichloromethane layer is found to neutralise 26.00 cm3 of


0.025 mol dm–3 hydrochloric acid while 25.0 cm3 of aqueous layer required
21.00 cm3 of 1.00 mol dm–3 hydrochloric acid for neutralisation.

The equilibrium constant of ammonia between the aqueous layer and the
trichloromethane layer is 25.0.

NH3 (aq) NH3 (CHCl3)

© DHS 2013 8872/02 [Turn Over


www.gradtutors.com
110
11

[ammonia] aqueous
K=
[ammonia] CHCl3
Calculate:
(i) the concentration of ammonia in the trichloromethane layer.

[NH3] = 26.00/25.0 x 0.025 = 0.026 mol dm–3

(ii) the concentration of free ammonia in the aqueous layer, using the equilibrium
constant.

[ammonia] aqueous
KPartition coefficient = = 25.0
[ammonia] CHCl3

[ammonia]aqueous = 0.026 x 25.0 = 0.65 mol dm–3

(iii) the total concentration of ammonia (free and in the cation [Cu(NH3)n]2+) in the
aqueous layer.

Total [ammonia] = 21.00/25.0 x 1.00 = 0.84 mol dm–3

(iv) the value of n in the formula of the cation [Cu(NH3)n]2+, stating one assumption
made in the calculation.
[7]
[ammonia] (complexed) = 0.84 – 0.65 = 0.19 mol dm–3

nCu2+ = 0.100 x 0.100 = 0.0100 mol

nammonia (complexed) = 0.19 x 200/1000 = 0.038 mol

n = 0.038/0.01 = 3.8
4

Assumption:
1. All the ammonia in the compound, [Cu(NH3)n]2+, reacted with hydrochloric
acid.
2. None of the copper complex ion is partitioned/ migrated to the organic layer.

(b) Both beryllium difluoride and boron trifluoride can also react with ammonia. However,
their mole ratios are different.

Beryllium difluoride and ammonia react in a 1:2 ratio while boron trifluoride and
ammonia react in a 1:1 ratio.

(i) Draw diagrams to show the bonding within these molecules and state the type of
bond formed in the products.

© DHS 2013 8872/02 [Turn Over


www.gradtutors.com
111
12

BeF2 and BF3 react with NH3 through dative (or co–ordinate) bonds.

(ii) Explain why the mole ratios are different for these two compounds.
[6]
Beryllium in BeF2 has only 4 valence electrons while boron in BF3 has 6
valence electrons. Hence beryllium in BeF2 needs to form two dative bonds
with two ammonia molecules while boron in BF3 only needs to form one
dative bond with one ammonia molecule, in order to achieve stable octet
electronic configuration.

(c) The graph below shows the variation in first ionisation energy of some elements in
Period 3.

First ionisation energy


X
Y

n n+1 n+2 n+3 n+4

Proton number
Elements X and Y have proton numbers n and n+1 respectively.

(i) By considering the trend in first ionisation energy, state and explain which Group
element Y belongs to.
Group III. A Group III metal has a lower first ionisation energy than a Group
II element X because less energy is required to remove a 3p electron in Y
than a 3s electron in X since a 3p electron is higher in energy and is more
strongly held by the nucleus.

(ii) Describe the behaviour of the oxide of X in dilute hydrochloric acid and sodium
hydroxide respectively.
A Group II metal (X) oxide is basic and undergoes neutralisation with acid
but does not react with dilute NaOH.

(iii) Sodium carbonate was added to a solution of the chloride of Y and


effervescence was observed. Explain this observation with the aid of relevant
equations.
[7]
[Total: 20]

Element Y is in Group III and is Al. AlCl3 undergoes hydrolysis to give an


acidic solution which reacts with sodium carbonate to form CO2 gas.
AlCl3 (s) + 6H2O (l)  [Al(H2O)6]3+ (aq) + 3Cl– (aq)
[Al (H2O)6]3+ (aq) [Al(H2O)5(OH)]2+(aq) + H+ (aq)
2H+ (aq) + CO32–(aq)  H2O(l) + CO2(g)

OR
YCl3 undergoes hydrolysis to give an acidic solution which reacts with sodium

© DHS 2013 8872/02 [Turn Over


www.gradtutors.com
112
13

carbonate to form CO2 gas.


[Y(H2O)6]3+ (aq) [Y(H2O)5(OH)]2+(aq) + H+ (aq)
2H+ (aq) + CO32–(aq)  H2O(l) + CO2(g)

3 (a) A tooth is made up of two parts: the crown and the root. Dental crown is the visible
part of the teeth which is made of enamel and dentine.

Enamel
Crown
Dentine

Root

The enamel is made of hydroxyapatite, Ca5(PO4)3OH. In the mouth, mineral


substances such as calcium ions and phosphate ions that are present in saliva
contribute to the formation and decomposition of the hydroxyapatite. These two
processes occur simultaneously until an equilibrium is reached. The formation process
is called mineralisation of the enamel, whereas the decomposition process is called
demineralisation.

Ca5(PO4)3OH(s) 5Ca2+(aq) + 3PO43–(aq) + OH–(aq)

Explain how an acidic medium can affect the demineralisation of teeth.


[2]
Ca5(PO4)3OH(s) 5Ca2+(aq) + 3PO43–(aq) + OH–(aq) --- (1)

In an acidic medium, the H+ ions can react with OH– or PO43– ions produced during
demineralisation of hydroxyapatite in eqm (1). By Le Chatelier’s Principle, this shifts
the equilibrium (1) forward (or to the right), encouraging/ accelerating the
demineralisation process which resulted in tooth decalcification.

(b) An organic compound R (Mr = 113.6) contains by mass 63.2% carbon, 8.7%
hydrogen, and 28.1% oxygen. R undergoes an addition reaction with bromine in a
1:1 mole ratio. It also reacts with sodium hydrogen carbonate, liberating carbon
dioxide gas.

(i) Calculate the empirical formula of R.

Element C H O
% by mass 63.2 8.7 28.1
63.2 / 12.0 8.7 / 1.0 28.1 / 16.0
No. of moles
= 5.27 = 8.7 = 1.76
Mole ratio 3 5 1
Empirical formula of R is C3H5O.

© DHS 2013 8872/02 [Turn Over


www.gradtutors.com
113
14

(ii) Determine the molecular formula of R.

Let the molecular formula of R be (C3H5O)n.


(3 x 12.0 + 5 x 1.0 + 16.0)n = 113.6
n=2
Molecular formula of R is C6H10O2.

(iii) Deduce with reasoning, a possible structure for compound R.

R contains a –COOH functional group since it undergoes acid–


base/neutralisation reaction with sodium hydrogen carbonate gives CO2.
R contains one C=C double bond as it reacts with bromine in a 1:1 mole ratio.
Therefore R is CH2=CHCH(CH2CH3)CO2H
[8]

(c) Suggest a suitable chemical test to distinguish between molecules S and T and
describe the expected observations for each molecule.
OH O OH O

CH3 CH2CH3

S T

Add aqueous iodine in aqueous NaOH to separate samples of S and T, and warm.
[1] S gives a yellow precipitate while T does not give a yellow precipitate.

(d) The elements of Group VII, especially chlorine, have played an important
role in the development of chemistry. The following describes a reaction
involving a chlorine containing compound.

An interhalogen compound A, with the formula IxCly, was dissolved in


aqueous potassium iodide, and a brown solution was obtained.

IxCly + I–  I2 + Cl– (not balanced)

The brown solution was titrated with 0.50 mol dm–3 sodium thiosulfate and
34.0 cm3 was required to reach the end–point. Careful addition of aqueous
silver nitrate produced a white precipitate of silver chloride, which
weighed 1.83 g when dried.
[4]

(i) Calculate the amount of iodine produced in the reaction shown above.

I2 + 2S2O32–  2I– + S4O62–

n(I2) produced when reacted with KI


= [0.50 x (34.0/1000)] / 2 = 8.50 x 10-3 mol

(ii) Calculate the amount of chloride ions produced in the reaction shown
above.

© DHS 2013 8872/02 [Turn Over


www.gradtutors.com
114
15

n(Cl–) = 1.83 / 143.5 = 0.0128 mol

(iii) Determine the formula of compound A and hence construct a balanced


equation between compound A and potassium iodide.

n(Cl–): n(I2) = 0.0128 : 8.50 x 10-3 = 3 : 2


 simplest mole ratio, x:y = 1:3
Hence formula of A is ICl3.
ICl3 + 3I–  2I2 + 3Cl–

(e) Anhydrous iron(III) chloride is a dark brown moisture sensitive solid (melting
point = 306 oC) which forms an acidic solution in water. It exists as a dimer,
Fe2Cl6 in the solid state. In the gaseous state, the dimer increasingly
o
dissociates into its monomer until dissociation is complete at 400 C.

Fe2Cl6(g) 2FeCl3(g)

0.450 g of Fe2Cl6 was placed in a 0.500 dm3 evacuated flask and heated to
350 oC. The amount of gaseous mixture at equilibrium was found to be 2.54
x 10–3 mol.

Using the data above, calculate the following:


 the initial amount of Fe2Cl6(g);
 the amount of dissociated Fe2Cl6(g);
 and hence the degree of dissociation of Fe2Cl6 at 350 oC.

Degree of dissociation = (fraction of dissociated Fe2Cl6(g)) x 100 %


[4]

0.450
Initial amount of Fe2Cl6 =  0.001386 mol
55.8x2  35.5x6

Let amount of Fe2Cl6 dissociated at equilibrium be a mol

Fe2Cl6(g) 2FeCl3(g)
Initial amount / mol 0.001386 0
Change in amount / mol –a + 2a
Equilibrium amount / mol 0.001386 – a 2a

Total amount of gases at equilibrium = 0.001386  a + 2a = 0.00254


Hence, 0.001386 + a = 0.00254
a = 0.001154
or [1] for any appropriate working to calculate the unknown value indicated
by student.
0.001154
Degree of dissociation = x100%  83.3 %
0.001386

© DHS 2013 8872/02 [Turn Over


www.gradtutors.com
115
16

(f) Iron(III) chloride is a catalyst used in the synthesis of chlorobenzene from


benzene.

Explain the catalytic effect of iron(III) chloride on the rate of reaction in terms of
the Maxwell-Boltzmann distribution.
Fraction of molecule
with energy, E No. of particles with E  Ea
for uncatalysed reaction

No. of particles with E  Ea’


for catalysed reaction

Energy, E
Ea1 Ea
(catalysed) (uncatalysed)

When a catalyst is present,

 number of reactant particles with at least the activation energy (Ea’(cat))


increases
 number of effective collisions taking place in the reaction increases
 rate of reaction is proportional to the frequency of effective collisions.
 rate of reaction increases
[3]

© DHS 2013 8872/02 [Turn Over


www.gradtutors.com
116
 
 
HWA CHONG
C INS
STITUTION
Prelimiinary Exam
mination
Higher 1

CANDIDATE
OUP
CT GRO 122S
NAME

CENTR
RE INDEX
NUMBE
ER NUMBEER

CHEMISTRY 8872/01
Paper 1 Multiple Choice
C 255 Septemb
ber 2013
50 min
Addition
nal Materialls: Multiple Choice Ansswer Sheet
Data Boooklet

READ T
THESE INS
STRUCTION
NS FIRST

Write in
n soft pencil.
Comple ete the inforrmation on the
t optical m
mark sheet (OMS) as shown
s below
w.

Write
W your na
ame

Write
W your CT
T group

Write
W and
shade
s yourr
NRIC
N
or
o FIN num
mber

There aare thirty qu


uestions on this paper.. Answer all questions. For each qquestion, th
here are
four posssible answ
wers A, B, C and D.
Choose e the one yoou considerr correct and our choice in
d record yo n soft penccil on the OMS.

Each co
orrect answ
wer will score educted for a wrong an
e one markk. A mark wiill not be de nswer.
Any rou
ugh working
g should be done in thiss booklet.

This document
d cconsists of 12
1 printed pages.
p
  www.gradtutors.com
117
2
 
Section A

For each question there are four possible answers, A, B, C, and D. Choose the one you consider to
be correct.

1 How many neutrons are present in 0.13g of 13C?


[L = the Avogadro constant]

A 0.06L B 0.07L C 0.13L D 0.91L

2 Phosphorus sulfide, P4S3, is used in small amounts in the tips of matches. On striking a
match, this compound burns to produce an oxide of phosphorus in the +5 oxidation state
and an oxide of sulfur in the +4 oxidation state.

How many moles of oxygen gas are needed to burn one mole of P4S3 in this way?

A 6 B 7.5 C 8 D 16

3 25.00 cm3 of a solution of acidified iron(II) sulfate, FeSO4, were titrated with
0.0200 mol dm–3 potassium manganate(VII). The mean titre was 27.40 cm3.

The equation for the reaction is shown.

MnO4– + 8H+ + 5Fe2+ → Mn2+ + 4H2O + 5Fe3+

What is the concentration of iron(II) sulfate solution?

A 4.38 ×10–3 mol dm–3


B 2.19 ×10–2 mol dm–3
C 9.12 ×10–2 mol dm–3
D 1.10 ×10–1 mol dm–3

4 Which factor helps to explain why the first ionisation energies of the Group I elements
decrease from lithium to sodium to potassium to rubidium?

A The nuclear charge of the elements increases.


B The outer electron is in an ‘s’ subshell.
C The repulsion between spin–paired electrons increases.
D The shielding effect of the inner shells increases.

2013 HCI C2 H1 Chemistry Preliminary Examination / Paper 1

  www.gradtutors.com
118
3
 
5 In which pair of molecules are the values of the bond angles the closest?

A BF3 and NH3


B C2H4 and BF3
C H2O and C2H4
D CH4 and H2O

6 In the last century the Haber process was sometimes run at pressures of 1000 atm and
higher. Now it is commonly run at pressures below 100 atm.

What is the reason for this change?

A An iron catalyst is used.


B Maintaining the higher pressures is more expensive.
C The equilibrium yield of ammonia is increased at lower pressures.
D The rate of the reaction is increased at lower pressures.

7 In the analysis of an oxide of nitrogen, 0.10 mol of the oxide were reacted with excess
hydrogen under suitable conditions.

3.6 g of water were formed in this reaction, while the ammonia produced required 100 cm3
of 1.0 mol dm–3 HCl(aq) for neutralisation.

What is the formula of the oxide of nitrogen analysed?

A NO2 B NO C N2O D N2O5

8 In the gas phase, ammonia reacts with chlorine.

8NH3+ 3Cl2 → N2+ 6NH4Cl

Which row indicates the correct combination of statements about this reaction?

ammonia acts as ammonia acts as a dative bond is


a reducing agent a base formed
A   
B   
C   
D   

2013 HCI C2 H1 Chemistry Preliminary Examination / Paper 1

  www.gradtutors.com
119
4
 
9 In which change would only van der Waals’ forces have to be overcome?

A evaporation of ethanol C2H5OH(l) → C2H5OH(g)


B melting of ice H2O(s) → H2O(l)
C melting of solid carbon dioxide CO2(s) → CO2(l)
D solidification of butane C4H10(l) → C4H10(s)

10 Which enthalpy change could not be correctly represented by the enthalpy diagram shown?

A standard enthalpy change of atomisation


B standard enthalpy change of combustion
C standard enthalpy change of hydration
D standard enthalpy change of neutralisation

11 In the diagram, curve X was obtained by observing the decomposition of 100 cm3 of 1.0 mol
dm–3 hydrogen peroxide, catalysed by manganese(IV) oxide.

Which alteration to the original experimental conditions would produce curve Y?

A adding some 0.1 mol dm–3 hydrogen peroxide


B adding water
C lowering the temperature
D using less manganese(IV) oxide

2013 HCI C2 H1 Chemistry Preliminary Examination / Paper 1

  www.gradtutors.com
120
5
 

12 Different Boltzmann distributions are shown in the diagrams.

In diagram 1, one curve P or Q corresponds to a temperature higher than that of the other
curve.

In diagram 2, one line X or Y corresponds to the activation energy for a catalysed reaction
and the other line corresponds to the activation energy of the same reaction when
uncatalysed.

Which combination gives the correct curve and line?

higher temperature presence of catalyst


A P X
B P Y
C Q X
D Q Y

13 Which set of solid elements contains a simple molecular structure, a giant covalent
(macromolecular) structure and a giant metallic structure?

A Mg, P, S
B P, Si, C
C S, P, Si
D S, Si, Al

2013 HCI C2 H1 Chemistry Preliminary Examination / Paper 1

  www.gradtutors.com
121
6
 
14 The diagram represents the energy changes for some reactions.

What are the natures of the conversions W → Y, Y → X and Z → W?

W→Y Y→X Z→W


A exothermic endothermic endothermic
B exothermic exothermic endothermic
C endothermic exothermic exothermic
D endothermic endothermic exothermic

15 Casein is a protein commonly found in cow’s milk. It exists in milk as a suspension of


particles where molecules will group together in three–dimensional spherical clusters.
Hydrocarbon chains fill the interior of the cluster and the ionic ends compose the outer
surface. These molecular conglomerations are known as micelles.

water 
a micelle 

What type of bonding is most likely to occur in the region labeled Y?

A Dispersion forces
B Hydrogen bonding
C Ion–dipole interactions
D Permanent dipole–permanent dipole interactions

2013 HCI C2 H1 Chemistry Preliminary Examination / Paper 1

  www.gradtutors.com
122
7
 
16 Three substances, R, S and T, have physical properties as shown.

substance R S T
mp / oC 801 2852 3550
bp / oC 1413 3600 4827
electrical conductivity of solid poor poor good

What could be the identities of R, S and T?

R S T
A MgO NaCl C [graphite]
B MgO NaCl SiO2
C NaCl MgO C [graphite]
D NaCl MgO SiO2

17 The sketch below shows the variation of first ionisation energy with proton number for six
elements of consecutive proton numbers between 1 and 18 (H to Ar).

What is the identity of the element X?

A Mg B Al C Si D P

18 When 0.20 mol of hydrogen gas and 0.15 mol of iodine gas are heated at 723 K until
equilibrium is established, the equilibrium mixture is found to contain 0.26 mol of hydrogen
iodide.
The equation for the reaction is as follows.
H2(g) + I2(g) 2HI(g)

What is the correct expression for the equilibrium constant Kc?

A 2  0.26 C (0.26)2
0.20  0.15 0.07  0.02
B (2  0.26)2 D (0.26)2
0.20  0.15 0.13  0.13

2013 HCI C2 H1 Chemistry Preliminary Examination / Paper 1

  www.gradtutors.com
123
8
 
19 In which class of compound, in its general formula, is the ratio of hydrogen atoms to carbon
atoms the highest?

A alcohols
B aldehydes
C carboxylic acids
D halogenoalkanes

20 The Russian composer Borodin was also a research chemist who discovered a reaction in
which two ethanal molecules combine to form a compound commonly known as aldol
(reaction I). Aldol forms another compound on heating (reaction II).

I 2CH3CHO  CH3CH(OH)CH2CHO
II CH3CH(OH)CH2CHO  CH3CH=CHCHO + H2O

Which of the following best describes reactions I and reaction II?

I I
A addition elimination
B addition reduction
C elimination reduction
D substitution elimination

21 A hydrocarbon, R on heating with an excess of hot concentrated acidic KMnO4(aq),


produces CH3COCH3 and (CH3)2CHCO2H as the only organic products.

What is R?

A (CH3)2C=C(CH3)CH2CH3
B (CH3)2C=CHCH2CH2CH3
C

2013 HCI C2 H1 Chemistry Preliminary Examination / Paper 1

  www.gradtutors.com
124
9
 

22 Which property of benzene may be directly attributed to the stability associated with its
delocalised electrons?

A It has a low boiling point.


B It does not conduct electricity.
C It tends to undergo substitution rather than addition reactions.
D It does not react with acidified KMnO4

23 Ibuprofen is an anti–inflammatory drug.

ibuprofen

What reaction would lead to its formation?

24 In its reaction with sodium, 1 mol of a compound X gives 1 mol of H2(g).

Which compound might X be?

A CH3CH2CH2CH2OH
B (CH3)3COH
C CH3CH2CH2CO2H
D CH3CH(OH)CO2H

2013 HCI C2 H1 Chemistry Preliminary Examination / Paper 1

  www.gradtutors.com
125
10
 

25 A compound P was boiled with aqueous sodium hydroxide and the resulting mixture was
cooled and acidified with dilute sulfuric acid. The final products included a compound
C3H6O2 and an alcohol which gave a positive triiodomethane test.

Which of the following formulae could represent P?

A CH3CH2CO2CH3

B CH3CH2OCOCH3

C CH2(OH)CH2CH2COCH3

D CH3CH2CO2CH2CH3

2013 HCI C2 H1 Chemistry Preliminary Examination / Paper 1

  www.gradtutors.com
126
11
 
Section B

For each of the questions in this section, one or more of the three numbered statements 1 to 3 may
be correct.

Decide whether each of the statements is or is not correct (you may find it helpful to put a tick
against the statements that you consider to be correct).

The responses A to D should be selected on the basis of

A B C D
1, 2 and 3 1 and 2 2 and 3 1 only
are are are is
correct correct correct correct

No other combination of statements is used as a correct response.

26 Use of the Data Booklet is relevant to this question.

In which pairs do both species have the same number of unpaired p electrons?

1 O and Cl+
2 F+ and Ga+
3 P and Ne+

27 Which are features of the structure of metallic copper?

1 a lattice of ions
2 delocalised electrons
3 ionic bonds

28 Which descriptions of the ammonium ion are correct?

1 It contains ten electrons.


2 It has a bond angle of 109.5°.
3 It has four bonding pairs of electrons.

2013 HCI C2 H1 Chemistry Preliminary Examination / Paper 1

  www.gradtutors.com
127
12
 
The responses A to D should be selected on the basis of

A B C D
1, 2 and 3 1 and 2 2 and 3 1 only
are are are is
correct correct correct correct

No other combination of statements is used as a correct response.

29 Deuterium, D, is the 21H isotope of hydrogen. DBr has the same chemical properties as HBr.

Which compounds could be made by the reaction of DBr with another compound in a single
reaction?

2 CHBr2CHDBr
3

30 How can the rate of reaction between ethanal and aqueous hydrogen cyanide be
increased?

1 by irradiation with ultraviolet light


2 by a rise in temperature
3 by the addition of a small quantity of aqueous sodium cyanide
 
 

2013 HCI C2 H1 Chemistry Preliminary Examination / Paper 1

  www.gradtutors.com
128

HWA CHONG INSTITUTION


Preliminary Examination
Higher 1

CANDIDATE
CT GROUP 12S
NAME

CHEMISTRY 8872/02
Paper 2 18 September 2013
2 hours
Candidates answer Section A on the Question Paper.
Additional Materials: Data Booklet
Writing paper

READ THESE INSTRUCTIONS FIRST


Write your name and CT group on all the work you hand in.
Write in dark blue or black pen.
You may use a pencil for any diagrams, graphs or rough working.
Do not use staples, paper clips, highlighters, glue, correction fluid or tapes.

Section A
Answer all questions.

Section B
Answer two questions on separate answer paper.
A Data Booklet is provided.
At the end of the examination, fasten all your work securely together.
The number of marks is given in brackets [ ] at the end of each question or part question.

FOR EXAMINERS’ USE ONLY

Paper 1 Paper 2 TOTAL


Multiple Choice Section A (Structured) Section B (Free Response)
Q1 /8 Q5 / 20
Q2 / 15 Q6 / 20
Q3 /5 Q7 / 20
Q4 / 12
110
/ 30 Subtotal / 40 Subtotal / 40

This question booklet consists of 13 printed pages and 1 blank page.

www.gradtutors.com
129
2

Section A

Answer all the questions in this section in the spaces provided.

1 (a) The elements of the third period of the Periodic Table, sodium to sulfur, all form chlorides
by direct combination.

(i) Sulfur forms a number of chlorides which are liquid at room temperature.
Which other element of the third period forms a chloride which is liquid at room
temperature?

…………………………………………………………………………………………………..

Aluminium chloride may be produced by passing a stream of chlorine over heated


aluminium powder in a long hard−glass tube.

(ii) Write a balanced equation, with state symbols, for this reaction of aluminium with
chlorine.

…………………………………………………………………………………………………..

(iii) No chloride of argon has ever been produced.


Suggest a reason for this.

…………………………………………………………………………………………………..

…………………………………………………………………………………………………..

[3]

(b) When chlorides of the elements of the third period are added to water, some simply
dissolve while others can be seen to react with the water.

Write equations for the reactions of the chlorides with water.

Sodium chloride

……………………………..…………………………………………………………………………..

Aluminium chloride

……………………………..…………………………………………………………………………..

Silicon tetrachloride

……………………………..…………………………………………………………………………..

[3]

2013 HCI C2 H1 Chemistry Prelim / Paper 2


www.gradtutors.com
130
3

(c) Sulfur forms the compound S4N4 with nitrogen. The structure of S4N4 is shown below.
Assume all bonds shown are single bonds.

(i) Determine the number of lone pairs of electrons around a nitrogen atom and a sulfur
atom in S4N4.

Nitrogen atom: ………………….…………………………

Sulfur atom: …………..……………………………………

(ii) Which bond angle, a or b, in the S4N4 molecule will be smaller? Explain your answer.

…………………………………………………………………………………………………..

…………………………………………………………………………………………………..

[2]
[Total: 8]

2013 HCI C2 H1 Chemistry Prelim / Paper 2


www.gradtutors.com
131
4

2 (a) E300 is an antioxidant used in white wines. It prevents dissolved oxygen reacting with the
ethanol to form an acid, X, which would produce a sour−tasting wine.

In the box below draw the displayed formula of the acid, X, responsible for the wine’s sour
taste.

[1]

(b) A student attempted to oxidise ethanol to acid X in the laboratory.

Give the chemical reagents and reaction conditions the student would need to use.

Reagents: ………………….……………………………………….………………………………..

Conditions: …………..……………………………………………………..………………………..

[2]

(c) Use the table of characteristic values for infra−red absorption in the Data Booklet to
answer this question.

Infra−red absorptions can be used to identify functional groups in organic compounds.


For example, ethyl ethanoate shows absorptions at 1000−1300 cm−1 and 1680−1750 cm−1.

The infra−red spectrum of the purified product the student actually obtained has strong
absorptions at about 1720−1740 cm−1; it has no broad peak at 2500−3200 cm−1.

Use the Data Booklet together with the information given to identify the product made by
the student. Give your reasoning.

Name or formula of product: ………..…………………………….………………………………..

Reasoning: …………..……………………………………………………..………………………..

…………………………………………………………………………………………………………

…………………………………………………………………………………………………………

[3]

2013 HCI C2 H1 Chemistry Prelim / Paper 2


www.gradtutors.com
132
5
(d) When E300 is added to water, an acid−base equilibrium is set up, as shown below.

(i) Explain what is meant by the term base in the Bronsted−Lowry theory.

…………………………………………………………………………………………………..

(ii) Circle two species in the equation above that are acting as bases.

(iii) When a solution of E300 reacts with calcium carbonate, fizzing occurs and a solid
product can be extracted from the resulting mixture.

Identify the gas produced and suggest a formula for the solid product.

Gas: ………………….……………………………………….………………………………..

Formula of solid product: …………………………………………..………………………..

[4]

(e) E300 has a C=C bond with two different groups on each carbon. It does not, however,
show geometrical isomerism whereas 1,2−dichloroethene does.

Explain why 1,2−dichloroethene shows geometrical isomerism and suggest a reason why
E300 does not.

……………….………..……………………………………………………..………………………..

…………………………………………………………………………………………………………

…………………………………………………………………………………………………………

……………….………..……………………………………………………..………………………..

……………….………..……………………………………………………..………………………..

[2]

2013 HCI C2 H1 Chemistry Prelim / Paper 2


www.gradtutors.com
133
6

(f) A primary alcohol group in E300 reacts with C17H35COOH to form another antioxidant.

(i) Circle a primary alcohol group on the structure of E300 below.

(ii) Draw the structural formula of the new functional group formed in this reaction.

(iii) What else must be added to a mixture of E300 and C17H35COOH, to make the new
antioxidant?

…………………………………………………………………………………………………..

[3]

[Total: 15]

2013 HCI C2 H1 Chemistry Prelim / Paper 2


www.gradtutors.com
134
7

3 (a) Dinitrogen tetroxide, N2O4(g) , and nitrogen dioxide, NO2(g), coexist in the following
dynamic equilibrium.

2NO2(g) N2O4(g) ∆H = −57 kJmol−1

A chemist adds 4.00 moles of NO2 to a container with a volume of 2.00 dm3. The container
is sealed, heated to a constant temperature and allowed to reach equilibrium.

The equilibrium mixture contains 3.20 moles of NO2.

(i) Explain what is meant by the term dynamic equilibrium.

…………………………………………………………………………………………………..

…………………………………………………………………………………………………..

(ii) Write an expression for the equilibrium constant Kc for this reaction.

(iii) Calculate the value of Kc under these conditions, giving the units.

[4]

(b) The experiment is repeated but the temperature is increased.

State the effect on the concentrations of NO2 and N2O4 when the mixture reached
equilibrium.

……………….………..……………………………………………………..………………………..

…………………………………………………………………………………………………………

[1]

[Total: 5]

2013 HCI C2 H1 Chemistry Prelim / Paper 2


www.gradtutors.com
135
8

4 (a) Simple esters are flammable liquids. Flammability is affected by volatility.

Write the following homologous series in order of boiling point, assuming molecular
masses are similar.

alcohols alkanes esters

highest boiling point …………….…………………………


…………….…………………………

lowest boiling point …………….…………………………


[1]

(b) (i) The structure of methyl ethanoate, C3H6O2, is shown below.

Write an equation for the complete combustion of methyl ethanoate.

…………………………………………………………………………………………………..

(ii) Define standard enthalpy change of formation.

…………………………………………………………………………………………………..

…………………………………………………………………………………………………..

…………………………………………………………………………………………………..

(iii) Use the standard enthalpy changes of combustion, ∆HcO, in Table 1 to calculate the
standard enthalpy change of formation of methyl ethanoate.

Table 1
∆HcO, / kJ mol−1
carbon –393.5
hydrogen –285.8
methyl ethanoate –1592.1

………………………….. kJ mol−1

[5]
2013 HCI C2 H1 Chemistry Prelim / Paper 2
www.gradtutors.com
136
9

(c) A student used the apparatus shown in Figure 1 to carry out experiments to determine the
standard enthalpy change of combustion for methyl ethanoate.

Figure 1

An initial experiment was carried out using methyl ethanoate. This ester was combusted in
a spirit burner underneath a copper can, which has a mass of 250 g, so that the flame from
the burner heated 300 cm3 of water in the can. It was found that 0.980 g of ester was
required to raise the temperature of the water in the can by 10.0 oC.

Calculate the total thermal energy in kJ gained by the water and the copper can in this
initial experiment.

The specific heat capacities of water and copper are 4.18 and 0.384 J g−1 K−1,
respectively.
Take the density of water to be 1.00 g cm−3. Assume that the water and copper are in
thermal equilibrium with each other. Express your answer to the appropriate number of
significant figures.
 
Thermal energy added to water =

Thermal energy added to copper =

Total thermal energy =

………………………….. kJ
[3]

2013 HCI C2 H1 Chemistry Prelim / Paper 2


www.gradtutors.com
137
10

(d) Using the ∆Hc value in Table 1, calculate the total theoretical thermal energy in kJ
released by the mass of methyl ethanoate combusted in this initial experiment.

………………………….. kJ
[2]

(e) In terms of the ease of lighting, how does methyl ethanoate compare to decyl ethanoate
(CH3COOC10H21)?

……………….………..……………………………………………………..………………………..

…………………………………………………………………………………………………………

[1]

[Total: 12]

2013 HCI C2 H1 Chemistry Prelim / Paper 2


www.gradtutors.com
138
11
Section B

Answer two questions from this section on separate answer paper.

5 (a) Chlorine consists of two isotopes, 35Cl and 37Cl, in the abundance ratio 3:1.

(i) Write the full electronic configuration of chlorine.

(ii) Define the term ‘isotope’.

(iii) State the number of protons, neutrons and electrons in a 37Cl atom.

(iv) Calculate the relative atomic mass of chlorine.

(v) Sketch the trend of the successive ionisation energies of the first 10 electrons of
a chlorine atom. Briefly explain the shape of your sketch.
[7]

(b) Chlorine forms compounds with almost all of the elements, exhibiting oxidation
numbers from –1 to +7.

(i) Explain in terms of oxidation numbers the reaction(s) undergone by chlorine in


the following equation.
Cl2 + 2NaOH  NaCl + NaOCl + H2O

(ii) Describe, in terms of orbital overlap, the bonding in a HCl molecule. Draw a
clearly labelled diagram to illustrate your answer.
[4]

(c) A solution of HCl has a pH of 1.3 while a solution of CH3CO2H, at the same
concentration, has a pH of 3.0.

(i) Calculate the concentration of H+ ions in both the acid solutions. Explain any
difference in your answers.

Both hydrochloric acid and CH3CO2H react with magnesium metal, giving off hydrogen
gas. For a fixed amount of magnesium, the rate equation for the reaction is as follows.
rate = k [H+(aq)]

When 20.0 cm3 of each of the acid solutions in part (c)(i) is separately reacted with an
excess of magnesium, the same volume of hydrogen is given off, but CH3CO2H
solution reacts more slowly.

(ii) Write an equation for the reaction between CH3CO2H and Mg.

(iii) Calculate the volume of hydrogen given off at room temperature and pressure.

(iv) Explain why the hydrogen gas is evolved more slowly from the CH3CO2H
solution but why, eventually, it produces the same volume of hydrogen gas.
[6]

(d) A solution containing CH3CO2H and its sodium salt acts as an acidic buffer. Buffers
are also present in human blood.
Explain what is meant by an acidic buffer. Use blood as an example to illustrate your
answer.
[3]
[Total: 20]

2013 HCI C2 H1 Chemistry Prelim / Paper 2


www.gradtutors.com
139
12
6 (a) The oxides of the Period 3 elements include the following: Na2O, Al2O3 and SO2.

(i) Draw dot–and–cross diagrams to illustrate the bonding in Na2O and SO2.

(ii) In terms of structure and bonding, explain the difference in the melting point of
Na2O and SO2.

(iii) Sketch a graph to show the variation in the melting point of the oxides in the third
period of the Periodic Table.

(iv) Describe and explain what you would see when a sample of Na2O(s) and
Al2O3(s) are separately added to a solution of Universal indicator. Write
equations for any reactions that occur.
[8]

(b) Al2O3 is used in the dehydration of alcohols to alkenes. Two isomeric alcohols, A and
B, are shown below.

CH3CH(OH)CH2CH3 (CH3)3COH
A B

(i) Draw the structural formula of one other alcohol isomeric with A and B.

(ii) Draw the structural formula of an alkene that is obtained by dehydrating A.

(iii) Suggest how alcohols, A and B, could be distinguished from each other. Give
the reagents, conditions and observations with each alcohol.
[4]

(c) Both alcohols and alkenes can be converted to halogenoalkanes.

(i) State the type of reaction undergone and name a reagent used to convert
CH3CH2CH2OH to CH3CH2CH2Cl .

(ii) Use values of bond energies from the Data Booklet to calculate the enthalpy
change for the reaction between propene and hydrogen chloride.

CH3CH=CH2 + HCl  CH3CHClCH3


[5]

(d) Halogenoalkanes such as CH3CHClCH3 undergo hydrolysis to form alcohols. The


relative rates of hydrolysis can be studied by dissolving the halogenoalkane in ethanol
and adding aqueous AgNO3.
With the aid of the Data Booklet, suggest an explanation for the following
observations.

Compound Observations after heating with AgNO3(aq)


CH3CHClCH3 ppt forms after 10 minutes
CH3CHBrCH3 ppt forms after 2 minutes
CH3CHICH3 ppt forms immediately

[3]
[Total: 20]

2013 HCI C2 H1 Chemistry Prelim / Paper 2


www.gradtutors.com
140
13
7 (a) A study on the kinetics of the reaction between hydrogen peroxide and iodide ions in
acidic solution was carried out.
H2O2 + 2I– + 2H+  2H2O + I2
In one experiment, both [H2O2] and [H+] were kept constant at 0.05 mol dm–3 and the
initial [I–] was 0.001 mol dm–3. It was found that the order with respect to [I–] is one.

(i) Sketch the shape of the graph of [I–] against time. Use your graph to show
clearly how you would determine the order with respect to [I–].

The following data was obtained after the experiment was repeated with different
[H2O2] and [H+].

Expt [H2O2] / mol dm–3 [H+] / mol dm–3 relative rate


1 0.05 0.05 1.0
2 0.07 0.05 1.4
3 0.09 0.07 1.8

(ii) Use the data to deduce the orders with respect to [H2O2] and [H+], explaining
your reasoning.

(iii) Write an expression for the rate equation and state the units of the rate constant.
[6]

(b) The decomposition of hydrogen peroxide is a first order reaction.


H2O2  H2O + ½O2 ∆HO = −98 kJ mol−1
The uncatalysed reaction has an activation energy of 79 kJ mol−1. Both platinum and
the enzyme catalase act as catalysts, and speed up the reaction.

(i) Use the data provided to construct a reaction pathway diagram for the
uncatalysed reaction.

(ii) On your sketch in part (b)(i), draw the energy profile of the catalysed reaction
and explain how the presence of a catalyst increases the rate of the reaction.

(iii) What effect will the presence of the catalyst have on the rate constant for this
reaction? Explain your answer.
[6]

(c) Three organic compounds, C, D and E each have the same empirical formula CH2O.
The numbers of carbon atoms in their molecules are shown in the table.

compound number of C atoms


C 1
D 2
E 3

C gives a silver mirror when treated with Tollens’ reagent. In D and in E, the carbon
atoms are bonded directly to one another. D and E each give a brisk effervescence
with Na2CO3(aq). When E is heated under reflux with acidified K2Cr2O7, the product, F,
gives an orange–yellow precipitate with 2,4–dinitrophenylhydrazine.

Identify and suggest structures for C, D, E and F and explain the reactions involved.
[8]
[Total: 20]

2013 HCI C2 H1 Chemistry Prelim / Paper 2


www.gradtutors.com
141
14

BLANK PAGE

2013 HCI C2 H1 Chemistry Prelim / Paper 2


www.gradtutors.com
142
1
2013 HCI H1 Chemistry Preliminary Examination Answer Scheme

Paper 1

1 B 6 B 11 A 16 C 21 C 26 D
2 C 7 A 12 C 17 B 22 C 27 B
3 D 8 A 13 D 18 C 23 B 28 A
4 D 9 C 14 B 19 A 24 D 29 B
5 B 10 A 15 A 20 A 25 D 30 C

A = 8; B = 8; C = 8; D = 6

Paper 2 (Section A)

1 a) i) Si or P

ii) 2Al(s) + 3Cl2(g)  Al2Cl6(s) or 2AlCl3(s)

(iii) argon has complete/ full outer shell of electrons or octet or IE too high

b)
NaCl(s) + aq  Na+(aq) + Cl-(aq)

AlCl3(s) + aq  Al3+(aq) or [Al(H2O)6]3+ + 3Cl-(aq)


[Al(H2O)6]3+ + H2O  [Al(H2O)5OH]2+ + H3O+

SiCl4(l) + 4H2O  SiO2.2H2O(s) + 4HCl(aq)

c)i)
nitrogen atom 1 lone pair

sulphur atom 2 lone pairs

ii) angle a or S because lone pair- lone pair repulsions are stronger than lone pair- bond pair
repulsions

Q2

b) K2Cr2O7(aq) + (acidified) /with H2SO4(aq)


heat under reflux

c) ethanal
reasoning: absorption at 1720-1740 indicate a carbonyl compound;
no broad peak at 25000-3200 suggest absence of –OH found in acid or alcohols.

2013 HCI C2 H1 Chemistry Prelim Answer Scheme


www.gradtutors.com
143
2
d) i) a proton/ H+ acceptor

ii)

iii) gas is CO2

-
(C6H7O6)2 Ca2+ or (C6H7O6)2Ca

e)
restricted rotation around C=C bond

the two –OH groups in E300 can only be on the same side of the C=C because the ring structure
will not allow them to be on opposites /cannot rotate.

f)

ii) –OCO- or -OOC-

iii) Concentrated sulfuric acid

3 a) i) rate of forward reaction is equal to rate of backward

ii)

Kc = [N2O4]/[NO2]2

-2
iii) Kc = (0.400/2) / {(3.2)/2 }2 =(0.200)/ (1.6)2 = 7.81 x 10 dm3 mol-1

b) concentration of N2O4 will decrease and conc. of NO2 will increase

2013 HCI C2 H1 Chemistry Prelim Answer Scheme


www.gradtutors.com
144
3
4. a) alkane → ester → alcohol

b (i) C3H6O2 + 7/2 O2 → 3CO2 + 3H2O or equation multiplied through by 2.

(ii) Enthalpy change when 1 mol of substance is formed from its elements.
1 mark for each of the points in bold
Mention of standard states or under standard conditions or 1 bar pressure.

(iii) ∆fH(C3H6O2) = 3∆cH(C) + 3 ∆cH(H2) – ∆cH(C3H6O2) =

= ((3×−393.5) + (3×−285.8) – (−1592.1)) kJ mol–1 = – 445.8 kJ mol–1

Correctly multiplying carbon and hydrogen values by 3 or a 3:3:1 ratio of


C:H2:C3H6O2 (1)

Correct signs, i.e. −∆cH(ester) + ∆cH(elements)


Correct final answer to 1 d.p. (1)
Allow ecf from an earlier penalised error if it has been worked through correctly.

c) Thermal energy added to water = 4.18 J K–1 g–1 × 10.0 K × 300 g = 12540 J (1)
Thermal energy added to copper = 0.384 J K–1 g–1 × 10.0 K × 250 g = 960 J (1)
Total energy = 13.5 kJ (3 s.f. required) (1)

Answer must be in kJ, not Joules, but no penalty for omitting to write kJ.

d)

Amount of ester = 0.980 g / 74.0 g mol–1 = 0.0132 mol (1)


Theoretical energy released = 0.0132 mol × 1592.1 kJ mol–1 = 21.1 kJ (1) 3 s.f.
Allow ecf with amount of ester.

e) The methyl ethanoate will be easier to light (more volatile)

2013 HCI C2 H1 Chemistry Prelim Answer Scheme


www.gradtutors.com
145
4

Paper 2 (Section B)

1 (a) (i) 1s2 2s2 2p6 3s2 3p5 [1]

(ii) Isotopes are atoms of the same element that has the same number of [1]
protons but different number of neutrons / or different mass number.

(iii) 17 protons, 20 neutrons and 17 electrons [1]

(iv) (1×37)+(3×35) [1]


Ar of Cl = = 35.5 (no units)
4

(v) I.E./ kJ mol1

No. of electron removed


1 2 3 4 5 6 7 8 9 10

Correct axes (units optional) and shape [1]

Successive I.E. increase as the remaining electrons are closer to the [1]
nucleus due to the decreasing number of electrons being attracted by the
same number of protons / or increasing effective nuclear charge.

There is a large increase from the 7th to 8th I.E. as the 8th electron is [1]
removed from the inner quantum shell which is much closer to and less
shielded from the nucleus.

(b) (i) Oxidation number increases from 0 in Cl2 to +1 in NaOCl [1]


and decreases from 0 to –1 in NaCl.

Cl2 undergoes disproportionation / both oxidation and reduction. [1]

(ii) [1]

The H–Cl bond consists of a  bond formed by head on overlap of s orbital [1]
of H with p orbital of Cl.

(c) (i) [H+] in HCl = 10–1.3 = 0.0501 mol dm–3 [1]


[H+] in CH3COOH = 10–3.0 = 0.001 mol dm–3

[H+] in CH3COOH is much lower as CH3COOH is a weak acid which


dissociates partially while HCl is a strong acid which dissociates completely.

Accept explanation in terms of strength of HCl [1]

(ii) 2 CH3COOH + Mg  (CH3COO)2Mg + H2


Accept ionic equation [1]
2013 HCI C2 H1 Chemistry Prelim Answer Scheme
www.gradtutors.com
146
5

(iii) H+ available = 0.0501 x 20/1000 = 1.002 x 10–3 mol

Volume of H2 = ½ x H+ x 24000 = ½ x 0.001002 x 24000 = 12.0 cm3 [1]

(iv) H2 is evolved more slowly due to lower [H+] in CH3COOH (since rate α [H+]). [1]

The equilibrium CH3COOH CH3COO– + H+ continually shifts to the [1]


+
right as H is used up.
The same volume of H2 is produced since the amount of H+ ions available
are the same in both acids.

(d) An acidic buffer consists of a weak acid and its conjugate base and is able to [1]
resist pH changes when small amount of acid or base is added to it.

The buffer in blood contains H2CO3 with HCO3–. Excess H+ and OH– ions are
removed as follows.

HCO3– + H+  CO2 + H2O [1]


or HCO3– + H3O+  CO2 + 2H2O
or HCO3– + H3O+  H2CO3 + H2O

H2CO3 + OH–  HCO3– + H2O [1]

2013 HCI C2 H1 Chemistry Prelim Answer Scheme


www.gradtutors.com
147
6

2 (a) (i) 2x[1]


2 Na+ [ O ] 2– O S O

(ii) Na2O has a giant ionic structure which consists of Na+ and O2– ions held by [1]
strong ionic bonds. Hence, Na2O has a high melting point.

SO2 has a simple covalent structure where SO2 molecules are held by weak [1]
pd–pd attractions. Hence, SO2 has a low melting point.

(iii) m.p/ K

Na2O MgO Al2O3 SiO2 P4O10 SO3

Correct axes (units optional) and shape [1]


Accept SO2 or SO3

(iv) Na2O(s) – solution turns blue as a strongly alkaline solution is formed. [1]

Na2O + H2O → 2NaOH(aq)


Accept ionic equation [1]

Al2O3(s) – solution remains purple as solid is insoluble in water. [1]

(b) (i) [1]

(ii) [1]

(iii) K2Cr2O7, dilute H2SO4, heat [1]


A – orange solution turns green; B – solution remains orange [1]
or
KMnO4, dilute H2SO4, heat
A – purple solution turns colourless; B – solution remains purple or
I2(aq), NaOH(aq), warm
A – yellow crystals/ ppt; B – no ppt

(c) (i) Type of reaction: (nucleophilic) substitution [1]


Reagents: PCl5 , (r.t.) / SOCl2, warm / conc. HCl, ZnCl2
[1]
(ii) CH3CH=CH2 + HCl  CH3CHClCH3

Hr = [BE(C=C) + BE(H–Cl)] – [BE(C–C) + BE(C–H) + BE(C–Cl)]


= (610 + 431) – (350 + 410 + 340) 2x[1]
= 1041 – 1100
= –59 kJ mol–1 (sign and units) [1]

2013 HCI C2 H1 Chemistry Prelim Answer Scheme


www.gradtutors.com
148
7

(d) BE(C–Cl) = 340 kJ mol–1


BE(C–Br) = 280 kJ mol–1
BE(C–I) = 240 kJ mol–1 [1]

The rate of hydrolysis increases as the C–X bond gets weaker. [1]

Hence, CH3CHICH3 hydrolyses most rapidly to give iodide ions which form a ppt [1]
with Ag+ immediately (while CH3CHClCH3 hydrolyses least rapidly hence ppt is
observed only after 10 minutes).
or
Rate of hydrolysis increases from CH3CHClCH3 to CH3CHBrCH3 to CH3CHICH3
and hence a ppt is observed immediately for CH3CHICH3.

2013 HCI C2 H1 Chemistry Prelim Answer Scheme


www.gradtutors.com
149
8

3 (a) (i) [I–]


0.001

0.0005

0.00025

time
t½ t½
Correct axes and shape [1]
Shows t½ is constant [1]

(ii) Compare Expt 1 and 2:


As [H2O2] increases by 0.07/0.05 = 1.4, so does rate
 order w.r.t. [H2O2] = 1
or
n
 0.07  1.4
    order w.r.t. [H2O2] = n = 1 [1]
 0.05  1.0

Compare Expt 1 and 3:


As [H2O2] increases by 0.09/0.05 = 1.8 and [H+] increases by 0.07/0.05 =
1.4, rate increases by 1.8 times.
 rate is independent of [H+] / order w.r.t. [H+] = 0
or
m
 0.07   0.09  1.8
    =1
 0.05   0.05  1.0
 order w.r.t. [H+] = m = 0 [1]

If both orders are correct but no working/explanation given – award [1]

(iii) Rate = k [H2O2] [I–] [1]


Units for k = mol–1 dm3 s–1 or mol–1 dm3 min–1 [1]
ecf from (a)(ii)
(b) (i)
Energy/ kJ mol–1
Ea=+79 kJ mol–1

catalysed reaction

HO = –98 kJ mol–1

Reaction Progress

Correct axes and shape [1]


Correct labels for H and Ea [1]

2013 HCI C2 H1 Chemistry Prelim Answer Scheme


www.gradtutors.com
150
9
(ii) Correct profile for catalysed reaction [1]

A catalyst provides a different reaction pathway with lower activation [1]


energy.
The proportion of molecules with energy greater or equal to activation
energy increases.
Number of effective collision increases and reaction rate increases.

(iii) The rate constant will increase. [1]

From the rate equation, if rate increases despite all concentrations [1]
remaining the same, then the value of k must have increased.

or

Using Arrhenius equation, k = A e−Ea/RT , if activation energy, Ea,


decreases, k will increase.

(c) 4x[1]

C gives a Ag mirror with Tollens’ reagent  C is an aldehyde [1]

D and E contains –COOH group which reacts with Na2CO3(aq) to give CO2 gas. [1]

E undergoes oxidation with acidified K2Cr2O7 to give F

F undergoes condensation with 2,4–DNPH to give an orange–yellow ppt


 F contains a ketone  E contains a 2o alcohol

F contains ketone + condensation [1]


E contains 2o alcohol + oxidation [1]
or award 1 mark for any 2 of the 4 points

2013 HCI C2 H1 Chemistry Prelim Answer Scheme


www.gradtutors.com
151

INNO
OVA JUNIOR COLLLEGE
JC 2 PRELIMINARY E
EXAMINA
ATION 2
in prep
paration for General Ce
ertificate of Education Advanced LLevel
Higher 1

CANDIDAT
TE
NAME

CLASS DEX NUMB


IND BER

CHEMIISTRY 88
872/01
Paper 1 M
Multiple Choiice 118 Septem
mber 2013
50 minutes
Additional Materials: Data Book klet
Multiple Choice Answ
wer Sheet

READ THE
ESE INSTR
RUCTIONS FIRST

Write yourr name and class on all the work yyou hand in.
Write in so
oft pencil.
Do not usee staples, paper clips, highlighterss, glue or co
orrection fluid.

There are thirty quesstions on th


his paper. A
Answer all questions. For each qquestion the
ere are fourr
possible answers A, B,
B C and D..
Choose thhe one you u consider correct and d record yoour choice in soft peencil on the separate
Answer Sh heet.

ns on the Answer
Read the instruction A eet very carefully.
She

Each corre
ect answer will
w score one mark. A mark will not
n be deduc
cted for a w
wrong answe
er.
hould be done in this bo
Any rough working sh ooklet.

This document
d cconsists of 12
1 printed pages.
p

PRELIM 2 
Innova Junio
INNOVA or College 8872/01/2
2012 [T
Turn over

www.gradtutors.com
152
2

Section A

For each question there are four possible answers, A, B, C, and D. Choose the one you consider
to be correct.
12 13
1 A giant molecule contains a large amount of carbon, mainly of isotopes C and C. It was
found that the relative atomic mass of carbon in the molecule is 12.20.

What is the ratio of 12C and 13C?

A 3:1 B 4:1 C 3:4 D 1:4

2 20.0 cm3 of 0.02 mol dm–3 aqueous sodium bromate(V), NaBrO3, was found to react
completely with 80.0 cm3 of 0.01 mol dm–3 hydroxylamine, NH2OH. The half equation for the
reduction of bromate (V) ions is given as shown.

BrO3–(aq) + 6H+(aq) +6e Br–(aq) + 3H2O(l)

Which of the following could be the nitrogen containing product in this reaction?

A N2O
B NO2
C NO
D NO3–

3 The successive ionisation energies (IE) of two elements, A and B, are given below.

IE/ kJ mol–1 1st 2nd 3rd 4th 5th 6th 7th 8th
A 1090 2350 4610 6220 37800 47000 - -
B 1251 2298 3822 5158 6542 9362 11018 33604

What is the likely formula of the compound that is formed when A reacts with B?

A AB B A2B3 C AB4 D A4B

4 Which electronic configuration represents an atom of an element that forms a simple ion with
a charge of –3?

A 1s2 2s2 2p6 3s2 3p1


B 1s2 2s2 2p6 3s2 3p3
C 1s2 2s2 2p6 3s2 3p6 3d1 4s2
D 1s2 2s2 2p6 3s2 3p6 3d3 4s2

PRELIM 2  INNOVA 8872/01/2013 [Turn over

www.gradtutors.com
153
3

5 Which of the following species has a square planar structure?

A BrF4–
B BF4–
C CH4
D C2H4

6 Which of the following will not form a hydrogen bond with another of its own molecule?

A CH3OH
B CH3CHO
C CH3NH2
D CH3COOH

7 The C2H2 molecule is linear.

What can be deduced from this about the numbers of σ and π bonds present in the
molecule?

 
A 2 2
B 2 3
C 3 1
D 3 2

8 The diagram below shows part of the structure of ordinary ice.

Which of the following statements is not true?

A The bond angle in ice is 104.5º.


B Ice has a cage-like “open” structure.
C The density of water is higher than that of ice.
D Both ice and water have covalent bonding as well as hydrogen bonding.

PRELIM 2  INNOVA 8872/01/2013 [Turn over

www.gradtutors.com
154
4

9 The equilibrium constant for reaction I below is K. What is the equilibrium constant for
reaction II?

1
reaction I SO2(g) + O2(g) SO3(g)
2
reaction II
2SO3(g) 2SO2(g) + O2(g)
A 2K C K–2
B 2K–1 D K2

10 A 10 cm3 sample of 0.010 mol dm–3 HCl is diluted by adding distilled water at constant
temperature.

Which one of the following items correctly shows the effect of the dilution on the
concentrations of H+ and OH− ions in the solution?

[H+] [OH−]
A decrease decrease
B decrease increase
C increase decrease
D increase increase

11 The table gives the concentrations and pH values of the aqueous solutions of two
compounds, C and D. Either compound could be an acid or a base.

C D
-3
Concentration 2 mol dm 2 mol dm-3
pH 6 9

Student E concluded that C is a strong acid.


Student F concluded that the extent of dissociation is lower in C(aq) than in D(aq).
Which of the students are correct?

A E only
B F only
C Both E and F
D Neither E nor F

PRELIM 2  INNOVA 8872/01/2013 [Turn over

www.gradtutors.com
155
5

12 The following data refers to the reaction

2NO(g) + O2(g) 2NO2(g)

[NO]/ mol dm-3 [O2]/ mol dm-3 relative initial rate


1.0 0.5 1.0
2.0 0.5 4.0
2.0 1.0 8.0

What is the rate equation for the reaction?

A rate = k[O2]
B rate = k[NO2]2
C rate = k[NO][O2]
D rate = k[NO]2[O2]

13 The reaction

2G + H J

is first order with respect to G and second order with respect to H. What is the unit of the rate
constant?

A s–1
B mol dm-3 s–1
C mol-1 dm3 s–1
D mol-2 dm6 s–1

14 Some enthalpy changes of combustion are given below.

∆Hc / kJ mol-1
CO(g) + ½ O2(g) CO2(g) –283
H2(g) + ½ O2(g) H2O(l) –286
CH3OH(l) + 1½ O2(g) CO2 (g) + 2H2O(l) –715

What is the enthalpy change of the following reaction?

CO(g) + 2H2(g) CH3OH(l)

A –146 kJ mol–1
B –140 kJ mol–1
C +140 kJ mol–1
D +146 kJ mol–1

PRELIM 2  INNOVA 8872/01/2013 [Turn over

www.gradtutors.com
156
6

15 The properties of the oxides of four elements K, L, M and N in the Third Period in the Periodic
Table are given below.
 The oxide of K is insoluble in water and in dilute acid but is soluble in hot and
concentrated sodium hydroxide.
 The oxide of L is amphoteric.
 The oxide of M reacts with dilute sodium hydroxide at room temperature.
 The oxide of N dissolves in water to form a strong alkaline solution.

Which of the following is correct in order of increasing proton number?

A N, L, K, M
B K, L, M, N
C N, K, L, M
D N, M, K, L

16 The size of Na+, Mg2+ and Al3+ is in the order: Na+ > Mg2+ > Al3+. Which of the following best
explain this trend?

A The number of electrons decreases while the number of protons increases.


B The number of electrons are the same but the number of protons increases.
C The number of electrons and protons decreases.
D The number of electrons and protons increases.

17 Element O is heated in chlorine. The product is then added to water. The resulting solution is
found to be neutral. What could O be?

A sodium
B aluminium
C phosphorus
D chlorine

18 Which pair of reactions could have the same common intermediate?

P CH3CH2CH3 intermediate (CH3)2CHCN


Q CH3CH(OH)CH3 intermediate (CH3)2C(OH)CN
R CH3CH=CH2 intermediate CH3CH(OH)CH3
S CH3CO2CH2CH2CH3 intermediate CH3CH2CH2Br

A P and Q C Q and S
B P and R D R and S

PRELIM 2  INNOVA 8872/01/2013 [Turn over

www.gradtutors.com
157
7

19 High energy radiation in the stratosphere produces free-radicals from chlorofluoroalkanes,


commonly known as CFCs.

Which free radical is most likely to result from the irradiation of CHFClCF2Cl?

A CHFClCFCl

B CHClCF2Cl

C CHFCF2Cl

D CFClCF2Cl

20 Santonin is a drug that was once widely used to expel parasitic worms from the body.

CH3

O CH3

CH3 O

O
santonin

When santonin is first treated with warm dilute H2SO4, and then the product of this reaction is
treated with cold acidified KMnO4, a final product T is obtained.

How many atoms of hydrogen in each molecule of product T can be displaced with sodium
metal?

A 2 C 5
B 4 D 6

PRELIM 2  INNOVA 8872/01/2013 [Turn over

www.gradtutors.com
158
8

21 Hydrocortisone is a steroid hormone produced by the adrenal gland and is released in


response to stress. It is commonly used as an active ingredient in anti-inflammatory creams.

Which of the following statements about hydrocortisone is incorrect?


A It does not show geometric isomerism.
B It turns hot acidified K2Cr2O7 from orange to green.
C When treated with an excess of hot concentrated acidified KMnO4, it forms a compound
containing five carbonyl groups.
D When treated with NaBH4 in the presence of methanol, it forms a compound containing
five hydroxy groups.

22 The reaction conditions for four different transformations are given below.

Which transformation has a set of conditions that is not correct?

UV light
A CH3CH3 + Cl2 (CCl4) CH3CH2Cl + HCl

KMnO4/H+
B CH2=CHCH=CH2 4CO2 + 3H2O
Heat

anhydrous
AlCl3
+ Cl2(aq) + HCl

heat
+ 2OH– + 2Cl– + H2O

PRELIM 2  INNOVA 8872/01/2013 [Turn over

www.gradtutors.com
159
9

23 An alcohol with molecular formula CnH2n+1OH does not react with MnO4– / H+.

What is the least number of carbon atoms such an alcohol could possess?

A 4
B 5
C 6
D 7

24 An organic compound U has the following properties.

 changes the colour of acidified sodium dichromate(VI) from orange to green,


 has no effect on Fehling’s reagent,
 gives a positive tri-iodomethane test

What could U be?

A CH3COCH2CHO C CH3COCH2CH2OH
B CH3COCH2COCH3 D HOCH2CH2CH2CH2OH

25 The following reaction scheme shows the synthesis of an amino acid, alanine.

O OH NH NH3+
NH3
C CH3 C H C CH3 C H
CH3 H I II CH3 H
I NH2 II COOH
alanine

Which of the following shows the correct types of reaction for steps I and II?

I II
A substitution elimination
B substitution condensation
C addition elimination
D addition addition

PRELIM 2  INNOVA 8872/01/2013 [Turn over

www.gradtutors.com
160
10

Section B

For each of the questions in this section, one or more of the three numbered statements 1 to 3 may
be correct.

Decide whether each of the statements is or is not correct (you may find it helpful to put a tick
against the statements that you consider to be correct.)

The responses A to D should be selected on the basis of

A B C D
1, 2 and 3 1 and 2 2 and 3 1 only
are only are only are is
correct correct correct correct

No other combination of statements is used as a correct response.

26 Which of the following ions contains half-filled d-orbitals?

1 Mn2+
2 Cu+
3 Fe2+

27 Which of the following processes is endothermic?

1 electrolysis of water
2 freezing of water
3 condensation of steam

28 Which of the following might alter the rate constant of a reaction?

1 temperature
2 concentration
3 pressure

PRELIM 2  INNOVA 8872/01/2013 [Turn over

www.gradtutors.com
161
11

29 Aldosterone is a steroid hormone that increases reabsorption of ions and water in the kidney.
It has the structure shown below.

Aldosterone was reacted in separate experiments with:

(i) 2,4-dinitrophenylhydrazine
(ii) Ag(NH3)2+(aq)
(iii) PCl5(s)

Which statements about these reactions are correct?

1 One mole of aldosterone could react with three moles of 2,4-dinitrophenylhydrazine.


2 One mole of aldosterone could form two moles of Ag.
3 One mole of aldosterone could react with 2 moles of PCl5(s).

PRELIM 2  INNOVA 8872/01/2013 [Turn over

www.gradtutors.com
162
12

The responses A to D should be selected on the basis of

A B C D
1, 2 and 3 1 and 2 2 and 3 1 only
are only are only are is
correct correct correct correct

No other combination of statements is used as a correct response.

30 Which of the following reactions would produce 1,4-dicarboxylic acid?

reactant reagents and conditions

1 hot acidified potassium manganate(VII)

2 hot acidified potassium dichromate(VI)

3 alkaline aqueous iodine and heat followed by acidification

PRELIM 2  INNOVA 8872/01/2013 [Turn over

www.gradtutors.com
163

INNOVA JUNIOR COLLEGE


JC2 PRELIMINARY EXAMINATION 2
in preparation for General Certificate of Education Advanced Level
Higher 1

CANDIDATE
NAME

CLASS INDEX NUMBER

CHEMISTRY 8872/02
18 September 2013
Section A: Structured Questions
Section B: Free Response Questions 2 hours

Candidates answer Section A on the Question Paper


Additional Materials: Writing Papers
Data Booklet
Cover Page

READ THESE INSTRUCTIONS FIRST


For Examiner’s Use
Write your index number, name and civics group.
Write in dark blue or black pen. Section A
You may use pencil for any diagrams, graphs or rough working.
Do not use staples, paper clips, highlighters, glue or correction fluid. 1 8
Answer all questions in the space provided. 2
A Data Booklet is provided. 9
3
8
You are advised to show all working in calculations.
You are reminded of the need for good English and 4
15
clear presentation in your answers.
Significant
You are reminded of the need for good handwriting.
figures
Your final answers should be in 3 significant figures.
Handwriting
You may use a calculator.
Total
The number of marks is given in brackets [ ] at the end of each 40
question or part question.

At the end of the examination, fasten all your work


securely together.

This document consists of 17 printed pages and 1 blank page.

Innova Junior College [Turn over

PRELIM 2  INNOVA 8872/02/2013 [Turn over


www.gradtutors.com
164
2
For
Section A Examiner’s
Use
Answer ALL questions on the spaces provided.

1 The kinetics of the reaction between acetone and potassium cyanide was investigated.

An experiment was performed in which 0.200 mol dm3 of acetone, CH3COCH3 was
reacted with an excess of acid and sodium cyanide, NaCN. A graph of concentration of
CH3COCH3 against time was plotted.

CH3COCH3 + H+ + NaCN CH3C(OH)(CN)CH3 + Na+

[CH3COCH3] / moldm-3

Time / seconds

(i) Using your graph, deduce the order of reaction with respect to acetone.

(ii) Why was the acid and sodium cyanide used in excess?

.....................................................................................................................................

…………………………………………………………………………………………………

…………………………………………………………………………………………………

PRELIM 2  INNOVA 8872/02/2013 [Turn over


www.gradtutors.com
165
3
For
(iii) The order of reaction with respect to the acid was zero. Examiner’s
Use
Sketch, on the axes provided, the shape of the graphs relating to the information
provided.

[acid] / mol dm3 Rate / mol dm3 s1

time / s [acid] / mol dm3

Graph 1 Graph 2

(iv) Given that the order of reaction with respect to sodium cyanide is one, write the rate
law for this reaction.

.....................................................................................................................................

(v) Suggest with a reason whether pentan-2-one reacts more or less rapidly with NaCN
as compared to acetone.

.....................................................................................................................................

…………………………………………………………………………………………………

…………………………………………………………………………………………………

…………………………………………………………………………………………………

[Total: 8]

PRELIM 2  INNOVA 8872/02/2013 [Turn over


www.gradtutors.com
166
4
For
2 Benzoic acid is a colourless crystalline solid that has antifungal abilities and is an Examiner’s
important precursor for the synthesis of many organic compounds. Use

(a) Explain, with the aid of a diagram, why benzoic acid has an apparent molecular
mass of 244 in an organic solvent such as benzene.

[2]

(b) Both benzoic acid and methylbenzene undergo aromatic substitution to produce the
chloro-compounds shown:

(i) Suggest why a harsher condition is required for the substitution of benzoic
acid.

..............................................................................................................................

……………………………………………………………………………………………

……………………………………………………………………………………………

……………………………………………………………………………………………

PRELIM 2  INNOVA 8872/02/2013 [Turn over


www.gradtutors.com
167
5
For
(ii) How would you expect the acidity of 3-chlorobenzoic acid to compare with that Examiner’s
of benzoic acid? Use

..............................................................................................................................

……………………………………………………………………………………………

……………………………………………………………………………………………

………...………………………………………………………………………………[3]

(c) Benzoic acid is sparingly soluble in water. However, it dissolves readily to give a
colourless solution upon the addition of a reagent, N. Identify reagent N.

…………………………………………………………………………………………….…[1]

(d) Benzene-1,2-dicarboxylic acid can be produced via the following synthetic pathway.

(i) What type of reaction is step I?

..............................................................................................................................

(ii) Identify reagent P.

..............................................................................................................................
(iii) Suggest a structure for compound Q and draw its structural formula in the box
provided.
[3]

[Total: 9]

PRELIM 2  INNOVA 8872/02/2013 [Turn over


www.gradtutors.com
168
6
For
3 Phosphorus(V) chloride, PCl5, is a white solid which sublimes at 160°C. Examiner’s
Use
When gaseous phosphorus(V) chloride is heated in a closed container, the following
equilibrium is established.

PCl5(g) PCl3(g) + Cl2(g)

(a) Sketch a graph to show how the rates of the forward and backward reactions
change from the time phosphorus pentachloride is heated to the time the reaction
reaches dynamic equilibrium. Label your two lines clearly.

[2]

(b) In an experiment, 1.00 mol of PCl5 vapour was heated in a closed 5.00 dm3 flask at
500K until equilibrium had been established. The amount of chlorine collected was
0.508 mol.

Write an expression for the equilibrium constant and calculate its value.

[3]

PRELIM 2  INNOVA 8872/02/2013 [Turn over


www.gradtutors.com
169
7
For
(c) The graph below shows how the percentage yield of chlorine from the above Examiner’s
equilibrium varies as the reaction progresses at two different pressures. Use

% yield of chlorine

2 atm

5 atm

progress of reaction

(i) Account for the differences in the two graphs at different pressures.

..............................................................................................................................

……………………………………………………………………………………………

……………………………………………………………………………………………

………...…………………………………………………………………………………

..............................................................................................................................

……………………………………………………………………………………………

(ii) On the same axes, sketch a graph to show how the percentage yield of
chlorine will vary at 5 atm in the presence of a catalyst.
[3]

[Total: 8]

PRELIM 2  INNOVA 8872/02/2013 [Turn over


www.gradtutors.com
170
8
For
4 (a) During the last Olympic Games, a torch relay around several countries took place Examiner’s
before the torch was used to ignite the Olympic cauldron to mark the start of the Use
Game. The fuel used to light the Olympic flame is propane. Propane is also used in
blow torches, portable stoves and in outdoor heaters.

A propane portable stove was used during a barbecue to boil water for making of
soup. It took 10 minutes to bring the water to boil.

Metal container

Water

Portable stove

Propane

The following information was known.

Mass of metal container / g 850


Mass of metal container containing water / g 1600
Initial temperature of water / oC 32
o
Final temperature of water / C 100
−1 −1
Specific heat capacity of water / J g K 4.18

Besides the above information, it was also known that the theoretical standard
enthalpy change of combustion of propane is −2220 kJ mol−1.

(i) Define the standard enthalpy change of combustion of propane.

.............................................................................................................................

…………………………………………………………………………………………...

…………………………………………………………………………………………..

(ii) Calculate the mass of propane needed to raise the temperature of the water
during the barbecue.

PRELIM 2  INNOVA 8872/02/2013 [Turn over


www.gradtutors.com
171
9
For
Another value of the standard enthalpy change of combustion of propane, Hc, can Examiner’s
be calculated using bond energies and the energy cycle below. Use

Hc
C3H8(g) + 5O2(g) 3CO2(g) + 4H2O(g)

+6460 kJ mol1 H1

3 C(g) + 8 H(g) + 10 O(g)

(iii) Use suitable bond energies given in the Data Booklet to calculate H1.

(iv) Hence, calculate Hc.

(v) Suggest why the value of Hc you calculated differs from the theoretical value.

.............................................................................................................................

…………………………………………………………………………………………...

………………………………………………………………………………………...[7]

PRELIM 2  INNOVA 8872/02/2013 [Turn over


www.gradtutors.com
172
10
For
(b) Most outdoor heaters are powered by the combustion of propane gas stored in small Examiner’s
cylinders. At room temperature and pressure, a typical outdoor heater designed to Use
produce 15 kW of energy runs from a cylinder containing 13 kg of propane.

As pure propane gas is odourless, small amounts of another compound are usually
added so that gas leaks from propane cylinders can be detected. An example of
such an odorant is ethanethiol, C2H5SH which has an odour that resembles that of
onion. Ethanethiol is chosen since the human nose can detect its presence at levels
of only about 0.02 moles of it per billion (109) moles of propane.

(i) Calculate the total amount of heat energy released by the complete
combustion of all the propane in a cylinder used in an outdoor heater.

(ii) Using your answer to (b)(i), calculate the time needed to empty one cylinder of
propane to produce 15 kW of energy. You are given that 15kW is equivalent to
15 kJ s1.

(iii) Hence, calculate the rate, in cm3 s1, at which propane must leave the cylinder
in order to produce 15 kW of energy.

PRELIM 2  INNOVA 8872/02/2013 [Turn over


www.gradtutors.com
173
11
For
(iv) Calculate the volume of carbon dioxide evolved when 13 kg of propane was Examiner’s
completely burnt in an outdoor heater. Use

(v) Calculate the minimum mass of ethanethiol which must be added to 13 kg of


propane so that it can be detected by the human nose in the event of a
propane gas leak.

[5]

(c) Besides being used as fuel, propane can also be used to synthesise other organic
compounds.

Outline the synthesis of propan-1-ol from propane. State clearly the reagents and
conditions used and draw any intermediate product(s) formed.

[3]

[Total: 15]

PRELIM 2  INNOVA 8872/02/2013 [Turn over


www.gradtutors.com
174

12

Section B

Answer two of the three questions in this section on separate answer paper.

5 (a) Ethanal is one of the most important aldehydes occurring widely in nature and is
also the cause of hangovers from alcohol consumption. It is produced on a large
scale industrially via the Wacker Process as shown.

2 CH2=CH2 + O2 2 CH3CHO

(i) State the hybridisation of each carbon atom in ethene and draw a diagram to
show the hybridised orbitals around one of these carbon atoms.

(ii) Describe, in terms of orbital overlap, the bonding of the two carbon atoms of
the C=C bond in ethene.
[4]

(b) The Wacker reaction is accelerated by palladium catalysts.

(i) Define catalyst.

(ii) Explain with the aid of a diagram that shows the distribution of molecular
energies, how the addition of a catalyst accelerates the reaction.

(iii) In a college laboratory, ethanal can be synthesised from ethanol. Briefly


outline how you would carry out this reaction.

(iv) Explain why ethanal can be prepared in the way you described in (b)(iii), in
terms of the volatility of ethanal and ethanol.
[8]

PRELIM 2  INNOVA 8872/02/2013 [Turn over


www.gradtutors.com
175

13

(c) (i) Another common carbonyl compound used in the industry is phenylacetone,
which is an intermediate used to produce pesticides and anticoagulants.

phenylacetone

Compound C can be synthesised from phenylacetone.

O
step I O
H+
C Intermediate A
C CH3 C
C O-
H H
H H
step II
H CH3
H
C CH3COOH
C
C O O Intermediate B
H
H
Compound C

Complete the synthetic route by giving the structures of intermediates A and


B as well as the reagents and conditions for steps I and II.

(ii) Suggest the structures of the product and type of reaction undergone when
phenylacetone reacts with the following reagents.

 HCN, trace OH–


 hydrazine, H2N-NH2
[8]

[Total: 20]

PRELIM 2  INNOVA 8872/02/2013 [Turn over


www.gradtutors.com
176

14

6 (a) The elements of the third period of the Periodic Table, sodium to sulfur, all form
chlorides by direct combination.

(i) When dry chlorine is passed over heated aluminium powder in a long
hard-glass tube, a vapour is produced which condenses to a solid in the
cooler parts of the tube. At low temperatures, the vapour has a Mr of 267.

State one observation you would make during this reaction.

(ii) Suggest the molecular formula of the vapour, and draw a dot-and-cross
diagram to illustrate its bonding.

(iii) The solid reacts with water in two separate ways.

 When a few drops of water are added to the solid, steamy white fumes
are evolved and a white solid remains, which is insoluble in water.
 When a large amount of water is added to the solid, a clear, weakly
acidic solution results.

Write equations, including state symbols, for these two reactions.


[7]

(b) Lattice energy can be used as a measure of the energetic stability of ionic
compounds.

(i) Define the lattice energy of silver fluoride.

The table below shows numerical values of lattice energies for silver fluoride and
silver iodide. These have been determined from experimental data or theoretically
calculated.

Experimental value Theoretical value


Compound
/ kJ mol–1 / kJ mol–1
AgF –967 –953
AgI –889 –808

(ii) By quoting appropriate data from the Data Booklet, explain why the lattice
energy of silver iodide is less exothermic than silver fluoride.

(iii) Silver fluoride and silver iodide have the same crystal structure. There is close
agreement between the experimental and theoretical values of lattice energy
for AgF but not for AgI. Suggest a reason for this.

[4]

PRELIM 2  INNOVA 8872/02/2013 [Turn over


www.gradtutors.com
177

15

(c) Compound C has the empirical formula CH2O and Mr of 90. There is no reaction
when C is treated with NaHCO3. When 0.600 g of C is reacted with an excess of Na,
160 cm3 of H2, measured at room temperature and pressure, is produced. Treatment
of C with 2,4-dinitrophenylhydrazine reagent produces an orange solid. When C is
warmed with Fehling’s reagent, a brick red precipitate is formed.

The structure of C has the following feature:


 No carbon atom has more than one oxygen atom joined to it.

Suggest a structure for C, explaining your reasoning.


[7]

(d) Compound C can be both oxidised and reduced.

(i) Give the structural formula of the compound formed when C is reacted with
NaBH4 under suitable conditions.

(ii) Give the structural formula of the compound formed when C is heated under
reflux with acidified K2Cr2O7.
[2]

[Total: 20]

PRELIM 2  INNOVA 8872/02/2013 [Turn over


www.gradtutors.com
178

16

7 (a) When sodium is burned in air, a mixture of sodium oxide, Na2O, and sodium peroxide
Na2O2, is formed. The mixture reacts with water according to the following equations.

Na2O + H2O 2NaOH


Na2O2 + 2H2O 2NaOH + H2O2

The following information will allow you to calculate the relative amounts of the two
oxides produced when sodium is burned.

 The mixture obtained by burning a sample of sodium was dissolved in distilled


water and made up to 100 cm3 to give solution D.

 A 25.0 cm3 portion of solution D was titrated with 0.100 mol dm–3 HCl.
22.5 cm3 of acid was required to reach the end-point.

 The H2O2 content of solution D was found by titration of another 25.0 cm3 portion
with 0.0200 mol dm–3 KMnO4.

The following reaction occurs.

2MnO4– + 5H2O2 + 6H+ 2Mn2+ + 5O2 8H2O

10.0 cm3 of KMnO4 solution was required to reach the end-point.

(i) Using the results of the HCl titration calculate the total amount in moles of NaOH
in 100 cm3 of solution D.

(ii) Using the results of the KMnO4 titration, calculate the amount in moles of H2O2 in
100 cm3 of solution D.

(iii) Hence calculate the amount in moles of Na2O2 formed during the burning of the
sodium sample.

(iv) Using your result in (a)(iii), calculate the amount in moles of Na2O formed during
the burning of the sodium sample.
[6]

(b) A buffer solution can be prepared by mixing 500 cm3 of 4 mol dm–3 of CH3COOH in
500 cm3 of 0.5 mol dm–3 of NaOH.

(i) Calculate the pH of NaOH before mixing is carried out.

(ii) By using suitable equations, describe how the above buffer solution works.
[4]

PRELIM 2  INNOVA 8872/02/2013 [Turn over


www.gradtutors.com
179

17

(c) The graph below shows the first ionisation energy of eight elements with consecutive
proton number.

1st Ionisation Energy kJ mol-1


L

J
E
G
I
F

Atomic
number

(i) State the valence electronic configuration of element F.

(ii) Give the formula of the oxide that F is likely to form.

(iii) Hence or otherwise, explain the drop in first ionisation energy from E to F.
[4]

(d) (i) Sketch a graph of the melting point of oxides of elements from sodium to
sulfur.

(ii) Explain as fully as you can why the melting point varies in the way shown.
[6]

[Total: 20]

PRELIM 2  INNOVA 8872/02/2013 [Turn over


www.gradtutors.com
180

18

BLANK PAGE

PRELIM 2  INNOVA 8872/02/2013 [Turn over


www.gradtutors.com
181

INNO
OVA JUNIOR COLLLEGE
JC 2 PRELIMINARY E
EXAMINA
ATION 2
in prep
paration for General Ce
ertificate of Education Advanced LLevel
Higher 1

CANDIDAT
TE
NAME

CLASS DEX NUMB


IND BER

CHEMIISTRY 88
872/01
Paper 1 M
Multiple Choiice 118 Septem
mber 2013
50 minutes
Additional Materials: Data Book klet
Multiple Choice Answ
wer Sheet

READ THE
ESE INSTR
RUCTIONS FIRST

Write yourr name and class on all the work yyou hand in.
Write in so
oft pencil.
Do not usee staples, paper clips, highlighterss, glue or co
orrection fluid.

There are thirty quesstions on th


his paper. A
Answer all questions. For each qquestion the
ere are fourr
possible answers A, B,
B C and D..
Choose thhe one you u consider correct and d record yoour choice in soft peencil on the separate
Answer Sh heet.

ns on the Answer
Read the instruction A eet very carefully.
She

Each corre
ect answer will
w score one mark. A mark will not
n be deduc
cted for a w
wrong answe
er.
hould be done in this bo
Any rough working sh ooklet.

This document
d cconsists of 12
1 printed pages.
p

Innova Junio
or College [T
Turn over

PRELIM 2  INNOVA 8872/01/2


2012

www.gradtutors.com
182
2

Section A

For each question there are four possible answers, A, B, C, and D. Choose the one you consider
to be correct.
12 13
1 A giant molecule contains a large amount of carbon, mainly of isotopes C and C. It was
found that the relative atomic mass of carbon in the molecule is 12.20.

What is the ratio of 12C and 13C?

A 3:1 B 4:1 C 3:4 D 1:4

2 20.0 cm3 of 0.02 mol dm–3 aqueous sodium bromate(V), NaBrO3, was found to react
completely with 80.0 cm3 of 0.01 mol dm–3 hydroxylamine, NH2OH. The half equation for the
reduction of bromate (V) ions is given as shown.

BrO3–(aq) + 6H+(aq) +6e Br–(aq) + 3H2O(l)

Which of the following could be the nitrogen containing product in this reaction?

A N2O
B NO2
C NO
D NO3–

3 The successive ionisation energies (IE) of two elements, A and B, are given below.

IE/ kJ mol–1 1st 2nd 3rd 4th 5th 6th 7th 8th
A 1090 2350 4610 6220 37800 47000 - -
B 1251 2298 3822 5158 6542 9362 11018 33604

What is the likely formula of the compound that is formed when A reacts with B?

A AB B A2B3 C AB4 D A4B

4 Which electronic configuration represents an atom of an element that forms a simple ion with
a charge of –3?

A 1s2 2s2 2p6 3s2 3p1


B 1s2 2s2 2p6 3s2 3p3
C 1s2 2s2 2p6 3s2 3p6 3d1 4s2
D 1s2 2s2 2p6 3s2 3p6 3d3 4s2

PRELIM 2  INNOVA 8872/01/2013 [Turn over

www.gradtutors.com
183
3

5 Which of the following species has a square planar structure?

A BrF4–
B BF4–
C CH4
D C2H4

6 Which of the following will not form a hydrogen bond with another of its own molecule?

A CH3OH
B CH3CHO
C CH3NH2
D CH3COOH

7 The C2H2 molecule is linear.

What can be deduced from this about the numbers of σ and π bonds present in the
molecule?

 
A 2 2
B 2 3
C 3 1
D 3 2

8 The diagram below shows part of the structure of ordinary ice.

Which of the following statements is not true?

A The bond angle in ice is 104.5º.


B Ice has a cage-like “open” structure.
C The density of water is higher than that of ice.
D Both ice and water have covalent bonding as well as hydrogen bonding.

PRELIM 2  INNOVA 8872/01/2013 [Turn over

www.gradtutors.com
184
4

9 The equilibrium constant for reaction I below is K. What is the equilibrium constant for
reaction II?
1
reaction I SO2(g) + O2(g) SO3(g)
2
reaction II
2SO3(g) 2SO2(g) + O2(g)
A 2K C K–2
B 2K–1 D K2

10 A 10 cm3 sample of 0.010 mol dm–3 HCl is diluted by adding distilled water at constant
temperature.

Which one of the following items correctly shows the effect of the dilution on the
concentrations of H+ and OH− ions in the solution?

[H+] [OH−]
A decrease decrease
B decrease increase
C increase decrease
D increase increase

11 The table gives the concentrations and pH values of the aqueous solutions of two
compounds, C and D. Either compound could be an acid or a base.

C D
-3
Concentration 2 mol dm 2 mol dm-3
pH 6 9

Student E concluded that C is a strong acid.


Student F concluded that the extent of dissociation is lower in C(aq) than in D(aq).
Which of the students are correct?

A E only
B F only
C Both E and F
D Neither E nor F

PRELIM 2  INNOVA 8872/01/2013 [Turn over

www.gradtutors.com
185
5

12 The following data refers to the reaction

2NO(g) + O2(g) 2NO2(g)

[NO]/ mol dm-3 [O2]/ mol dm-3 relative initial rate


1.0 0.5 1.0
2.0 0.5 4.0
2.0 1.0 8.0

What is the rate equation for the reaction?

A rate = k[O2]
B rate = k[NO2]2
C rate = k[NO][O2]
D rate = k[NO]2[O2]

13 The reaction

2G + H J

is first order with respect to G and second order with respect to H. What is the unit of the rate
constant?
A s–1
B mol dm-3 s–1
C mol-1 dm3 s–1
D mol-2 dm6 s–1

14 Some enthalpy changes of combustion are given below.

∆Hc / kJ mol-1
CO(g) + ½ O2(g) CO2(g) –283
H2(g) + ½ O2(g) H2O(l) –286
CH3OH(l) + 1½ O2(g) CO2 (g) + 2H2O(l) –715

What is the enthalpy change of the following reaction?

CO(g) + 2H2(g) CH3OH(l)

A –146 kJ mol–1
B –140 kJ mol–1
C +140 kJ mol–1
D +146 kJ mol–1

PRELIM 2  INNOVA 8872/01/2013 [Turn over

www.gradtutors.com
186
6

15 The properties of the oxides of four elements K, L, M and N in the Third Period in the Periodic
Table are given below.
 The oxide of K is insoluble in water and in dilute acid but is soluble in hot and
concentrated sodium hydroxide.
 The oxide of L is amphoteric.
 The oxide of M reacts with dilute sodium hydroxide at room temperature.
 The oxide of N dissolves in water to form a strong alkaline solution.

Which of the following is correct in order of increasing proton number?

A N, L, K, M
B K, L, M, N
C N, K, L, M
D N, M, K, L

16 The size of Na+, Mg2+ and Al3+ is in the order: Na+ > Mg2+ > Al3+. Which of the following best
explain this trend?

A The number of electrons decreases while the number of protons increases.


B The number of electrons are the same but the number of protons increases.
C The number of electrons and protons decreases.
D The number of electrons and protons increases.

17 Element O is heated in chlorine. The product is then added to water. The resulting solution is
found to be neutral. What could O be?

A sodium
B aluminium
C phosphorus
D chlorine

18 Which pair of reactions could have the same common intermediate?

P CH3CH2CH3 intermediate (CH3)2CHCN


Q CH3CH(OH)CH3 intermediate (CH3)2C(OH)CN
R CH3CH=CH2 intermediate CH3CH(OH)CH3
S CH3CO2CH2CH2CH3 intermediate CH3CH2CH2Br

A P and Q C Q and S
B P and R D R and S

PRELIM 2  INNOVA 8872/01/2013 [Turn over

www.gradtutors.com
187
7

19 High energy radiation in the stratosphere produces free-radicals from chlorofluoroalkanes,


commonly known as CFCs.

Which free radical is most likely to result from the irradiation of CHFClCF2Cl?

A CHFClCFCl

B CHClCF2Cl

C CHFCF2Cl

D CFClCF2Cl

20 Santonin is a drug that was once widely used to expel parasitic worms from the body.

CH3

O CH3

CH3 O

O
santonin

When santonin is first treated with warm dilute H2SO4, and then the product of this reaction is
treated with cold acidified KMnO4, a final product T is obtained.

How many atoms of hydrogen in each molecule of product T can be displaced with sodium
metal?

A 2 C 5
B 4 D 6

PRELIM 2  INNOVA 8872/01/2013 [Turn over

www.gradtutors.com
188
8

21 Hydrocortisone is a steroid hormone produced by the adrenal gland and is released in


response to stress. It is commonly used as an active ingredient in anti-inflammatory creams.

Which of the following statements about hydrocortisone is incorrect?


A It does not show geometric isomerism.
B It turns hot acidified K2Cr2O7 from orange to green.
C When treated with an excess of hot concentrated acidified KMnO4, it forms a compound
containing five carbonyl groups.
D When treated with NaBH4 in the presence of methanol, it forms a compound containing
five hydroxy groups.

22 The reaction conditions for four different transformations are given below.

Which transformation has a set of conditions that is not correct?

UV light
A CH3CH3 + Cl2 (CCl4) CH3CH2Cl + HCl

KMnO4/H+
B CH2=CHCH=CH2 4CO2 + 3H2O
Heat

anhydrous
AlCl3
+ Cl2(aq) + HCl

heat

+ 2OH + 2Cl– + H2O

PRELIM 2  INNOVA 8872/01/2013 [Turn over

www.gradtutors.com
189
9

23 An alcohol with molecular formula CnH2n+1OH does not react with MnO4– / H+.

What is the least number of carbon atoms such an alcohol could possess?

A 4
B 5
C 6
D 7

24 An organic compound U has the following properties.

 changes the colour of acidified sodium dichromate(VI) from orange to green,


 has no effect on Fehling’s reagent,
 gives a positive tri-iodomethane test

What could U be?

A CH3COCH2CHO C CH3COCH2CH2OH
B CH3COCH2COCH3 D HOCH2CH2CH2CH2OH

25 The following reaction scheme shows the synthesis of an amino acid, alanine.

O OH NH NH3+
NH3
C CH3 C H C CH3 C H
CH3 H I II CH3 H
I NH2 II COOH
alanine

Which of the following shows the correct types of reaction for steps I and II?

I II
A substitution elimination
B substitution condensation
C addition elimination
D addition addition

26 Which of the following ions contains half-filled d-orbitals?

1 Mn2+
2 Cu+
3 Fe2+

PRELIM 2  INNOVA 8872/01/2013 [Turn over

www.gradtutors.com
190
10

27 Which of the following processes is endothermic?

1 electrolysis of water
2 freezing of water
3 condensation of steam

28 Which of the following might alter the rate constant of a reaction?

1 temperature
2 concentration
3 pressure

29 Aldosterone is a steroid hormone that increases reabsorption of ions and water in the kidney.
It has the structure shown below.

Aldosterone was reacted in separate experiments with:

(i) 2,4-dinitrophenylhydrazine
(ii) Ag(NH3)2+(aq)
(iii) PCl5(s)

Which statements about these reactions are correct?

1 One mole of aldosterone could react with three moles of 2,4-dinitrophenylhydrazine.


2 One mole of aldosterone could form two moles of Ag.
3 One mole of aldosterone could react with 2 moles of PCl5(s).

PRELIM 2  INNOVA 8872/01/2013 [Turn over

www.gradtutors.com
191
11

30 Which of the following reactions would produce 1,4-dicarboxylic acid?

reactant reagents and conditions

1 hot acidified potassium manganate(VII)

2 hot acidified potassium dichromate(VI)

alkaline aqueous iodine and heat followed by acidification

PRELIM 2  INNOVA 8872/01/2013 [Turn over

www.gradtutors.com
192

INNOVA JUNIOR COLLEGE


JC2 PRELIMINARY EXAMINATION 2
in preparation for General Certificate of Education Advanced Level
Higher 1

CANDIDATE
NAME

CLASS INDEX NUMBER

CHEMISTRY 8872/02
18 September 2013
Section A: Structured Questions
Section B: Free Response Questions 2 hours

Candidates answer Section A on the Question Paper


Additional Materials: Writing Papers
Data Booklet
Cover Page

READ THESE INSTRUCTIONS FIRST


For Examiner’s Use
Write your index number, name and civics group.
Write in dark blue or black pen. Section A
You may use pencil for any diagrams, graphs or rough working.
Do not use staples, paper clips, highlighters, glue or correction fluid. 1 8
Answer all questions in the space provided. 2
A Data Booklet is provided. 9
3
8
You are advised to show all working in calculations.
You are reminded of the need for good English and 4
15
clear presentation in your answers.
Significant
You are reminded of the need for good handwriting.
figures
Your final answers should be in 3 significant figures.
Handwriting
You may use a calculator.
Total
The number of marks is given in brackets [ ] at the end of each 40
question or part question.

At the end of the examination, fasten all your work


securely together.

This document consists of 17 printed pages and 1 blank page.

Innova Junior College [Turn over

PRELIM 2  INNOVA 8872/02/2013 [Turn over


www.gradtutors.com
193
2
For
Section A Examiner’s
Use
Answer ALL questions on the spaces provided.

1 The kinetics of the reaction between acetone and potassium cyanide was investigated.

An experiment was performed in which 0.200 mol dm3 of acetone, CH3COCH3 was
reacted with an excess of acid and sodium cyanide, NaCN. A graph of concentration of
CH3COCH3 against time was plotted.

CH3COCH3 + H+ + NaCN CH3C(OH)(CN)CH3 + Na+

[CH3COCH3] / moldm-3

Time / seconds

(i) Using your graph, deduce the order of reaction with respect to acetone.
Workings of two t½ shown on graph

t½ is approximately constant at 24 s

 order of reaction w.r.t. propanone is 1

(ii) Why was the acid and sodium cyanide used in excess?
To ensure their concentration remain relatively constant during the reaction and not
affect the rate of reaction

PRELIM 2  INNOVA 8872/02/2013 [Turn over


www.gradtutors.com
194
3
For
(iii) The order of reaction with respect to the acid was zero. Examiner’s
Use
Sketch, on the axes provided, the shape of the graphs relating to the information
provided.

[acid] / mol dm3 Rate / mol dm3 s1

time / s [acid] / mol dm3

Graph 1 Graph 2

(iv) Given that the order of reaction with respect to sodium cyanide is one, write the rate
law for this reaction.
rate = k [(CH3)2CO] [NaCN]

(v) Suggest with a reason whether pentan-2-one reacts more or less rapidly with NaCN
as compared to acetone.
Less rapidly

There is a bulkier propyl group/ CH3CH2CH2 group in pentan-2-one / there is steric


hindrance present in pentan-2-one

[Total: 8]

PRELIM 2  INNOVA 8872/02/2013 [Turn over


www.gradtutors.com
195
4
For
2 Benzoic acid is a colourless crystalline solid that has antifungal abilities and is an Examiner’s
important precursor for the synthesis of many organic compounds. Use

(a) Explain, with the aid of a diagram, why benzoic acid has an apparent molecular
mass of 244 in an organic solvent such as benzene.

  
 

  

Benzoic acid forms a dimer via hydrogen bonds with another of its molecule.
[2]

(b) Both benzoic acid and methylbenzene undergo aromatic substitution to produce the
chloro-compounds shown:

(i) Suggest why a harsher condition is required for the substitution of benzoic
acid.
–COOH is a deactivating group / electron-withdrawing group and hence makes
the benzene ring less reactive towards substitution reactions/ reduces the
electron density of the ring

(ii) How would you expect the acidity of 3-chlorobenzoic acid to compare with that
of benzoic acid?
3-chlorobenzoic acid is a stronger acid than benzoic acid. The –Cl atom is an
electron withdrawing group and hence will disperse the negative charge on the
O atom of 3-chlorobenzoate ion. Thus stabilising 3-chlorobenzoate ion.
[3]
(c) Benzoic acid is sparingly soluble in water. However, it dissolves readily to give a
colourless solution upon the addition of a reagent, N. Identify reagent N.
Reagent N: NaOH(aq) or Na2CO3(aq)
[1]

PRELIM 2  INNOVA 8872/02/2013 [Turn over


www.gradtutors.com
196
5
For
(d) Benzene-1,2-dicarboxylic acid can be produced via the following synthetic pathway. Examiner’s
Use

(i) What type of reaction is step I?


Substitution

(ii) Identify reagent P.

KMnO4, dilute H2SO4, (heat with reflux)

(iii) Suggest a structure for compound Q and draw its structural formula in the box
provided.

[3]

[Total: 9]

PRELIM 2  INNOVA 8872/02/2013 [Turn over


www.gradtutors.com
197
6
For
3 Phosphorus(V) chloride, PCl5, is a white solid which sublimes at 160°C. Examiner’s
Use
When gaseous phosphorus(V) chloride is heated in a closed container, the following
equilibrium is established.

PCl5(g) PCl3(g) + Cl2(g)

(a) Sketch a graph to show how the rates of the forward and backward reactions
change from the time phosphorus pentachloride is heated to the time the reaction
reaches dynamic equilibrium. Label your two lines clearly.

Reaction Rate Dynamic Equlibrium is attained

forward reaction
Backward rxn

Backward reaction
Forward rxn

teqm Time

[2]

(b) In an experiment, 1.00 mol of PCl5 vapour was heated in a closed 5.00 dm3 flask at
500K until equilibrium had been established. The amount of chlorine collected was
0.508 mol.

Write an expression for the equilibrium constant and calculate its value.

[ PCl3 ][Cl2 ]
Kc = mol dm 3
[ PCl5 ]

PCl5 PCl3 + Cl2


Initial amount/ mol 1.00 0 0

Change in amount/ mol –0.508 +0.508 +0.508


Eqm amount/ mol 0.492 0.508 0.508

 0.508  0.508 
  
Kc = 
5  5 
= 0.105 mol dm-3
 0.492 
 
 5 

[3]

PRELIM 2  INNOVA 8872/02/2013 [Turn over


www.gradtutors.com
198
7
For
(c) The graph below shows how the percentage yield of chlorine from the above Examiner’s
equilibrium varies as the reaction progresses at two different pressures. Use

% yield of chlorine

2 atm

catalyst 5 atm

progress of reaction

(i) Account for the differences in the two graphs at different pressures.

At higher pressure (5atm), system responds to decrease the pressure by


shifting position of equilibrium to the left to favour the formation of less
gaseous molecules. Thus, equilibrium yield of chlorine decreases.

Rate increases at higher pressure, thus gradient becomes steeper.

OR

At lower pressure (2atm), system responds to increase the pressure by shifting


position of equilibrium to the right to favour the formation of more gaseous
molecules. Thus, equilibrium yield of chlorine increases.

Rate decrease at lower pressure, thus gradient becomes gentler.

(ii) On the same axes, sketch a graph to show how the percentage yield of
chlorine will vary at 5 atm in the presence of a catalyst.
Steeper gradient , same equilibrium yield
[3]

[Total: 8]

PRELIM 2  INNOVA 8872/02/2013 [Turn over


www.gradtutors.com
199
8
For
4 (a) During the last Olympic Games, a torch relay around several countries took place Examiner’s
before the torch was used to ignite the Olympic cauldron to mark the start of the Use
Game. The fuel used to light the Olympic flame is propane. Propane is also used in
blow torches, portable stoves and in outdoor heaters.

A propane portable stove was used during a barbecue to boil water for making of
soup. It took 10 minutes to bring the water to boil.

Metal container

Water

Portable stove

Propane

The following information was known.

Mass of metal container / g 850


Mass of metal container containing water / g 1600
Initial temperature of water / oC 32
o
Final temperature of water / C 100
−1 −1
Specific heat capacity of water / J g K 4.18

Besides the above information, it was also known that the theoretical standard
enthalpy change of combustion of propane is −2220 kJ mol−1.

(i) Define the standard enthalpy change of combustion of propane.


The standard enthalpy change of combustion of propane is the enthalpy
change (heat evolved) when 1 mole of propane is completely burnt in oxygen /
burnt in excess oxygen under standard conditions of 298K and 1 atm.
(ii) Calculate the mass of propane needed to raise the temperature of the water
during the barbecue.
Amount of heat gained by water
= (1600 – 850) × 4.18 × (100  32)
= 2.1318 x 105 J
= 213 kJ

Amount of propane needed = 213.18 ÷ 2220 = 0.09603 mol

Mass of propane required = 0.09603 × 44.0 = 4.225 = 4.23 g

PRELIM 2  INNOVA 8872/02/2013 [Turn over


www.gradtutors.com
200
9
For
Another value of the standard enthalpy change of combustion of propane, Hc, can Examiner’s
be calculated using bond energies and the energy cycle below. Use

Hc
C3H8(g) + 5O2(g) 3CO2(g) + 4H2O(g)

+6460 kJ mol1 H1

3 C(g) + 8 H(g) + 10 O(g)

(iii) Use suitable bond energies given in the Data Booklet to calculate H1.

Hc
C3H8(g) + 5O2(g) 3CO2(g) + 4H2O(g)

+6460 kJ mol1 H1

3 C(g) + 8 H(g) + 10 O(g)

H1 = 6E(C=O) + 8E(OH)


= 6 × 740 + 8 × 460
= + 8120
= +8.12 x 103 kJ mol1

(iv) Hence, calculate Hc.


Hc = +6460  (+8120)
= 1660
= -1.66 x 103 kJ mol1

(v) Suggest why the value of Hc you calculated differs from the theoretical value.

The bond energy values obtained from the Data Booklet are average (mean)
bond energy values derived from a full range of molecules that contain the
particular bond and would differ from the experimental values.
[7]

PRELIM 2  INNOVA 8872/02/2013 [Turn over


www.gradtutors.com
201
10
For
(b) Most outdoor heaters are powered by the combustion of propane gas stored in small Examiner’s
cylinders. At room temperature and pressure, a typical outdoor heater designed to Use
produce 15 kW of energy runs from a cylinder containing 13 kg of propane.

As pure propane gas is odourless, small amounts of another compound are usually
added so that gas leaks from propane cylinders can be detected. An example of
such an odorant is ethanethiol, C2H5SH which has an odour that resembles that of
onion. Ethanethiol is chosen since the human nose can detect its presence at levels
of only about 0.02 moles of it per billion (109) moles of propane.

(i) Calculate the total amount of heat energy released by the complete
combustion of all the propane in a cylinder used in an outdoor heater.
Total amount of heat energy released =
13000 × 2220
44.0
= 6.55 × 105 kJ

(ii) Using your answer to (b)(i), calculate the time needed to empty one cylinder of
propane to produce 15 kW of energy. You are given that 15kW is equivalent to
15 kJ s1.
Total amount of time needed to empty one cylinder
5
= 6.55×10
15
= 4.37 × 104 s

(iii) Hence, calculate the rate, in cm3 s1, at which propane must leave the cylinder
in order to produce 15 kW of energy.

Rate at which propane must leave the container


13000
×24000
= 44.0
4.37×10 4
= 162 cm3 s1

(iv) Calculate the volume of carbon dioxide evolved when 13 kg of propane was
completely burnt in an outdoor heater.
C3H8(g) + 5O2(g)  3CO2(g) + 4H2O(g)
Amount of propane burnt = 13000 ÷ 44 = 295.45 mol
Amount of CO2 evolved = 3 x 295.45 = 886.36 mol
Volume of CO2 evolved = 886.36 x 24 = 21273 dm3
= 2.13 x 104 dm3

(v) Calculate the minimum mass of ethanethiol which must be added to 13 kg of


propane so that it can be detected by the human nose in the event of a
propane gas leak.
x
Minimum mass of ethanethiol required = 62.1 × 109 = 0.02
13000
44.0
Solve for x = 3.67 × 107 g
[5]

PRELIM 2  INNOVA 8872/02/2013 [Turn over


www.gradtutors.com
202
11
For
Examiner’s
(c) Besides being used as fuel, propane can also be used to synthesise other organic Use
compounds.

Outline the synthesis of propan-1-ol from propane. State clearly the reagents and
conditions used and draw any intermediate product(s) formed.
limited Cl2 in CCl4 H H H
uv light
CH3CH2CH3 H C C C H

H H Cl [1]

aq NaOH
heat with reflux

H H H

H C C C H

H H O
H
[3]
[Total: 15]

PRELIM 2  INNOVA 8872/02/2013 [Turn over


www.gradtutors.com
203

12

Section B

Answer two of the three questions in this section on separate answer paper.

5 (a) Ethanal is one of the most important aldehydes occurring widely in nature and is
also the cause of hangovers from alcohol consumption. It is produced on a large
scale industrially via the Wacker Process as shown.

2 CH2=CH2 + O2 2 CH3CHO

(i) State the hybridisation of each carbon atom in ethene and draw a diagram to
show the hybridised orbitals around one of these carbon atoms.
Sp2 hydrisation

(ii) Describe, in terms of orbital overlap, the bonding of the two carbon atoms of
the C=C bond in ethene.
The two carbon atoms are sp2 hybridised. One sp2 orbital from each carbon
atom overlap head-on to give a sigma bond. The unhybridised p orbital from
each carbon atom overlap sideway to give π bond.
[4]

(b) The Wacker reaction is accelerated by palladium catalysts.

(i) Define catalyst.


A catalyst is defined as a substance which alters the rate of reaction by
providing an alternative pathway with a lower activation energy and itself
remains chemically unchanged at the end of the reaction.

(ii) Explain with the aid of a diagram that shows the distribution of molecular
energies, how the addition of a catalyst accelerates the reaction.

 Catalyst provides an alternative pathway with a lower activation energy.


 Fraction of particles with energy equal to or greater than the activation
energy of the catalysed reaction, Ea(cat) increases.
 Frequency of effective collisions taking place in the reaction increases.
 Since rate of reaction is proportional to the frequency of effective
collisions, rate of reaction increases.

PRELIM 2  INNOVA 8872/02/2013 [Turn over


www.gradtutors.com
204

13

(iii) In a college laboratory, ethanal can be synthesised from ethanol. Briefly


outline how you would carry out this reaction.
Ethanal can be prepared by heating ethanol under distillation with potassium
dichromate and dilute sulfuric acid.

(iv) Explain why ethanal can be prepared in the way you described in (b)(iii), in
terms of the volatility of ethanal and ethanol.
[8]
 Both ethanal and ethanol have simple molecular structures.
 There exists hydrogen bonding between ethanol molecules
but van der waals forces of attraction between ethanal
molecules.
 Less energy is needed to overcome the weaker van der waals
forces of attraction between ethanal molecules.
 Hence volatility of ethanal is higher and will distil out before it
can continue to get oxidised to ethanoic acid.

(c) (i) Another common carbonyl compound used in the industry is phenylacetone,
which is an intermediate used to produce pesticides and anticoagulants.

phenylacetone

Compound C can be synthesised from phenylacetone.

O
step I O
H+
C Intermediate A
C CH3 C
-
C O
H H
H H
step II
H CH3
H
C CH3COOH
C
C O O Intermediate B
H
H
Compound C

Complete the synthetic route by giving the structures of intermediates A and


B as well as the reagents and conditions for steps I and II.
step I
I2 (aq), NaOH(aq), warm

step II
LiAlH4, dry ether, room temperature

PRELIM 2  INNOVA 8872/02/2013 [Turn over


www.gradtutors.com
205

14

Intermediate A

Intermediate B

(ii) Suggest the structures of the product and type of reaction undergone when
phenylacetone reacts with the following reagents.

 HCN, trace OH–


 hydrazine, H2N-NH2

Type of reaction: Addition

Type of reaction: condensation


[8]

[Total: 20]

PRELIM 2  INNOVA 8872/02/2013 [Turn over


www.gradtutors.com
206

15

6 (a) The elements of the third period of the Periodic Table, sodium to sulfur, all form
chlorides by direct combination.

(i) When dry chlorine is passed over heated aluminium powder in a long
hard-glass tube, a vapour is produced which condenses to a solid in the
cooler parts of the tube. At low temperatures, the vapour has a Mr of 267.

State one observation you would make during this reaction.


Any one observations:
 (yellow) White solid, AlCl3 formed/ white fumes, Al2Cl6 seen
 Chlorine gas decolourised
 Aluminium glows

(ii) Suggest the molecular formula of the vapour, and draw a dot-and-cross
diagram to illustrate its bonding.
MF of vapour: Al2Cl6

Al2Cl6 is formed through two dative covalent bonding

(iii) The solid reacts with water in two separate ways.

 When a few drops of water are added to the solid, steamy white fumes
are evolved and a white solid remains, which is insoluble in water.
 When a large amount of water is added to the solid, a clear, weakly
acidic solution results.

Write equations, including state symbols, for these two reactions.

Few drops of water:

2AlCl3 (s) + 3H2O (l)  Al2O3 (s) + 6HCl (g) OR

AlCl3 (s) + 3H2O (l)  Al(OH)3 (s) + 3HCl (g)

Excess water:
AlCl3 (s) + 6H2O(l)  [Al(H2O)6]3+ (aq) + 3Cl- (aq)

[Al(H2O)6]3+ (aq)  [Al(H2O)5(OH)]2+ (aq) + H+ (aq)

OR
The 2 equations can be combined:
AlCl3 (s) + 6H2O (l) [Al(H2O)5(OH)]2+ (aq) + H+ (aq) + 3Cl- (aq)
[7]

PRELIM 2  INNOVA 8872/02/2013 [Turn over


www.gradtutors.com
207

16

(b) Lattice energy can be used as a measure of the energetic stability of ionic
compounds.

(i) Define the lattice energy of silver fluoride.


Lattice energy of silver fluoride is the enthalpy change which occurs when 1 mole of solid ionic
silver fluoride is formed from its constituent gaseous ions (Ag+(g) & F-(g) under standard
conditions of 298K, 1 atm.

The table below shows numerical values of lattice energies for silver fluoride and
silver iodide. These have been determined from experimental data or theoretically
calculated.

Experimental value Theoretical value


Compound
/ kJ mol–1 / kJ mol–1
AgF –967 –953
AgI –889 –808

(ii) By quoting appropriate data from the Data Booklet, explain why the lattice
energy of silver iodide is less exothermic than silver fluoride.
q  q
 Lattice energy 
r  r
 F– and l– have the same charges

 but ionic radius of F– is smaller than I–


 quote ionic radius of both F–(0.136nm) and I– (0.216nm)
 lattice energy of AgI is less exothermic than AgF

(iii) Silver fluoride and silver iodide have the same crystal structure. There is close
agreement between the experimental and theoretical values of lattice energy
for AgF but not for AgI. Suggest a reason for this.

AgI is an ionic compound with some covalent character due to large electron
cloud of I– that can be polarized by small size Ag+.
[4]

(c) Compound C has the empirical formula CH2O and Mr of 90. There is no reaction
when C is treated with NaHCO3. When 0.600 g of C is reacted with an excess of Na,
160 cm3 of H2, measured at room temperature and pressure, is produced. Treatment
of C with 2,4-dinitrophenylhydrazine reagent produces an orange solid. When C is
warmed with Fehling’s reagent, a brick red precipitate is formed.

The structure of C has the following feature:


 No carbon atom has more than one oxygen atom joined to it.

Suggest a structure for C, explaining your reasoning.


 Since C has no reaction with NaHCO3 => carboxylic acid absent

 Since C undergoes redox/acid-metal reaction with Na =>


 alcohol present
1 4
6 0
0 0
2

 n(H2) produced from 0.600 g of C = = 6.67 x 10-3 mol

PRELIM 2  INNOVA 8872/02/2013 [Turn over


www.gradtutors.com
208

17

 n(H atoms) = 2 (6.67 x 10-3 ) = 1.33 x 10-2 mol


0
.
6 9
0 0
0
 n(C) = = 6.67 x 10-3 mol

 n(C) : n(H atoms) = 6.67 x 10-3 : 1.33 x 10-2 = 1 : 2

 since each –OH group produces one H atom, there are two -OH groups.
 since C undergoes condensation with 2,4-DNPH =>
 aldehyde / ketone present

 since C undergoes (mild) oxidation with Fehling’s =>


 aldehyde present

structure of C:

CH2(OH)CH(OH)CHO OR

[7]

(d) Compound C can be both oxidised and reduced.

(i) Give the structural formula of the compound formed when C is reacted with
NaBH4 under suitable conditions.
H OH

H2C C CH2

OH OH

(ii) Give the structural formula of the compound formed when C is heated under
reflux with acidified K2Cr2O7.

[2]

[Total: 20]

PRELIM 2  INNOVA 8872/02/2013 [Turn over


www.gradtutors.com
209

18

7 (a) When sodium is burned in air, a mixture of sodium oxide, Na2O, and sodium peroxide
Na2O2, is formed. The mixture reacts with water according to the following equations.

Na2O + H2O 2NaOH


Na2O2 + 2H2O 2NaOH + H2O2

The following information will allow you to calculate the relative amounts of the two
oxides produced when sodium is burned.

 The mixture obtained by burning a sample of sodium was dissolved in distilled


water and made up to 100 cm3 to give solution D.

 A 25.0 cm3 portion of solution D was titrated with 0.100 mol dm–3 HCl.
22.5 cm3 of acid was required to reach the end-point.

 The H2O2 content of solution D was found by titration of another 25.0 cm3 portion
with 0.0200 mol dm–3 KMnO4.

The following reaction occurs.

2MnO4– + 5H2O2 + 6H+ 2Mn2+ + 5O2 8H2O

10.0 cm3 of KMnO4 solution was required to reach the end-point.

(i) Using the results of the HCl titration calculate the total amount in moles of NaOH
in 100 cm3 of solution D.
(i) 22.5
No. of moles of H+ = x 0.100 = 2.25 x 10-3 mol
1000
No. of moles of OH- in 25.0 cm3 of solution D
= 2.25 x 10-3 mol

No. of moles of OH- in 100 cm3 of solution D


= 2.25 x 10-3 x 4
= 9.00 x 10-3 mol (3sf)

(ii) Using the results of the KMnO4 titration, calculate the amount in moles of H2O2 in
100 cm3 of solution D.
(ii) 10.0
No. of moles of KMnO4 = x 0.0200 = 2.00 x 10-4 mol
1000
No. of moles of H2O2 in 25.0 cm3 of solution D
5
= 2.00 x 10-4 x
2
= 5.00 x 10-4 mol

No. of moles of H2O2 in 100 cm3 of solution D


= 5.00 x 10-4 x 4
= 2.00 x 10-3 mol (3sf)

PRELIM 2  INNOVA 8872/02/2013 [Turn over


www.gradtutors.com
210

19

(iii) Hence calculate the amount in moles of Na2O2 formed during the burning of the
sodium sample.
(iii) In 100 cm3 of solution D,
No. of moles of Na2O2 = No. of moles of H2O2
= 2.00 x 10-3 mol (3sf)

(iv) Using your result in (a)(iii), calculate the amount in moles of Na2O formed during
the burning of the sodium sample.
No. of moles of NaOH from dissolving Na2O2 in water
= 2.00 x 10-3 x 2
= 4.00 x 10-3 mol

No. of moles of NaOH from dissolving Na2O in water


= 9.00 x 10-3 – 4.00 x 10-3
= 5.00 x 10-3

No. of moles of Na2O = 5.00 x 10-3  2 = 2.50 x 10-3 mol (3sf)

[6]

(b) A buffer solution can be prepared by mixing 500 cm3 of 4 mol dm–3 of CH3COOH in
500 cm3 of 0.5 mol dm–3 of NaOH.

(i) Calculate the pH of NaOH before mixing is carried out.


pOH = –log [OH-]
= –log(0.5)
= 0.301

pH = 14 – pOH
= 14 – 0.301
= 13.7

(ii) By using suitable equations, describe how the above buffer solution works.
[4]
When a small amount of acid, H+ ions, is added to the buffer solution,

CH3COO- (aq) + H+ (aq)  CH3COOH (aq)

The additional H+ ions are removed by the large concentration of CH3COO-


present in the buffer. Hence, the pH remains almost unchanged.

When a small amount of base, OH– ions, is added to the buffer solution,

CH3COOH (aq) + OH– (aq)  CH3COO- (aq) + H2O (l)

The additional OH– ions are removed by the large concentration of CH3COOH
present in the buffer. Hence, the pH again remains almost unchanged.
(c) The graph below shows the first ionisation energy of eight elements with consecutive
proton number.

PRELIM 2  INNOVA 8872/02/2013 [Turn over


www.gradtutors.com
211

20

1st Ionisation Energy kJ mol-1


L

J
E
G
I
F

Atomic
number

(i) State the valence electronic configuration of element F.


ns2np1

(ii) Give the formula of the oxide that F is likely to form.


F2O3

(iii) Hence or otherwise, explain the drop in first ionisation energy from E to F.
E: ns2
F: ns2np1

For F, the outermost electron is removed from the (n)p subshell, which is further away from
nucleus than (n)s subshell in E. Hence the outermost electron in F is less strongly attracted by the
nucleus and requires less energy to remove.
[4]

(d) (i) Sketch a graph of the melting point of oxides of elements from sodium to
sulfur.

PRELIM 2  INNOVA 8872/02/2013 [Turn over


www.gradtutors.com
212

21

(ii) Explain as fully as you can why the melting point varies in the way shown.
Na2O, MgO and Al2O3 are giant ionic structure with strong electrostatic forces of attraction
between the ions. High amount of energy needed to break the strong electrostatic attraction.

MgO has a higher melting point than Na2O because of the stronger electrostatic attraction
between the highly charged Mg2+ and the O2 ions.

Al2O3 has a slightly lower melting point than MgO because Al2O3 has some degree of covalent
character due to the highly charged density of Al3+ ion.

SiO2 has a giant covalent structure with strong extensive covalent bond between Si and O
atom. High amount of energy needed to break the strong extensive covalent bonds between the
Si and O atoms

Both P4O10 and SO3 has a simple molecular structure with weak van der Waals’ forces of
attraction between the molecules. P4O10 has a larger electron cloud as compared to SO3.
More energy is needed to overcome the forces of attraction between P4O10 molecules leading to
a higher melting point as compared to SO3.
[6]

[Total: 20]

BLANK PAGE

PRELIM 2  INNOVA 8872/02/2013 [Turn over


www.gradtutors.com
213

22

PRELIM 2  INNOVA 8872/02/2013 [Turn over


www.gradtutors.com
214

JURONG JUNIOR COLLEGE


JC 2 PRELIMINARY EXAMINATION
Higher 1

CHEMISTRY 8872/01
Paper 1 Multiple Choice 19 September 2013
50 minutes
Additional Materials: Multiple Choice Answer Sheet
Data Booklet

READ THESE INSTRUCTIONS FIRST

Write in soft pencil.


Do not use staples, paper clips, highlighters, glue or correction fluid.
Write your name, class and shade your exam index number on the Answer Sheet in the spaces
provided.

There are thirty questions on this paper. Answer all questions. For each question there are four
possible answers A, B, C and D.
Choose the one you consider correct and record your choice in soft pencil on the separate Answer
Sheet.

Read the instructions on the Answer Sheet very carefully.

Each correct answer will score one mark. A mark will not be deducted for a wrong answer.
Any rough working should be done in this booklet.

A Data Booklet is provided. Do not write anything on the Data Booklet.

This document consists of 13 printed pages.


[Turn over

www.gradtutors.com
215
2
SECTION A
For each question there are four possible answers, A, B, C and D. Choose the one you consider
to be correct.

1. The relative abundances of the isotopes of a sample of titanium are shown in the table
below.

Relative Isotopic Mass 46 47 48 49 50


Relative Abundance 11.2 10.1 100.0 7.3 7.0
What is the relative atomic mass of titanium in this sample?

A 47.92
B 47.91
C 47.90
D 47.89

2. How many atoms are there in 500 cm3 of oxygen at 0 oC and 1 atm?

A 1.25 1022
B 1.34 1022
C 2.50 1022
D 2.68 1022

3. The radioactive isotope 210


84 Po was said to be the agent that poisoned the former Russian
security agent Alexander Litvinenko in London in November 2006.
210
84 Po decays to give an element X and emits a high energy -particle (which is a helium
nucleus, 42 He ). No other particle is produced. -particles cause irreparable damage to the
tissues of internal organs.
210
84 Po  42 He + X
210
Which row in the table correctly describes the composition of 84 Po and element X?

210
84 Po X

number of number of number of


neutrons protons neutrons
A 126 80 122
B 126 82 124
C 210 80 206
D 210 82 208

Jurong Junior College www.gradtutors.com


8872/01/Prelim 2013 [Turn Over
216
3
4. In the body, cellular respiration produces energy from the oxidation of glucose.
The diagram shows the structure of glucose.

A new artificial sweetener has been produced by replacing all of the hydroxyl groups
attached directly to the ring carbon atoms in glucose with chlorine atoms.
What is the empirical formula of this chlorinated glucose?

A CHClO C C6H7Cl5O
B C3H4Cl2O D C6H8Cl4O2

5. What is the electronic configuration of an element with first ionisation energy higher than
that of each of its neighbours in the Periodic Table?

A 1s2 2s2 2p6 3s2 3p1


B 1s2 2s2 2p6 3s2 3p2
C 1s2 2s2 2p6 3s2 3p3
D 1s2 2s2 2p6 3s2 3p5

6. Which one of the following does not contain any unpaired s or p electron?

A Cr B Ge

C S D Sc

Jurong Junior College www.gradtutors.com


8872/01/Prelim 2013 [Turn Over
217
4
7. Which graph correctly shows the first four ionisation energies of a Group III element?

A B

C D

8. BF3.CH3OH is used to produce methyl esters from compounds containing acyl groups. In
the diagrams, x, and o represent electrons from B, F and O respectively.
Which tetrahedral structure correctly shows the electron pairs around the boron atom?

A B

C D

9. Which process involves the breaking of hydrogen bonds?

A H2(l)  H2(g) B NH3(l)  NH3(g)


C 2HI(g)  H2(g) + I2(g) D CH3COOH (aq)  CH3COO― (aq) + H+(aq)

Jurong Junior College www.gradtutors.com


8872/01/Prelim 2013 [Turn Over
218
5
10. Which diagram best represents the structure of solid magnesium oxide?

A C

B D

11. Which of the following is involved in determining the enthalpy change of a chemical
reaction?

A the number of stages involved in the chemical reaction


B the activation energy of the reaction
C the initial and final state of the reacting system
D the intermediates of the overall chemical reaction

12. Which class of reaction always has an endothermic enthalpy change?

A combustion C formation
B ionisation D lattice energy

13. If the reaction P + Q  R + S is described as being zero order with respect to P,


it means that

A P is a catalyst in this reaction.


B no P molecules possess sufficient energy to react.
C the concentration of P does not change during the reaction.
D the rate of reaction is independent of the concentration of P.

Jurong Junior College www.gradtutors.com


8872/01/Prelim 2013 [Turn Over
219
6
14. The reaction of a compound RS is shown below.
RS (g)  R (g) + S (g)
The rate equation for the reaction is rate = k[RS] and the rate constant is found to be
3.6 10–3 s–1.
If the initial concentration of RS is 2.0 10–2 mol dm–3, what will be the concentration of
RS and R after 385 seconds?

[RS] / mol dm–3 [R] / mol dm–3

A 2.5 × 10–3 2.5 × 10–3


B 2.5 × 10–3 1.75 × 10–2
C 5.0 × 10–3 5.0 × 10–3
D 5.0 × 10–3 1.5 × 10–2

Jurong Junior College www.gradtutors.com


8872/01/Prelim 2013 [Turn Over
220
7
15. The diagram shows the reaction pathway diagram for an uncatalysed reaction.

The reaction is then catalysed.

What are the changes in the rate constant and the reaction pathway diagram?

rate constant energy profile

A decrease

B decrease

C increase

D increase

Jurong Junior College www.gradtutors.com


8872/01/Prelim 2013 [Turn Over
221
8
16. The product of the concentrations of X and Y, [X][Y], is plotted against time, t, for the
following second-order reaction.
X+Y  Z
Which graph would be obtained?

[X][Y] [X][Y]

A B

0 time 0 time

[X][Y] [X][Y]

C D

0 time 0 time

17. An equilibrium can be represented by the following equation.


P(aq) + Q(aq) 2R(aq) + S(aq)
In a certain mixture, the equilibrium concentration of S is 10 mol dm‒3.

What will be the new equilibrium concentration of S if 5 mol of Q is dissolved in the


mixture?

A 15 mol dm‒3
B Between 10 mol dm‒3 and 15 mol dm‒3
C 10 mol dm‒3
D Between 5 mol dm‒3 and 10 mol dm‒3

18. Which of the following is an incorrect trend of the elements of the Period 3 of the
Periodic Table?

A The radii of the atoms decrease.


B The oxides of the elements change from basic to acidic.
C The melting points of the elements decrease steadily.
D The compounds of the elements change from ionic to covalent.

Jurong Junior College www.gradtutors.com


8872/01/Prelim 2013 [Turn Over
222
9
19. The oxide of G and the chloride of H are separately mixed with water. The two resulting
mixtures have the same effect on litmus. Which of the following represents the correct
elements?

G H
A Al Si
B Mg P
C P Al
D S Na

20. The table below shows the rate of hydrolysis of halogen-containing


compounds P to S. The rate of hydrolysis is measured by the speed at which the silver
halide precipitate is formed.

P Q R S

Cl CH2Cl CH2Br CH2I

No precipitate. Precipitate formed Precipitate formed after 2 Precipitate forms


after 10 minutes. minutes. instantaneously.

Which statement explains the rate of reaction?

A The C–X bond length increases from compound P to S.


B The solubility of the compounds increases from P to S.
C The mechanism for the reaction changes from P to S.
D The reaction conditions are increasingly milder from P to S.

21. Phenylethanoic acid, C6H5CH2COOH can be synthesised from (bromomethyl)benzene,


C6H5CH2Br in the laboratory in two steps.

What are reagents and conditions that can be used for this synthesis?

Step 1 Step 2

A NaOH(aq), heat Acidified K2Cr2O7, heat

B Ethanolic NaOH, heat Acidified KMnO4, heat

C Ethanolic NaCN, heat HCl(aq), heat

D Cold HCN, trace amount of NaCN H2SO4(aq), heat

Jurong Junior College www.gradtutors.com


8872/01/Prelim 2013 [Turn Over
223
10
22. Jasmone is an active ingredient of jasmine. It is extracted from jasmine flowers for
perfume.

Jasmone

Which of the following statements is correct?

A It exists as 1 pair of cis-trans isomers.

B There are 4 sp3 carbon atoms in Jasmone.

C Effervescence is observed when Jasmone is heated with acidified KMnO4.

D A reddish brown precipitate is observed when hot alkaline Cu2+ complex solution
was added to Jasmone.

23. Which transformation can be readily achieved by only one substitution reaction?

A CH3CH=CH2  CH3CH=CHCl

B (CH3)3CCH2F  (CH3)3CCH2NH2

24. Which of the following functional group is found in the molecule below?

A Tertiary alcohol B Amide


C Ketone D Phenyl group

Jurong Junior College www.gradtutors.com


8872/01/Prelim 2013 [Turn Over
224
11
25. Ethanal reacts with HCN as shown below.

When the reaction is repeated with propanone, the reaction is slower.

Which of the following helps to explain the above observation?

A C=O in propanone is stronger.


B Propanone molecules are held by stronger permanent-dipole permanent-dipole
interaction.
C ‒CH3 is electron-donating, it intensifies the negative charge in propanone.
D ‒CH3 is electron-donating, it makes the carbonyl carbon in propanone less
electron-deficient.

SECTION B
For each of the questions in this section, one or more of the three numbered statements 1 to 3
may be correct.
Decide whether each of the statements is or is not correct.
The responses A to D should be selected on the basis of

A B C D
1, 2 and 3 are 1 and 2 only are 2 and 3 only are 1 only is
correct correct correct correct

No other combination of statements is used as a correct response.

26. An ionic compound Q


 Has an empirical formula NH2O.
 Reacts with NaOH(aq) to produce ammonia gas
Which statements about the anion in this salt are correct?

1 It has a single negative charge


2 It contains an atom with an oxidation number of +3.
3 It contains 20 electrons.

Jurong Junior College www.gradtutors.com


8872/01/Prelim 2013 [Turn Over
225
12
The responses A to D should be selected on the basis of

A B C D
1, 2 and 3 are 1 and 2 only are 2 and 3 only are 1 only is
correct correct correct correct

No other combination of statements is used as a correct response.

27. Calcium reacts with water to form calcium hydroxide and hydrogen.
Ca(s) + 2H2O(l)  Ca(OH)2(s) + H2(g)
The standard enthalpy change for this reaction can be measured in the laboratory.
What further information is needed in order to calculate the standard enthalpy change of
formation of calcium hydroxide, Hf(Ca(OH)2)?

1 Hf of H2O(l)
2 Hf for H2(g)
3 First and second ionisation energies of Ca

28. A catalytic converter is part of the exhaust system of many modern cars.
Which reactions occur in a catalytic converter?

1 2CO + 2NO  2CO2 + N2


2 CO2 + NO  CO + NO2
3 2SO2 + 2NO  2SO3 + N2

29. Which of the following tests can be used to distinguish between the two organic
compounds below?

1 Na metal
2 Aqueous bromine, in the dark
3 2,4-Dinitrophenylhydrazine

Jurong Junior College www.gradtutors.com


8872/01/Prelim 2013 [Turn Over
226
13
The responses A to D should be selected on the basis of

A B C D
1, 2 and 3 are 1 and 2 only are 2 and 3 only are 1 only is
correct correct correct correct

No other combination of statements is used as a correct response.

30. A derivative of Rofecoxib, an efficient drug against arthritis, has the following structure.

Which of the following statements about this compound are correct?

1 It undergoes hydrogenation with hydrogen gas and nickel catalyst.


2 It reacts with cold dilute alkaline KMnO4 to give a diol.
3 The molecule is planar.

Jurong Junior College www.gradtutors.com


8872/01/Prelim 2013 [Turn Over
227

JC2 H1 2013 Chemistry Prelim Paper 1 Suggested Worked Solution  

1.  A  11.  C  21. C 


2.  D  12.  B  22. A 
3.  B  13.  D  23. D 
4.  B  14.  D  24. B 
5.  C  15.  D  25. D 
6.  D  16.  B  26. B 
7.  B  17.  B  27. D 
8.  B  18.  C  28. D 
9.  B  19.  C  29. A 
10.  C  20.   
A  30. B 
 

JC2 H1 Chemistry Prelim 2013 www.gradtutors.com


Paper 1 Suggested Worked Solutions Page 1 of 1
228
1

JURONG JUNIOR COLLEGE


JC 2 PRELIMINARY EXAMINATION
Higher 1

CANDIDATE
NAME

EXAM INDEX
CLASS NUMBER

CHEMISTRY 8872/02
Paper 2 Structured Questions 30 August 2013
2 hours
Candidates answer Section A on the Question Paper.

Additional Materials: Answer Paper


Data Booklet

READ THESE INSTRUCTIONS FIRST


Write your name, class and index number on all the work you hand in.
Write in dark blue or black pen on both sides of the paper.
You may use a soft pencil for any diagrams, graphs or rough working.
Do not use staples, paper clips, highlighters, glue or correction fluid.

Section A
Answer all questions.

Section B
Answer two questions on separate answer paper.

A Data Booklet is provided. Do not write anything on the Data Booklet.

At the end of the examination, fasten all your work securely together.
The number of marks is given in brackets [ ] at the end of each question or part question.

For Examiner’s Use

Section A Section B

1 7

2 8

3 9

Total

This document consists of 18 printed pages.


Jurong Junior College 8872/02/Prelim 2013 [Turn over
www.gradtutors.com
229
2
Section A

Answer all questions in this section in the spaces provided.

1. (a) (i) When separate beams of 14O2 and 24Mg2+ are passed through an
electric field in the set-up shown below, they behave differently.

source

24
Sketch on the diagram above to show the path of the beam of Mg2+ as
it enters the electric field. Label the beam.

(ii) Given that the angle of deflection for 24Mg2+ is 2.5o, calculate the angle of
deflection of the 14O2 beam.

[2]

Jurong Junior College 8872/02/Prelim 2013 [Turn over


www.gradtutors.com
230
3

1. (b) H1
MgCl2(s) Mg2+(aq) + 2Cl (aq)

H2 H3

Mg2+(g) + 2Cl (g)

(i) What enthalpy change does H2 represent?

…………………………………………………………………………………

(ii) Reaction Enthalpy change, H

Mg2+(g)  Mg2+(aq) 1920 kJ mol1

Cl (g)  Cl (aq) 381 kJ mol1

Mg2+(g) + 2Cl (g)  MgCl2(s) 2326 kJ mol1

Using the above values, calculate H3 and hence H1.

(iii) How would you expect the magnitude of the lattice energy of magnesium
oxide to compare with magnesium chloride. Explain your answer.

[5]

[Total: 7]

Jurong Junior College 8872/02/Prelim 2013 [Turn over


www.gradtutors.com
231
4
2. In aqueous solution, methanoic acid (HCO2H) reacts with bromine (Br2).

HCO2H(aq) + Br2(aq)  2H+(aq) + 2Br (aq) + CO2(g)

Student X carried out an investigation on the rate of this reaction. He uses a large
excess of methanoic acid (0.500 mol dm3). During the reaction, bromine is used up
and loses its brown colour. The colour intensity of the bromine colour can be
measured using a colorimeter to give the concentration of Br2.

Time/s 0 50 100 150 230 300 400

[Br2(aq)]/ 0.0100 0.0080 0.0067 0.0055 0.0040 0.0030 0.0022


mol dm3

(a) Explain the purpose of using large excess of methanoic acid.

[1]

(b) Using the graph paper on the next page, plot a graph and determine the order
of reaction with respect to bromine.

[3]

(c) Given that the reaction is first order with respect to methanoic acid, write the
rate equation for the reaction.

[1]

(d) Determine the initial rate of the experiment carried out by Student X. Hence
calculate the rate constant, stating its units. You should show your clear
working on your graph.

[2]

Jurong Junior College 8872/02/Prelim 2013 [Turn over


www.gradtutors.com
232
5

Jurong Junior College 8872/02/Prelim 2013 [Turn over


www.gradtutors.com
233
6
2. (e) Student Y also carried out an experiment to study the reaction between
methanoic acid (HCO2H) and bromine (Br2).

HCO2H(aq) + Br2(aq)  2H+(aq) + 2Br (aq) + CO2(g)

He reacted 50 cm3 of 0.500 mol dm3 HCO2H with 50 cm3 of 0.500 mol dm3
Br2 at 25oC and measured the volume of gas collected every 10 minutes. Then
he plotted Curve R in Figure 2.1.

Sketch on Figure 2.1 to show how the volume of CO2 collected changes with
time if he repeated the experiment using 50 cm3 of 0.500 mol dm3 HCO2H
with 50 cm3 of 0.500 mol dm3 Br2 at 100oC.

Volume of CO2/ cm3

Curve R

time
Figure 2.1

[1]

[Total: 8]

Jurong Junior College 8872/02/Prelim 2013 [Turn over


www.gradtutors.com
234
7
3. (a) On the following diagrams, sketch graphs to show the variation in the named
property of some Period 3 elements or compounds.

Melting point

Na Mg Al Si P S Cl
pH of solution formed
after adding water

Na2O MgO Al2O3 SiO2 P4O10 SO3


State the pH values formed by each oxide on your sketch. [2]

(b) Explain why sodium and magnesium have different melting point.

[2]

(c) Write equations to explain the difference in the pH of the solution formed after
water is added to P4O10 and MgO.

[2]

[Total: 6]

Jurong Junior College 8872/02/Prelim 2013 [Turn over


www.gradtutors.com
235
8

4. Haber process is an industrial process to manufacture ammonia from nitrogen.


N2(g) + 3H2(g)  2NH3(g)

(a) Give two features of a reaction at equilibrium.

Feature 1:

Feature 2:

[2]

(b) (i) What are the typical values of pressure and temperature used
industrially?

Temperature ………………………………………….

Pressure ………………………………………………

(ii) Explain why that particular temperature in 4b(i) is chosen.

[3]

Jurong Junior College 8872/02/Prelim 2013 [Turn over


www.gradtutors.com
236
9

4. (c) (i) Sketch a graph on the axes given below to show how the % yield of NH3
changes with pressure.

yield of NH3

pressure

(ii) Explain the shape of your graph in 4(c)(i).

[2]

[Total: 6]

Jurong Junior College 8872/02/Prelim 2013 [Turn over


www.gradtutors.com
237
10

5. The Dissolved Oxygen Concentration (DOC) in rivers and lakes is important for
aquatic life. If the DOC falls below 5 ppm, most species of fish cannot survive.

Environmental chemists can determine the DOC in water using the procedure below.
 Step 1: A sample of river water is shaken with aqueous Mn2+ and aqueous alkali.
The dissolved oxygen oxidises the Mn2+ to Mn3+, forming a pale brown
precipitate of Mn(OH)3.
O2(aq) + 4Mn2+(aq) + 8OH(aq) + 2H2O(l)  4Mn(OH)3(s)

 Step 2: Mn(OH)3 precipitate is then reacted with an excess of aqueous


potassium iodide, which is oxidised to iodine.
Mn(OH)3(s) + e  Mn(OH)2(s) + OH(aq)
I2(aq) + 2e  2I(aq)

 Step 3: The iodine formed is then determined by titration with aqueous sodium
thiosulfate, Na2S2O3(aq).
2S2O32(aq) + I2(aq)  S4O62(aq) + 2I(aq)

25.0 cm3 of a sample of river water was analysed using the above procedure.
The titration requires 24.60 cm3 of 0.00100 mol dm–3 Na2S2O3(aq).

(a) Calculate the amount of iodine reacted during the titration in Step 3.

[1]

(b) Using the two half equations given above in Step 2, construct a balanced
equation for the reaction that occurred in Step 2 of the procedure.

[1]

(c) Using the equation you constructed in (b), calculate the amount of Mn(OH)3
reacted in Step 2.

[1]

(d) Hence calculate the amount of O2(aq) in 25.0cm3 of the sample of river water.

[1]

Jurong Junior College 8872/02/Prelim 2013 [Turn over


www.gradtutors.com
238
11
5. (e) Calculate the Dissolved Oxygen Concentration (DOC), in g dm–3, in the river
water.

[1]

(f) Assume the density of the river water is 1 g cm–3, calculate the Dissolved
Oxygen Concentration (DOC), in ppm, in the river water.

[1 ppm of dissolved oxygen = 1 g of dissolved oxygen in 106 g of river water]

[1]

(f) Hence. comment on whether there is enough oxygen for fish to survive in that
river.

[1]

[Total: 7]

Jurong Junior College 8872/02/Prelim 2013 [Turn over


www.gradtutors.com
239
12
6. Shown below are some active ingredients in essential oils.

CH3 CH3

OH
H3C OH

CH3 H3C CH3

Terpinen-4-ol Menthol
(Found in lavender essential oil) (Found in peppermint essential oil)

(a) Identify all the functional groups in terpinen-4-ol.

…………………………………………………………………………………………..
[1]
(b) (i) When subjected to dehydration, terpinen-4-ol forms x isomeric organic
products while menthol forms y isomeric organic products.
What are the numbers x and y?

x = …………………………… y = ………………………………….
(ii) State the reagents and conditions required for the dehydration.
…………………………………………………………………………………….
(iii) Draw the structural formula of one organic product that is obtained when
menthol undergoes dehydration.

[3]
(c) Both terpinen-4-ol and menthol decolourise purple KMnO4. Draw the structural
formula of the products formed.
Oxidation product of terpinen-4-ol:

Oxidation product of menthol:

[2]

[Total:6]

Jurong Junior College 8872/02/Prelim 2013 [Turn over


www.gradtutors.com
240
13
Section B

Answer two questions from this section on separate answer paper.

7. (a) Burning of fossil fuels emitted large amounts of sulfur oxides (SOx) and
nitrogen oxides (NOy). When exposed to the atmosphere, these oxides react
with water to form sulfuric acid and nitric acid which are the components of
acid rain.

(i) In the atmosphere, nitric oxide (NO) is oxidised by ozone (O3) to form
nitrogen dioxide (NO2), which in turn reacts with OH radical to form
nitric acid (HNO3).

Draw dot-and-cross diagrams for NO2 molecule and OH radical.

Hence explain why they react readily to form nitric acid (HNO3).

(ii) Besides forming nitric acid, NO contributes to acid rain by being a


homogeneous catalyst in the oxidation of SO2 to SO3.

The catalysed reaction occur in two steps:

NO(g) + ½O2(g)  NO2(g) H = 57 kJ mol1

NO2(g) + SO2(g)  SO3(g) + NO(g) H = 88 kJ mol1

The activation energy of the first step, Ea1, is lower than that of second
step, Ea2.

Use the information above to construct a fully-labelled reaction pathway


diagram for this two-step reaction.

(iii) Explain the term “catalyst”. [7]

(b) A sample of carbon disulphide, CS2, reacts with excess nitrogen monoxide,
NO, to form a yellow residue, sulfur, and two gases CO2 and N2. The gaseous
products occupied a total volume of 130 cm3. However, when passed through
aqueous sodium hydroxide, the volume of the gas decreased.

All gases were measured at room temperature.

(i) Write a balanced equation for the reaction between CS2 and NO.

(ii) Calculate the mass of carbon disulphide, CS2, in the sample.

(iii) Write an equation to show why the volume of gaseous product


decreases when passed through aqueous sodium hydroxide.

(iv) Describe, in terms of orbital overlap, the bonding between sulfur and
carbon atoms in a CS2 molecule. A clearly labelled diagram may clarify
your answer. [6]

Jurong Junior College 8872/02/Prelim 2013 [Turn over


www.gradtutors.com
241
14
7. (c) Both chloroethane and iodoethane react with NH3 in ethanol under pressure to
give ethylamine.

NH3 in ethanol
CH3CH2Cl CH3CH2NH2
Heat in sealed tube
ethylamine

NH3 in ethanol
Heat in sealed tube

CH3CH2I

(i) What conditions must be used in the above reaction so that ethylamine
can be obtained as a major product?

(ii) In the above reaction with NH3, is CH3CH2Cl more or less reactive
compared to CH3CH2I?

Explain your answer. [3]

(d) Chloroethane can also be converted to propylamine.

Step I Step II CH3CH2CH2NH2


CH3CH2Cl X
propylamine
(i) State the reagents and conditions needed for Step I and II.

(ii) Give the structural formula of X. [3]

(e) Chloroethane reacts with methylamine, CH3NH2, to form compound Y, which is


an isomer of propylamine.

CH3NH2 in ethanol
CH3CH2Cl Y
Heat in sealed tube

Give the displayed formula of Y. [1]

[Total: 20]

Jurong Junior College 8872/02/Prelim 2013 [Turn over


www.gradtutors.com
242
15
8. This question is about ethanoic acid and two other naturally occurring carboxylic
acids.

(a) Some information about two of the processes used to make ethanoic acid is
given below.

Process 1:
This is a one-step process that involves the reaction of methanol with carbon
monoxide.
CH3OH + CO  CH3COOH
The conditions used are 180 °C and 30 atmospheres pressure and a
rhodium/iodine catalyst is used.
The percentage yield for this process is 99%.

Process 2:
This involves the oxidation of naphtha, a fraction obtained from crude oil.
Liquid naphtha is oxidised using air at a temperature of 180 °C and 50
atmospheres pressure. No catalyst is needed.
A large variety of other products are also formed in this oxidation.

(i) Suggest one advantage of making ethanoic acid using Process 1 rather
than Process 2.

(ii) Other products formed in Process 2 are carboxylic acids, aldehydes and
ketones.

One such product is compound L.

One mole of L, C3H4O3, forms half a mole of carbon dioxide when


reacted with aqueous sodium carbonate. L decolourises acidified KMnO4
and forms red-brown precipitate with Fehling’s solution.

Draw the structural formulae of L and the product formed when L reacts
with Fehling’s solution. [3]

(b) In gaseous state just above boiling point, the monomer and dimer forms of
ethanoic acid exist together in equilibrium.

2CH3COOH(g)  (CH3COOH)2

Draw a fully-labelled diagram to illustrate the bond formed when ethanoic acid
dimerise. [1]

Jurong Junior College 8872/02/Prelim 2013 [Turn over


www.gradtutors.com
243
16
8. (c) pKa values of some naturally occurring Bronsted-Lowry acids are shown in
Table 8.1 below.

Common name IUPAC name Structural Formula pKa value

Acetic acid Ethanoic acid CH3CO2H 4.76


(from vinegar)

pyruvic acid 2-oxopropanoic acid CH3COCO2H 2.39


(formed during
metabolism

Lactic acid 2-hydroxypropanoic CH3CH(OH)CO2H 3.86


(from milk) acid

Table 8.1

(i) What is meant by the term Bronsted-Lowry acid?

(ii) Arrange in increasing order the relative strength of the three acids in
Table 8.1.

(iii) Explain why pyruvic acid has a lower pKa than ethanoic acid.

(iv) Pyruvic acid can be converted to 2-chloropropanoic acid using the


following two-step synthesis.

O OH Cl
Step I Step II

H3C COOH H3C COOH H3C COOH


pyruvic acid 2-chloropropanoic acid

For both Step I and II, state the type of reaction occurred and the
reagents and conditions required. [7]

(d) (i) Define standard enthalpy change of neutralisation.

(ii) Explain why the standard enthalpy change of neutralisation of ethanoic


acid is less exothermic than that of HCl. [2]

Jurong Junior College 8872/02/Prelim 2013 [Turn over


www.gradtutors.com
244
17

8. (e) A student pipetted 25.0 cm3 0.250 mol dm3 of lactic acid into a conical flask.
He added NaOH(aq) solution from a burette and monitor the pH of the reaction
mixture in the conical flask using a pH meter.

The pH curve obtained by the student is shown below.

(i) Using the pKa value in Table 8.1, determine the value of Ka of lactic acid.

Hence calculate the initial pH of the lactic acid in the conical flask before
NaOH(aq) was added.

(ii) Calculate the concentration of the NaOH(aq) solution.

(iii) At point K, the solution in the conical flask contains a mixture of lactic
acid and its sodium salt. This mixture can be used as buffer.

What do you understand by the term buffer? Using a balanced equation,


show how the above buffer works when some dilute HCl is added to it.

(iv) Choose the most suitable indicator for the titration from the table below.
State the colour change of the solution at endpoint.

pH at which Acid Base


Indicator
colour changes colour colour

Congo red 3–5 blue red

Brilliant yellow 6–8 yellow orange

Thymol blue 8 – 10 yellow blue [7]

[Total: 20]

Jurong Junior College 8872/02/Prelim 2013 [Turn over


www.gradtutors.com
245
18
9. (a) P, Q and R are three structural isomers with the molecular formula of C8H8O2.

P and Q react with aqueous NaOH at room temperature.

R does not react with aqueous NaOH at room temperature but when heated
with aqueous NaOH, R forms compound S, C7H5O2Na and compound T which
has a Mr of 32.

T forms white fumes with PCl5.

When heated with acidified KMnO4, P and R oxidised to form the same
product, benzoic acid.

However, when heated with acidified KMnO4, Q formed 1,3-benzene


dicarboxylic acid.
COOH

COOH

1,3-benzene dicarboxylic acid

(i) Suggest structures of P, Q, R, S, and T. Show how you deduce these


structures and suggest the types of reactions that occur.

(ii) Write balanced equation for the following reactions:

 T reacts with PCl5.

 Q reacts with hot acidified KMnO4. [11]

(b) Phosphorus reacts with chlorine to form liquid PCl3 and solid PCl5. Unlike
phosphorus, nitrogen only forms NCl3.

(i) Explain why phosphorus can form PCl5 but nitrogen does not form NCl5.

(ii) By considering the number of bond pairs and lone pairs around P,
predict and explain bond angle in PCl3.

(iii) Explain why PCl5 is a solid but PCl3 is a liquid. [6]

(c) Student G added a few drops of water to PCl5 solid. To the same amount of
PCl5 solid, student H added excess cold water which contains a few drops of
universal indicator.

(i) Student G and H make different observations.

Describe what does student G and H see.

(ii) Write a balanced equation, with state symbols, for the reaction that
student G carried out. [3]

[Total: 20]

Jurong Junior College 8872/02/Prelim 2013 [Turn over


www.gradtutors.com
246
1
Suggested Mark Scheme for 2013 H1 Chemistry Prelim Paper 2

1. (a) (i)

+
Deflection only starts 
source when there is an electric 
field. Beam resume its 
straight path once it is out 
24
Mg2+   of the electric field. 

2.5 2
(ii) Angle of deflection =  = 4.29 o
2 / 24 14

(b) (i) Lattice energy of MgCl2

(ii) H3 = 1920 + 2(381) = 2682 kJ mol1

H1 = H3  H2 = (2326)+ (2682) = 356 kJ mol1

(iii) Lattice energy MgO is more exothermic than that of MgCl2


or magnitude of the lattice energy of MgO is greater.

This is because O2 is smaller ionic radius and has higher charge than Cl  .

2. (a) Methanoic acid is used in large excess so as to keep its concentration relatively
constant throughout the experiment so that we can determine the order of reaction
with respect to Br2 and the rate is independent of concentration of HCOOH.

(b) (t1/2)1= 175 s, (t1/2)2= 180 s

Average (t1/2)1= 177.5 s

Successive half-life approximately constant, the reaction is first order with


respect Br2.

(c) Rate = k[methanoic acid][Br2]

0.010  0.0057
(d) Initial rate = gradient = = 4.3  105 mol dm3 s1
0  100

From rate equation, 4.3  105 = k(0.500)(0.01)

k = 8.6  103 mol1 dm3 s1


Jurong Junior College 8872/02/Prelim 2013 [Turn over
www.gradtutors.com
247
2
[Br2(aq)] /mol dm–3

0.0057

(t½)1 = 175 s (t½)2 = 180 s


time /s

175 355

Jurong Junior College 8872/02/Prelim 2013 [Turn over


www.gradtutors.com
248
3
2. (e)
Volume of CO2/ cm3  reaches eqm faster  
BUT yield remain unchange 
Curve R
 faster rate   [1]

time
Figure 2.1

3. (a)
Melting point

Na Mg Al Si P S Cl
pH of solution formed

14
after adding water

9
7

Na2O MgO Al2O3 SiO2 P4O10 SO3

Mg2+ has a greater charge and a smaller radius than Na+.


(b)
Moreover, in Mg there are 2 valence electrons per Mg atom available for metallic
bonding while in Na there is only one valence electron per Na atom available for
metallic bonding.

Thus more energy is needed to overcome the stronger electrostatic attraction


between Mg2+ and its valence electrons.

(c) P4O10 + 6H2O  4H3PO4 ; MgO + H2O  Mg(OH)2

Jurong Junior College 8872/02/Prelim 2013 [Turn over


www.gradtutors.com
249
4
Haber process is an industrial process to manufacture ammonia from nitrogen.
N2(g) + 3H2(g)  2NH3(g)

4. (a) Feature 1 : rate of forward reaction = rate of backward reaction.

Feature 2 : Concentration of reactants and products remains constant when a


reaction is at equilibrium.

(b) (i) Temperature 450 oC

Pressure 250 atm

(ii) At low temperature, equilibrium position shifts to the right to favour the
exothermic reaction and thus increasing the yield of ammonia.
However, reaction is slow at low temperature. Therefore, a moderately high
temperature of about 450 C is used.

(c) (i) yield of NH3

pressure

(ii) When pressure increase, equilibrium position of N2(g) + 3H2(g)  2NH3(g)


shifts to the right to decrease/reduce the number of gaseous molecules.
Thus yield of NH3 increases.

Jurong Junior College 8872/02/Prelim 2013 [Turn over


www.gradtutors.com
250
5

0.00100
5. (a) Amount of Na2S2O3 reacted in titration =  24.60  2.46 105 mol
1000

2S2O32  I2

Amount of iodine reacted = 2.46  10-5  ½ = 1.23  10-5 mol

(b) 2Mn(OH)3 + 2I  2Mn(OH)2(s) + 2OH + I2

(c) 2Mn(OH)3  I2

amount of Mn(OH)3 reacted in Step 2 = 1.23  10-5  2 = 2.46  10-5 mol

(d) Hence calculate the amount of O2(aq) in 25.0cm3 of the sample of river water.

O2  4Mn(OH)3
amount of O2(aq) in 25.0cm3 of river water = 2.46  10-5  4 = 6.15  10-6 mol

6.15 106
(e) Concentration of O2 in mol dm-3 = 1000  2.46 104 mol dm-3
25

DOC = 2.46  10-4  2(16.0) = 0.00787 g dm-3

(f) Since density of river water is 1 g cm–3,

1000 g river water ----- 0.00787 g O2(aq)

0.00787
106 river water ------  106  7.87 g of O2(aq)
1000

DOC in ppm = 7.87

(f) Yes enough O2 because DOC > 5ppm.

6. (a) Alkene, tertiary alcohol

(b) (i) x=3 y=2

(ii) Excess concentrated H2SO4 heat (or 170oC)

Jurong Junior College 8872/02/Prelim 2013 [Turn over


www.gradtutors.com
251
6
CH3 CH3

or
6. (b) (iii)

H3C CH3 H3C CH3

(c) Oxidation product of terpinen-4-ol:


CH3

O
COOH
COOH O

H3C OH CH3

CH3 or H3C OH

CH3

Oxidation product of menthol:

CH3

H3C CH3

Jurong Junior College 8872/02/Prelim 2013 [Turn over


www.gradtutors.com
252
7
Section B

7. (a) (i) O N O O H

React readily so that N and O can achieve octet configuration .

(ii) energy /kJ mol–1

Ea,2 > Ea,1

NO + ½O2 Ea,1 Ea,2


+ SO2
–57

NO2 + SO2
–88 NO + SO3

Progress of reaction

(iii) A catalyst increases rate of reaction by providing an alternative reaction


pathway with lowered activation energy It takes part in the reaction but
remain chemically unchanged at the end of the reaction.

(b) (i) CS2 + 2NO  CO2 + N2 + 2S

Or

4CS2 + 8NO  4CO2 + 4N2 + S8

(ii) Total volume of CO2 and N2 = 130 cm3

Volume of CO2 (or N2) = 65 cm3

65
Amount of CO2 (or N2) = = 0.00271 mol
24000

CS2  N2  CO2 
Amount of CS2 in sample = 0.00271 mol

Mass of CS2 in sample = 0.00271 [2.0  2(32.1)] = 0.207 g

(iii) CO2(g) + NaOH(aq)  NaHCO3(aq)

Or

CO2(g) + 2NaOH(aq)  Na2CO3(aq) + H2O(l)

Jurong Junior College 8872/02/Prelim 2013 [Turn over


www.gradtutors.com
253
8
7. (b) (iv)  




(c) (i) Excess NH3 or limited CH3CH2X

(ii) Since Cl has a smaller atomic radius than I, C-Cl bond is shorter and hence
stronger. It requires more energy to break C-Cl bond.
Thus CH3CH2Cl is less reactive.

(d) (i) Step I: ethanolic NaCN (or KCN), heat under reflux.

Step II: LiAlH4 dry ether or H2, Ni catalyst, heat

CH3CH2CN
(ii)

(e) H H
H
H C C N
C H
H H H
H

8. (a) (i) Process 1 uses a lower pressure which means lower cost or safer method.

Or

Process 1 produces high yield of ethanoic acid while Process 2 produces a


large variety of products.

(ii) O

L : H C CH2COOH Product of L : 

(b) - + -
O: HO
CH3C C CH3
- +
OH :O-

Hydrogen bonds

Jurong Junior College 8872/02/Prelim 2013 [Turn over


www.gradtutors.com
254
9
8. (c) (i) Bronsted-Lowry acid is a proton donor.

(ii) CH3COOH < CH3CH(OH)CO2H < CH3COCO2H

(iii) Since O is more electronegative than C, ‒CO group is electron


withdrawing.

Thus the negative charge on O atom of CH3COCO2 is more dispersed than


that of CH3CO2.

Hence CH3COCO2 is more stable and CH3COCO2H is a stronger acid.

(iv) Reagents and Conditions Type of reaction

Step I H2, Ni catalyst, heat Reduction

Step II HCl, ZnCl2 catalyst, heat Substitution

(d) (i) Standard enthalpy change of neutralisation is the energy released when 1
mol of water is formed from an acid-base reaction under standard
conditions.

(ii) Ethanoic acid is a weak acid which partially dissociates.

Energy released during neutralisation is reabsorbed to dissociate ethanoic


acid completely.

Thus standard enthalpy change of neutralisation of ethanoic acid is less


exothermic.

(e) (i) Ka = 10-3.86 = 1.38 x 104 mol dm-3

[H+] = (0.25)(1.38 104 ) = 5.87  103 mol dm-3

pH = 2.23

25.0
(ii) Amount of lactic acid used =  0.25 = 6.25  103 mol
1000

Equivalence point is reached when 22 cm3 NaOH is added.

Amount of NaOH in 22 cm3 = 6.25  103 mol

6.25  103
Concentration of NaOH = 1000 = 0.284 mol dm-3
22

Jurong Junior College 8872/02/Prelim 2013 [Turn over


www.gradtutors.com
255
10
8. (e) (iii) A buffer is a solution that resists pH changes when small amount of acid
or base is added.

CH3CH(OH)CO2 + H+  CH3CH(OH)CO2H

(iv) Thymol blue

Colour change : yellow to green

Jurong Junior College 8872/02/Prelim 2013 [Turn over


www.gradtutors.com
256
11
9. (a) P, Q and R are three structural isomers with the molecular formula of C8H8O2.
 C: H = 1 : 1  presence of benzene ring in P, Q and R

P and Q react with aqueous NaOH at room temperature.


 Acid base reaction  P and Q contains –COOH

R does not react with aqueous NaOH at room temperature but when heated with
aqueous NaOH, R forms compound S, C7H5O2Na and compound T which has a Mr of
32.
 Alkaline hydrolysis  R is an ester
T is an alcohol. Since Mr of T= 32, T is CH3OH
- +
COO Na COOCH3

S R

T forms white fumes with PCl5.


  substitution

When heated with acidified KMnO4, P and R oxidised to form the same product,
benzoic acid.
  P and R are monosubstituted benzene
CH2COOH

P is

However, when heated with acidified KMnO4, Q formed 1,3-benzene dicarboxylic acid.
COOH

COOH
1,3-benzene dicarboxylic acid

R COOH

Q has the structure Q is

 R CH3

Jurong Junior College 8872/02/Prelim 2013 [Turn over


www.gradtutors.com
257
12
9. (a) (ii) CH3OH + PCl5  CH3Cl + POCl3 + HCl
COOH COOH

+ 3[O] + H2O
CH3 COOH

(b) (i) P can expand octet because it has energetically accessible vacant 3d
orbitals. Hence it forms PCl5.

N cannot expand octet because its next available vacant orbital is the 3s
orbital. Promotion of electron to another principal quantum shell requires
too much energy to be feasible.

(ii) Bond angle in PCl3 is 107o.

In PCl3, the P has 3 bond pairs and 1 lone pair.


The 4 electron pairs will arrange in a tetrahedral arrangement.

However, since lone pair-bond pair repulsion is greater than the bond pair-
bond pair repulsion, the bond angle is reduced from 109.5o to 107o.

(iii) PCl5 has a larger number of electrons per molecule.

Thus more energy is needed to overcome the stronger van der Waals’ forces
between PCl5 molecules than the weaker permanent dipole-permanent
dipole interaction between the PCl3 molecules.

As a result, PCl5 has a higher melting point.

(c) (i) Student G sees the formation of white fumes.

Student H sees the green solution turns red.

(ii) PCl5(s) + H2O(l)  POCl3(l) + 2HCl(g)

Jurong Junior College 8872/02/Prelim 2013 [Turn over


www.gradtutors.com
258
Class Adm No

Candidate Name:

2013 Promotional Examination II


Pre-University 2
H1 CHEMISTRY 8872 / 01
24 September 2013

50 minutes
Additional Materials: Multiple Choice Answer Sheet
Data Booklet

READ THESE INSTRUCTIONS FIRST

INSTRUCTIONS TO CANDIDATES

1. Do not turn over this question paper until you are told to do so.
2. Write your name, class and admission number in the spaces provided at the top of
this page and on the Answer Sheet provided.
3. Answer ALL questions and shade the correct answers on the Answer Sheet provided
using a soft pencil.
4. No extra time will be given for shading.
5. Hand in the question paper and the Answer Sheet separately.

INFORMATION FOR CANDIDATES


Marks will not be deducted for wrong answers; your total score will be the number of correct
answers given.

FOR EXAMINER’S USE

TOTAL (30 marks)

This question paper consists of 12 printed pages and 2 blank pages.

[Turn over
www.gradtutors.com
259
2

Answer all questions on the OMR form provided (30 Marks)


Section A
For each question, there are four possible answers, A, B, C and D.
Choose the one you consider correct.

1. Which of the following have the same number of particles as 32.0 g of oxygen gas?
A 2.0 g of hydrogen gas
B 11.5 g of sodium metal
C 24.0 g of carbon
D 35.5 g of chlorine gas

2. 10 cm3 of a hydrocarbon CxHy was exploded with 100 cm3 of oxygen. After cooling to
room temperature, the resultant gaseous mixture has a volume of 80 cm3. When the
resultant gaseous mixture was treated with a solution of potassium hydroxide, the
volume of the gaseous mixture decreased to 40 cm3. What is the molecular formula of
the hydrocarbon?
[All gas volumes are measured at r.t.p.]
A C3H6
B C3H8
C C4H8
D C4H10

3. The metallic ion, Mx+, is oxidised to MO4- by acidified K2Cr2O7 solution. A sample of
25.0 cm3 of 0.280 mol dm-3 aqueous Mx+ requires 23.30 cm3 of 0.200 mol dm-3 acidified
K2Cr2O7 solution for complete reaction. What is the value of x in Mx+?
[Cr2O72- + 14H+ + 6e¯ 2Cr3+ + 7H2O]
A 1
B 2
C 3
D 4

www.gradtutors.com
260
3

4. Which of the following species in their ground states have three unpaired electrons?
A Si
B S
C Fe3+
D Cr3+

5. Which of the following species contains ionic, covalent and dative bonding within itself?
A Al2Cl6
B NH4SO4
C H2SO4
D H3O+

6. Which of the following series of substances does not show a decreasing trend in
boiling point?
A BaCl2 > BCl3 > BF3
B CH3I > CH3Br > CH3Cl
C AsH3 > PH3 > NH3
D CH3COONa > CH3COOH > CH3CH2OH

7. Which of the following reactions has a negative enthalpy change of reaction?


A KOH(aq) + HCl(aq) → KCl(aq) + H2O(l)
B H2O(l) → H2O(g)
C NaF(s) → Na+(g) + F (g)
D Cl2(g)  2Cl(g)

[Turn over
www.gradtutors.com
261
4

8. Dinitrogen oxide, N=N=O, reacts with ethyne, H-C≡C-H, in the gaseous phase to
produce water vapour, carbon dioxide and nitrogen gases as the only products.
5N2O (g) + C2H2 (g)  H2O (g) + 2CO2 (g) + 5N2 (g)
The enthalpy change of the reaction is -1710 kJ mol-1.
Using appropriate information from the Data Booklet, calculate the N=O bond energy,
in kJ mol-1, in dinitrogen oxide.
A 390
B 686
C 784
D 1370

9. The enthalpy change of formation of aluminium oxide and copper(II) oxide is


-1676 kJ mol-1 and -155 kJ mol-1 respectively. What is the enthalpy change of the
following reaction?
2Al(s) + 3CuO(s)  Al2O3(s) + 3Cu(s)
-1
A -1521 kJ mol
B -1211 kJ mol-1
C +1211 kJ mol-1
D +2141 kJ mol-1

10. For the following equilibrium:


3Fe (s) + 4H2O (g) Fe3O4 (s) + 4H2 (g)
Which is the correct expression for the equilibrium constant, Kc?
3 4
[Fe] [H 2O]
A 4
[Fe 3O 4] [H 2]
4
[Fe3O4] [H 2]
B 3 4
[Fe] [H2O]
4
[H 2O]
C 4
[H 2]
4
[H 2]
D 4
[H 2O]

www.gradtutors.com
262
5

11. Consider the following equilibrium:


PCl5 (g) PCl3 (g) + Cl2 (g) ∆H > 0
Which of the following statements is correct?
A Increasing the temperature will decrease the yield of Cl2.
B Increasing the pressure will decrease the yield of Cl2.
C The equilibrium constant has a unit of mol2 dm-6.
D The presence of a catalyst will increase the yield of Cl2.

12. A compound Z is formed during a gas phase reaction. The graphs below show how the
percentage of Z present at equilibrium varies with temperature and pressure.

%Z %Z

Temperature Pressure

Which of the following responses concerning the equilibrium is correct?


No. of gaseous particles in product
Enthalpy change of reaction
No. of gaseous particles in reactant
A Exothermic >1
B Exothermic <1
C Endothermic >1
D Endothermic <1

13. What is the pH of a solution prepared by dissolving 5.0 g of NaOH in 500 cm3 of water?
A 0.6
B 3.6
C 10.4
D 13.4

[Turn over
www.gradtutors.com
263
6

14. The following data was obtained from the studies of the reaction between O2 and N2O5
in a vessel at constant temperature.

Experiment 1 2 3
Initial total volume of O2 and N2O5 / cm3 1.00 1.60 2.00
3
Initial volume of O2 / cm 0.60 1.20 1.20
3 -1
Initial rate of reaction / cm s 2.06 4.12 16.48

Which of the following statements is correct regarding the above system?


A The rate equation is rate = k [O2]2.
B The overall order of the reaction is three.
C The rate constant k has units of cm-3s-1.
D The reaction is first order with respect to N2O5.

15. In the diagram below, curve I was obtained from the decomposition of 250 cm3 of
2.0 mol dm-3 hydrogen peroxide in the presence of manganese(IV) oxide as a catalyst
at 25 C.
Volume of O2
collected / cm3

II
I

Time / s

Which alteration to the original experiment would produce curve II?


A Using more manganese(IV) oxide.
B Increasing the temperature.
C Adding 50 cm3 of 1.0 mol dm-3 hydrogen peroxide.
D Adding 200 cm3 of water.

www.gradtutors.com
264
7

16. W
Which of the following sttatements a
about the ra
ate constantt of a chemiical reaction
n is not
tru
ue?
A Increasse in concen
ntration of rreactants wiill increase the rate connstant.
B The ratte constant decreases when the activation
a en
nergy of thee reaction is
increassed.
C The ratte constant of different reactions can
c have diffferent unitss.
D The ratte constant increases w
when a cata
alyst is used
d.

17. he graph sh
Th hows how a property off the elements Na to Cl
C varies acrross the perriod.

W
What is the property?
p
A Electriccal conductivity
B Atomic radius
C Melting
g point
D Electronegativity

18. A mixture of an oxide and


a a chlor ide of elem
ments in Period 3 is addded to water. The
esulting solu
re ution has a pH of 2.
W
What could be
b the cons
stituents off the mixturre?
A Na2O and
a MgCl2
B NaCl and Na2O
C SiCl4 and SiO2
D Al2O3 and
a NaCl

[Tu
urn over
www.gradtutors.com
265
8

19. Methyl butanoate, CH3OCOCH2CH2CH3, is responsible for the smell of pineapples.


What are the products formed when methyl butanoate undergoes alkaline hydrolysis?
A CH3COOH and CH3CH2CH2OH
B CH3CH2CH2COOH and CH3OH
C CH3CH2CH2COO- and CH3O-
D CH3CH2CH2COO- and CH3OH

20. Which of the following reagents can be used to distinguish between compounds R and
S shown below?
O H O H H

C C O C C C OH
HO
C C C HO H H
H
H H H
Compound R Compound S
A Na2CO3
B PCl5
C LiAlH4 in dry ether
D acidified K2Cr2O7

21. 0.02 mol of alkene F requires 1.92 dm3 of gaseous HBr for complete reaction at
room temperature and pressure. How many double bonds are present in one
molecule of F?
A 2
B 3
C 4
D 5

www.gradtutors.com
266
9

22. Which of the following is not correct for compound A?

CHO H

C C
H

O C CH3

O
compound A
A It decolourises aqueous bromine.
B It can be reduced by NaBH4 in ethanol.
C It reacts with Fehling’s solution to give a brick-red ppt.
D It reacts with hot ethanolic KOH.

23. Three different halogenoalkanes, R, S and T were separately hydrolysed in hot


aqueous alkali, followed by treatment with acidified silver(I) nitrate solution. The mass
of the precipitate formed was collected and weighed. The following graph was
obtained.

mass of precipitate /g
R

Time /min

Which of the following sets gives the possible identities of compounds R, S and T?
R S T
A CH3CH(CH3)CH2F CH3CH(CH3)CH2Cl CH3CH(CH3)CH2Br
B CH3CH(CH3)CH2Cl CH3CH(CH3)CH2Br CH3CH(CH3)CH2F
C CH3CH(CH3)CH2Br CH3CH(CH3)CH2F CH3CH(CH3)CH2Cl
D CH3CH(CH3)CH2Br CH3CH(CH3)CH2Cl CH3CH(CH3)CH2F

[Turn over
www.gradtutors.com
267
10

24. How many cis-trans isomers would be possible for the following molecule?
CH3
CH3CH CHCH2CH2C CHCH3

A 2
B 4
C 6
D 8

25. Which of the following compounds could undergo an elimination reaction when treated
with hot ethanolic sodium hydroxide?
A C(CH3)3OH
B
Cl

C CH2=CHOH
D CH3Br

Section B
For each of the questions in this section, one or more of the three numbered statements 1 to
3 may be correct.
Decide whether each of the statements is or is not correct (you may find it helpful to put a
tick against the statements that you consider to be correct).

The responses A to D should be selected on the basis of


A B C D
1, 2 and 3 are 1 and 2 only are 2 and 3 only are 1 only is correct
correct correct correct

26. The boiling point of ethylamine, CH3CH2NH2, is 16.6°C.


Which bond(s) is/are broken when ethylamine is boiled?
1 Covalent bonds
2 Hydrogen bonds
3 Temporary dipole-induced dipole interactions

www.gradtutors.com
268
11

The responses A to D should be selected on the basis of


A B C D
1, 2 and 3 are 1 and 2 only are 2 and 3 only are 1 only is correct
correct correct correct

27. A metal hydroxide dissolves partially in water as shown:


M(OH)2 (s) + aq M2+ (aq) + 2OH– (aq) H > 0
Which of the following statements is/are true as temperature increases?
1 pH increases
2 More M(OH)2 (s) dissolves.
3 Equilibrium is reached at a faster rate

28. Which of the following series of species show(s) an increase in the radii?
1 Ca2+ < K+ < Ar
-
2 Cl < S2- < P3-
3 Na < Mg < Al

29. 2-methylbuta-1,3-diene is used to make synthetic rubber. The structure of the molecule
is shown.
CH3
CH2 C CH CH2

2-methylbuta-1,3-diene

Which of the following statements about 2-methylbuta-1,3-diene is/are not correct?


1 It has two C atoms which are sp2 hybridised.
2 It reacts with hot acidified K2Cr2O7 to give CO2.
3 It can exhibit cis-trans isomerism.

[Turn over
www.gradtutors.com
269
12

The responses A to D should be selected on the basis of


A B C D
1, 2 and 3 are 1 and 2 only are 2 and 3 only are 1 only is correct
correct correct correct

30. Which of the following synthetic routes can produce ethanoic acid?

2 alcoholic KCN dilute H+


CH3Br
reflux heat

END OF PAPER

www.gradtutors.com
270
13

BLANK PAGE

[Turn over
www.gradtutors.com
271
14

BLANK PAGE

www.gradtutors.com
272

MCQ
1 A 7 A 13 D 19 D 25 B
2 C 8 B 14 B 20 A 26 C
3 C 9 B 15 C 21 C 27 A
4 D 10 D 16 A 22 D 28 B
5 B 11 B 17 A 23 D 29 A
6 C 12 C 18 C 24 B 30 A

www.gradtutors.com
273
2
 

2013 MJC H1 Prelim Paper 1 QP and Answers


1 The polyoxometallate W72Mn12Si7O26840- was recently identified in a crystal
garden.

Which row gives oxidation states of tungsten and manganese that are
consistent with this formula?

oxidation state oxidation state of


of tungsten manganese
A +3 +2
B +3 +3
C +6 +2
D +6 +3

2 Copper (II) ions react with iodide ions to release iodine as shown in the
following equation.

2Cu2+ + 4I-  2CuI + I2

The iodine released can be determined by titration using a standardised


solution of sodium thiosulfate. The equation for this reaction is shown below,

2S2O32- + I2  S4O62- + 2I-

25.00 cm3 of a solution containing copper (II) ions was treated with excess
iodide ions. The resulting iodine required 26.50 cm3 of sodium thiosulfate of
concentration 0.150 mol dm-3 for complete reaction.

What was the concentration of copper (II) ions in the solution used?

A 0.0795 mol dm-3


B 0.0912 mol dm-3
C 0.159 mol dm-3
D 0.318 mol dm-3

©MJC Chemistry 
www.gradtutors.com
274
3
 

3 In a reaction, 0.20 mol of an oxide, NxOy was reacted with 20 dm3 of hydrogen
gas at room temperature and pressure and passed over a heated catalyst. At
the end of the reaction, 0.80 dm3 of hydrogen gas remained. The ammonia
produced required 0.200 mol of sulphuric acid for complete neutralisation.

The reaction of the oxide with hydrogen can be represented by the following
equation:

NxOy (g) + 3x  2y H (g)  x NH (g) + y H2O (l)


2 3
2

What is the molecular formula of the oxide?

A NO
B NO2
C N2O
D N2O4

4 Which compound is composed of a cation and anion(s) that do not contain


the same number of electrons as each other?

A LiH
B NaOH
C NH4F
D TiCl3

5 The ionic radius of the elements in Period 3 (Na to Cl) changes across the
period.

Which of the following statements does not explain the trend of the ionic
radius across the period?

A Across the period, there is an increase in the nuclear charge.


B The anions have greater shielding effect than the cations.
C Screening effect is approximately constant among the cations.
D The effective nuclear charge decreases from P3- to Cl-.

 
www.gradtutors.com
275
4
 

6 Which of the following statements describe a phenomenon which cannot be


explained by hydrogen bonding?

A Ice floats on water.


B The boiling point of methanol is lower than that of ethanol.
C Ethanal dissolves in water.
D Ethanoic acid molecules form dimers when dissolved in benzene.

7 A covalent molecule contains

 14 electrons,
 one lone pair of electrons,
 two  bonds.

What is the molecule?

A C2H4
B HCN
C H2O2
D N2

8 Titanium occurs naturally as the mineral rutile, TiO2. One possible method of
extraction of titanium is to reduce the rutile by heating with carbon.

TiO2(s) + 2C(s)  Ti(s) + 2CO(g) ∆H = +720 kJ mol-1

Given that the enthalpy change of formation of CO(g) is -110 kJ mol-1, what is
enthalpy change of formation of TiO2(s)?

A -830 kJ mol-1
B +830 kJ mol-1
C -940 kJ mol-1
D +940 kJ mol-1

 
www.gradtutors.com
276
5
 

9 The curvee Y and thhe value E a represent the disttribution oof energies
s of the
moleculess and the activation e
energy for an
a uncataly
ysed gase ous reactioon.

What is a possible outcome if tthe reactio


on is catalysed?

A The distribution
d n of energ ies will be
e given by curve X aand the ac
ctivation
energ
gy by value
e V.
B The distribution
d n of energ ies will be
e given by curve Y aand the ac
ctivation
energ
gy by value
e V.
C The distribution
d n of energ ies will be
e given by curve Y aand the ac
ctivation
energ
gy by value
e W.
D The distribution
d n of energ ies will be
e given by curve Z aand the ac
ctivation
energ
gy by value
e W.

10 Two mole es of comp pound J w


were placedd in a ves ssel. The ccompound
d J was
partly deco
omposed byb heating . A dynamic equilibrium betweeen chemica
als J, K
and L wass establishe
ed.

At equilibrrium, x mol of L were


e present and
a the total numberr of moles present
was (2+x). What is thhe equatioon for this equilibrium
e ?

A J 2K + L

B 2J 2K + L

C 2J K+L

D 2J K + 2L
L

 
www.gradtutors.com
277
6
 

11 Bromine is formed by the reaction between bromate(V) ions and bromide ions
and acid:

BrO3- (aq) + 5Br - (aq) + 6H+ (aq)  3Br2 (aq) + 3H2O (l)

The results of some experiments on the reaction are shown below.

Concentration / mol dm-3 Initial rate /


Experiment
BrO3 -
Br-
H + mol dm-3 s-1
1 0.10 0.10 0.10 1.2 x 10-3
2 0.10 0.30 0.10 3.6 x 10-3
3 0.20 0.10 0.10 2.4 x 10-3
4 0.20 0.10 0.20 9.6 x 10-3

Which statement is true based on the above data?

A The overall order of reaction is three.


B The rate of reaction is independent of [BrO3-].
C The unit of the rate constant is mol-3 dm9 s-1.
D The time taken for the concentration of Br- to decrease to half its initial
value is the same for Experiment 1 to 4. 
12 Citric acid is a monobasic acid.

Which of the following pair of solutions, when mixed at equal volume, will give
a best buffer solution that resists a pH change when a small amount of acid or
base is added?

A 0.5 mol dm-3 of citric acid and 0.5 mol dm-3 of NaOH
B 0.5 mol dm-3 citric acid and 0.125 mol dm-3 of Ba(OH)2
C 1.0 mol dm-3 sodium citrate and 0.25 mol dm-3 of H2SO4
D 1.0 mol dm-3 sodium citrate and 0.5 mol dm-3 of H2SO4

 
www.gradtutors.com
278
7
 

13 A dilute solution of hydrogen peroxide decomposes slowly in aqueous solution


according to the following equation:
2H2O2(aq) → 2H2O(l) + O2(g)
A solution with an original concentration of 3.00 mol dm-3 was placed in a
bottle contaminated with transition metal ions, which act as catalyst for the
decomposition. The rate of decomposition was measured by withdrawing
10.0 cm3 portion at various times and titrating with acidified 0.05 mol dm-3
KMnO4(aq).
The following results were obtained.

Volume of KMnO4(aq) / cm3


30

25

20

15

10

0 Time / min
0 5 10 15 20 25 30

How long has the solution in the bottle been contaminated before the first
portion was withdrawn for titration? Assume that 5 mol H2O2 reacts with 2 mol
KMnO4.
A 14 minutes

B 42 minutes

C 84 minutes

D 112 minutes

 
www.gradtutors.com
279
8
 

14 Which of the following statement is correct for the sequence of compounds


below considered from left to right?

NaF MgO AlN SiC

A The bonding becomes decreasingly covalent.


B The formula-units of these compounds are not isoelectronic.
C Electronegativity difference between the elements in each compound
decreases.
D When water is added to these compounds, the resultant solution
becomes increasingly acidic.

15 An element X in Period 3 is a metalloid and semiconductor and has a chloride


which reacts with water to form an acidic solution.

An element Y has an atomic number one less than element X.

What is a property of the oxide of element Y?

A It is a gas at room temperature.


B It is amphoteric.
C It is covalent.
D Its formula is YO2.

16 The information relates to element Z.

 Z is in Period 3 of the Periodic Table.


 Z has a lower electrical conductivity than Mg.
 A Z atom has half-filled subshell in its ground state.
 Z forms an acidic oxide on exposure to air.

What is Z?

A Na
B Si
C P
D Cl

 
www.gradtutors.com
280
9
 

17 Ethoxyethane, C2H5OC2H5, is the most well known of a class of compounds


called ethers. Ethers have the general formula R1–O–R2 where R1 and R2 are
hydrocarbon groups.

How many structural isomers does ethoxyethane have?

A 2
B 3
C 6
D 7
 
18 Cholesterol is the most common steroid alcohol. It has a molecular formula of
C27H46O and has the structure shown.

How many carbon atoms are in the hydrocarbon group R?

A 7
B 8
C 9
D 10

19 Experiments are carried out on three compounds X, Y, and Z.

CH3(CH2)16CO 2 CH2

CH3(CH2)16CO 2 CH BrCH2CH2CH2Br
CH3(CH2)16CO 2 CH2 CN

X Y Z
A sample of 0.01 mol of each compound is heated under reflux with 100 cm3
of 0.5 mol dm-3 NaOH (in excess) until hydrolysis is complete and any
ammonia produced is expelled from solution. The excess NaOH is then
titrated in each case and is found to require 40 cm3, 60 cm3 and 80 cm3 of 0.5
mol dm-3 HCl for neutralisation.

 
www.gradtutors.com
281
10
 

Which sequence of compounds matches these results?

40 cm3 60 cm3 80 cm3

A X Y Z

B X Z Y

C Y X Z

D Z Y X

20 1 mole of an organic compound E undergoes elimination on reaction with


ethanolic sodium hydroxide to form 2 moles of HBr.

What could E be?

A   Br
CH2CHCH3
Br

B   CH3
Br C CH2CH2Br
CH3

C   Br
CH2Br

D   CH3
BrCH2 C CH2Br
CH3

 
www.gradtutors.com
282
11
 

21 Compound M has the structural formula as shown below.

CH(CN)OCOCH3

Compound M

Which of the following statement regarding M is correct?

A 1 mole of M reacts with hot dilute HCl to produce 2 moles of carboxylic


acid.
B M reacts with 2,4-dinitrophenylhydrazine give an orange precipitate.
C M reacts with I2 in NaOH to produce a yellow precipitate.
D M is a product of an addition reaction.

22 The diagram shows a reaction scheme.

Which statement about this reaction scheme is not correct?

A Step 1 involves addition.


B Step 2 involves hydrolysis.
C P forms a yellow precipitate with alkaline aqueous iodine.
D P has the molecular formula C3H4O4.

 
www.gradtutors.com
283
12
 

23 Terephthalaldehyde is used as an intermediate for manufacturing of dyes and


fluorescent whitening agents. It can be synthesised from 4-methylbenzoic acid
via a 3-step synthesis.

3 steps

4-methylbenzoic acid Terephthalaldehyde

Which of the following are the reagents and conditions needed for the
conversion?

Step 1 Step 2 Step 3


A Cr2O72-/H+, heat to distill LiAlH4, r.t.p Hot KMnO4/H+
B Cr2O72-/H+, heat to distill NaBH4, r.t.p Hot KMnO4/H+
C Hot KMnO4/H+ LiAlH4, r.t.p Cr2O72-/H+, heat to distill
D Hot KMnO4/H+ NaBH4, r.t.p Cr2O72-/H+, heat to distill

24 An industrial preparation of alcohols involves the following step.

H H H H
catalyst
C C + CO + H2 H C C H
H H H CHO

What is the likely structure of compound X produced by the reaction below?


(D = 21H )

H CH2CH3
catalyst
C C + CO + D2 X
H3C H

 
www.gradtutors.com
284
13
 

H CH2CH3
H3C C C H

A H CHO  

H CH2CH3
H3C C C D

B H CDO  

H CH2CH3
H3C C C H

C D CHO  

H CH2CH3
H3C C C H

D D CDO  
 

25 Sucrose is highly soluble, as shown in the graph, and it dissolves according to

C12H22O11(s) C12H22O11(aq) ∆H = +4.75 kJ mol-1

Sucrose Solubility
600
Solubility (g/ 100g H2O)

500

400

300

200

100

0
40 50 60 70 80 90 100
o
Temperature / C

A cup of coffee was made with 200 cm3 of water at 80oC and as much sugar
as would dissolve. It was left to cool to 60oC.

 
www.gradtutors.com
285
14
 

During cooling from 80oC to 60oC,

A the fraction of [C12H22O11(aq)]/[C12H22O11(s)] remains unchanged.


B the system responds to the change in temperature by absorbing more
heat.
C the rate of forward reaction increases as the system can release heat
more easily.
D the position of equilibrium shifts left as it is an endothermic reaction.

26 Most elements contain more than one isotope, atoms that have the same
chemical properties but different masses.

Which expressions correctly define the term relative atomic mass of an


element?

1 The mass of 1 mole of atoms of an element relative to the mass of 1


mole of 12C atoms.
2 The mass of 1 atom of an element relative to the mass of 1 atom of
12
C.
3 The mass of 1 mole of atoms of an element divided by 6.02 x 1023.

27 Given that,
(CH3)3N + CH3COOH (CH3)3NH+ + CH3COO-

Which of the following statements are not correct?

1 CH3COOH acts as a Bronsted acid.


2 (CH3)3N is the conjugate base of CH3COOH.
3 (CH3)3N and CH3COOH can act as a buffer pair to resist pH changes.

28 Hydrogen reacts with gaseous bromine to form hydrogen bromide,

H2(g) + Br2(g) 2HBr(g)

and with gaseous iodine to form hydrogen iodide

H2(g) + I2(g) 2HI(g)

For the first reaction, the rate equation is

 
www.gradtutors.com
286
15
 

k1[H2 ][Br2]1.5
rate =
[Br2 ]k2[HBr]

For the second reaction, the rate equation is

rate = k[H2][I2]

What can be deduced from this information only?

1 For the hydrogen/ bromine reaction, the formation of HBr slows down
the rate of the forward reaction.
2 Only the hydrogen/ iodine reaction could be a single step reaction.
3 The overall order of reaction for the hydrogen/ bromine reaction is 2.

29 Compound X has the following structure.

OH
O O
HO
H3C CH3 O OH

Which of the following statements about Compound X are incorrect?

1 Compound X reacts with 2,4-dinitrophenylhydrazine to form an orange


precipitate.
2 One mole of Compound X completely reacts with three moles of
Na2CO3.
3 One mole of Compound X completely reacts with three moles of KOH
at room temperature and pressure.

30 Which pair of organic compounds cannot be distinguished by a chemical test?

CH3

and
1  

CH2CH3 C(CH3)3

and
2

 
www.gradtutors.com
287
16
 

O O

C C
and O

3 O  

 
www.gradtutors.com
288

Name: ___________________ Class: ________ Reg Number: _____

MERIDIAN JUNIOR COLLEGE


JC2 Preliminary Examination
Higher 1

_____________________________________________________________________

Chemistry 8872/02
Paper 2 17 September 2013
2 hours
Additional Materials: Data Booklet
Writing Papers
_____________________________________________________________________
READ THESE INSTRUCTIONS FIRST

Write your name, class and register number in the spaces at the top of this page.

This booklet contains Section A and Section B.

Section A : Pg 2 to 13

Answer all questions in Section A in the spaces provided on the question paper. You are advised to spend
about 1 h on Section A.
Examiner’s Use

Section B : Pg 14 to 20 Paper 1 MCQ / 30 m

Answer 2 out of 3 questions in Section B. You are / 33 %


advised to spend about 1 h on Section B.
Paper 2 Q1 / 12 m
Hand in Section B separately from Section A. Section A
Q2 / 17 m
Fasten your answers for Section B behind the given
Cover Page. Detach the Cover Page from the last page
Q3 / 11 m
behind this booklet.
Paper 2
/ 40 m
Section B
INFORMATION FOR CANDIDATES
Paper 2 / 80 m
The number of marks is given in brackets [ ] at the end Total / 67 %
of each question or part question.
Grand
/ 100 %
Total
Grade

_____________________________________________________________________
This document consists of 20 printed pages (including this cover page)

©MJC Chemistry
www.gradtutors.com
289

Section A : Structured Questions


Answer all the questions in this section in the spaces provided.

1 An early Periodic Table devised by Mendeleev listed the elements known then in
order of their relative atomic mass.

(a) (i) When Mendeleev created the table, there were uncertainties regarding the
relative atomic mass of tellurium. It is now known that there are eight
isotopes of tellurium. Complete the following table. Give your answers to
four significant figures.

isotope percentage abundance isotopic mass x


percentage abundance
tellurium-120 0.09 11.00

tellurium-122 2.46 300.0

tellurium-123 0.87 107.0

tellurium-124 4.61 572.0

tellurium-125 6.99 874.0

tellurium-126 18.71 2357

tellurium-128 31.79

tellurium-130

Hence, calculate the relative atomic mass of tellurium to 1 decimal place.

[Turn Over
www.gradtutors.com
290

(ii) The electronic configuration of tellurium and antimony are as follows:

Te: [Kr]5s25p4
Sb: [Kr]5s25p3

Explain why the first ionisation energy of tellurium is lower than that of
antimony.

……………………………………………………………………………………..…

………………………………………………………………………………..………

………………………………………………………………………………….…[4]

(b) Tellurium (IV) chloride, TeCl4, exists as a vapour at 380°C. Gaseous TeCl4 has a
structure similar to sulfur tetrafluoride, SF4.

(i) Draw a dot-and-cross diagram of TeCl4.

(ii) Predict, with reasoning, the shape of TeCl4.

………………………………………………………………………………..………

………………………………………………………………………………..………

……………………………………………………………………………………..…

……………………………………………………………………………………..…

………………………………………………………………………………..………

(iii) The compound TeCl4 exists, but OCl4 does not.


By considering the possible types of bonding in the two compounds,
suggest a reason for this difference. (Assume that Te and O atoms occupy
a central position in each of these molecules.)

………………………………………………………………………………..………

…………………………………………………………………………..……………

…………………………………………………………………………….………[4]

[Turn Over
www.gradtutors.com
291

(c) Tellurium resembles silicon in many aspects. For example, both are metalloids and
are used as semiconductors. Tellurium (IV) chloride, TeCl4, dissolves in water in
the same way as silicon (IV) chloride, SiCl4.

Describe the reaction, if any, of the chlorides of aluminium and tellurium with
water. Give equations for any reactions and suggest the pH values of the resulting
solution.

…………………………………………………………………………………..……………

……………………………………………………………………………………………..…

…………………………………………………………………………………..……………

……………………………………………………………………………………………..…

…………………………………………………………………………………..……………

……………………………………………………………………………………………..…

…………………………………………………………………………………..……………

……………………………………………………………………………………………..…

…………………………………………………………………………………..……………

……………………………………………………………………………………………..…

…………………………………………………………………………………..……………

…………………………………………………………………………………………….[4]

[Total : 12]

[Turn Over
www.gradtutors.com
292

2 Steel is an interstitial alloy, as carbon atoms fit in


between iron atoms in the lattice. Brass is a substitution
alloy, as zinc atoms replace the copper atoms in the
lattice. Stainless steel is a combination of interstitial and
substitution alloys, because carbon atoms fit into the
interstices, but some of the iron atoms are replaced with interstitial substitution
nickel atoms.

The alloy’s structure will determine properties such as hardness, which measures
the material’s resistance to permanent shape change when a force is applied.
These properties rely on how much electrostatic attraction is present between ions
and electrons in any given volume of the lattice.

q
Lattice Strength  
r

(a) The ionic radius and ionic charge of the following element are given in the
following table.

Element C V Cr Fe Ni Cu Zn W
Ionic radius (10-12 m) 16 64 62 64 60 73 74 66
Ionic charge +4 +3 +3 +3 +3 +2 +2 +4

(i) Adding a small amount of which metallic element is likely to increase the
lattice strength of iron most? Give a reason for your answer.

…………………………………………………………………………………......…

……………………………………………………………………………………..…

(ii) From the table above, suggest another element that can be added to iron to
form a substitution alloy.

…………………………………………………………………………………….[2]

(b) The hardness of iron is enhanced when carbon atoms are added to iron atoms in
the lattice. Explain how adding carbon atoms enhance the hardness of iron in
steel.

…………………………………………………………………………………………..……

…………………………………………………………………………………………..……

…………………………………………………………………………………………….[1]

[Turn Over
www.gradtutors.com
293

(c) Addition of a minimum 13% chromium by mass into steel will form
stainless steel which is resistant to oxidation. Suggest how adding
chromium helps to prevent oxidation of the metal.

…………………………………………………………………………………………..……

…………………………………………………………………………………………….[1]

(d) With the aid of a labelled diagram, describe the structure and bonding in copper.

…………………………………………………………………………………………..……

…………………………………………………………………………………………..……

.…………………………………………………………………………………………....[2]

[Turn Over
www.gradtutors.com
294

(e) The diagram below is a Pourbaix diagram of iron. Any point on the Pourbaix
diagram will give most stable form of the element for that electrode potential, E o
and pH. The lines in the Pourbaix diagram show the equilibrium conditions where
the activities are equal for the species on each side of that line.

A positive value of Eo denotes a strongly oxidising environment where iron is


oxidised. For example, at pH 14 and Eo 0.8 V, Fe will be oxidised to FeO42-.

Pourbaix diagrams are useful in predicting the predominant species of that


element in different environments.

Electrode
potential, Eo/ V

FexO4
HFeO2-

(i) Use the Pourbaix diagram to answer the following questions:

1 Which species of iron is the strongest oxidising agent?

………………………………………………………………………………

2 What is the range of the Eo values when oxidation of Fe(OH)2 to


FexO4 take places from pH 9 to 14?

………………………………………………………………………………...

[Turn Over
www.gradtutors.com
295

(ii) A lake containing FeO42- (aq) is very well aerated with oxygen at a pH
of 5. However, due to continuous NaOH discharge from a
chemical industry, reddish brown sediments of Fe2O3 is observed in
the lake.

Write a half-equation to explain the observation above.

[3]

(f) Oxides of iron can exist in several forms: FeO, Fe2O3 or FexO4.

A sample of FexO4 is oxidised to Fe3+ (aq) using 10.0 cm3 of 0.0860 mol dm-3 of
acidified KMnO4 solution. 25.85 cm3 of 0.500 mol dm-3 potassium iodide solution
was needed to reduce all the Fe3+ (aq) to Fe2+ (aq).

(i) Calculate the number of moles of Fe3+.

(ii) Hence, or otherwise, calculate the value of x in FexO4.

The following mole ratio is given: KMnO4 ≡ 5 FexO4

[Turn Over
www.gradtutors.com
296

(iii) From your answer in (f)(ii), the average oxidation number of an iron atom in
FexO4 is not a whole number. Hence, suggest the oxidation states of the iron
atoms in FexO4.

…………………………………………………………………………………….[4]

(g) Cations of some of the metals such as Fe and Cu can act as catalysts for specific
reactions. Using the concept of activation energy together with an appropriate
sketch of the Maxwell-Boltzmann distribution, explain how the catalysis reaction
increases the rate of reaction.

…………………………………………………………………………………………..……

…………………………………………………………………………………………..……

…………………………………………………………………………………………..……

…………………………………………………………………………………………..……

.………………………………………………………………………………………...….[4]

[Total: 17]

[Turn Over
www.gradtutors.com
297

10

3 The spice cinnamon has been used as a medicine for many hundreds of years.
However, recent research has suggested that cinnamon could also be an effective
pesticide against the larvae of mosquitoes, thus helping in the fight against
malaria.

Each of the following four compounds, which are present in cinnamon, appears to
be effective as a pesticide.

(a) Only one of the compounds A, B or C will react with each of the following reagents
under suitable conditions. In each case, identify the compound concerned and
draw the structural formula of the organic product formed.

Each compound may be used once, more than once, or not at all.

Compound Structural formula of the


Reagent
A, B or C organic product

Na2CO3 (aq)

2,4-DNPH

[Turn Over
www.gradtutors.com
298

11

Compound Structural formula of the


Reagent
A, B or C organic product

Tollens’
reagent

[5]

(b) Draw the structural formulae of the organic compounds formed when

(i) compound B is heated with aqueous iodine in sodium hydroxide;

(ii) compound C is heated under reflux with hot acidified concentrated


manganate (VII) ions.

[2]

[Turn Over
www.gradtutors.com
299

12

(c) Compound E may be synthesised from compound A via a two-stage process as


shown below.

(i) What is the structural formula of the intermediate, D, in this conversion?

(ii) For each step, in the reaction sequence, give reagent(s) and conditions.

Step I
Reagent(s): …………………………………………………………………………

Conditions: ………………………………………………………………………….

Step II
Reagent(s): …………………………………………………………………………

Conditions: …………………………………….………………………………...[3]

[Turn Over
www.gradtutors.com
300

13

(d) Unlike compound B, glucose is very soluble in water.

Glucose

Draw a labelled diagram to show the interaction between a molecule of glucose


and a molecule of water. You can illustrate your answer on the sketch given below.

[1]

[Total: 11]

End of Section A

[Turn Over
www.gradtutors.com
301

14

Section B : Free Response Questions


Answer two of the following three questions.
Answer these questions on separate answer papers.

4 Gasoline is the petroleum fraction used as motor car fuel. Gasoline additives are
often added to increase its octane rating and to reduce pollution. One such
additive is liquid ethyl-tertiary-butyl ether (ETBE).

CH3

CH3 C O

CH3 CH2 CH3

Ethyl-tertiary-butyl ether (ETBE)

(a) Define standard enthalpy change of formation. [1]

(b) While the enthalpy change of formation of ETBE cannot be determined


experimentally, the enthalpy change of combustion of ETBE can be determined
using a bomb calorimeter. The experimental set-up is as shown.

thermometer electrical supply

gas outlet

wire
insulation jacket cotton
sample boat

bomb calorimeter
water

A known volume of water was added and ETBE was introduced into the sample
boat. A spark was produced when the electrical supply was switched on. The
temperature was recorded at every 1 minute interval for 20 minutes. At the end of
the experiment, the apparatus was allowed to cool.

The following results were recorded:

Mass of sample boat + ETBE (start of experiment) / g 108.01


Mass of sample boat + ETBE (after leaving to cool) / g 102.51
Volume of water heated / cm3 500
Initial temperature / °C 28.0
Final maximum temperature / °C 95.5

The instrumentation manual of the calorimeter states that the efficiency factor is
70%.

[Turn Over
www.gradtutors.com
302

15

Calculate the enthalpy change of combustion of ETBE. Assume that the specific
heat capacity of water is 4.18 J g-1 K-1. Express your answers to the appropriate
number of significant figures.
[4]

(c) Using your answer in part (b), the energy cycle below and the data provided,
calculate the enthalpy change of formation of ETBE.

1
6C (s) + 7H2 (g) + O2 (g) C6H14O (l)
2
O2 (g) O2 (g)
6O2 (g)

1
6CO2 (g) + 7H2O (l) + O2 (g)
2
Data provided: ΔHf(CO2) = -393 kJ mol-1, ΔHf(H2O) = -286 kJ mol-1
[1]

(d) When gasoline burns in an internal combustion engine, there are some
undesirable exhaust gases emitted such as nitrogen monoxide. Briefly explain how
nitrogen oxide is formed in an internal combustion engine and state the
environmental damage it causes. [2]

(e) Compound K is a structural isomer of ETBE. K produces a yellow precipitate with


warm alkaline iodine solution and produces effervescence with sodium metal.
When K is heated under reflux conditions with concentrated sulfuric acid, two
isomeric compounds L and M are obtained. Upon reaction with hot acidified
potassium manganate (VII), L produces N, C4H8O. N gives a precipitate when
treated with 2,4-dinitrophenylhydrazine but not with Tollen’s reagent.

(i) Deduce the structures of K, L, M and N. Explain the chemistry of the


reactions involved.

(ii) State the type of structural isomerism between ETBE and K.


[7]

The oil refineries and petrochemical industry demand high performance refractory
products. Magnesium and aluminium oxides are commonly used as refractory materials
as they are both physically and chemically stable at high temperatures.

(f) The acid-base behavior of aluminium oxide, Al2O3, shows similarities to that of
magnesium oxide, MgO, on the one hand, and phosphorus (V) oxide, P4O10, on
the other.

Describe what these similarities are, and explain why aluminum oxide occupies
this in-between position. Write equations for all the reactions you choose to
illustrate your answer.
[5]

[Total: 20]

[Turn Over
www.gradtutors.com
303

16

5 The “OXO” reaction (reaction I in the scheme below) is industrially important for
synthesis of alcohols, aldehydes and carboxylic acids. For example, butan-1-ol,
butanal and butanoic acid can all be synthesised from propene, C3H6, according to
the following scheme.

(a) Write an expression for Kc for reaction I, stating its units.


[2]

(b) 10 g of H2 is mixed with C3H6 and CO in a 1: 1: 1 mole ratio in a vessel of 800 cm3
at 500 K. At equilibrium, the concentration of butanal is found to be 3.00 mol dm -3.

(i) Calculate the equilibrium concentrations of C3H6, CO and H2.

(ii) Hence, calculate the value for Kc at 500 K.

(iii) Hence, draw a fully labelled graph for the reaction above, showing how the
concentration of the reactants and product change from the time the gases
are mixed to the time the reaction reaches equilibrium at time t.

(iv) An approximate value for ∆H can be calculated using bond energy values.
Taking the bond energy for the CO bond in carbon monoxide to be
1077 kJ mol-1, and using other appropriate bond energies given in the Data
Booklet, calculate the value of ∆H for reaction I.

(v) Suggest, with a reason, whether a high temperature would favour the
formation of butanal.
[8]

(c) (i) State the reagents and conditions needed for


 reaction II,
 reaction III.

(ii) State the reagents and conditions needed in the conversion of butan-1-ol
into
 1-chlorobutane
 2-chlorobutane (in two steps)
[5]

[Turn Over
www.gradtutors.com
304

17

(d) The compound 2-chlorobutane is a useful intermediate for making other organic
compounds.

(i) It forms butan-2-ol by heating with NaOH (aq).

How would you expect the rate of this reaction to compare to that of the
reaction of 2-iodobutane with NaOH (aq)? Explain your answer.

(ii) State, with the help of relevant equations, how 2-chlorobutane could be
distinguished by a simple chemical test from butan-2-ol through a positive
test for 2-chlorobutane.
[4]

(e) When butane is treated with a small quantity of chlorine in the presence of
ultraviolet light, both 1-chlorobutane and 2-chlorobutane are produced. Suggest
the approximate ratio in which they are formed.
[1]

[Total: 20]

[Turn Over
www.gradtutors.com
305

18

6 Lactic acid (2-hydroxypropanoic acid), CH3CH(OH)CO2H, is present in milk. It is a


chemical compound that plays a role in several biochemical processes. A
chemistry student conducted the following experiment in the laboratory.

(a) 25.0 cm3 of an aqueous solution of lactic acid is titrated against 0.120 mol dm 3 of
aqueous sodium hydroxide and the following graph is obtained.

13.00

12.00

11.00

10.00

9.00

8.00
pH

7.00

6.00

5.00

4.00

3.00

2.00
0 10 20 30 40
Volume of aq NaOH added/ cm3

[Turn Over
www.gradtutors.com
306

19

(i) Calculate the concentration of the lactic acid used.

(ii) Determine the initial concentration of H+ dissociated from lactic acid and
hence calculate the acid dissociation constant of lactic acid.

(iii) The student is provided with some acid-base indicators.

Indicator pH range at which colour change occurs


Methyl violet 0.5 – 1.5
Chlorophenol red 4.8 – 6.4
Alizarin Yellow 10.1 – 12.0

His classmate suggested that they could use chlorophenol red to detect the
end-point of this titration.

Explain whether chlorophenol red is a suitable indicator for this titration.

(iv) Explain, with the aid of equations, how a mixture of lactic acid and its
conjugate base, CH3CH(OH)COO– acts as a buffer on the addition of small
amounts of acid and bases. [7]

(b) When lactic acid is refluxed with a catalytic amount of concentrated sulfuric acid, a
neutral compound D with the molecular formula of C6H8O4 is formed. Compound D
does not react with sodium. Deduce the structural formula of D, giving reasons for
your answer. [3]

(c) E is an isomer of lactic acid. It reacts with Fehling’s solution to give a brick red
precipitate. 0.01 mol of E reacts with sodium to give 0.24 dm3 of hydrogen gas
under room temperature and pressure conditions. One mole of E reacts with two
moles of PCl5.

(i) Suggest a structural formula of E, giving reasons for your answer.

(ii) Explain why lactic acid is more acidic than E.


[6]

(d) Maleic acid and fumaric acid are naturally occurring acids found in fruits. They
have the following features:
 Same molecular formula of C4H4O4
 Are dibasic acids
 Are cis-trans isomers of each other
 Fumaric acid has a higher boiling point than maleic acid

(i) What do you understand by the term cis-trans isomers?

[Turn Over
www.gradtutors.com
307

20

(ii) The following are the structural formula of maleic acid and fumaric acid.

H H HOOC H
C C C C
HOOC COOH H COOH

maleic acid fumaric acid

Using bonding and structure, account for the differences in the boiling points
observed.
[4]

[Total: 20]

End of Section B

[Turn Over
www.gradtutors.com
308
21

Cover Page for


`` Section B

Name: _____________________ Class: _________ Reg Number: _____

Meridian Junior College


2013 JC2 Preliminary Examinations

H1 Chemistry 8872

17 September 2013

INSTRUCTION TO CANDIDATES

Write your name, class and register number in the spaces provided at the top of
this page.

Circle the two questions that you have attempted

Fasten your answers for Section B behind this Cover Page.

You are advised to spend about 1 h for Section B.

INFORMATION FOR CANDIDATES

The number of marks is given in brackets [ ] at the end of each question or part
question.

You are reminded the need for good English and clear presentation in your
answers.

Examiner’s Use

Q4 / 20 m

Section B Q5 / 20 m

Q6 / 20 m

Total / 40 m
[Turn Over
www.gradtutors.com
309
1

2013 MJC H1 Suggested Answers


 
Section A: Structured Questions

1(a) (i) Percentage abundance of Te-130


= 34.48

Isotopic mass x % abundance of Te-128


= 4069

Isotopic mass x % abundance of Te-130


= 4482

Relative atomic mass of Te


= 127.7

(ii) There is inter-electron repulsion between the paired electrons in the 5p


orbital of Te.

(b) (i) xx xx

Cl x Cl
x
xx

xx

xx
Te xx
x x

Cl Cl
xx

xx

xx
xx

xx xx

(ii) There are 4 bond pairs and 1 lone pair around Te atom.
To minimise repulsion, the 5 electron pairs are directed to the corners
of a trigonal bipyramid.
Since lone pair-bond pair repulsion > bone pair-bond pair repulsion, the
shape of TeCl4 is distorted tetrahedral.

(iii) Te has empty and energetically assessible 5d orbitals to accept


electrons and expands its octet structure.

(c) TeCl4 undergoes complete hydrolysis to form strongly acidic solutions


with pH= 2

TeCl4 + 2H2O → TeO2 + 4HCl

AlCl3 undergoes hydration and slight hydrolysis to form an acidic


solution with pH=3.0

AlCl3 (s) 


H O
Al3+ (aq) + 3Cl- (aq)
2

[Al(H2O)6]3+ [Al(H2O)5(OH)]2+ + H+

©MJC Chemistry
www.gradtutors.com
310
2

q
2(a) (i) W. It has the highest  value
r
(ii) V (or Cr or W)

(b) Adding carbon atoms into the space between the iron atoms in the
lattice will prevent the iron atoms from sliding over each other easily
and hence do not go out of shape easily.

(c) Cr forms an oxide layer and prevents oxygen from reacting with the
iron.

(d) Metallic bonds are strong electrostatic forces of attraction between the
cations and sea of delocalised electrons in a giant metallic lattice
structure.

delocalised
+ + + + e electrons
e e e
+ + + +
e metal cations
+ + + +
+ +
e + +
e
e e
2-
(e) (i) FeO4
-0.4 to -0.6V

(ii) 6e + 5H2O + 2FeO42-  Fe2O3 + 10OH

(f) (i) No of moles of I-= 1.293 x 10-2


No of mole of Fe3+ = 1.29x 10-2

(ii) x = 3.

(iii) +2 and +3

©MJC Chemistry
www.gradtutors.com
311
3

(g)

Ea is lowered due to an alternative reaction pathway.

Number of reactant particles with E ≥ Ea increases.

Frequency of effective collisions increases.

Rate of reaction is proportional to frequency of effective collisions.

3(a)
Compound Structural formula of the
Reagent
A, B or C organic product
CH=CHCH3

Na2CO3 (aq) C

COO-Na+

©MJC Chemistry
www.gradtutors.com
312
4

O2N
NO2

H
N

A N
2,4-DNPH
CH=CHC
H

C
CH=C O-
Tollens’ A H
reagent

(b)(i)

CHI3

(ii) COOH

COOH

©MJC Chemistry
www.gradtutors.com
313
5

(c)(i)

(ii) Step I
HCN Conditions: trace amount of NaOH or NaCN, cold
Step II
Dilute H2SO4 heat

(d)

CHO

CHOH H
-
- + O
CHOH
H
CHOH Hydrogen bonding

CHOH

CH2OH

Section B: Free Response Questions C

4(a) Standard enthalpy change of formation is the energy change when one
mole of product is formed from its elements at 298K and 1 atm.

(b) Hc (ETBE) = - 3.75 x 103 kJ mol-1

(c) - 610 kJ mol-1

(d) In the internal combustion engine, where the temperature is very high,
nitrogen react with oxygen in the air to form nitrogen oxide. Nitrogen
oxides cause acid rain.

(e)(i) K has a molecular formula of C6H14O


K undergoes oxidation with warm alkaline iodine. K contains the
structure CH3-CH(OH)

©MJC Chemistry
www.gradtutors.com
314
6

K undergoes acid-metal displacement with sodium. K is an alcohol


K undergoes elimination to give alkenes L and M.
L undergoes oxidation with acidified KMnO4 to produce N. N is a
carbonyl compound.
Since N undergoes condensation with 2,4-DNPH, but does not
undergo oxidation with Tollen’s reagent,N is not an aldehyde or is a
ketone

OH CH3

H CH3
H3C C C CH2CH3
H3C C C CH2CH3
H H

K L

CH3 H 3C

C O
H3CH2C C C CH2
CH 3 CH 2
H H

M N

(ii) Functional group isomerism.

(f) MgO is a basic oxide.


MgO + 2H+ Mg2+ + H2O

P4O10 is an acidic oxide.


P4O10 + 12OH- 4PO43- + 6H2O

Al2O3 is an amphoteric oxide.


Al2O3 + 6H+ 2 Al3+ + 3H2O

Al2O3 + 2OH- + 3H2O 2[Al(OH)4]-

The in-between behavior of Al2O3 is due to the high charge density of


Al3+ ion polorising the O2- anion hence resulting in partial covalent
nature of Al-O interaction.

[CH3CH2CH2CHO]
5(a) Kc  Unit: mol-2 dm6
[CH3CH  CH2 ][CO][H2 ]

©MJC Chemistry
www.gradtutors.com
315
7

(b)(i)
CH3CH=CH2 + CO + H2  CH3CH2CH2CHO
Initial conc / mol dm-3 6.25 6.25 6.25 0
Change in conc / mol dm-3 -3 -3 -3 +3
Final conc / mol dm-3 3.25 3.25 3.25 3.00

(ii) 0.0874 mol-2 dm6

(iii)
concentration

6.25

reactant
3.25
product
3

time
t

(iv) -137 kJ mol-1

(v) By Le Chatelier’s Principle, an increase in temperature would cause the


equilibrium position to shift to the left towards the endothermic reaction
to absorb heat.

The formation of butanal is not favoured.

(c) (i) Reaction II: KMnO4, H2SO4 (aq), heat


Or K2Cr2O7, H2SO4 (aq), heat

Reaction III: LiAlH4 in dry ether, r.t.p.


Or NaBH4 in ethanol, r.t.p.

(ii) Reagents & conditions: PCl5, r.t.p. (absence of water)

Reagents & conditions for step 1: Conc H2SO4 at 180 oC

Reagents & conditions for step 2: HCl gas, r.t.p.

©MJC Chemistry
www.gradtutors.com
316
8

(d) (i) Bond strength: C-Cl bond > C-I bond .Bond energy: C-I < C-Cl

(ii) Heat each mixture with aqueous NaOH followed by the addition of
dilute HNO3 and AgNO3(aq).If a white ppt of AgCl is formed, the
compound is 2-chlorobutane

CH3CH(Cl)CH2CH3 + NaOH CH3CH(OH)CH2CH3+ Na+ + Cl-

Ag+ + Cl- AgBr

(e) 3 : 2.

6(a) (i) 0.0960 mol dm3

(ii) 1.72 x 104 mol dm3

(iii) No. Since this is a strong base – weak acid titration, the pH at
equivalence point is basic. The pH transition range of chlorophenol red
does not lie within the range of rapid pH change over the equivalence
point.

(iv)

When a small amount of acid, H+ is added:

CH3CH(OH)COO- + H+  CH3CH(OH)COOH

The added H+ is removed as CH3 CH(OH)COOH. Hence pH remains fairly


constant

When a small amount of base, OH- is added:

CH3CH(OH)COOH + OH -  CH3CH(OH)COO- + H2O

The added OH- is removed as CH3CH(OH)COO- and H2O. Hence pH remains


fairly constant

©MJC Chemistry
www.gradtutors.com
317
9

(b) D does not undergo acid-metal displacement with Na hence does not contain an alcohol
or carboxylic acid group.
Lactic acid undergoes self- esterification to form D with no of C atoms doubled
hence D is a diester.

O C
C
H
3H
C O

O C
HC
C O

H
3
(c) (i) E undergoes oxidation with Fehling’s solution. E is an aliphatic
aldehyde.

E Ξ H2 There are two alcohol groups.


One mole of E undergoes substitution with two moles of PCl5.
There are two alcohol groups.
H C O
H

O H

E is
C O

C
H

H
H

(ii) In the carboxylate anion, the negative charge is delocalised over the 2
O atoms. Thereby stabilising the carboxylate ion relative to lactic acid

(d) (i) Cis-trans isomerism arises when rotation of a double bond is restricted
due to the presence of  bond It also arises since different substituent
groups are bonded to each C atom of the C=C bond.

(ii) Due to the proximity of the two –COOH groups, cis-somer is capable of
intramolecular hydrogen bonding. Hence the cis-isomer possesses less
extensive intermolecular hydrogen bonding.

Since more energy is required to overcome the more extensive


hydrogen bonds between the trans-butenedioic acid molecules, Trans-
butenedioic acid has a higher boiling point.

©MJC Chemistry
www.gradtutors.com
318
1

NATIONAL JUNIOR COLLEGE


SH2 PRELIMINARY EXAM
Higher 1

CANDIDATE
NAME

SUBJECT REGISTRATION
CLASS NUMBER

CHEMISTRY 8872/01
Paper 1 Multiple Choice
Thursday 19 Sept 2013
Additional Materials: Multiple Choice Answer Sheet
50 minutes
Data Booklet

READ THESE INSTRUCTIONS FIRST

Write in soft pencil.

Do not use staples, paper clips, highlighters, glue or correction fluid.

Write your name, subject class and registration number on the Answer Sheet in the spaces
provided unless this has been done for you.

There are 30 questions on this paper. Answer all questions. For each question there are four
possible answers A, B, C and D.

Choose the one you consider correct and record your choice in soft pencil on the separate
Answer Sheet.

Read the instructions on the Answer Sheet very carefully.

Each correct answer will score one mark. A mark will not be deducted for a wrong answer.
Any rough working should be done in this booklet.

This paper consists of 13 printed pages and 1 blank page.

NJC/H1 Chem/01/2013 [Turn over

www.gradtutors.com
319
2

Section A

For each question there are four possible answers A, B, C, and D. Choose the one you
consider to be correct.

1 Use of the Data Booklet is relevant to this question.

A typical solid fertiliser for use with household plants and shrubs contains the elements
N, P, and K in the ratio of 15 g: 30 g : 15 g per 100 g of fertiliser. The recommended
usage of fertiliser is 14 g of fertiliser per 5 dm3 of water.

What is the concentration of nitrogen atoms in this solution?

A 0.03 mol dm–3


B 0.05 mol dm–3
C 0.42 mol dm–3
D 0.75 mol dm–3

2 In which species are the numbers of electrons and neutrons equal?


A
18
8 O 2

19
B 9 F

C
23
11 Na 

9
D 4 Be

3 Which one of the following represents the configuration of the three electrons of
highest energy for the ground state of an element in group III?

A 1s2 2s1
B 1s1 2s1 2p1
C 2s1 2p2
D 4s2 4p1

NJC/H1 Chem/01/2013 [Turn over

www.gradtutors.com
320
3

4 A pure hydrocarbon is used in bottled gas for cooking and heating. When 10 cm3 of the
hydrocarbon is burned in 70 cm3 of oxygen (an excess), the final gaseous mixture
contains 30 cm3 of carbon dioxide and 20 cm3 of unreacted oxygen. All gaseous
volumes were measured under identical conditions.

What is the formula of the hydrocarbon?

A C2H6
B C3H6
C C3H8
D C4H10

5 The ions P3–, S2– and Cl– have radii 0.212nm, 0.184nm and 0.181nm respectively.
Which one of the following correctly explains the decreases in radius going from P 3– to
Cl–?

A Increases in both the total number of electrons and the nuclear charge
B An increase in the total number of electrons with the nuclear charge remaining
constant
C A constant total number of electrons and an increase in the nuclear charge
D An decrease in the total number of electrons with the nuclear charge remaining
constant

6 Which of the following molecules and ions does not have a bond angle of 120o around
the central atom(s)?

A AlCl3
B PH3
C CH3+
D N2O4

7 In which process are hydrogen bonds broken?

A 2H2O(l) → 2H2(g) + O2(g)


B NH3(aq) + H2O(l) → NH4+(aq) + OH‒(aq)
C CH3CH2OH(l) → CH3CH2OH(g)
D CH3COOH(aq) + NaOH(aq) → CH3COONa(aq) + H2O(l)

NJC/H1 Chem/01/2013 [Turn over

www.gradtutors.com
321
4

8 Which of the following statements about lattice structures is correct?

A The lattice energy of an ionic crystal tends to be large if the sizes of the two types
of ions involved are small.
B The particles occupying lattice sites may be atoms or ions but not molecules.
C The forces holding the lattice together are so strong that the particles occupying
lattice sites are held stationary.
D The lattice energy of an ionic crystal is always larger than that of a metal.

9 The gecko, a small lizard, can climb up a smooth glass window. The gecko has
millions of microscopic hairs on its toes and each hair has thousands of pads at its tip.
The result is that the molecules in the pads are extremely close to the glass surface on
which the gecko is climbing.
What is the attraction between the gecko’s toe pads and the glass surface?

A co-ordinate bonds
B covalent bonds
C ionic bonds
D van der Waals’ forces

10 The equation below represents the combination of gaseous atoms of non-metal X and
of hydrogen to form gaseous X2H6 molecules.
2 X(g) + 6 H(g) → X2H6(g) ∆H = –2775kJmol–1
The bond energy of an X–H bond is 395kJmol–1.
What is the bond dissociation energy of an X–X bond?

A –405.0 kJmol–1
B –202.5 kJmol–1
C +202.5 kJmol–1
D +405.0 kJmol–1

NJC/H1 Chem/01/2013 [Turn over

www.gradtutors.com
322
5

11 Two moles of compound P were placed in a vessel. The compound P was partly
decomposed by heating. A dynamic equilibrium between chemicals P, Q and R was
established.

At equilibrium, x mol of R were present and the total number of moles present was
(2 + x).

What is the equation for this equilibrium?

A P 2Q+R

B 2P 2Q+R

C 2P Q+R

D 2P Q+2R

12 The table gives the concentrations and pH values of the aqueous solutions of two
compounds, X and Y. Either compound could be an acid or a base.

X Y

Concentration / mol dm–3 10-4 10-2

pH 6 9

Student P concluded that X is a strong acid.


Student Q concluded that the extent of dissociation is lower in X(aq) than in Y(aq).

Which of the students are correct?

A both P and Q
B neither P nor Q
C P only
D Q only

NJC/H1 Chem/01/2013 [Turn over

www.gradtutors.com
323
6

13 The curve Y and the value Ea represent the distribution of energies of the molecules
and the activation energy for an uncatalysed gaseous reaction.

What is a possible outcome if the reaction is catalysed?

A The distribution of energies will be given by curve Y and the activation energy by
value V.
B The distribution of energies will be given by curve Y and the activation energy by
value W.
C The distribution of energies will be given by curve X and the activation energy by
value W.
D The distribution of energies will be given by curve Z and the activation energy by
value V.

14 The following data is obtained during the first order thermal decomposition of
2A(g)  B(g) + C(s)
at constant volume and temperature.
Time Total pressure in Pascal
At the end of 10 minutes of reaction 600
After completion of reaction 400

What is the rate constant for this reaction in min1?

A cannot be determined due to insufficient data


B 0.0346
C 0.0693
D 0.138

NJC/H1 Chem/01/2013 [Turn over

www.gradtutors.com
324
7

15 An enzyme is completely denatured at 50 °C. A fixed concentration of this enzyme is


added to a fixed concentration of its substrate. The time taken for completion of the
reaction is measured at different temperatures.

Which graph shows the results?

16 The information relates to element Z.

 Z is in Period 3 of the Periodic Table.


 Z has a lower electrical conductivity than Mg.
 A Z atom has a half-filled subshell in its ground state.
 Z forms an acidic oxide on exposure to air.

What is Z?

A Na

B Si

C P

D Cl

NJC/H1 Chem/01/2013 [Turn over

www.gradtutors.com
325
8

17 One of the reactions taking place in a catalytic converter in a car exhaust system is
between nitrogen oxide and octane (unburned petrol). The products of this reaction are
non-toxic.

Which is the correct equation for the reaction?

A C8H16 + 16 NO → 8 CO + 8 N2 + 8 H2O
B C8H16 + 24 NO → 8 CO2 + 12 N2 + 8 H2O

C C8H18 + 17 NO → 8 CO + 8 ½ N2 + 9 H2O

D C8H18 + 25 NO → 8 CO2 + 12 ½N2 + 9 H2O

18 The products obtained by cracking an alkane, M, are methane, ethene and propene.
The mole fraction of ethene in the products is 0.5.
What is the identity of M?
A C6H14
B C8H18
C C9H20
D C11H24

19 What is the total number of structural isomers for an organic compound, C4H9F?
A 3
B 4

C 5

D 6

20 Use of the Data Booklet is relevant to this question.


In which reaction is the relative molecular mass of the organic product the largest?
A bromoethane + aqueous sodium hydroxide
B bromoethane + alcoholic sodium hydroxide

C ethane + bromine

D ethanol + phosphorus pentachloride

NJC/H1 Chem/01/2013 [Turn over

www.gradtutors.com
326
9

21 Compound X

 has the molecular formula C10H14O;


 is unreactive towards mild oxidising agents.

What is the structure of the compound formed by dehydration of X?

22 An organic substance S reacts with sodium to give hydrogen, but does not react with
2,4-dinitrophenylhydrazine and does not react with ethanol in the presence of
concentrated sulfuric acid.

What can be deduced from this information?

A S is a carboxylic acid.
B S is a ketone.
C S is an alcohol.
D S is an alkane.

23 Acrylic acid is produced from propene, a gaseous product of oil refineries.

Which statement about acrylic acid is not correct?

A Both bond angles x and y are approximately 120 .

B It decolourises aqueous bromine.

C It gives an orange precipitate with 2,4-dinitrophenylhydrazine reagent.


D It reacts with an alcohol to give an ester.

NJC/H1 Chem/01/2013 [Turn over

www.gradtutors.com
327
10

24 Fumaric acid can be converted in to oxaloacetic acid by a two-step process involving the
intermediate Q.

Each of these steps can be achieved in the laboratory by a single reagent.

What could be the intermediate Q and the reagent for step 2?

Q Reagent for step 2


A HO2CCH(OH)CH2COOH Warm acidified K2Cr2O7
B HO2CCHBrCH(OH)COOH Warm NaOH(aq)
C HO2CCH(OH)CH2COOH Fehling’s solution
D HO2CCHBrCH2COOH Warm acidified KMnO4

25 Santonin is a drug that was once widely used to expel parasitic worms from the body.

Santonin

When santonin is first treated with warm dilute H2SO4, followed by cold acidified KMnO4,
X is obtained.

How many atoms of hydrogen in each molecule of X can be displaced with sodium
metal?

A 2 B 4 C 5 D 6

NJC/H1 Chem/01/2013 [Turn over

www.gradtutors.com
328
11

Section B

For each of the questions in this section, one or more of the three numbered statements 1 to
3 may be correct. Decide whether each of the statements is or is not correct (you may find it
helpful to put a tick against the statements that you consider to be correct.)

The responses A to D should be selected on the basis of

A B C D

1,2, and 3 are 1 and 2 only are 2 and 3 only are


1 only is correct
correct correct correct

No other combination of statements is used as a correct response.

26 The first ionisation energies of twenty successive elements in the Periodic Table are
represented in the graph.
The letters given are not the normal symbols for these elements.

first ionisation
energy
/kJ mol1

Which statements about the elements and their compounds in this graph are correct?

1 Elements B, J and R are in Group 0 of the Periodic Table.

2 The second ionisation energy of K is larger than the second ionisation energy of
L.

3 Oxide and hydroxide of element M are amphoteric.

NJC/H1 Chem/01/2013 [Turn over

www.gradtutors.com
329
12

27 When light is shone for a fixed period into a solution of CH2ICH2I and I2 in
tetrachloromethane at 100oC, the following reaction occurs:

CH2ICH2I  CH2=CH2 + I2

Relative initial concentration Relative initial


Relative light
experiment rate of formation
C2H4I2 I2 intensity
of I2
1 2 1 4 2
2 2 1 1 1
3 1 1 4 1
4 1 2 4 1

What conclusions about the reaction can be drawn from the data in the table?

1 The rate of formation of iodine is proportional to initial concentration of C2H4I2.


2 The rate of formation of iodine is proportional to (light intensity).
3 Iodine is not involved in the rate determining step.

28 The equilibrium constant Kp for the reaction


X(g) + Y(g) Z(g)
varies with temperature as shown in the diagram below.

Which conclusions can be drawn from this information?

1 The equilibrium mixture contains a greater proportion of Z at higher


temperatures.
2 The equilibrium mixture contains a greater proportion of Z at higher pressures.
3 The activation energy for the forward reaction is smaller than the activation
energy for the backward reaction.

NJC/H1 Chem/01/2013 [Turn over

www.gradtutors.com
330
13

The responses A to D should be selected on the basis of

A B C D
1,2, and 3 are 1 and 2 only are 2 and 3 only are
1 only is correct
correct correct correct

No other combination of statements is used as a correct response.

29 Which pairs of reagents will react together in a redox reaction?

1 CH4 + Cl2

2 CH3CH2CHO + Fehling’s reagent

3 CH3CH2COCH3 + Tollens’ reagent

30 The compound shown is a hormone produced during pregnancy to suppress ovulation.

CH3
C O
CH3

CH3

HO

Which statements are correct?

1 When reacted with alkaline aqueous iodine, a yellow precipitate is observed.

2 When reacted with LiAlH4, the product formed has 3 carbon atoms converted
from sp2 hybridised to sp3 hybridised.
3 When reacted with an excess of hot acidified K2Cr2O7, the product formed does
not react with aqueous with aqueous Br2.

– End of Paper–

NJC/H1 Chem/01/2013 [Turn over

www.gradtutors.com
331
14

SH2 Prelim Paper 1 Answers

1 A 11 B 21 D

2 A 12 B 22 C

3 D 13 A 23 C

4 C 14 C 24 A

5 C 15 C 25 D

6 B 16 C 26 A

7 C 17 D 27 A

8 A 18 B 28 B

9 D 19 B 29 B

10 D 20 C 30 D

NJC/H1 Chem/01/2013 [Turn over

www.gradtutors.com
332
2013 NJC H1 Prelims Paper 2 Answers

1(a) Cr3+: 1s22s22p63s23p63d3


Mn2+: 1s22s22p63s23p63d5

(b) (i) CH3CH2OH CH3CH(OH)CH3

(ii) I: Orange to green


II: No. of moles of K2Cr2O7 =

= 0.160
III: No. of moles of C3H6O =

=0.3
C3H6O is the limiting reagent.

Theoretical mass of C3H6O produced = 0.3 x (36+6+16)


= 17.4g

% yield =

= 30%

(iii) I: Impurity – carboxylic acid


Reason – Chemical shift of 9.0-13.0ppm represents the presence
Of R-COOH group.

II: CH3CH2OH

www.gradtutors.com
333
2013 NJC H1 Prelims Paper 2 Answers

2(a) Order: 1st order


Explanation: The same amount of time taken gives twice the
concentration for curve A when compared to curve B.

(b)
(c)

(d) Reaction between I- and S2O82- has high activation energy as they are
of the same charge and energy is required to overcome the repulsive
forces.

3(a)

www.gradtutors.com
334
2013 NJC H1 Prelims Paper 2 Answers

(b) (1450+736+2x150+76+642) – (2x349)

= -2506 kJmol-1

(c) | |

Since rCl- > rF-, therefore the lattice energy of magnesium fluoride is more
exothermic than magnesium chloride.
Hence more energy is required to break the ionic bond between
magnesium ion and fluoride ion.

4(a) C4H8O

(b) (i) Test 3: alkene

(ii) Test 4: alcohol

(c) (i) Carbonyl

(ii) 1 alcohol

(d) Free rotation about the bond is restricted by the presence of


multiple bonds.
The carbon atoms on both ends of the C=C double bond contain
two non-identical groups.
(e)
CH2OH
H3C CH2OH H
C C C C
H H H3C H
Cis trans

www.gradtutors.com
335
2013 NJC H1 Prelims Paper 2 Answers

5(a) (i) -2

(ii) A: HCHO
B: HCOOH
C: CO2

(iii) A: K2Cr2O7, dilute sulfuric acid and heat with distillation

B: K2Cr2O7 or KMnO4 dilute sulfuric acid and heat

C: Heat with excess hydrogen or KMnO4 dilute sulfuric acid and


heat

(b) (i) Q50%= 100 x 4.18 x 30


= 12 540J

nethanol = 1.5/46 = 0.0326 moles

ΔH=-Q100%/nethanol
= -769 kJmol-1

(ii) There will be heat lost to the surrounding.


(c) (i) Kc = [CH3OH]/[H2]2[CO] mol-2dm6

(ii) Kc = (0.8/1.5) / (0.4/1.5)2(0/4/1.5)


= 56,25
(iii) [2(436)+1077]-[3(410)+360+460]
ΔH = -101 kJmol-1

(d) (i) D: CH3CH(CH3)CH2OH Primary


(ii) E: CH3CH2CH2CH2OH Primary
F: CH3CH2CH(OH)CH3 Secondary
G: (CH3)3COH Tertiary
(iii) F
(iv) F

6(a) The total number of protons and neutron in the nucleus


(b) Deflection to the positive plate
The angle of deflection is greater.
(c) (i)

O ...N.. O O ...O... O
xx xx xx xx
x x x x x x x
x x x x x x x
xx

(ii) Bond angle of O3 is between 110° to 120° because lone pair-bond


pair replusion is greater than bond pair-bond pair replusion.

The bond angle of NO2 is between the bond angle of O3 to 170° as


a single unpaired electron will be less effective in diminishing the
basic angle, and so the bond angle should be greater than that for
O3.

www.gradtutors.com
336
2013 NJC H1 Prelims Paper 2 Answers

(iii) Chlorine is able to expand its octet of electrons through the 3d


orbital which is accessible for electron occuptation.
Fluorine cannot expand its octer and is unable to form dative bond
as it is too electronegative.
(d) (i) For magnesium to react with oxygen it is necessary to apply heat,
ideally by holding a piece of the metal in tongs in an atmosphere of
oxygen, such as in a gas jar.

Magnesium burns on heating with a brilliant white flame.


(ii) NaCl: dissolves but do not under go hydrolysis
MgCl2: undergoes partial hydrolysis
AlCl3: Undergoes hydrolysis
SiCl4: Undergoes hydrolysis and release HCl
PCl5: Undergoes hydrolysis and release HCl
(e) (i) No. of moles of S2O32- = 0.0015
No. of moles of I2 = 0.00075
(ii) No. of moles of O3 = 0.00075
Volume of O3 = 16.8 cm3
% of O3 present = 3.36%
(f) (i) Aldehyde: K2Cr2O7, dilute H2SO4, heat with immediate distillation
Carboxylix acid: K2Cr2O7/KMnO4 dilute H2SO4, heat

(ii) H H
H H C H CH3
C C C
H H H
C C
CH3 H H HO CH3
L M N

7(a) (i) NaCN, dilute H2SO4, trace amount of NaCN


(ii) Dilute H2SO4, heat
(b) (i) Assuming lactic acid is a strong acid, pH = -lg[H+] = 0.699

Since the pH of lactic acid is 1.7, it will be a weak acid as it is only


partially dissociated.
(c) (i) It is a solution that will resist pH changes when small amount of
acid or alkali is added.

CH3CH(OH)COOH + OH- → CH3CH(OH)COO- + H2O


CH3CH(OH)COO- + H+ → CH3CH(OH)COOH
(ii) Phenolphthalein, colourless to pink
(d) (i) Ester
(ii) Hydrolysis
(e) (i) NaOH(aq), heat
(ii) O O
C C C O C C C C C O C C
Cl Cl
K L

www.gradtutors.com
337
2013 NJC H1 Prelims Paper 2 Answers

The COCl is more reactive than C-Cl due to extra electronegative


oxygen in COCl will polarise the C-Cl bond to a greater extent
which make it easier to break and hence more reactive.

(f) 1) Test: Na2CO3(aq)


Observation: N – effervescence
M – no effervescence

2) Test: 2, 4-dinitrophenylhydrazine
Observation: N – no orange precipitate
M – orange precipitate

3) Test: [Ag(NH3)2]+
Observation: N – no silver mirror
M – silver mirror

4) Test: Alkaline solution of copper(II) tartarate


Observation: N – Blue solution
M – reddish brown precipiate

www.gradtutors.com
338

NATIONAL JUNIOR COLLEGE


PRELIMINARY EXAMINATION
Higher 1

CANDIDATE
NAME

SUBJECT REGISTRATION
CLASS NUMBER

CHEMISTRY 8872/02
Paper 2 Structured Questions and Wednesday 5 Sep 2013
Free Response Questions 2 hours
Additional Materials: Writing Paper
Data Booklet

READ THE INSTRUCTION FIRST For Examiner’s Use


Write your subject class, registration number and name on all the
work you hand in. 1 /13
Do not open this booklet until you are told to do so.
2 /9
Write in dark blue or black pen on both sides of the paper. Section
You may use a soft pencil for any diagrams, graphs or rough A
working. Do not use paper clips, highlighters, glue or correction 3 /7
fluid.
4 /11
Section A:
Answer ALL questions in the spaces provided on the question
paper. 6 /20
Section B:
Section
Answer two questions on separate answer paper. 7 /20
B

The number of marks is given in brackets [ ] at the end of each 8 /20


question or part question.

At the end of the examination, fasten all your work together. Total /80

You are advised to spend one hour on each section of the paper.

This document consists of 13 pages and 0 blank page.

www.gradtutors.com
339
2

For
Section A: Structured Questions [Total: 40 marks] Examiner’s
Use
Answer ALL questions in the space provided.

1 (a) Complete the electronic structures of the Cr3+ and Mn2+ ions.

Cr3+ 1s22s22p6 .....................

Mn2+ 1s22s22p6 ....................


[2]

(b) A student was given the following instructions for the oxidation of an alcohol,
C3H8O.

To 20 cm3 of water in a round bottom flask, carefully add 5 cm3 of concentrated


sulfuric acid and set up the distillation apparatus as shown below.

K2Cr2O7(aq) + C3H8O(l)

water out

H2SO4(aq)

water in

heat ice bath

product

Make up a solution containing 47.2 g of potassium dichromate(VI), K2Cr2O7, in a


15 cm3 of water, add 18.0 g of the alcohol, C3H8O, and pour this mixture into the
dropping funnel.

Boil the acid in the flask. Add the mixture from the dropping funnel at such a rate
that the product is slowly collected.

Re-distil the product and collect the fraction that boils between 48oC and 50oC.

NJC/H1 Chem/02/2013 [Turn over

www.gradtutors.com
340
3

(i) Identify the possible isomers of the alcohol, C3H8O. For


Examiner’s
Use

[2]

(ii) The balanced equation of the reaction is


3 C3H8O + K2Cr2O7 + 4 H2SO4 → 3 C3H6O + Na2SO4 + Cr2(SO4)3 + 7 H2O

I State the colour change that the student would observe during the
reaction.
from……………………..… to ………………………………

II Calculate the amount of K2Cr2O7 used.

III The student obtained 5.22 g of the carbonyl compound, C3H6O.


Calculate the percentage yield of the product obtained by the student
assuming that the reaction goes to completion.

[5]

NJC/H1 Chem/02/2013 [Turn over

www.gradtutors.com
341
4

(iii) Use the table of characteristic values for Typical proton chemical shift For
values in the Data Booklet to answer this question. Examiner’s
Use

Nuclear magentic resonance can be used to identity functional groups in


organic compounds. For example, ethyl ethanoate shows a chemical shift at
2.0ppm.
O
CH3 C

OCH2CH3
ethyl ethanoate

An impure sample of C3H6O obtained by a student was analysed using


nuclear magnetic resonance. The nuclear magnetic resonance recorded a
chemical shift at 9.0-13.0 ppm.

I Identity the functional group present in the impurity. Explain your


answer.

Impurity …………………………….

Reason…………………………………………………………………………

…………………………………………………………………………………..

………………………………………………………………………………….

II Suggest a structure for the alcohol used.

[4]

[Total:13]

NJC/H1 Chem/02/2013 [Turn over

www.gradtutors.com
342
5

2 Iodide ions are oxidised to iodine by peroxodisulphate ions, S2O82–. The reaction can be For
Examiner’s
catalysed by Fe2+(aq) ions and by Fe3+(aq) ions. Use

In an experiment to investigate the uncatalysed reaction, the concentration of iodine


was determined at different times. Curve A shown below was obtained.

The experiment was repeated using half the original concentration of iodide ions but
keeping other conditions the same. Curve B was obtained.

(a) Use these curves and the dotted lines to deduce the order of the reaction with
respect to iodide ions. Explain how you deduced the order.

Order ……………………………………………………………………………………….

Explanation ………………………………………………………………………………...

……………………………………………………………………………………………....

……………………………………………………………………………………………….

………………………………………………………………………………………………. [3]

(b) On the axes above, sketch a curve to show how the results will change if the
experiment leading to curve B is repeated under the same conditions of
concentration but at a lower temperature. Label this curve X. [2]

(c) On the axes above, sketch a curve to show how the results will change if the
experiment leading to curve A is repeated in the presence of a catalyst containing
Fe2+(aq) ions. Label this curve Y. [2]

NJC/H1 Chem/02/2013 [Turn over

www.gradtutors.com
343
6

(d) Suggest why the reaction needs to be catalysed by Fe2+(aq) ions and by Fe3+(aq) For
Examiner’s
ions. Use

[2]

[Total:9]

3 Lattice enthalpy can be used as a measure of ionic bond strength. Lattice enthalpies
are determined indirectly using an enthalpy cycle called a Born-Haber cycle.

The table below shows the enthalpy changes that are needed to determine the lattice
enthalpy of magnesium chloride, MgCl2.

Letter Enthalpy change Energy/ kJ mol–1


A 1st electron affinity of chlorine -349
B 1st ionization energy of magnesium +736
C 2nd ionization energy of magnesium +1450
D Atomization of chlorine +150
E Atomistation of magnesium +76
F Formation of magnesium chloride -642
G Lattice enthalpy of magnesium chloride

(a) On the cycle below, write the correct letter in each empty box.

[3]

NJC/H1 Chem/02/2013 [Turn over

www.gradtutors.com
344
7

(b) Use the Born-Haber cycle to calculate the lattice enthalpy of magnesium chloride. For
Examiner’s
Use

lattice enthalpy =……………… kJ mol–1 [2]

(c) Magnesium chloride melts at 714oC while magnesium fluoride melts at 1255oC
Explain the difference in the melting points of the magnesium halides.

……………………………………………………………………………………………….

……………………………………………………………………………………………….

……………………………………………………………………………………………….

[2]

[Total:7]

4 A student obtained the following results when analysing an organic compound H.

Test Observation
Test 1 Relative molecular mass 72
Test 2 % composition by mass C, 66%; H, 11.1%; O, 22.2%
Test 3 Reactions with Br2(aq) Br2 decolourised
Test 4 Reactions with Na(s) H2(g) evolved
Test 5 Reactions with warm Cr2O72-/H+ Green colour observed

The student allowed test 5 to go to completion and then investigated the product of
test 5 with the following result.

Test 6 Reaction with 2, 4- No reaction


dinitrophenylhydrazine

(a) Calculate the molecular formula of H.

[2]
NJC/H1 Chem/02/2013 [Turn over

www.gradtutors.com
345
8

(b) What can be deduced about the nature of H by the following tests? For
Examiner’s
Use
(i) Test 3 ...............................................................................................................

(ii) Test 4................................................................................................................


[2]

(c) (i) What functional group would have given a positive test result in test 6?

...........................................................................................................................

(ii) What functional group is shown to be present in H by tests 5 and 6?

........................................................................................................................... [2]

(d) On testing a sample of H, the student found that it was not chiral.

H did, however, show cis-trans isomerism.

How does cis-trans isomerism arise in an organic molecule?

...................................................................................................................................

...................................................................................................................................

................................................................................................................................... [2]

(e) Use all of the information above to draw labelled, displayed formulae of the
stereoisomers of compound H.

[3]

[Total:11]
NJC/H1 Chem/02/2013 [Turn over

www.gradtutors.com
346
9

Section B: Free Response Questions [Total: 40 marks]

Answer two of three questions in this section on separate paper.

5 (a) (i) Calculate the oxidation number of carbon in methanol, CH3OH.

(ii) Using only the elements C, H and O, draw the structural formulae of three
compounds, A, B and C, each containing a single carbon atom with an
oxidation number of zero, +2 and +4 respectively.

(iii) Suggest reagents and conditions for converting methanol into each of the
three compounds A, B and C. [6]

(b) In an experiment to determine the enthalpy change of combustion of ethanol,


∆Hc, a quantity of the fuel was burned underneath a copper can containing 100g
of water. It was found that the temperature of the water rose by 30.0°C after
1.50g of ethanol had been burned.

Assume that 50% of the heat obtained from burning the methanol is lost to the
surroundings.

(i) Calculate the apparent ∆Hc of ethanol from these figures. Ignore the heat
capacity of the copper can, and use the figure of 4.18 J g-1 K-1 for the
specific heat capacity of water.

(ii) The true value of ∆Hc of ethanol is -1367 kJmol-1. Compare this value to
the one you have calculated in (i) and suggest a reason for the
discrepancy. [4]

(c) Methanol can be produced by using a reversible reaction between carbon


monoxide and hydrogen.

2H2(g) + CO(g) CH3OH(g)

When 2.00 mol of hydrogen and 1.00 mol of carbon monoxide are mixed and
heated to a high temperature in a container of volume 1.50 dm3, the equilibrium
yield of methanol is 0.80 mol.

(i) Write the equilibrium constant, Kc, for this reaction and give its units.
(ii) Calculate a value for the equilibrium constant, Kc, for this reaction at this
temperature and give its units.

(iii) Taking the bond energy for the C-O bond in carbon monoxide to be 1077
kJ mol-1, and using other appropriate bond energies given in the Data
Booklet, calculated the entalpy change for this reaction.
[6]
Hence suggest why a pressure of 100 atm and 300°C are used in the
reaction.

NJC/H1 Chem/02/2013 [Turn over

www.gradtutors.com
347
10

(d) (i) Draw the structural formulae of four alcohols with the molecular formula
C4H10O. Label your structures D, E, F and G.
(ii) Classify these alcohols as primary, secondary or tertiary.

(iii) Identify which alcohol reacts with alkaline aqueous iodine, giving the
structural formulae of the products
(iv) Identify which alcohol is oxidised by an excess of acidified K2Cr2O7 to give
a non-acidic organic product, giving the structural formula of the product. [6]

[Total:20]

6 (a) Describe what is meant by the term nucleon number. [1]

(b) State two ways in which the behaviour of electrons in an electric field differs from [1]
that of protons.

(c) (i) Draw dot-and-cross diagrams to show the bonding in the molecules of NO2
and O3.
(ii) Suggest a value for the bond angle in each of the above two molecules,
giving reasons for your choice.
(iii) The compound FO2 does not exist, but ClO2 does.
By considering the possible types of bonding in the two compounds,
suggest reasons for this difference. [6]

(d) (i) Describe how you would carry out the reaction between magnesium and
oxygen, and state what you would observe during this reaction.

(ii) The pH values of the solutions formed when the chlorides of Period III are
separately shaken with water decrease from sodium to phosphorus.

Explain why this is the case. [4]

(e) Ozone is usually made by passing oxygen gas through a tube between two
highly charged electrical plates.

3O2(g) → 2O3(g)

The reaction does not go to completion, so a mixture of the two gases results.
The concentration of O3 in the mixture can be determined by its reaction with
aqueous KI.
O3 + 2KI + H2O → I2 + O2 + 2KOH
The iodine formed can be estimated by its reaction with sodium thiosulfate.

2Na2S2O3 + I2 → Na2S4O6 + 2NaI

NJC/H1 Chem/02/2013 [Turn over

www.gradtutors.com
348
11

When 500 cm3 of an oxygen/ozone gaseous mixture at s.t.p. was passed into an
excess of aqueous KI, and the iodine titrated, 15.0 cm3 of 0.100 mol dm-3
Na2S2O3 was required to discharge the iodine colour.

(i) Calculate the amount in moles of iodine produced.


(ii) Hence calculate the percentage of O3 in the gaseous mixture. [3]

(f) Oxidation is an important reaction in organic chemistry. Both aldehydes and


carboxylic acids can be prepared by the oxidation of alcohols with acidified
K2Cr2O7.
(i) Describe how you could ensure that only either the aldehyde or the
carboxylic acid is produced during the oxidation process.
(ii) Compounds L and M, both C9H10, and compound N, C9H12O, are all
oxidised by hot concentrated alkaline KMnO4, followed by acidification, to
give benzene-1, 2-dicarboxylic acid, P.

COOH

COOH
P

Compound L reacts with Br2(aq), but compound M does not.


Compound N reacts with alkaline aqueous iodine.
Suggest structures for compounds L, M and N. [5]

[Total:20]

NJC/H1 Chem/02/2013 [Turn over

www.gradtutors.com
349
12

7 This question is about hydroxyacids.


One of the simplest hydroxyacids is lactic acid, 2-hydroxypropanoic acid. It can be
made in the laboratory by the following route.

I II
CH3CHO CH3CH(OH)CN CH3CH(OH)COOH
Lactic acid

(a) State the reagents and conditions needed for


(i) Reaction I,
(ii) Reaction II.
[3]

The pH of 0.20 mol dm-3 lactic acid is 1.7.


(b) Explain whether lactic acid is a strong or weak acid with relevant calculation. [2]

(c) (i) Explain what is meant by a buffer solution.

Write equations that occur to illustrate the action of a buffer using a


solution of lactic acid and sodium lactate as an example.

(ii) Suggest a suitable indicator for the titration of lactic acid with sodium
hydroxide.

State the colour change at the end point. [4]

Poly(lactic acid) (PLA) is a biodegradable polymer used for food packaging and
cosmetic bottles. A section of the PLA molecule is shown below.

( OCH(CH3)CO )
n

PLA

(d) (i) Name the functional group present in PLA


(ii) Suggest the type of reaction that might occur during the biodegradation [2]
of PLA.

NJC/H1 Chem/02/2013 [Turn over

www.gradtutors.com
350
13

(e) Another route to lactic acid acid uses the Hell-Volhard-Zelinsky (HVZ) reaction,
followed by hydrolysis.
Cl2 + PCl3 III
CH3CH2COOH CH3CHClCOCl CH3CH(OH)COOH
HVZ J
(i) Suggest the reagents and conditions for reaction III.

(ii) When one mole of the dichloride J is treated with one mole of ethanol,
C2H5OH, a neutral product K is obtained. Treatment of K with NH3 in
ethanol under pressure produces L, C5H11NO2.
Suggest structures for K and L and explain the different reactivities of the
two chlorine atoms in J.
[5]

(f) 2-hydroxyphenylethanoic acid, N, can be prepared from compound M by the


following route.

O H OH H OH
C NaOH(aq) C H+(aq) C
CHO COONa COOH

M N

Suggest two tests (stating reagents and observations) that would enable the
compounds M and N to be distinguished from each other. [4]

[Total:20]

NJC/H1 Chem/02/2013 [Turn over

www.gradtutors.com
351

NANYANG JUNIOR COLLEGE


JC 2 PRELIMINARY EXAMINATION
Higher 1

CHEMISTRY 8872/01
Paper 1 Multiple Choice 19 September 2013
50 minutes
Additional Materials: Multiple Choice Answer Sheet
Data Booklet

READ THESE INSTRUCTIONS FIRST

Write in soft pencil.


Do not use staples, paper clips, highlighters, glue or correction fluid.
Write your name, class and tutor’s name on the Answer Sheet in the spaces provided unless
this has been done for you.

There are thirty questions on this paper. Answer all questions. For each question there are
four possible answers A, B, C and D.
Choose the one you consider correct and record you choice in soft pencil on the separate
Answer Sheet.

Read the instructions on the Answer Sheet very carefully.

Each correct answer will score one mark. A mark will not be deducted for a wrong answer.
Any rough working should be done in this booklet.

This document consists of 14 printed pages.

[Turn over

www.gradtutors.com
352

Section A

For each question there are four possible answers, A, B, C and D. Choose the one
you consider to be correct.

1 Morphine, C17H19NO, is a strong narcotic drug that doctors prescribe as a


painkiller. What mass of morphine has the same number of carbon atoms as
9.0 g of CO2?

A 0.529 g B 3.04 g C 11.2 g D 51.7 g

2 Carbon disulfide is a by-product found in the combustion of plastic. When 1


mole of carbon disulfide is burnt in 4 moles of oxygen, carbon dioxide and sulfur
dioxide are formed.

CS2(g) + 3O2(g)  CO2(g) + 2SO2(g)

Which of the following statements about the reaction is true?

A It is a disproportionation reaction.
B Carbon in CS2 is oxidised to CO2.
C 6 moles of electrons are transferred during the reaction.
D When the resultant gas mixture was passed through NaOH(aq), volume
decreased by 75 %.

3 When passed through an electric field, the 1H nucleus is deflected as follows.


Which one of the above beams represents the deflection for an ion 2X2– ?

(4o)

(4o)

4 A and B are in Group V and Group VI respectively. Which of the following


comparisons between the first and second ionisation energies of A and B is
correct?

1st I.E. 2nd I.E.


A A>B A<B
B A>B A>B
C A<B A<B
D A<B A>B
H1 Chemistry 8872/01 NYJC J2/2013 Prelim

www.gradtutors.com
353
3

5 In which of the following pairs of compounds is the second compound more


polar than the first compound?

A HCN BeCl2
B NH3 PH3
C SO2 SeO2
D CH3Cl CCl4

6 Which of the following statements about the properties associated with ionic
and covalent bonds is correct?

A Only ionic compounds are soluble in water.


B All covalent compounds have low melting points.
C Both ionic and covalent bonds can occur in the same compound.
D All ionic compounds can conduct electricity under standard conditions.

7 In an experiment, 50 cm3 of water at 30 oC was brought to boiling point by


burning butan-1-ol in excess of oxygen. Given that the enthalpy change of
combustion of butan-1-ol is -w kJ mol-1, calculate the percentage efficiency if
m g of butan-1-ol was used in this process.

Assume that the heat capacity of water is 4.2 J K-1 cm-3 and the experiment is
done under room temperature and pressure.

(50 × 4.2 × 70) × 74


A %
10mw

10mw
B %
10my(50 × 4.2 × 70) × 74

[(50 + y) × 4.2 × 70] × 74


C %
10mw

(50 × 4.2 × 70) × 74 × 100


D %
mw

H1 Chemistry 8872/01 NYJC J2/2013 Prelim [Turn over


www.gradtutors.com
354
4

8 What is the value of the enthalpy change for the following process equal to?

Al(s)  Al3+(g) + 3e−

A the third ionisation energy of alumnium


B the enthalpy change of vaporisation of aluminium
C the sum of the first ionisation energy, second ionization energy and the
third ionization energy of aluminium
D the sum of the enthalpy change of atomization of aluminium, the first
ionisation energy, second ionization enery and the third ionization
energy of aluminium

9 The graphs below show the variation of the percentage of gaseous reactants
present at equilibrium, with temperature and pressure.

% reactants (T + 20) oC
at
equilibrium
ToC

pressure

Which one of the following systems could the graphs represent?

A 2Fe(s) + 3 O2(g) ⇌ Fe2O3(s) ∆H = –822 kJ mol–1


2
B 3O2(g) + 4NH3(g) ⇌ 2N2(g) + 6H2O(g) ∆H = –1248 kJ mol–1
C N2O4(g) ⇌ 2NO2 (g) ∆H = +57 kJ mol–1
D CO2(g) + C(s) ⇌ 2CO(g) ∆H = +173 kJ mol–1

H1 Chemistry 8872/01 NYJC J2/2013 Prelim

www.gradtutors.com
355

10 Two gases, X2 and Y, react as follows:

X2(g) + 3Y(g)  X2Y3(g)

A mixture containing 0.50 mol of X2 and 0.50 mol of Y was heated in a 0.50 dm3
closed container. When the reaction reached equilibrium, 0.10 mol of X2Y3 was
produced.

What is the value of the equilibrium constant, Kc, for this reaction?

A 0.98 B 1.25 C 3.91 D 31.2

11 The pH of normal human blood is 7.4. Strenuous exercise can cause the
condition called acidosis in which the pH falls. If the pH drops to 6.8, death may
occur.

How many times greater is the hydrogen ion concentration in blood at pH 6.8
compared with that at pH 7.4?

A 0.25 B 0.6 C 2.0 D 4.0

12 The decomposition of ammonia is a zero order reaction.

2NH3 (g)  N2 (g) + 3H2 (g)

Which of these graphs correspond to the decomposition of ammonia?

A B

C D

H1 Chemistry 8872/01 NYJC J2/2013 Prelim [Turn over


www.gradtutors.com
356
6

13 The decomposition of hydrogen peroxide, H2O2, is a first-order reaction.

It took 30 min for [H2O2] to drop from 1.00 mol dm-3 to 0.25 mol dm-3. At the
same temperature, how long will it take for [H2O2] to drop from 0.80 mol dm-3 to
0.10 mol dm-3?

A 28 min
B 30 min
C 45 min
D 60 min

14 The diagram represents the distribution of the kinetic energy of the particles
within a gas at temperature T.

Number of
particles

Kinetic
Energy

Which of the following statements is correct?

A When the temperature is decreased, the area under the curve will
decrease.

B When the temperature is increased, the proportion of particles with any


given energy will increase.

C When the temperature is decreased, the maximum of the curve will be


displaced to the right and downwards.

D When the temperature is increased, the proportion of particles with


energies above any given value will increase.

H1 Chemistry 8872/01 NYJC J2/2013 Prelim

www.gradtutors.com
357
7

15 Mixtures I, II, and III were prepared by adding 10 g of NaCl, SiCl4 and SiO2
respectively to 1 dm3 of water and stirring thoroughly.

Will there be effervescence when aqueous sodium carbonate is added to these


mixtures?

I II III
A no no yes
B no yes no
C no yes yes
D yes yes no

16 When heated with chlorine, the hydrocarbon 3,3–dimethylpentane undergoes


substitution.

How many different monochloro–substituted products can be obtained?

A 1 B 2 C 3 D 4

17 Given the following bond energies:

bond bond energy / kJ mol–1


C–C 350
C=C 610

Which of the following statements is true?

A The π bond is stronger than the σ bond.

B The C≡C bond energy is estimated to be 870 kJ mol–1.

C The π bond energy cannot be determined from the data.

D The C≡C bond energy is the sum of the C-C bond energy and C=C bond
energy.

H1 Chemistry 8872/01 NYJC J2/2013 Prelim [Turn over


www.gradtutors.com
358
8

18 A hydrocarbon on heating with cold dilute potassium manganate(VII) produces


the structure below as the only organic product.

HO CH3

HO OH

HO

What could be a possible structure of the hydrocarbon?

A CH3 B CH3

C CH3 D CH2

19 Ozone depletion potential (ODP) is a measure of the effectiveness of


chlorofluoroalkanes in destroying stratospheric ozone.

In which sequence are compounds listed in increasing order of their ODPs?

A CHClF2 < CH3CCl2F < CCl2FCClF2


B CHClF2 < CCl2FCClF2 < CH3CCl2F
C CH3CCl2F < CCl2FCClF2 < CHClF2
D CCl2FCClF2 < CHClF2 < CH3CCl2F

H1 Chemistry 8872/01 NYJC J2/2013 Prelim

www.gradtutors.com
359
9

20 (CH3)2C(OH)COOH, may be synthesised from (CH3)2CHBr, through a series of


reactions.

Which set of reagents, used in sequential order, would be the most suitable for
this synthesis?

A aqueous KOH, acidified KMnO4


B HCN with NaOH catalyst, dilute H2SO4
C HCN with NaOH catalyst, acidified KMnO4
D aqueous KOH, acidified KMnO4, HCN with NaOH catalyst, dilute H2SO4

21 Which of the following halides will react most rapidly with aqueous sodium
hydroxide?

A (CH3)3CF
B (CH3)3CCl
C (CH3)3CBr
D (CH3)3CI

22 Which of the following chemical tests can distinguish between 1-chlorobutane


and 2-chlorobutane?

A Heating with excess NaOH(aq), followed by adding aqueous iodine.


B Heating with limited NaOH(aq), followed by adding aqueous silver
nitrate.
C Heating with excess NaOH(aq), followed by adding aqueous potassium
dichromate (VI).
D Heating with excess NaOH(aq), followed by refluxing with acidified
aqueous potassium manganate (VII).

H1 Chemistry 8872/01 NYJC J2/2013 Prelim [Turn over


www.gradtutors.com
360
10

23 In the presence of a dilute alkali, some aldehydes and ketones undergo the
‘aldol reaction’ where they dimerise to form a hydroxylcarbonyl compound
(an aldol). For example, ethanal dimerises in this way to form 3-hydroxybutanal.

OH
CH3CHO CH3CHCH2CHO

OH

Which of the following compounds will undergo the aldol reaction to produce the
aldol shown below?

CH3 CH3

CH3CCH2C O

OH

A CH3COCH3
B CH3CH2CHO
C (CH3)2CHCHO
D CH3CH2COCH3

24 In which of the following sequence is the value of pKa decreasing for the
following compounds?

O O H3C
CH3 CH2
CH3 C CH3 OH Cl C H OH
C OH C C OH C

CH2Cl CH3 CH3 CH3

C D E F

A F, D, E, C
B D, F, C, E
C C, E, D, F
D E, C, F, D

H1 Chemistry 8872/01 NYJC J2/2013 Prelim

www.gradtutors.com
361
11

25 Compound G has the following structure.

CH2OH

COOCH3 COOH

Which of the following represents the product of its reaction with NaOH (aq) at
standard conditions?

A CH2OH B CH2O

O O O
C COOH C
C

O O O

C CH2OH D CH2O

O O
COOCH3 C COOCH3 C

O O

H1 Chemistry 8872/01 NYJC J2/2013 Prelim [Turn over


www.gradtutors.com
362
12

Section B

For each of the questions in this section one or more of the three numbered
statements 1 to 3 may be correct.

Decide whether each of the statements is or is not correct (you may find it helpful to
put a tick against the statements which you consider to be correct).

The responses A to D should be selected on the basis of

A B C D
1, 2 and 3 1 and 2 2 and 3 1 only
are only are only are is
correct correct correct correct

No other combination of statements is used as a correct response.

26 Element J has the following successive ionisation energies in kJ mol-1.

1st 2nd 3rd 4th 5th 6th 7th


1300 3400 5300 7400 11000 13000 21000

Which of the following statements about J is true?

1 J has 2 unpaired electrons in its valence orbital.


2 J forms JO2.
3 The first IE of J is lower than its succeeding element.

27 Which of the following mixtures of solutions can function as a buffer?

1 50 cm3 of 0.2 mol dm–3 of CH3COOH and 25 cm3 of 0.2 mol dm–3
Ba(OH)2
2 40 cm3 of 0.1 mol dm–3 of NH3 and 15 cm3 of 0.2 mol dm–3 HCl
3 25 cm3 of 0.1 mol dm–3 of CH3COO–Na+ and 25 cm3 of 0.05 mol dm–3
HCl

28 The chloride of element L dissolves in aqueous NaOH and its oxide has a high
melting point.

What could element L be?

1 aluminum
2 sodium
3 phosphorous

H1 Chemistry 8872/01 NYJC J2/2013 Prelim

www.gradtutors.com
363
13

The responses A to D should be selected on the basis of

A B C D
1, 2 and 3 1 and 2 2 and 3 1 only
are only are only are is
correct correct correct correct

No other combination of statements is used as a correct response.

29 Which of the following reagents can be used to distinguish the following two
compounds?

O OH

C O CH2
H2C OH C

U V

1 2,4-DNPH, warm
2 Na2CO3 (aq)
3 NaHB4 (aq)

H1 Chemistry 8872/01 NYJC J2/2013 Prelim [Turn over


www.gradtutors.com
364
14

The responses A to D should be selected on the basis of

A B C D
1, 2 and 3 1 and 2 2 and 3 1 only
are only are only are is
correct correct correct correct

No other combination of statements is used as a correct response.

30 An organic compound M undergoes the following reactions:

• It decolourises a solution of bromine in tetrachloromethane.


• It reacts with sodium bromide in concentrated sulfuric acid.
• It reacts with aqueous KOH to produce a compound with two alcohol
functional groups.

Which compounds could be M?

1 OH

Br

2 Br

Br

3 OH

Br

H1 Chemistry 8872/01 NYJC J2/2013 Prelim

www.gradtutors.com
365

NANYANG JUNIOR COLLEGE


2013 JC 2 PRELIMINARY EXAMINATION
Higher 1

1 B 11 D 21 D
2 D 12 C 22 A
3 D 13 C 23 A
4 A 14 D 24 B
5 C 15 B 25 C
6 C 16 C 26 A
7 A 17 B 27 C
8 D 18 B 28 B
9 A 19 A 29 B
10 C 20 D 30 D

www.gradtutors.com
366

NANYANG JUNIOR COLLEGE


JC 2 PRELIMINARY EXAMINATION
Higher 1

CANDIDATE
NAME

TUTOR’S
CLASS
NAME

CHEMISTRY 8872/02
Paper 2 19 September 2013
2 hours
Candidates answer Section A on the Question Paper.
Additional Materials: Answer Paper
Data Booklet
Graph Paper

READ THESE INSTRUCTIONS FIRST

Write your name and class on all the work you hand in.
Write in dark blue or black pen on both sides of the paper.
You may use a soft pencil for any diagrams, graphs or rough working.
Do not use staples, paper clips, highlighters, glue or correction fluid.

Section A
Answer all the questions.

Section B
Answer two questions on separate answer paper.

At the end of the examination, fasten all your work securely together.
The number of marks is given in brackets [ ] at the end of each question or part
question.

For Examiner’s Use

Section A

B5

B6

B7

Total

This document consists of 15 printed pages.

[Turn over
www.gradtutors.com
367
2 For
Examiner’s
Use
Section A

Answer all the questions in this section in the spaces provided.

1 (a) Silicon and sulfur are among the most abundant period 3 elements on Earth.

(i) Describe the variations in melting points of the elements silicon and
sulfur. Explain the variations in terms of their structures and bonding.

(ii) Describe the action of water on the oxides of silicon and sulfur, write
equations for any reactions that occur, and suggest the pH of each
solution formed.

[5]

H1 Chemistry 8872/02 NYJC J2/2013 Prelim

www.gradtutors.com
368
3 For
Examiner’s
Use
(b) Sulfur hexafluoride (SF6) is a colourless and odourless gas that is poorly
soluble in water. It is hypothesised that sulfur pentafluoride ion ( SF5− ) is very
soluble in water.

(i) Draw a dot-and-cross diagram to show the bonding in a sulfur


pentafluoride ion.

(ii) State and explain in detail how the shape of sulfur pentafluoride ion
arises.

shape

explanation

(iii) Explain with the aid of a diagram why sulfur pentafluoride ion is very
soluble in water.

[5]

[Total: 10]

H1 Chemistry 8872/02 NYJC J2/2013 Prelim [Turn over

www.gradtutors.com
369
4 For
Examiner’s
Use

2 (a) Given the following information, explain why the different compounds have
different pKa values at 298 K.

Compound pKa
Benzoic acid 4.19
2-methylbutanoic acid 4.80
Butan-2-ol 17.6

[4]

H1 Chemistry 8872/02 NYJC J2/2013 Prelim

www.gradtutors.com
370
5 For
Examiner’s
Use
(b) 25.0 cm3 of 2-methylbutanoic acid was titrated against 0.15 mol dm-3 of KOH
and the following titration curve was obtained.

(i) Explain what is meant by the term acid dissociation constant, Ka, of
2-methylbutanoic acid.

(ii) Calculate the concentration of 2-methylbutanoic acid used in this


experiment.

(iii) Using relevant values from (a) and b (ii), calculate the initial pH of
2-methylbutanoic acid acid.

H1 Chemistry 8872/02 NYJC J2/2013 Prelim [Turn over

www.gradtutors.com
371
6 For
Examiner’s
Use
(iv) Suggest and explain a suitable indicator for the above titration.

indicator

explanation

(v) With the aid of an equation, explain why the pH of the titration curve at
region X remains fairly constant.

[6]

[Total: 10]

H1 Chemistry 8872/02 NYJC J2/2013 Prelim

www.gradtutors.com
372
7 For
Examiner’s
Use
3 2-methylbutane is a component of natural gas. It is used as a fuel and also as an
active ingredient in toothpaste for sensitive teeth.

2-methylbutane (Mr = 72.0) can be synthesised from 3-methylbut-1-ene via a


reaction with hydrogen under suitable conditions. The enthalpy change of this
reaction is –116 kJ mol-1.

∆Hrxn = –116 kJ mol-1


+ H2

(a) (i) Suggest the conditions for this reaction.

(ii) Calculate the enthalpy change of combustion of 2-methylbutane given


the following enthalpy changes:
1
H2(g) + O2(g)  H2O(l) ∆H1 = –125 kJ mol-1
2
15
(CH3)2CHCH=CH2(g) + O2  5CO2(g) + 5H2O (l) ∆H2 = –3503 kJ mol-1
2

(iii) Calculate the minimum mass of 2-methylbutane required to heat 1000 g


of water from 25 °C to boiling point.

The specific heat capacity of water is 4.2 J g-1 K-1.

[5]

H1 Chemistry 8872/02 NYJC J2/2013 Prelim [Turn over

www.gradtutors.com
373
8 For
Examiner’s
Use
(b) Refer to the reaction scheme of the synthesis of 2-methylbut-2-ene from
2–methylbutane below.

(b) (i)

Step 1 Step 2

(i) Draw the structure of a possible intermediate for the reaction in the
space above.

(ii) State the type of reaction occuring in steps 1 and 2.

(iii) Give reagents and conditions for steps 1 and 2.

Step 1

Step 2

(iv) Will the yield of 2-methylbut-2-ene be high? Explain your answer.

[5]

[Total: 10]

H1 Chemistry 8872/02 NYJC J2/2013 Prelim

www.gradtutors.com
374
9 For
Examiner’s
Use
4 Pheremones are compounds an animal produces which can influence a change in
the behavior of another animal or insect of the same species. Compound A is an
isomer of an insect pheromone with a distinct peanut and hay-like odour
characteristic.

OH

Compound A

(a) In not more than 3 steps, suggest a synthetic route for the formation of B from
A. State clearly the reagents and conditions for each reaction step.

OH

CN
OH

Compound B

[5]

H1 Chemistry 8872/02 NYJC J2/2013 Prelim [Turn over

www.gradtutors.com
375
10 For
Examiner’s
Use

(b) Compound B can undergo a single reaction to form an amine. Draw the
displayed formula of this amine and state the type of reaction and the
reagents and conditions used in this reaction.

Type of reaction

Reagents and conditions

[3]

(c) Suggest the reagents and conditions for a chemical test that could be used to
distinguish between compounds A and B.

Reagents and conditions

Observations

[2]

[Total: 10]

H1 Chemistry 8872/02 NYJC J2/2013 Prelim

www.gradtutors.com
376
11

Section B

Answer two questions from this section on separate answer paper.

5 (a) Sketch a graph to show how ionic radius changes across period 3 elements and
explain the shape of your graph. [3]

(b) (i) With the aid of an equation, define the 2nd ionisation energy of gallium.

The graph below shows the 1st 8 ionisation energies of element D.

lg I.E.

1 2 3 4 5 6 7 8
Number of electrons removed

(ii) Prove with reasoning that Element D belongs to the same group as
gallium in the Periodic Table. Write the electronic configuration of the
other shell of element D.

(iii) Gallium(III) oxide is used as a precursor in the manufacturing of


semiconductor materials. Draw the dot-and-cross diagram of
gallium(III) oxide. [6]

(c) Period 3 oxides can be formed by combustion of the elements in oxygen.


Describe the formation of sodium oxide and phosphorus pentoxide and write
down the relevant equations of the reactions. [3]

www.gradtutors.com
H1 Chemistry 8872/02 NYJC J2/2013 Prelim [Turn over
377
12

(d) The addition of Cl2 to water forms hydrochloric acid and hypochlorous acid
according to the following equation.
Reaction I: Cl2 + H2O HCl + HClO

Hypochlorous acid hydrolyses in water to form a hypochlorite ion, OCl − . The


hypochlorite ion formed is the main component in bleach and disinfectant
agents.
Reaction II: HClO OCl − + H+

(i) Explain why the formation of the stable hypochlorite ion is facilitated by
dissolving chlorine gas in basic solutions, such as sodium hydroxide.

(ii) 1 mole of Cl2 was dissolved in 1 dm3 of water with a dissociation


coefficient of 0.75 in reaction I. Calculate the value of Kc, giving its units.

(iii) There are two unlabelled solutions X and Y, each containing the same
concentration of HCl (aq) and HOCl (aq). To identify the solutions a
student mixed equal volumes of NaOH (aq) with each solution. The
following results are obtained:

Reaction ∆T / °C
NaOH + X 4.8
NaOH + Y 5.6

Deduce the acid present in solutions X and Y and explain your answer. [8]

[Total: 20]

www.gradtutors.com
H1 Chemistry 8872/02 NYJC J2/2013 Prelim
378
13

6 (a) Methylbenzene can become 2-chlorobenzoic acid and 3-chlorobenzoic acid via
different reaction synthesis. Give the reagents and conditions required for the
synthesis of 2-chlorobenzoic acid, write the chemical equations and name the
type of reactions that occur. State and explain how 3-chlorobenzoic acid may be
synthesised instead of 2-chlorobenzoic acid. [6]

(b) Explain why 2-chlorobenzoic acid is a stronger acid than benzoic acid. [2]

(c) The equation for the formation of methylbenzene from benzene and
chloromethane is shown.
CH3

+ CH3Cl
+ HCl

A student performed an experiment to determine the order of reaction with


respect to chloromethane. The initial concentration of chloromethane was
0.0100 mol dm–3 and benzene was used in large excess. The concentration of
chloromethane at various times in the reaction mixture was found as follows:

Time / s [chloromethane] / mol dm–3


0 0.0100
40 0.0079
80 0.0062
120 0.0049
160 0.0038
200 0.0030

(i) Explain what is meant by order of reaction.

(ii) Plot the above data on suitable axes and use your graph to determine the
order of reaction with respect to chloromethane.

(iii) The initial concentration of chloromethane was 0.0100 mol dm–3 and
benzene was used in large excess. Suggest why benzene was used in
large excess in this experiment?

(iv) Explain with the aid of a suitable diagram, how the presence of a catalyst
can cause an increase in the rate of a chemical reaction. [12]

[Total: 20]

www.gradtutors.com
H1 Chemistry 8872/02 NYJC J2/2013 Prelim [Turn over
379
14

7 Maleic acid is one of two stereoisomers of butenedioic acid, HOOC(CH)2COOH.


Maleic acid is the cis-isomer whereas fumaric acid is the trans-isomer. This question
examines the chemistry of maleic acid and its derivatives.

(a) (i) Draw the full structural formula of maleic acid.

(ii) The two stereoisomers have different melting points as intramolecular


hydrogen bonding is possible only in one isomer. Identify this isomer and
hence, explain whether it has a higher or lower melting point, compared to
the other isomer. [4]

(b) Maleic acid is not an approved additive as long term consumption of high levels
of maleic acid can cause kidney damage. In May 2013, the Agri-food &
Veterinary Authority of Singapore recalled several food products imported from
Taiwan that were found to contain maleic acid. Most of the affected products
were tapioca starch balls used in the preparation of bubble tea beverages.

Some physical and chemical properties of maleic acid are given below.

Molecular formula: C4H4O4


Physical state and appearance: Powdered solid
Colour: White.
Mr: 116.08
Density: 1.59 g cm-3
Acid/Base properties: Dibasic
Water/Oil Distribution Coefficient: The product is more soluble in water;
[in oil ]
lg = −0.5
[in water ]

In a volumetric analysis to determine percentage by mass of maleic acid present


in tapioca starch balls, a 5.0 g sample was first powdered and dissolved in 100.0
cm3 of an oil-based solvent. Most of the maleic acid was extracted from the oil-
based solvent into an aqueous medium with 100.0 cm3 distilled water. 25.0 cm3
of the aqueous solution required 7.10 cm3 of 0.05 mol dm-3 aqueous sodium
hydroxide for complete neutralisation.

www.gradtutors.com
H1 Chemistry 8872/02 NYJC J2/2013 Prelim
380
15

(i) Using the data provided, determine the concentration of maleic acid
present in 100.0 cm3 of aqueous solution. You may assume the only acid
present in tapioca starch balls is maleic acid.

(ii) When a solute is added to two solvents, A and B, which do not mix, some
of the solute dissolves in each of the solvents and an equilibrium is set up
between the two solvents. It has been shown that for dilute solutions, the
ratio of the two concentrations in each of the solvents at equilibrium is a
constant known as the Dissociation Coefficient.

Using your answer in (i) and the data provided, determine the
concentration of maleic acid remaining in the oil-based solvent after
extraction.

(iii) Hence, using your answer in (i) and (ii) and data provided, determine the
percentage by mass of maleic acid in the sample of tapioca starch balls.
[7]

(c) (i) Maleic acid can be reduced to a compound S with molecular formula
C4H8O2. State the reagents required for this conversion and draw the full
structural formula of S.

(ii) Compound S has other structural isomers T and U all having the
molecular formula C4H8O2.

1 mole of T gives 1 mole of yellow precipitate with aqueous alkaline


iodine. T also decolourises aqueous bromine and exhibits geometric
isomerism.

1 mole of U also gives 1 moles of yellow precipitate with aqueous alkaline


iodine. However U does not decolourise aqueous bromine.

Deduce the structures of compounds T and U. Explain your reasoning


clearly. [9]

[Total: 20]

www.gradtutors.com
H1 Chemistry 8872/02 NYJC J2/2013 Prelim [Turn over
381

NANYANG JUNIOR COLLEGE


JC 2 PRELIMINARY EXAMINATIONo
Higher 1

CANDIDATE
NAME
ANSWERS

TUTOR’S
CLASS
NAME

CHEMISTRY 8872/02
Paper 2 19 September 2013
2 hours
Candidates answer Section A on the Question Paper.
Additional Materials: Answer Paper
Data Booklet
Graph Paper

READ THESE INSTRUCTIONS FIRST

Write your name and class on all the work you hand in.
Write in dark blue or black pen on both sides of the paper.
You may use a soft pencil for any diagrams, graphs or rough working.
Do not use staples, paper clips, highlighters, glue or correction fluid.

Section A
Answer all the questions.

Section B
Answer two questions on separate answer paper.

At the end of the examination, fasten all your work securely together.
The number of marks is given in brackets [ ] at the end of each question or part
question.

For Examiner’s Use

Section A

B5

B6

B7

Total

This document consists of 16 printed pages.

[Turn over
www.gradtutors.com
382
2 For
Examiner’s
Use
Section A

Answer all the questions in this section in the spaces provided.

1 (a) Silicon and sulfur are among the most abundant period 3 elements on Earth.
(i) Describe the variations in melting points of the elements silicon and
sulfur. Explain the variations in terms of their structures and bonding.
Silicon has giant molecular structure held together by numerous strong
covalent bonds between atoms. As large amount of energy is required
to break the strong covalent bonds, silicon has a higher melting point.
[1m]

Sulfur has simple molecular structure with weak van der Waals forces of
attractions between molecules. Lesser amounts of energy is required to
break the weak van der Waals forces of attraction hence sulfur has
lower melting point. [1m]
(ii) Describe the action of water on the oxides of silicon and sulfur, write
equations for any reactions that occur, and suggest the pH of each
solution formed. [5]
SiO2 is insoluble in water because energy from the weak van der Waals
forces between SiO2 and H2O molecules are unable to overcome the
strong covalent bonds between Si and O atoms (pH = 7). [1m]

Oxides of sulfur reacts with water to give a highly acidic solution (pH =
1-2). [1m]
SO2 (g) + H2O (l)  H2SO3 (aq) [1m for both]
SO3 (g) + H2O (l)  H2SO4 (aq)
(b) Sulfur hexafluoride (SF6) is a colourless and odourless gas that is poorly
soluble in water. It is hypothesised that sulfur pentafluoride ion ( SF5− ) is very
soluble in water.
(i) Draw a dot-and-cross diagram to show the bonding in a sulfur
pentafluoride ion.
- -
xx
x x
x F xx x
xx
x
xx xx x F xx xx
x
x
F x F
x x x
x x
x
F x xx F
x x x
x
xx x xx x

xx
S x xx
S
x xx
x x xx
x
x F x F x x
F F xx
x xx x
xx or xx xx

[1m]

www.gradtutors.com
H1 Chemistry 8872/02 NYJC J2/2013 Prelim
383
3 For
Examiner’s
Use
(ii) State and explain in detail how the shape of sulfur pentafluoride ion
arises.
There are 5 bond pairs and 1 lone pair of electrons around the central
sulfur atom. [1m]
Hence shape is square pyramidal. [1m]
(iii) Explain with the aid of a diagram why sulfur pentafluoride ion is very
soluble in water. [5]
SF5− is a molecular ion. The ion-dipole interaction between the ion and
water is strong enough to overcome the strong hydrogen bonds between
water molecules. [1m] Hence it is very soluble in water.

δ+
δ− H δ+
O H
l ll l
l l l l l l l l l ll l l

O δ−
l l l

δ+ H lllllll SF -
l

5 lllllllHδ+
llllll

H
l

δ+ Hδ+ Ion-dipole interaction


O
δ− [1m]

[Total: 10]

2 (a) Given the following information, explain why the different compounds have
different pKa values at 298 K.

Compound pKa
Benzoic acid 4.19
2-methylbutanoic acid 4.80
Butan-2-ol 17.6
[4]
The strength of an acid depends on the stability of the conjugate base. The
more stable the conjugate base, the stronger the acid.

The higher the pKa, the lower the Ka, the weaker the acid. Hence benzoic
acid is the strongest acid.

Hence, the order of stability of the conjugate base is


benzoate ion > 2-methylbutanoate ion > butoxide ion [1m]
In CH3(CH2)3O–, the presence of electron-donating –(CH2)3CH3 group
intensifies the negative charge on oxygen. Hence CH3(CH2)3O – is
destabilised. Butan-2-ol is the weakest acid. [1m]

www.gradtutors.com
H1 Chemistry 8872/02 NYJC J2/2013 Prelim [Turn over
384
4 For
Examiner’s
Use
In CH3CH2CH(CH3)CO2– and C6H5CO2–, the p-orbital of O– overlaps with the p
orbitals of the C=O bond. Hence the negative charge is effectively dispersed
over one carbon atom and two electronegative oxygen atoms. [1m] However,
C6H5CO2– is more stabilised than CH3CH2CH(CH3)CO2– as the negative
charge can be dispersed into benzene (more effectively) unlike CH3CO2–.
Benzoic acid is therefore a stronger acid than ethanoic acid and stronger
thanethanol. [1m]
(b) 25.0 cm3 of 2-methylbutanoic acid was titrated against 0.15 mol dm-3 of KOH
and the following titration curve was obtained.

(i) Explain what is meant by the term acid dissociation constant, Ka, of
2-methylbutanoic acid.
The acid dissociation constant is defined as Ka
H+   CH3CH2CH ( CH3 ) CO2 – 
= [1m]
[CH3CH2CH ( CH3 ) CO2H]

(ii) Calculate the concentration of 2-methylbutanoic acid used in this


experiment.
(0.15)(24.50)
[acid] = = 0.1470 mol dm-3 [1m]
(25.0)

(iii) Using relevant values from (a) and b (ii), calculate the initial pH of
2-methylbutanoic acid acid.
Given pKa = 4.80:
[H+] = (10 ) (0.1470) = 2.329 x 10-6 mol dm-3
-4.80

pH = 5.63 [1m]

www.gradtutors.com
H1 Chemistry 8872/02 NYJC J2/2013 Prelim
385
5 For
Examiner’s
Use
(iv) Suggest and explain a suitable indicator for the above titration.

Phenolphthalein is a suitable indicator. [1m] This is a weak acid-strong


base titration. The pH range of the indicator (8-10) falls within the pH
change at equivalence point (7-11) [1m]

(v) With the aid of an equation, explain why the pH of the titration curve at
region X remains fairly constant. [6]
When base is added, the added OH is removed by reacting with the
large reservoir of 2-methylbutanoic acid molecules present.

OH + CH3CH2CH(CH3)CO2H  CH3CH2CH(CH3)CO2–Na+ + H2O


[1m equation & explain]
[Total: 10]

3 2-methylbutane is a component of natural gas. It is used as a fuel and also as an


active ingredient in toothpaste for sensitive teeth.

2-methylbutane (Mr = 72.0) can be synthesised from 3-methylbut-1-ene via a


reaction with hydrogen under suitable conditions. The enthalpy change of this
reaction is –116 kJ mol-1.

∆Hrxn = –116 kJ mol-1


+ H2

(i) Suggest the conditions for this reaction.


Ni/Heat or Pt/rt or Pd/rt [1m]
(ii) Calculate the enthalpy change of combustion of 2-methylbutane given
the following enthalpy changes:
1
H2(g) + O2(g)  H2O(l) ∆H1 = –125 kJ mol-1
2
15
(CH3)2CHCH=CH2(g) + O2  5CO2(g) + 5H2O (l) ∆H2 = –3503 kJ mol-1
2

∆Hrxn = Σ∆Hc(reactants) – Σ∆Hc(products)


-116 = (-125) + (-3503) – ∆Hc(2-methylbutane) [1m]
∆Hc(2-methylbutane) = –3510 kJ mol-1 (3sf) [1m]

www.gradtutors.com
H1 Chemistry 8872/02 NYJC J2/2013 Prelim [Turn over
386
6 For
Examiner’s
Use
(iii) Calculate the minimum mass of 2-methylbutane required to heat 1000 g
of water from 25 °C to boiling point. The specific heat capacity of water
is 4.2 J g-1 K-1. [5]
Heat absorbed by water = mc∆T = (1000)(4.2)(100-25) = 315000 J [1m]

∆H = -Q/n
(-3510x1000) = -315000/(m/72)
m = 6.46 g (3sf) [1m]

(b) Refer to the reaction scheme of the synthesis of 2-methylbut-2-ene from


2–methylbutane below.

Step 1 Step 2

(i) Draw the structure of a possible intermediate for the reaction in the
space above.

or [1m]

(ii) State the type of reaction occuring in steps 1 and 2.


Step 1: substitution Step 2: elimination [1m for both]

(iii) Give reagents and conditions for steps 1 and 2.


Step 1: (limited) Cl2, UV light [1m]
Step 2: ethanolic NaOH, heat under reflux [1m]

(iv) Will the yield of 2-methylbut-2-ene be high? Explain your answer. [5]
No. Substitution of alkanes is uncontrollable and there will be a mixture
of various substitution products. [1m]
[Total: 10]

www.gradtutors.com
H1 Chemistry 8872/02 NYJC J2/2013 Prelim
387
7 For
Examiner’s
Use
4 Pheremones are compounds an animal produces which can influence a change in
the behavior of another animal or insect of the same species. Compound A is an
isomer of an insect pheromone with a distinct peanut and hay-like odour
characteristic.

OH

Compound A

(a) In not more than 3 steps, suggest a synthetic route for the formation of B from
A. State clearly the reagents and conditions for each reaction step.

OH

CN
OH

Compound B
[5]

O O
Excess conc. H2SO4
170 °C
OH

300 oC, 60 atm,


conc. H3PO4
catalyst,
OH H2O (g)
O

CN
HCN(g) in the
Br
presence of a
OH
trace amount of
NaCN
10 – 20 °C

www.gradtutors.com
H1 Chemistry 8872/02 NYJC J2/2013 Prelim [Turn over
388
8 For
Examiner’s
Use
(b) Compound B can undergo a single reaction to form an amine. Draw the
displayed formula of this amine and state the type of reaction and the
reagents and conditions used in this reaction. [3]
H

H H O H H

H C C C C C H

H H C O H

N H
H H

Type of reaction: Reduction


Reagents and conditions: LiAlH4 in dry ether

(c) Suggest the reagents and conditions for a chemical test that could be used to
distinguish between compounds A and B. [2]

Test: NaOH (aq), I2, warm


Observations with A: A yellow ppt is observed.
Observations with B: A yellow ppt is not observed.

OR

Test: 2,4-dinitrophenylhydrazine, warm


Observations with A: An orange ppt is observed.
Observations with B: An orange ppt is not observed.
[Total: 10]

www.gradtutors.com
H1 Chemistry 8872/02 NYJC J2/2013 Prelim
389
9

Section B

Answer two questions from this section on separate answer paper.

5 (a) Sketch a graph to show how ionic radius changes across period 3 elements and
explain the shape of your graph. [3]

Ionic
Radius










Na+ Mg2+ Al3+ Si4+ P3- S2- Cl- [1]

Anions have an additional shell of electrons than cations of the same period thus
have larger radii. [1]

Across the period, ionic radius decreases for cations and anions (ions which are
isoelectronic) as nuclear charge increase while shielding effect remains the
same. Hence effective nuclear charge increases and the outermost electron
shell is pulled slightly closer, giving rise to the decreasing trend. [1]

(b) (i) With the aid of an equation, define the 2nd ionisation energy of gallium.
Ga+ (g)  Ga2+(g) + e- [1m]

The graph below shows the 1st 8 ionisation energies of element D.

lg I.E.

1 2 3 4 5 6 7 8
Number of electrons removed

www.gradtutors.com
H1 Chemistry 8872/02 NYJC J2/2013 Prelim [Turn over
390
10

(ii) Prove with reasoning that Element D belongs to the same group as
gallium in the Periodic Table. Write the electronic configuration of the
outer shell of element D.
The 4th electron is from an inner quantum shell [1m] and it experiences
increased electrostatic attraction with the nucleus. A large amount of
energy is required to remove the 4th electron. [1m]
ns2 np1 [1m]
(iii) Gallium(III) oxide is used as a precursor in the manufacturing of
semiconductor materials. Draw the dot-and-cross diagram of
gallium(III) oxide. [6]

[2m; number of ions, charge on ions, number and position of valence


electrons]
(c) Period 3 oxides can be formed by combustion of the elements in oxygen.
Describe the formation of sodium oxide and phosphorus pentoxide and write
down the relevant equations of the reactions. [3]
2 Na (s) + ½ O2 (g)  Na2O (s)
Sodium burns vigorously with an intense yellow flame to form a white ionic solid
of sodium oxide. [1m]
P4 (s) + 5 O2 (g)  P4O10 (s) [1m for both]
Phosphorus reacts rapidly with O2 with a pale buish green flame to form a white
P4O10 solid. [1m]
(d) The addition of Cl2 to water forms hydrochloric acid and hypochlorous acid
according to the following equation.
Reaction I: Cl2 + H2O HCl + HClO

Hypochlorous acid hydrolyses in water to form a hypochlorite ion, OCl − . The


hypochlorite ion formed is the main component in bleach and disinfectant
agents.
Reaction II: HClO OCl − + H+

(i) Explain why the formation of the stable hypochlorite ion is facilitated by
dissolving chlorine gas in basic solutions, such as sodium hydroxide.
NaOH will react with HCl, reducing the concentration of HCl. The position
of equilibrium of reaction 1 will shift to the right to increase the [HCl] and
increases the [HClO]. [1m]
When [HClO] increases, the position of equilibrium will shift to the right in
reaction 2 to reduce the [HClO] and forms OCl-. This stabilises the
hypochlorite in NaOH. [1m]

www.gradtutors.com
H1 Chemistry 8872/02 NYJC J2/2013 Prelim
391
11

(ii) 1 mole of Cl2 was dissolved in 1 dm3 of water with a dissociation


coefficient of 0.75 in reaction I. Calculate the value of Kc, giving its units.

Cl2 + H2O HCl + HClO


Initial 1 0 0
Change -0.75 +0.75 +0.75
Equilibrium 0.25 0.75 0.75 [1m for equilibrium values]

0.75 0.75
( )( )
Kc = 1 1 = 2.25 mol dm-3
0.25
( )
1
[1m for correct Kc expression, 1m for values and units]

(iii) There are two unlabelled solutions X and Y, each containing the same
concentration of HCl (aq) and HOCl (aq). To identify the solutions a
student mixed equal volumes of NaOH (aq) with each solution. The
following results are obtained:

Reaction ∆T / °C
NaOH + X 4.8
NaOH + Y 5.6

Deduce the acid present in solutions X and Y and explain your answer. [8]

X contains HOCl. Y contains HCl. [1m]

In the neutralisation reactions, the enthalpy change of reaction of the


weak acid is less exothermic [1m] than that of a strong acid as it will
involve the enthalpy change of dissociation of H+ [1m] from the weak acid
as it undergoes partial dissociation. Energy is required for the
dissociation.
[Total: 20]

www.gradtutors.com
H1 Chemistry 8872/02 NYJC J2/2013 Prelim [Turn over
392
12

6 (a) Methylbenzene can become 2-chlorobenzoic acid and 3-chlorobenzoic acid via
different reaction synthesis. Give the reagents and conditions required for the
synthesis of 2-chlorobenzoic acid, write the chemical equations and name the
type of reactions that occur. State and explain how 3-chlorobenzoic acid may be
synthesised instead of 2-chlorobenzoic acid. [6]
Step 1: Substitution of methylbenzene to form 2-chloromethylbenzene.

Reagent & Condition: Cl2 (l) with anhydrous FeCl3

CH3 CH3
Cl
+ Cl 2 + HCl

[2m for name of reaction, reagent & condition and equation]

Step 2: Oxidation of 2-chloromethylbenzene to form 2-chlorobenzoic acid.

Reagent & Condition: KMnO4 (aq), H2SO4 (aq), Heat under reflux.

CH3 COOH
Cl 3 O Cl
+ H 2O

[2m for name of reaction, reagent & condition and equation]

3-chlorobenzoic acid can be synthesised from methylbenzene by changing the


sequence of reactions; oxidation followed by substitution. [1m]

The products are different as different existing substituent affects the position of
the incoming substituent. [1m] Methyl substituent is 2,4-directing while
carboxylic acid substituent is 3,5-directing.

(b) Explain why 2-chlorobenzoic acid is a stronger acid than benzoic acid. [2]
The electronegative Cl atom of chlorobenzenoic acid will exert an electron–
withdrawing effect on the corresponding carboxylate anion. [1m]
The negative charge becomes more dispersed and the conjugate base
becomes more stable. [1m]

Hence the dissociation of chlorobenzoic acid to give H+ is more favoured.

www.gradtutors.com
H1 Chemistry 8872/02 NYJC J2/2013 Prelim
393
13

(c) The equation for the formation of methylbenzene from benzene and
chloromethane is shown.

CH3

+ CH3Cl
+ HCl

A student performed an experiment to determine the order of reaction with


respect to chloromethane. The initial concentration of chloromethane was
0.0100 mol dm–3 and benzene was used in large excess. The concentration of
chloromethane at various times in the reaction mixture was found as follows:

Time / s [chloromethane] / mol dm–3


0 0.0100
40 0.0079
80 0.0062
120 0.0049
160 0.0038
200 0.0030

(i) Explain what is meant by order of reaction.


The term “order of reaction” refers to the power to which the
concentration of the reactant is raised in the experimentally determined
rate equation. [1m]

Or
For a general rate law, rate = k[A]m experimentally determined
[A] is the concentration of the reactant A,
m is the order of the reaction with respect to the reactant A

(ii) Plot the above data on suitable axes and use your graph to determine the
order of reaction with respect to chloromethane.

www.gradtutors.com
H1 Chemistry 8872/02 NYJC J2/2013 Prelim [Turn over
394
14

[1m for points plotted accurately and smooth curved line drawn, 1m for
axes labeled correctly, 1m for showing clearly on the graph dotted lines to
find two t1/2]

From the graph, half-lives are approximately constant at 120s. [1m]


Hence, order of reaction with respect to CH3Cl is 1. [1m]

(iii) The initial concentration of chloromethane was 0.0100 mol dm–3 and
benzene was used in large excess. Suggest why benzene was used in
large excess in this experiment?
Benzene was used in large excess so that its concentration will remain
almost constant throughout the experiment. As such, the rate of reaction
will only depend on chloromethane concentration [1m] thus enabling the
determination of the order of reaction with respect to chloromethane. [1m]

(iv) Explain with the aid of a suitable diagram, how the presence of a catalyst
can cause an increase in the rate of a chemical reaction. [12]

[1m for correct shape and axes, 1m for labelling and legend]

www.gradtutors.com
H1 Chemistry 8872/02 NYJC J2/2013 Prelim
395
15

In the presence of a catalyst, reaction has a different mechanism with a


lower activation energy compared to the uncatalysed reaction. [1m]
More molecules will possess energy greater than this lowered activation
energy, hence frequency of effective collisions will increase. [1m]
[Total: 20]

7 Maleic acid is one of two stereoisomers of butenedioic acid, HOOC(CH)2COOH.


Maleic acid is the cis-isomer whereas fumaric acid is the trans-isomer. This question
examines the chemistry of maleic acid and its derivatives.
(a) (i) Draw the full structural formula of maleic acid.

O O
H O C C O H
C C
H H

Must be cis and trigonal planar wrt C [1m]


Full structural/displayed formula [1m]
(ii) The two stereoisomers have different melting points as intramolecular
hydrogen bonding is possible only in one isomer. Identify this isomer and
hence, explain whether it has a higher or lower melting point, compared to
the other isomer. [4]
Maleic acid [1m] has intramolecular hydrogen bonding (as the two
COOH groups are closer to each other).

Maleic acid should have a lower melting point as it can form less
extensive intermolecular hydrogen bonds (between molecules) hence
less energy is required to break the weak hydrogen bonds. [1m]
(b) Maleic acid is not an approved additive as long term consumption of high levels
of maleic acid can cause kidney damage. In May 2013, the Agri-food &
Veterinary Authority of Singapore recalled several food products imported from
Taiwan that were found to contain maleic acid. Most of the affected products
were tapioca starch balls used in the preparation of bubble tea beverages.

Some physical and chemical properties of maleic acid are given below.
Molecular formula: C4H4O4
Physical state and appearance: Powdered solid
Colour: White.
Mr: 116.08
Density: 1.59 g cm-3
Acid/Base properties: Dibasic
Water/Oil Distribution Coefficient: The product is more soluble in water;
[in oil ]
lg = −0.5
[in water ]
www.gradtutors.com
H1 Chemistry 8872/02 NYJC J2/2013 Prelim [Turn over
396
16

In a volumetric analysis to determine percentage by mass of maleic acid present


in tapioca starch balls, a 5.0 g sample was first powdered and dissolved in 100.0
cm3 of an oil-based solvent. Most of the maleic acid was extracted from the oil-
based solvent into an aqueous medium with 100.0 cm3 distilled water. 25.0 cm3
of the aqueous solution required 7.10 cm3 of 0.05 mol dm-3 aqueous sodium
hydroxide for complete neutralisation.
(i) Using the data provided, determine the concentration of maleic acid
present in 100.0 cm3 of aqueous solution. You may assume the only acid
present in tapioca starch balls is maleic acid.
7.10
nNaOH = ×0.05
1000
= 3.550×10-4 mol
Since maleic acid is diprotic,
1
nacid in 25.0 cm3 = ×3.550×10-4
2
= 1.775×10-4 mol [1m]
100.0
nacid in 100.0 cm3 = ×1.775×10-4
25.0
= 7.100×10-4 mol [ecf 1m]
7.100 × 10-4
[acid] = = 7.100×10-3 mol dm-3 [ecf 1m]
0.100
*If student just find concentration in 25.0cm3 sample [2m]
(ii) When a solute is added to two solvents, A and B, which do not mix, some
of the solute dissolves in each of the solvents and an equilibrium is set up
between the two solvents. It has been shown that for dilute solutions, the
ratio of the two concentrations in each of the solvents at equilibrium is a
constant known as the Dissociation Coefficient.

Using your answer in (i) and the data provided, determine the
concentration of maleic acid remaining in the oil-based solvent after
extraction.
[in oil ]
lg = −0.5
[in water ]
[in oil ]
lg = −0.5
[7.100 × 10 −3 ]
[in oil ]
−2
= 10 −0.5
[7.100 × 10 ]
[in oil] =2.245x10-3 mol dm-3 [ecf1m]

www.gradtutors.com
H1 Chemistry 8872/02 NYJC J2/2013 Prelim
397
17

(iii) Hence, using your answer in (i) and (ii) and data provided, determine the
percentage by mass of maleic acid in the sample of tapioca starch balls.
[7]
nacid total = nacid (in water)+nacid (in oil)
100 100
= 7.100x10-3 × + 2.245x10-3 × [ecf1m]
1000 1000
= 9.345 × 10−4 mol
macid = 9.345×10-4 × 116.08 [ecf1m]
= 0.108 g
0.108
%mass = × 100% = 2.17% [ecf1m]
5.0
(c) (i) Maleic acid can be reduced to a compound S with molecular formula
C4H8O2. State the reagents required for this conversion and draw the full
structural formula of S.
Reagents and conditions: LiAlH4 in dry ether [1m]

S: H O O H

H C C C C H

H H H H [1m]

(ii) Compound S has other structural isomers T and U all having the
molecular formula C4H8O2.

1 mole of T gives 1 mole of yellow precipitate with aqueous alkaline


iodine. T also decolourises aqueous bromine and exhibits geometric
isomerism.

1 mole of U also gives 1 moles of yellow precipitate with aqueous alkaline


iodine. However U does not decolourise aqueous bromine.

Deduce the structures of compounds T and U. Explain your reasoning


clearly. [9]

OH H O
T: OH H OH U:
H C C C CH 3
H C C C CH 3
H H
H
[2m]

www.gradtutors.com
H1 Chemistry 8872/02 NYJC J2/2013 Prelim [Turn over
398
18

Deductions

OH

H C CH 3

H
in T undergoes oxidation with aq I2 to form a yellow ppt. [1m]

alkene in T undergoes addition with aq Br2. [1m]

T exhibits geometric isomerism because each C in C=C is bonded to 2


different groups. [1m]

alkene is absent in U since it does not react with aq Br2. [1m]

H C CH 3 in U undergoes oxidation with aq I2 to form a yellow ppt. [1m]

OH

H C CH 3

Since only 1 mole of CHI3 is formed, H cannot also be


present in U [1m], hence alcohol should be on the terminal carbon.

*6 Marking points but MAX 5 for deductions.


*If students write methyl alcohol or methyl carbonyl present in T/U… give
[1m]
[Total: 20]

www.gradtutors.com
H1 Chemistry 8872/02 NYJC J2/2013 Prelim
399

NANYANG JUNIOR COLLEGE


JC 2 PRELIMINARY EXAMINATION
Higher 1

CHEMISTRY 8872/01
Paper 1 Multiple Choice 19 September 2013
50 minutes
Additional Materials: Multiple Choice Answer Sheet
Data Booklet

READ THESE INSTRUCTIONS FIRST

Write in soft pencil.


Do not use staples, paper clips, highlighters, glue or correction fluid.
Write your name, class and tutor’s name on the Answer Sheet in the spaces provided unless
this has been done for you.

There are thirty questions on this paper. Answer all questions. For each question there are
four possible answers A, B, C and D.
Choose the one you consider correct and record you choice in soft pencil on the separate
Answer Sheet.

Read the instructions on the Answer Sheet very carefully.

Each correct answer will score one mark. A mark will not be deducted for a wrong answer.
Any rough working should be done in this booklet.

This document consists of 14 printed pages.

[Turn over

www.gradtutors.com
400

Section A

For each question there are four possible answers, A, B, C and D. Choose the one
you consider to be correct.

1 Morphine, C17H19NO, is a strong narcotic drug that doctors prescribe as a


painkiller. What mass of morphine has the same number of carbon atoms as
9.0 g of CO2?

A 0.529 g B 3.04 g C 11.2 g D 51.7 g

2 Carbon disulfide is a by-product found in the combustion of plastic. When 1


mole of carbon disulfide is burnt in 4 moles of oxygen, carbon dioxide and sulfur
dioxide are formed.

CS2(g) + 3O2(g)  CO2(g) + 2SO2(g)

Which of the following statements about the reaction is true?

A It is a disproportionation reaction.
B Carbon in CS2 is oxidised to CO2.
C 6 moles of electrons are transferred during the reaction.
D When the resultant gas mixture was passed through NaOH(aq), volume
decreased by 75 %.

3 When passed through an electric field, the 1H nucleus is deflected as follows.


Which one of the above beams represents the deflection for an ion 2X2– ?

(4o)

(4o)

4 A and B are in Group V and Group VI respectively. Which of the following


comparisons between the first and second ionisation energies of A and B is
correct?

1st I.E. 2nd I.E.


A A>B A<B
B A>B A>B
C A<B A<B
D A<B A>B
H1 Chemistry 8872/01 NYJC J2/2013 Prelim

www.gradtutors.com
401
3

5 In which of the following pairs of compounds is the second compound more


polar than the first compound?

A HCN BeCl2
B NH3 PH3
C SO2 SeO2
D CH3Cl CCl4

6 Which of the following statements about the properties associated with ionic
and covalent bonds is correct?

A Only ionic compounds are soluble in water.


B All covalent compounds have low melting points.
C Both ionic and covalent bonds can occur in the same compound.
D All ionic compounds can conduct electricity under standard conditions.

7 In an experiment, 50 cm3 of water at 30 oC was brought to boiling point by


burning butan-1-ol in excess of oxygen. Given that the enthalpy change of
combustion of butan-1-ol is -w kJ mol-1, calculate the percentage efficiency if
m g of butan-1-ol was used in this process.

Assume that the heat capacity of water is 4.2 J K-1 cm-3 and the experiment is
done under room temperature and pressure.

(50 × 4.2 × 70) × 74


A %
10mw

10mw
B %
10my(50 × 4.2 × 70) × 74

[(50 + y) × 4.2 × 70] × 74


C %
10mw

(50 × 4.2 × 70) × 74 × 100


D %
mw

H1 Chemistry 8872/01 NYJC J2/2013 Prelim [Turn over


www.gradtutors.com
402
4

8 What is the value of the enthalpy change for the following process equal to?

Al(s)  Al3+(g) + 3e−

A the third ionisation energy of alumnium


B the enthalpy change of vaporisation of aluminium
C the sum of the first ionisation energy, second ionization energy and the
third ionization energy of aluminium
D the sum of the enthalpy change of atomization of aluminium, the first
ionisation energy, second ionization enery and the third ionization
energy of aluminium

9 The graphs below show the variation of the percentage of gaseous reactants
present at equilibrium, with temperature and pressure.

% reactants (T + 20) oC
at
equilibrium
ToC

pressure

Which one of the following systems could the graphs represent?

A 2Fe(s) + 3 O2(g) ⇌ Fe2O3(s) ∆H = –822 kJ mol–1


2
B 3O2(g) + 4NH3(g) ⇌ 2N2(g) + 6H2O(g) ∆H = –1248 kJ mol–1
C N2O4(g) ⇌ 2NO2 (g) ∆H = +57 kJ mol–1
D CO2(g) + C(s) ⇌ 2CO(g) ∆H = +173 kJ mol–1

H1 Chemistry 8872/01 NYJC J2/2013 Prelim

www.gradtutors.com
403

10 Two gases, X2 and Y, react as follows:

X2(g) + 3Y(g)  X2Y3(g)

A mixture containing 0.50 mol of X2 and 0.50 mol of Y was heated in a 0.50 dm3
closed container. When the reaction reached equilibrium, 0.10 mol of X2Y3 was
produced.

What is the value of the equilibrium constant, Kc, for this reaction?

A 0.98 B 1.25 C 3.91 D 31.2

11 The pH of normal human blood is 7.4. Strenuous exercise can cause the
condition called acidosis in which the pH falls. If the pH drops to 6.8, death may
occur.

How many times greater is the hydrogen ion concentration in blood at pH 6.8
compared with that at pH 7.4?

A 0.25 B 0.6 C 2.0 D 4.0

12 The decomposition of ammonia is a zero order reaction.

2NH3 (g)  N2 (g) + 3H2 (g)

Which of these graphs correspond to the decomposition of ammonia?

A B

C D

H1 Chemistry 8872/01 NYJC J2/2013 Prelim [Turn over


www.gradtutors.com
404
6

13 The decomposition of hydrogen peroxide, H2O2, is a first-order reaction.

It took 30 min for [H2O2] to drop from 1.00 mol dm-3 to 0.25 mol dm-3. At the
same temperature, how long will it take for [H2O2] to drop from 0.80 mol dm-3 to
0.10 mol dm-3?

A 28 min
B 30 min
C 45 min
D 60 min

14 The diagram represents the distribution of the kinetic energy of the particles
within a gas at temperature T.

Number of
particles

Kinetic
Energy

Which of the following statements is correct?

A When the temperature is decreased, the area under the curve will
decrease.

B When the temperature is increased, the proportion of particles with any


given energy will increase.

C When the temperature is decreased, the maximum of the curve will be


displaced to the right and downwards.

D When the temperature is increased, the proportion of particles with


energies above any given value will increase.

H1 Chemistry 8872/01 NYJC J2/2013 Prelim

www.gradtutors.com
405
7

15 Mixtures I, II, and III were prepared by adding 10 g of NaCl, SiCl4 and SiO2
respectively to 1 dm3 of water and stirring thoroughly.

Will there be effervescence when aqueous sodium carbonate is added to these


mixtures?

I II III
A no no yes
B no yes no
C no yes yes
D yes yes no

16 When heated with chlorine, the hydrocarbon 3,3–dimethylpentane undergoes


substitution.

How many different monochloro–substituted products can be obtained?

A 1 B 2 C 3 D 4

17 Given the following bond energies:

bond bond energy / kJ mol–1


C–C 350
C=C 610

Which of the following statements is true?

A The π bond is stronger than the σ bond.

B The C≡C bond energy is estimated to be 870 kJ mol–1.

C The π bond energy cannot be determined from the data.

D The C≡C bond energy is the sum of the C-C bond energy and C=C bond
energy.

H1 Chemistry 8872/01 NYJC J2/2013 Prelim [Turn over


www.gradtutors.com
406
8

18 A hydrocarbon on heating with cold dilute potassium manganate(VII) produces


the structure below as the only organic product.

HO CH3

HO OH

HO

What could be a possible structure of the hydrocarbon?

A CH3 B CH3

C CH3 D CH2

19 Ozone depletion potential (ODP) is a measure of the effectiveness of


chlorofluoroalkanes in destroying stratospheric ozone.

In which sequence are compounds listed in increasing order of their ODPs?

A CHClF2 < CH3CCl2F < CCl2FCClF2


B CHClF2 < CCl2FCClF2 < CH3CCl2F
C CH3CCl2F < CCl2FCClF2 < CHClF2
D CCl2FCClF2 < CHClF2 < CH3CCl2F

H1 Chemistry 8872/01 NYJC J2/2013 Prelim

www.gradtutors.com
407
9

20 (CH3)2C(OH)COOH, may be synthesised from (CH3)2CHBr, through a series of


reactions.

Which set of reagents, used in sequential order, would be the most suitable for
this synthesis?

A aqueous KOH, acidified KMnO4


B HCN with NaOH catalyst, dilute H2SO4
C HCN with NaOH catalyst, acidified KMnO4
D aqueous KOH, acidified KMnO4, HCN with NaOH catalyst, dilute H2SO4

21 Which of the following halides will react most rapidly with aqueous sodium
hydroxide?

A (CH3)3CF
B (CH3)3CCl
C (CH3)3CBr
D (CH3)3CI

22 Which of the following chemical tests can distinguish between 1-chlorobutane


and 2-chlorobutane?

A Heating with excess NaOH(aq), followed by adding aqueous iodine.


B Heating with limited NaOH(aq), followed by adding aqueous silver
nitrate.
C Heating with excess NaOH(aq), followed by adding aqueous potassium
dichromate (VI).
D Heating with excess NaOH(aq), followed by refluxing with acidified
aqueous potassium manganate (VII).

H1 Chemistry 8872/01 NYJC J2/2013 Prelim [Turn over


www.gradtutors.com
408
10

23 In the presence of a dilute alkali, some aldehydes and ketones undergo the
‘aldol reaction’ where they dimerise to form a hydroxylcarbonyl compound
(an aldol). For example, ethanal dimerises in this way to form 3-hydroxybutanal.

OH
CH3CHO CH3CHCH2CHO

OH

Which of the following compounds will undergo the aldol reaction to produce the
aldol shown below?

CH3 CH3

CH3CCH2C O

OH

A CH3COCH3
B CH3CH2CHO
C (CH3)2CHCHO
D CH3CH2COCH3

24 In which of the following sequence is the value of pKa decreasing for the
following compounds?

O O H3C
CH3 CH2
CH3 C CH3 OH Cl C H OH
C OH C C OH C

CH2Cl CH3 CH3 CH3

C D E F

A F, D, E, C
B D, F, C, E
C C, E, D, F
D E, C, F, D

H1 Chemistry 8872/01 NYJC J2/2013 Prelim

www.gradtutors.com
409
11

25 Compound G has the following structure.

CH2OH

COOCH3 COOH

Which of the following represents the product of its reaction with NaOH (aq) at
standard conditions?

A CH2OH B CH2O

O O O
C COOH C
C

O O O

C CH2OH D CH2O

O O
COOCH3 C COOCH3 C

O O

H1 Chemistry 8872/01 NYJC J2/2013 Prelim [Turn over


www.gradtutors.com
410
12

Section B

For each of the questions in this section one or more of the three numbered
statements 1 to 3 may be correct.

Decide whether each of the statements is or is not correct (you may find it helpful to
put a tick against the statements which you consider to be correct).

The responses A to D should be selected on the basis of

A B C D
1, 2 and 3 1 and 2 2 and 3 1 only
are only are only are is
correct correct correct correct

No other combination of statements is used as a correct response.

26 Element J has the following successive ionisation energies in kJ mol-1.

1st 2nd 3rd 4th 5th 6th 7th


1300 3400 5300 7400 11000 13000 21000

Which of the following statements about J is true?

1 J has 2 unpaired electrons in its valence orbital.


2 J forms JO2.
3 The first IE of J is lower than its succeeding element.

27 Which of the following mixtures of solutions can function as a buffer?

1 50 cm3 of 0.2 mol dm–3 of CH3COOH and 25 cm3 of 0.2 mol dm–3
Ba(OH)2
2 40 cm3 of 0.1 mol dm–3 of NH3 and 15 cm3 of 0.2 mol dm–3 HCl
3 25 cm3 of 0.1 mol dm–3 of CH3COO–Na+ and 25 cm3 of 0.05 mol dm–3
HCl

28 The chloride of element L dissolves in aqueous NaOH and its oxide has a high
melting point.

What could element L be?

1 aluminum
2 sodium
3 phosphorous

H1 Chemistry 8872/01 NYJC J2/2013 Prelim

www.gradtutors.com
411
13

The responses A to D should be selected on the basis of

A B C D
1, 2 and 3 1 and 2 2 and 3 1 only
are only are only are is
correct correct correct correct

No other combination of statements is used as a correct response.

29 Which of the following reagents can be used to distinguish the following two
compounds?

O OH

C O CH2
H2C OH C

U V

1 2,4-DNPH, warm
2 Na2CO3 (aq)
3 NaHB4 (aq)

H1 Chemistry 8872/01 NYJC J2/2013 Prelim [Turn over


www.gradtutors.com
412
14

The responses A to D should be selected on the basis of

A B C D
1, 2 and 3 1 and 2 2 and 3 1 only
are only are only are is
correct correct correct correct

No other combination of statements is used as a correct response.

30 An organic compound M undergoes the following reactions:

• It decolourises a solution of bromine in tetrachloromethane.


• It reacts with sodium bromide in concentrated sulfuric acid.
• It reacts with aqueous KOH to produce a compound with two alcohol
functional groups.

Which compounds could be M?

1 OH

Br

2 Br

Br

3 OH

Br

H1 Chemistry 8872/01 NYJC J2/2013 Prelim

www.gradtutors.com
413

NANYANG JUNIOR COLLEGE


2013 JC 2 PRELIMINARY EXAMINATION
Higher 1

1 B 11 D 21 D
2 D 12 C 22 A
3 D 13 C 23 A
4 A 14 D 24 B
5 C 15 B 25 C
6 C 16 C 26 A
7 A 17 B 27 C
8 D 18 B 28 B
9 A 19 A 29 B
10 C 20 D 30 D

www.gradtutors.com
414

NANYANG JUNIOR COLLEGE


JC 2 PRELIMINARY EXAMINATION
Higher 1

CANDIDATE
NAME

TUTOR’S
CLASS
NAME

CHEMISTRY 8872/02
Paper 2 19 September 2013
2 hours
Candidates answer Section A on the Question Paper.
Additional Materials: Answer Paper
Data Booklet
Graph Paper

READ THESE INSTRUCTIONS FIRST

Write your name and class on all the work you hand in.
Write in dark blue or black pen on both sides of the paper.
You may use a soft pencil for any diagrams, graphs or rough working.
Do not use staples, paper clips, highlighters, glue or correction fluid.

Section A
Answer all the questions.

Section B
Answer two questions on separate answer paper.

At the end of the examination, fasten all your work securely together.
The number of marks is given in brackets [ ] at the end of each question or part
question.

For Examiner’s Use

Section A

B5

B6

B7

Total

This document consists of 15 printed pages.

[Turn over
www.gradtutors.com
415
2 For
Examiner’s
Use
Section A

Answer all the questions in this section in the spaces provided.

1 (a) Silicon and sulfur are among the most abundant period 3 elements on Earth.

(i) Describe the variations in melting points of the elements silicon and
sulfur. Explain the variations in terms of their structures and bonding.

(ii) Describe the action of water on the oxides of silicon and sulfur, write
equations for any reactions that occur, and suggest the pH of each
solution formed.

[5]

H1 Chemistry 8872/02 NYJC J2/2013 Prelim

www.gradtutors.com
416
3 For
Examiner’s
Use
(b) Sulfur hexafluoride (SF6) is a colourless and odourless gas that is poorly
soluble in water. It is hypothesised that sulfur pentafluoride ion ( SF5− ) is very
soluble in water.

(i) Draw a dot-and-cross diagram to show the bonding in a sulfur


pentafluoride ion.

(ii) State and explain in detail how the shape of sulfur pentafluoride ion
arises.

shape

explanation

(iii) Explain with the aid of a diagram why sulfur pentafluoride ion is very
soluble in water.

[5]

[Total: 10]

H1 Chemistry 8872/02 NYJC J2/2013 Prelim [Turn over

www.gradtutors.com
417
4 For
Examiner’s
Use

2 (a) Given the following information, explain why the different compounds have
different pKa values at 298 K.

Compound pKa
Benzoic acid 4.19
2-methylbutanoic acid 4.80
Butan-2-ol 17.6

[4]

H1 Chemistry 8872/02 NYJC J2/2013 Prelim

www.gradtutors.com
418
5 For
Examiner’s
Use
(b) 25.0 cm3 of 2-methylbutanoic acid was titrated against 0.15 mol dm-3 of KOH
and the following titration curve was obtained.

(i) Explain what is meant by the term acid dissociation constant, Ka, of
2-methylbutanoic acid.

(ii) Calculate the concentration of 2-methylbutanoic acid used in this


experiment.

(iii) Using relevant values from (a) and b (ii), calculate the initial pH of
2-methylbutanoic acid acid.

H1 Chemistry 8872/02 NYJC J2/2013 Prelim [Turn over

www.gradtutors.com
419
6 For
Examiner’s
Use
(iv) Suggest and explain a suitable indicator for the above titration.

indicator

explanation

(v) With the aid of an equation, explain why the pH of the titration curve at
region X remains fairly constant.

[6]

[Total: 10]

H1 Chemistry 8872/02 NYJC J2/2013 Prelim

www.gradtutors.com
420
7 For
Examiner’s
Use
3 2-methylbutane is a component of natural gas. It is used as a fuel and also as an
active ingredient in toothpaste for sensitive teeth.

2-methylbutane (Mr = 72.0) can be synthesised from 3-methylbut-1-ene via a


reaction with hydrogen under suitable conditions. The enthalpy change of this
reaction is –116 kJ mol-1.

∆Hrxn = –116 kJ mol-1


+ H2

(a) (i) Suggest the conditions for this reaction.

(ii) Calculate the enthalpy change of combustion of 2-methylbutane given


the following enthalpy changes:
1
H2(g) + O2(g)  H2O(l) ∆H1 = –125 kJ mol-1
2
15
(CH3)2CHCH=CH2(g) + O2  5CO2(g) + 5H2O (l) ∆H2 = –3503 kJ mol-1
2

(iii) Calculate the minimum mass of 2-methylbutane required to heat 1000 g


of water from 25 °C to boiling point.

The specific heat capacity of water is 4.2 J g-1 K-1.

[5]

H1 Chemistry 8872/02 NYJC J2/2013 Prelim [Turn over

www.gradtutors.com
421
8 For
Examiner’s
Use
(b) Refer to the reaction scheme of the synthesis of 2-methylbut-2-ene from
2–methylbutane below.

(b) (i)

Step 1 Step 2

(i) Draw the structure of a possible intermediate for the reaction in the
space above.

(ii) State the type of reaction occuring in steps 1 and 2.

(iii) Give reagents and conditions for steps 1 and 2.

Step 1

Step 2

(iv) Will the yield of 2-methylbut-2-ene be high? Explain your answer.

[5]

[Total: 10]

H1 Chemistry 8872/02 NYJC J2/2013 Prelim

www.gradtutors.com
422
9 For
Examiner’s
Use
4 Pheremones are compounds an animal produces which can influence a change in
the behavior of another animal or insect of the same species. Compound A is an
isomer of an insect pheromone with a distinct peanut and hay-like odour
characteristic.

OH

Compound A

(a) In not more than 3 steps, suggest a synthetic route for the formation of B from
A. State clearly the reagents and conditions for each reaction step.

OH

CN
OH

Compound B

[5]

H1 Chemistry 8872/02 NYJC J2/2013 Prelim [Turn over

www.gradtutors.com
423
10 For
Examiner’s
Use

(b) Compound B can undergo a single reaction to form an amine. Draw the
displayed formula of this amine and state the type of reaction and the
reagents and conditions used in this reaction.

Type of reaction

Reagents and conditions

[3]

(c) Suggest the reagents and conditions for a chemical test that could be used to
distinguish between compounds A and B.

Reagents and conditions

Observations

[2]

[Total: 10]

H1 Chemistry 8872/02 NYJC J2/2013 Prelim

www.gradtutors.com
424
11

Section B

Answer two questions from this section on separate answer paper.

5 (a) Sketch a graph to show how ionic radius changes across period 3 elements and
explain the shape of your graph. [3]

(b) (i) With the aid of an equation, define the 2nd ionisation energy of gallium.

The graph below shows the 1st 8 ionisation energies of element D.

lg I.E.

1 2 3 4 5 6 7 8
Number of electrons removed

(ii) Prove with reasoning that Element D belongs to the same group as
gallium in the Periodic Table. Write the electronic configuration of the
other shell of element D.

(iii) Gallium(III) oxide is used as a precursor in the manufacturing of


semiconductor materials. Draw the dot-and-cross diagram of
gallium(III) oxide. [6]

(c) Period 3 oxides can be formed by combustion of the elements in oxygen.


Describe the formation of sodium oxide and phosphorus pentoxide and write
down the relevant equations of the reactions. [3]

www.gradtutors.com
H1 Chemistry 8872/02 NYJC J2/2013 Prelim [Turn over
425
12

(d) The addition of Cl2 to water forms hydrochloric acid and hypochlorous acid
according to the following equation.
Reaction I: Cl2 + H2O HCl + HClO

Hypochlorous acid hydrolyses in water to form a hypochlorite ion, OCl − . The


hypochlorite ion formed is the main component in bleach and disinfectant
agents.
Reaction II: HClO OCl − + H+

(i) Explain why the formation of the stable hypochlorite ion is facilitated by
dissolving chlorine gas in basic solutions, such as sodium hydroxide.

(ii) 1 mole of Cl2 was dissolved in 1 dm3 of water with a dissociation


coefficient of 0.75 in reaction I. Calculate the value of Kc, giving its units.

(iii) There are two unlabelled solutions X and Y, each containing the same
concentration of HCl (aq) and HOCl (aq). To identify the solutions a
student mixed equal volumes of NaOH (aq) with each solution. The
following results are obtained:

Reaction ∆T / °C
NaOH + X 4.8
NaOH + Y 5.6

Deduce the acid present in solutions X and Y and explain your answer. [8]

[Total: 20]

www.gradtutors.com
H1 Chemistry 8872/02 NYJC J2/2013 Prelim
426
13

6 (a) Methylbenzene can become 2-chlorobenzoic acid and 3-chlorobenzoic acid via
different reaction synthesis. Give the reagents and conditions required for the
synthesis of 2-chlorobenzoic acid, write the chemical equations and name the
type of reactions that occur. State and explain how 3-chlorobenzoic acid may be
synthesised instead of 2-chlorobenzoic acid. [6]

(b) Explain why 2-chlorobenzoic acid is a stronger acid than benzoic acid. [2]

(c) The equation for the formation of methylbenzene from benzene and
chloromethane is shown.
CH3

+ CH3Cl
+ HCl

A student performed an experiment to determine the order of reaction with


respect to chloromethane. The initial concentration of chloromethane was
0.0100 mol dm–3 and benzene was used in large excess. The concentration of
chloromethane at various times in the reaction mixture was found as follows:

Time / s [chloromethane] / mol dm–3


0 0.0100
40 0.0079
80 0.0062
120 0.0049
160 0.0038
200 0.0030

(i) Explain what is meant by order of reaction.

(ii) Plot the above data on suitable axes and use your graph to determine the
order of reaction with respect to chloromethane.

(iii) The initial concentration of chloromethane was 0.0100 mol dm–3 and
benzene was used in large excess. Suggest why benzene was used in
large excess in this experiment?

(iv) Explain with the aid of a suitable diagram, how the presence of a catalyst
can cause an increase in the rate of a chemical reaction. [12]

[Total: 20]

www.gradtutors.com
H1 Chemistry 8872/02 NYJC J2/2013 Prelim [Turn over
427
14

7 Maleic acid is one of two stereoisomers of butenedioic acid, HOOC(CH)2COOH.


Maleic acid is the cis-isomer whereas fumaric acid is the trans-isomer. This question
examines the chemistry of maleic acid and its derivatives.

(a) (i) Draw the full structural formula of maleic acid.

(ii) The two stereoisomers have different melting points as intramolecular


hydrogen bonding is possible only in one isomer. Identify this isomer and
hence, explain whether it has a higher or lower melting point, compared to
the other isomer. [4]

(b) Maleic acid is not an approved additive as long term consumption of high levels
of maleic acid can cause kidney damage. In May 2013, the Agri-food &
Veterinary Authority of Singapore recalled several food products imported from
Taiwan that were found to contain maleic acid. Most of the affected products
were tapioca starch balls used in the preparation of bubble tea beverages.

Some physical and chemical properties of maleic acid are given below.

Molecular formula: C4H4O4


Physical state and appearance: Powdered solid
Colour: White.
Mr: 116.08
Density: 1.59 g cm-3
Acid/Base properties: Dibasic
Water/Oil Distribution Coefficient: The product is more soluble in water;
[in oil ]
lg = −0.5
[in water ]

In a volumetric analysis to determine percentage by mass of maleic acid present


in tapioca starch balls, a 5.0 g sample was first powdered and dissolved in 100.0
cm3 of an oil-based solvent. Most of the maleic acid was extracted from the oil-
based solvent into an aqueous medium with 100.0 cm3 distilled water. 25.0 cm3
of the aqueous solution required 7.10 cm3 of 0.05 mol dm-3 aqueous sodium
hydroxide for complete neutralisation.

www.gradtutors.com
H1 Chemistry 8872/02 NYJC J2/2013 Prelim
428
15

(i) Using the data provided, determine the concentration of maleic acid
present in 100.0 cm3 of aqueous solution. You may assume the only acid
present in tapioca starch balls is maleic acid.

(ii) When a solute is added to two solvents, A and B, which do not mix, some
of the solute dissolves in each of the solvents and an equilibrium is set up
between the two solvents. It has been shown that for dilute solutions, the
ratio of the two concentrations in each of the solvents at equilibrium is a
constant known as the Dissociation Coefficient.

Using your answer in (i) and the data provided, determine the
concentration of maleic acid remaining in the oil-based solvent after
extraction.

(iii) Hence, using your answer in (i) and (ii) and data provided, determine the
percentage by mass of maleic acid in the sample of tapioca starch balls.
[7]

(c) (i) Maleic acid can be reduced to a compound S with molecular formula
C4H8O2. State the reagents required for this conversion and draw the full
structural formula of S.

(ii) Compound S has other structural isomers T and U all having the
molecular formula C4H8O2.

1 mole of T gives 1 mole of yellow precipitate with aqueous alkaline


iodine. T also decolourises aqueous bromine and exhibits geometric
isomerism.

1 mole of U also gives 1 moles of yellow precipitate with aqueous alkaline


iodine. However U does not decolourise aqueous bromine.

Deduce the structures of compounds T and U. Explain your reasoning


clearly. [9]

[Total: 20]

www.gradtutors.com
H1 Chemistry 8872/02 NYJC J2/2013 Prelim [Turn over
429

NANYANG JUNIOR COLLEGE


JC 2 PRELIMINARY EXAMINATIONo
Higher 1

CANDIDATE
NAME
ANSWERS

TUTOR’S
CLASS
NAME

CHEMISTRY 8872/02
Paper 2 19 September 2013
2 hours
Candidates answer Section A on the Question Paper.
Additional Materials: Answer Paper
Data Booklet
Graph Paper

READ THESE INSTRUCTIONS FIRST

Write your name and class on all the work you hand in.
Write in dark blue or black pen on both sides of the paper.
You may use a soft pencil for any diagrams, graphs or rough working.
Do not use staples, paper clips, highlighters, glue or correction fluid.

Section A
Answer all the questions.

Section B
Answer two questions on separate answer paper.

At the end of the examination, fasten all your work securely together.
The number of marks is given in brackets [ ] at the end of each question or part
question.

For Examiner’s Use

Section A

B5

B6

B7

Total

This document consists of 16 printed pages.

[Turn over
www.gradtutors.com
430
2 For
Examiner’s
Use
Section A

Answer all the questions in this section in the spaces provided.

1 (a) Silicon and sulfur are among the most abundant period 3 elements on Earth.
(i) Describe the variations in melting points of the elements silicon and
sulfur. Explain the variations in terms of their structures and bonding.
Silicon has giant molecular structure held together by numerous strong
covalent bonds between atoms. As large amount of energy is required
to break the strong covalent bonds, silicon has a higher melting point.
[1m]

Sulfur has simple molecular structure with weak van der Waals forces of
attractions between molecules. Lesser amounts of energy is required to
break the weak van der Waals forces of attraction hence sulfur has
lower melting point. [1m]
(ii) Describe the action of water on the oxides of silicon and sulfur, write
equations for any reactions that occur, and suggest the pH of each
solution formed. [5]
SiO2 is insoluble in water because energy from the weak van der Waals
forces between SiO2 and H2O molecules are unable to overcome the
strong covalent bonds between Si and O atoms (pH = 7). [1m]

Oxides of sulfur reacts with water to give a highly acidic solution (pH =
1-2). [1m]
SO2 (g) + H2O (l)  H2SO3 (aq) [1m for both]
SO3 (g) + H2O (l)  H2SO4 (aq)
(b) Sulfur hexafluoride (SF6) is a colourless and odourless gas that is poorly
soluble in water. It is hypothesised that sulfur pentafluoride ion ( SF5− ) is very
soluble in water.
(i) Draw a dot-and-cross diagram to show the bonding in a sulfur
pentafluoride ion.
- -
xx
x x
x F xx x
xx
x
xx xx x F xx xx
x
x
F x F
x x x
x x
x
F x xx F
x x x
x
xx x xx x

xx
S x xx
S
x xx
x x xx
x
x F x F x x
F F xx
x xx x
xx or xx xx

[1m]

www.gradtutors.com
H1 Chemistry 8872/02 NYJC J2/2013 Prelim
431
3 For
Examiner’s
Use
(ii) State and explain in detail how the shape of sulfur pentafluoride ion
arises.
There are 5 bond pairs and 1 lone pair of electrons around the central
sulfur atom. [1m]
Hence shape is square pyramidal. [1m]
(iii) Explain with the aid of a diagram why sulfur pentafluoride ion is very
soluble in water. [5]
SF5− is a molecular ion. The ion-dipole interaction between the ion and
water is strong enough to overcome the strong hydrogen bonds between
water molecules. [1m] Hence it is very soluble in water.

δ+
δ− H δ+
O H
l ll l
l l l l l l l l l ll l l

O δ−
l l l

δ+ H lllllll SF -
l

5 lllllllHδ+
llllll

H
l

δ+ Hδ+ Ion-dipole interaction


O
δ− [1m]

[Total: 10]

2 (a) Given the following information, explain why the different compounds have
different pKa values at 298 K.

Compound pKa
Benzoic acid 4.19
2-methylbutanoic acid 4.80
Butan-2-ol 17.6
[4]
The strength of an acid depends on the stability of the conjugate base. The
more stable the conjugate base, the stronger the acid.

The higher the pKa, the lower the Ka, the weaker the acid. Hence benzoic
acid is the strongest acid.

Hence, the order of stability of the conjugate base is


benzoate ion > 2-methylbutanoate ion > butoxide ion [1m]
In CH3(CH2)3O–, the presence of electron-donating –(CH2)3CH3 group
intensifies the negative charge on oxygen. Hence CH3(CH2)3O – is
destabilised. Butan-2-ol is the weakest acid. [1m]

www.gradtutors.com
H1 Chemistry 8872/02 NYJC J2/2013 Prelim [Turn over
432
4 For
Examiner’s
Use
In CH3CH2CH(CH3)CO2– and C6H5CO2–, the p-orbital of O– overlaps with the p
orbitals of the C=O bond. Hence the negative charge is effectively dispersed
over one carbon atom and two electronegative oxygen atoms. [1m] However,
C6H5CO2– is more stabilised than CH3CH2CH(CH3)CO2– as the negative
charge can be dispersed into benzene (more effectively) unlike CH3CO2–.
Benzoic acid is therefore a stronger acid than ethanoic acid and stronger
thanethanol. [1m]
(b) 25.0 cm3 of 2-methylbutanoic acid was titrated against 0.15 mol dm-3 of KOH
and the following titration curve was obtained.

(i) Explain what is meant by the term acid dissociation constant, Ka, of
2-methylbutanoic acid.
The acid dissociation constant is defined as Ka
H+   CH3CH2CH ( CH3 ) CO2 – 
= [1m]
[CH3CH2CH ( CH3 ) CO2H]

(ii) Calculate the concentration of 2-methylbutanoic acid used in this


experiment.
(0.15)(24.50)
[acid] = = 0.1470 mol dm-3 [1m]
(25.0)

(iii) Using relevant values from (a) and b (ii), calculate the initial pH of
2-methylbutanoic acid acid.
Given pKa = 4.80:
[H+] = (10 ) (0.1470) = 2.329 x 10-6 mol dm-3
-4.80

pH = 5.63 [1m]

www.gradtutors.com
H1 Chemistry 8872/02 NYJC J2/2013 Prelim
433
5 For
Examiner’s
Use
(iv) Suggest and explain a suitable indicator for the above titration.

Phenolphthalein is a suitable indicator. [1m] This is a weak acid-strong


base titration. The pH range of the indicator (8-10) falls within the pH
change at equivalence point (7-11) [1m]

(v) With the aid of an equation, explain why the pH of the titration curve at
region X remains fairly constant. [6]
When base is added, the added OH is removed by reacting with the
large reservoir of 2-methylbutanoic acid molecules present.

OH + CH3CH2CH(CH3)CO2H  CH3CH2CH(CH3)CO2–Na+ + H2O


[1m equation & explain]
[Total: 10]

3 2-methylbutane is a component of natural gas. It is used as a fuel and also as an


active ingredient in toothpaste for sensitive teeth.

2-methylbutane (Mr = 72.0) can be synthesised from 3-methylbut-1-ene via a


reaction with hydrogen under suitable conditions. The enthalpy change of this
reaction is –116 kJ mol-1.

∆Hrxn = –116 kJ mol-1


+ H2

(i) Suggest the conditions for this reaction.


Ni/Heat or Pt/rt or Pd/rt [1m]
(ii) Calculate the enthalpy change of combustion of 2-methylbutane given
the following enthalpy changes:
1
H2(g) + O2(g)  H2O(l) ∆H1 = –125 kJ mol-1
2
15
(CH3)2CHCH=CH2(g) + O2  5CO2(g) + 5H2O (l) ∆H2 = –3503 kJ mol-1
2

∆Hrxn = Σ∆Hc(reactants) – Σ∆Hc(products)


-116 = (-125) + (-3503) – ∆Hc(2-methylbutane) [1m]
∆Hc(2-methylbutane) = –3510 kJ mol-1 (3sf) [1m]

www.gradtutors.com
H1 Chemistry 8872/02 NYJC J2/2013 Prelim [Turn over
434
6 For
Examiner’s
Use
(iii) Calculate the minimum mass of 2-methylbutane required to heat 1000 g
of water from 25 °C to boiling point. The specific heat capacity of water
is 4.2 J g-1 K-1. [5]
Heat absorbed by water = mc∆T = (1000)(4.2)(100-25) = 315000 J [1m]

∆H = -Q/n
(-3510x1000) = -315000/(m/72)
m = 6.46 g (3sf) [1m]

(b) Refer to the reaction scheme of the synthesis of 2-methylbut-2-ene from


2–methylbutane below.

Step 1 Step 2

(i) Draw the structure of a possible intermediate for the reaction in the
space above.

or [1m]

(ii) State the type of reaction occuring in steps 1 and 2.


Step 1: substitution Step 2: elimination [1m for both]

(iii) Give reagents and conditions for steps 1 and 2.


Step 1: (limited) Cl2, UV light [1m]
Step 2: ethanolic NaOH, heat under reflux [1m]

(iv) Will the yield of 2-methylbut-2-ene be high? Explain your answer. [5]
No. Substitution of alkanes is uncontrollable and there will be a mixture
of various substitution products. [1m]
[Total: 10]

www.gradtutors.com
H1 Chemistry 8872/02 NYJC J2/2013 Prelim
435
7 For
Examiner’s
Use
4 Pheremones are compounds an animal produces which can influence a change in
the behavior of another animal or insect of the same species. Compound A is an
isomer of an insect pheromone with a distinct peanut and hay-like odour
characteristic.

OH

Compound A

(a) In not more than 3 steps, suggest a synthetic route for the formation of B from
A. State clearly the reagents and conditions for each reaction step.

OH

CN
OH

Compound B
[5]

O O
Excess conc. H2SO4
170 °C
OH

300 oC, 60 atm,


conc. H3PO4
catalyst,
OH H2O (g)
O

CN
HCN(g) in the
Br
presence of a
OH
trace amount of
NaCN
10 – 20 °C

www.gradtutors.com
H1 Chemistry 8872/02 NYJC J2/2013 Prelim [Turn over
436
8 For
Examiner’s
Use
(b) Compound B can undergo a single reaction to form an amine. Draw the
displayed formula of this amine and state the type of reaction and the
reagents and conditions used in this reaction. [3]
H

H H O H H

H C C C C C H

H H C O H

N H
H H

Type of reaction: Reduction


Reagents and conditions: LiAlH4 in dry ether

(c) Suggest the reagents and conditions for a chemical test that could be used to
distinguish between compounds A and B. [2]

Test: NaOH (aq), I2, warm


Observations with A: A yellow ppt is observed.
Observations with B: A yellow ppt is not observed.

OR

Test: 2,4-dinitrophenylhydrazine, warm


Observations with A: An orange ppt is observed.
Observations with B: An orange ppt is not observed.
[Total: 10]

www.gradtutors.com
H1 Chemistry 8872/02 NYJC J2/2013 Prelim
437
9

Section B

Answer two questions from this section on separate answer paper.

5 (a) Sketch a graph to show how ionic radius changes across period 3 elements and
explain the shape of your graph. [3]

Ionic
Radius










Na+ Mg2+ Al3+ Si4+ P3- S2- Cl- [1]

Anions have an additional shell of electrons than cations of the same period thus
have larger radii. [1]

Across the period, ionic radius decreases for cations and anions (ions which are
isoelectronic) as nuclear charge increase while shielding effect remains the
same. Hence effective nuclear charge increases and the outermost electron
shell is pulled slightly closer, giving rise to the decreasing trend. [1]

(b) (i) With the aid of an equation, define the 2nd ionisation energy of gallium.
Ga+ (g)  Ga2+(g) + e- [1m]

The graph below shows the 1st 8 ionisation energies of element D.

lg I.E.

1 2 3 4 5 6 7 8
Number of electrons removed

www.gradtutors.com
H1 Chemistry 8872/02 NYJC J2/2013 Prelim [Turn over
438
10

(ii) Prove with reasoning that Element D belongs to the same group as
gallium in the Periodic Table. Write the electronic configuration of the
outer shell of element D.
The 4th electron is from an inner quantum shell [1m] and it experiences
increased electrostatic attraction with the nucleus. A large amount of
energy is required to remove the 4th electron. [1m]
ns2 np1 [1m]
(iii) Gallium(III) oxide is used as a precursor in the manufacturing of
semiconductor materials. Draw the dot-and-cross diagram of
gallium(III) oxide. [6]

[2m; number of ions, charge on ions, number and position of valence


electrons]
(c) Period 3 oxides can be formed by combustion of the elements in oxygen.
Describe the formation of sodium oxide and phosphorus pentoxide and write
down the relevant equations of the reactions. [3]
2 Na (s) + ½ O2 (g)  Na2O (s)
Sodium burns vigorously with an intense yellow flame to form a white ionic solid
of sodium oxide. [1m]
P4 (s) + 5 O2 (g)  P4O10 (s) [1m for both]
Phosphorus reacts rapidly with O2 with a pale buish green flame to form a white
P4O10 solid. [1m]
(d) The addition of Cl2 to water forms hydrochloric acid and hypochlorous acid
according to the following equation.
Reaction I: Cl2 + H2O HCl + HClO

Hypochlorous acid hydrolyses in water to form a hypochlorite ion, OCl − . The


hypochlorite ion formed is the main component in bleach and disinfectant
agents.
Reaction II: HClO OCl − + H+

(i) Explain why the formation of the stable hypochlorite ion is facilitated by
dissolving chlorine gas in basic solutions, such as sodium hydroxide.
NaOH will react with HCl, reducing the concentration of HCl. The position
of equilibrium of reaction 1 will shift to the right to increase the [HCl] and
increases the [HClO]. [1m]
When [HClO] increases, the position of equilibrium will shift to the right in
reaction 2 to reduce the [HClO] and forms OCl-. This stabilises the
hypochlorite in NaOH. [1m]

www.gradtutors.com
H1 Chemistry 8872/02 NYJC J2/2013 Prelim
439
11

(ii) 1 mole of Cl2 was dissolved in 1 dm3 of water with a dissociation


coefficient of 0.75 in reaction I. Calculate the value of Kc, giving its units.

Cl2 + H2O HCl + HClO


Initial 1 0 0
Change -0.75 +0.75 +0.75
Equilibrium 0.25 0.75 0.75 [1m for equilibrium values]

0.75 0.75
( )( )
Kc = 1 1 = 2.25 mol dm-3
0.25
( )
1
[1m for correct Kc expression, 1m for values and units]

(iii) There are two unlabelled solutions X and Y, each containing the same
concentration of HCl (aq) and HOCl (aq). To identify the solutions a
student mixed equal volumes of NaOH (aq) with each solution. The
following results are obtained:

Reaction ∆T / °C
NaOH + X 4.8
NaOH + Y 5.6

Deduce the acid present in solutions X and Y and explain your answer. [8]

X contains HOCl. Y contains HCl. [1m]

In the neutralisation reactions, the enthalpy change of reaction of the


weak acid is less exothermic [1m] than that of a strong acid as it will
involve the enthalpy change of dissociation of H+ [1m] from the weak acid
as it undergoes partial dissociation. Energy is required for the
dissociation.
[Total: 20]

www.gradtutors.com
H1 Chemistry 8872/02 NYJC J2/2013 Prelim [Turn over
440
12

6 (a) Methylbenzene can become 2-chlorobenzoic acid and 3-chlorobenzoic acid via
different reaction synthesis. Give the reagents and conditions required for the
synthesis of 2-chlorobenzoic acid, write the chemical equations and name the
type of reactions that occur. State and explain how 3-chlorobenzoic acid may be
synthesised instead of 2-chlorobenzoic acid. [6]
Step 1: Substitution of methylbenzene to form 2-chloromethylbenzene.

Reagent & Condition: Cl2 (l) with anhydrous FeCl3

CH3 CH3
Cl
+ Cl 2 + HCl

[2m for name of reaction, reagent & condition and equation]

Step 2: Oxidation of 2-chloromethylbenzene to form 2-chlorobenzoic acid.

Reagent & Condition: KMnO4 (aq), H2SO4 (aq), Heat under reflux.

CH3 COOH
Cl 3 O Cl
+ H 2O

[2m for name of reaction, reagent & condition and equation]

3-chlorobenzoic acid can be synthesised from methylbenzene by changing the


sequence of reactions; oxidation followed by substitution. [1m]

The products are different as different existing substituent affects the position of
the incoming substituent. [1m] Methyl substituent is 2,4-directing while
carboxylic acid substituent is 3,5-directing.

(b) Explain why 2-chlorobenzoic acid is a stronger acid than benzoic acid. [2]
The electronegative Cl atom of chlorobenzenoic acid will exert an electron–
withdrawing effect on the corresponding carboxylate anion. [1m]
The negative charge becomes more dispersed and the conjugate base
becomes more stable. [1m]

Hence the dissociation of chlorobenzoic acid to give H+ is more favoured.

www.gradtutors.com
H1 Chemistry 8872/02 NYJC J2/2013 Prelim
441
13

(c) The equation for the formation of methylbenzene from benzene and
chloromethane is shown.

CH3

+ CH3Cl
+ HCl

A student performed an experiment to determine the order of reaction with


respect to chloromethane. The initial concentration of chloromethane was
0.0100 mol dm–3 and benzene was used in large excess. The concentration of
chloromethane at various times in the reaction mixture was found as follows:

Time / s [chloromethane] / mol dm–3


0 0.0100
40 0.0079
80 0.0062
120 0.0049
160 0.0038
200 0.0030

(i) Explain what is meant by order of reaction.


The term “order of reaction” refers to the power to which the
concentration of the reactant is raised in the experimentally determined
rate equation. [1m]

Or
For a general rate law, rate = k[A]m experimentally determined
[A] is the concentration of the reactant A,
m is the order of the reaction with respect to the reactant A

(ii) Plot the above data on suitable axes and use your graph to determine the
order of reaction with respect to chloromethane.

www.gradtutors.com
H1 Chemistry 8872/02 NYJC J2/2013 Prelim [Turn over
442
14

[1m for points plotted accurately and smooth curved line drawn, 1m for
axes labeled correctly, 1m for showing clearly on the graph dotted lines to
find two t1/2]

From the graph, half-lives are approximately constant at 120s. [1m]


Hence, order of reaction with respect to CH3Cl is 1. [1m]

(iii) The initial concentration of chloromethane was 0.0100 mol dm–3 and
benzene was used in large excess. Suggest why benzene was used in
large excess in this experiment?
Benzene was used in large excess so that its concentration will remain
almost constant throughout the experiment. As such, the rate of reaction
will only depend on chloromethane concentration [1m] thus enabling the
determination of the order of reaction with respect to chloromethane. [1m]

(iv) Explain with the aid of a suitable diagram, how the presence of a catalyst
can cause an increase in the rate of a chemical reaction. [12]

[1m for correct shape and axes, 1m for labelling and legend]

www.gradtutors.com
H1 Chemistry 8872/02 NYJC J2/2013 Prelim
443
15

In the presence of a catalyst, reaction has a different mechanism with a


lower activation energy compared to the uncatalysed reaction. [1m]
More molecules will possess energy greater than this lowered activation
energy, hence frequency of effective collisions will increase. [1m]
[Total: 20]

7 Maleic acid is one of two stereoisomers of butenedioic acid, HOOC(CH)2COOH.


Maleic acid is the cis-isomer whereas fumaric acid is the trans-isomer. This question
examines the chemistry of maleic acid and its derivatives.
(a) (i) Draw the full structural formula of maleic acid.

O O
H O C C O H
C C
H H

Must be cis and trigonal planar wrt C [1m]


Full structural/displayed formula [1m]
(ii) The two stereoisomers have different melting points as intramolecular
hydrogen bonding is possible only in one isomer. Identify this isomer and
hence, explain whether it has a higher or lower melting point, compared to
the other isomer. [4]
Maleic acid [1m] has intramolecular hydrogen bonding (as the two
COOH groups are closer to each other).

Maleic acid should have a lower melting point as it can form less
extensive intermolecular hydrogen bonds (between molecules) hence
less energy is required to break the weak hydrogen bonds. [1m]
(b) Maleic acid is not an approved additive as long term consumption of high levels
of maleic acid can cause kidney damage. In May 2013, the Agri-food &
Veterinary Authority of Singapore recalled several food products imported from
Taiwan that were found to contain maleic acid. Most of the affected products
were tapioca starch balls used in the preparation of bubble tea beverages.

Some physical and chemical properties of maleic acid are given below.
Molecular formula: C4H4O4
Physical state and appearance: Powdered solid
Colour: White.
Mr: 116.08
Density: 1.59 g cm-3
Acid/Base properties: Dibasic
Water/Oil Distribution Coefficient: The product is more soluble in water;
[in oil ]
lg = −0.5
[in water ]
www.gradtutors.com
H1 Chemistry 8872/02 NYJC J2/2013 Prelim [Turn over
444
16

In a volumetric analysis to determine percentage by mass of maleic acid present


in tapioca starch balls, a 5.0 g sample was first powdered and dissolved in 100.0
cm3 of an oil-based solvent. Most of the maleic acid was extracted from the oil-
based solvent into an aqueous medium with 100.0 cm3 distilled water. 25.0 cm3
of the aqueous solution required 7.10 cm3 of 0.05 mol dm-3 aqueous sodium
hydroxide for complete neutralisation.
(i) Using the data provided, determine the concentration of maleic acid
present in 100.0 cm3 of aqueous solution. You may assume the only acid
present in tapioca starch balls is maleic acid.
7.10
nNaOH = ×0.05
1000
= 3.550×10-4 mol
Since maleic acid is diprotic,
1
nacid in 25.0 cm3 = ×3.550×10-4
2
= 1.775×10-4 mol [1m]
100.0
nacid in 100.0 cm3 = ×1.775×10-4
25.0
= 7.100×10-4 mol [ecf 1m]
7.100 × 10-4
[acid] = = 7.100×10-3 mol dm-3 [ecf 1m]
0.100
*If student just find concentration in 25.0cm3 sample [2m]
(ii) When a solute is added to two solvents, A and B, which do not mix, some
of the solute dissolves in each of the solvents and an equilibrium is set up
between the two solvents. It has been shown that for dilute solutions, the
ratio of the two concentrations in each of the solvents at equilibrium is a
constant known as the Dissociation Coefficient.

Using your answer in (i) and the data provided, determine the
concentration of maleic acid remaining in the oil-based solvent after
extraction.
[in oil ]
lg = −0.5
[in water ]
[in oil ]
lg = −0.5
[7.100 × 10 −3 ]
[in oil ]
−2
= 10 −0.5
[7.100 × 10 ]
[in oil] =2.245x10-3 mol dm-3 [ecf1m]

www.gradtutors.com
H1 Chemistry 8872/02 NYJC J2/2013 Prelim
445
17

(iii) Hence, using your answer in (i) and (ii) and data provided, determine the
percentage by mass of maleic acid in the sample of tapioca starch balls.
[7]
nacid total = nacid (in water)+nacid (in oil)
100 100
= 7.100x10-3 × + 2.245x10-3 × [ecf1m]
1000 1000
= 9.345 × 10−4 mol
macid = 9.345×10-4 × 116.08 [ecf1m]
= 0.108 g
0.108
%mass = × 100% = 2.17% [ecf1m]
5.0
(c) (i) Maleic acid can be reduced to a compound S with molecular formula
C4H8O2. State the reagents required for this conversion and draw the full
structural formula of S.
Reagents and conditions: LiAlH4 in dry ether [1m]

S: H O O H

H C C C C H

H H H H [1m]

(ii) Compound S has other structural isomers T and U all having the
molecular formula C4H8O2.

1 mole of T gives 1 mole of yellow precipitate with aqueous alkaline


iodine. T also decolourises aqueous bromine and exhibits geometric
isomerism.

1 mole of U also gives 1 moles of yellow precipitate with aqueous alkaline


iodine. However U does not decolourise aqueous bromine.

Deduce the structures of compounds T and U. Explain your reasoning


clearly. [9]

OH H O
T: OH H OH U:
H C C C CH 3
H C C C CH 3
H H
H
[2m]

www.gradtutors.com
H1 Chemistry 8872/02 NYJC J2/2013 Prelim [Turn over
446
18

Deductions

OH

H C CH 3

H
in T undergoes oxidation with aq I2 to form a yellow ppt. [1m]

alkene in T undergoes addition with aq Br2. [1m]

T exhibits geometric isomerism because each C in C=C is bonded to 2


different groups. [1m]

alkene is absent in U since it does not react with aq Br2. [1m]

H C CH 3 in U undergoes oxidation with aq I2 to form a yellow ppt. [1m]

OH

H C CH 3

Since only 1 mole of CHI3 is formed, H cannot also be


present in U [1m], hence alcohol should be on the terminal carbon.

*6 Marking points but MAX 5 for deductions.


*If students write methyl alcohol or methyl carbonyl present in T/U… give
[1m]
[Total: 20]

www.gradtutors.com
H1 Chemistry 8872/02 NYJC J2/2013 Prelim
447

Name: Index No.: CT Group: 12

PIONEER JUNIOR COLLEGE

2013 JC2 Preliminary Examination


HIGHER 1

CHEMISTRY 8872/01
Paper 1
26 September 2013
Additional Material: Data Booklet
Multiple Choice Answer Sheet 50 minutes

READ THESE INSTRUCTIONS FIRST

Write in soft pencil.


Do not use staples, paper clips, highlighters, glue or correction fluid.
Write your name, Centre number and index number on the Answer Sheet in the spaces provided
unless this has been done for you.

There are thirty questions on this paper. Answer all questions. For each question, there are four
possible answers A, B, C and D.
Choose the one you consider correct and record your choice in soft pencil on the separate
Answer Sheet.

Read the instructions on the Answer Sheet very carefully.

Each correct answer will score one mark. A mark will not be deducted for a wrong answer.
Any rough working should be done in this booklet.

This document consists of 12 printed pages.

www.gradtutors.com
448
2
Section A
For each question, there are four possible answers labelled A, B, C and D. Choose the
one you consider to be correct.

1 Group I and II ionic hydrides react with water:

H– + H2O  OH– + H2

In an experiment, 1 g samples of each of the following four ionic hydrides are


treated with an excess of water.

Which sample produces the greatest volume of hydrogen gas?

A MgH2 B CaH2
C LiH D NaH

2 A 25 cm3 sample of a 0.1 mol dm–3 ASO4 solution requires 10 cm3 of 0.1 mol dm–3 of
acidified potassium manganate(VII) solution for complete reaction.

What is the final oxidation state of element A?

A +3 B +4
C +5 D +6

3 Which electronic configuration represents an atom of an element, E, that forms a


simple ion, E2–?

A 1s2 2s2 2p6 3s2 3p2


B 1s2 2s2 2p6 3s2 3p4
C 1s2 2s2 2p6 3s2 3p6
D 1s2 2s2 2p6 3s2 3p6 3d5 4s1

4 Which of the following sets consists of three compounds with different types of
structure?

A P4O10, Na2O, SiO2


B AlCl3, Cu2O, SiCl4
C AlF3, MgCl2, BN
D BeO, SO2, BH3

www.gradtutors.com
449
3
5 Which statement on the chemical properties of the oxides of the third period of the
Periodic Table is true?

A P4O10 reacts with water to give OH-.


B Al2O3 is soluble in both NaOH and HCl.
C SiO2 forms a solution of pH 6 when dissolved in water at room temperature.
D Na2O and MgO can be mixed in water to give an approximately neutral solution.

6 An element J does not react with cold water. However J reacts vigorously with
oxygen to give an oxide which reacts readily with water forming an acidic solution.

What is element J?

A magnesium
B aluminium
C silicon
D phosphorus

7 Warfarin is used as a rat poison.

O O

  
CH CH2 C CH3
OH O

Warfarin

Which of the following gives the hybridisation of the carbon atoms at positions α, β
and γ respectively?

α β γ
2
A sp sp sp
B sp2 sp3 sp2
C sp3 sp2 sp
D sp3 sp3 sp3

www.gradtutors.com
450
4
8 A student used the apparatus below to heat a beaker containing 300 g of water.

thermometer
beaker containing
300 g of water

burner containing
propan-1-ol
The following data were recorded:

Mass of propan-1-ol burnt = mg


Change in temperature of water = ∆T °C

Given that:

Relative molecular mass of propan-1-ol = 60.0


Enthalpy change of combustion of propan-1-ol = 2021 kJ mol−1
Specific heat capacity of water = c kJ kg−1 K−1

What is the efficiency of this heating process?

m  2021 1000 300  c  ∆T  60.0


A  100% B  100%
300  c  ∆T  60.0 m  2021
m  c  ∆T  60.0 300  c  ∆T  60.0
C  100% D  100%
300  2021 1000 m  2021 1000

9 Use of the Data Booklet is relevant to this question.

Phosphorus, P4, has the following molecular structure:

P P 

Assume that nitrogen were to form a similar molecule, N4.

2N2(g)  N4(g)

By considering the bonds broken and the bonds formed, what would be the value of
∆H for the above reaction?

A 1028 B 1328 C 1954 D 2628

www.gradtutors.com
451
5
10 The following equilibrium exists in a system containing carbon monoxide and
hydrogen gases.

CO(g) + 2H2(g) ⇌ CH3OH(g) ∆H < 0

Which of the following conditions would result in an increase in amount of methanol


produced?

A adding a catalyst
B heating the system
C adding of carbon monoxide
D lowering the pressure of the system

11 Given that
A2(g) + 4C(g) ⇌ 2AC2(g) Kc = 4.8 (numerical value)

it follows that, for the reaction,

AC2(g) ⇌ ½A2(g) + 2C(g) Kc = X (numerical value)

What would be the value of X?

1
A B 2.4
4 .8
1 1
C D
2. 4 4 .8

12 In which reaction is ammonia acting as an acid?

A 2NH3(l) + 2Na(s)  2NaNH2(s) + H2(g)


B 2NH3(g) + 3CuO(s)  3Cu(s) + N2(g) + 3H2O(g)
C NH3(g) + HCl(g) → NH4Cl(s)
D NH3(g) + H2O(l) ⇌ NH4+(aq) + OH-(aq)

13 The rate equation for the reaction W + 2X + Y → Z is given below:

Rate = k[X]2[Y]

If the concentration of W is tripled, the concentration of X is halved and the


concentration of Y is doubled, by what factor is the rate of reaction being changed?

A 0.5 B 1.0 C 2.0 D 4.0

www.gradtutors.com
452
6
14 Hydrogen and nitrogen monoxide can react to form nitrogen and steam.

2H2(g) + 2NO(g)  N2(g) + 2H2O(g)

The rate of reaction is first order with respect to hydrogen and second order with
respect to nitrogen monoxide.

0.100 mol dm-3 of H2 and 5.00 mol dm-3 of NO were put into a sealed flask of fixed
volume. It was found that there was 0.0250 mol dm-3 of H2 left 40 minutes later.

Which of the following statement is correct?

A An excess concentration of 5.00 mol dm-3 NO(g) is used so that the rate of
reaction will remain constant as reaction proceeds.
B There was 0.0250 mol dm-3 of N2(g) formed after 20 minutes.
C There was 0.0125 mol dm-3 of H2(g) left after 80 minutes.
D The unit of the rate constant is s-1.

15 Alkanes undergo substitution with Cl2 in the presence of ultraviolet light.

Which of the following could have been a possible starting material for the synthesis
of 2-chloro-2-methylpentane?

A CH3CH2CH2CH2CH3
B CH3CH(CH3)CH2CH2CH3
C CH3CH2CH(CH3)CH2CH3
D CH3CH2CH2CH2CH2CH3

16 Bromine in an inert solvent is added separately to hexane, hexene, benzene and


methylbenzene in the presence of sunlight.

Which of the following will not give the same observation as the others?

A hexane
B hexene
C benzene
D methylbenzene

www.gradtutors.com
453
7
17 Ethanedioic acid can be produced by the following steps:

H H
H H
Step I Step II
C C H C C H HO C C OH
H H
OH OH O O

Which reagents are used for steps I and II?

Step I Step II
A cold alkaline KMnO4 acidified K2Cr2O7, heat under reflux
B cold alkaline KMnO4 acidified KMnO4, distil
C hot alkaline KMnO4 acidified K2Cr2O7, distil
D hot alkaline KMnO4 acidified KMnO4, heat under reflux

18 How many different isomeric alkenes, including geometric isomers, could be


produced when C6H5CH2CHBrCH3 reacts with hot ethanolic KOH?

A 3 B 4 C 5 D 6

19 Aqueous silver nitrate was added at the same time to separate solutions of
chloroethane and iodoethane. The first signs of a reaction were in the sample
containing iodoethane.

Why was the reaction with iodoethane noticed first?

A The chloroethane also reacted with aqueous silver nitrate but gave a soluble
product.
B The chloroethane reacted more slowly because the carbon-chlorine bond is less
polar than the carbon-iodine bond.
C The iodoethane reacted more quickly because the carbon-iodine bond is weaker
than the carbon-chlorine bond.
D The chloroethane reacted more slowly because the carbon-chlorine bond is
longer than the carbon-iodine bond.

www.gradtutors.com
454
8
20 In 1911, Weizmann carried out a historically important conversion, changing starch
into a mixture of butan-1-ol and propanone.

Which of the following reagents will not be able to distinguish the products of this
conversion?

A acidified manganate(VII) ions


B 2,4-dinitrophenylhydrazine
C alkaline aqueous iodine
D Fehling’s solution

21 Methylbenzene can be used as an additive in unleaded petrol. It can be reacted with


chlorine in two ways, depending on the conditions of the reaction, as shown below.
AgNO3(aq)
CH3
I P white ppt observed
heat

AgNO3(aq)
II no white ppt observed
Q
heat

Which of the following set of information is correct?

I P II Q

CH2C l
A Cl2, uv CH3 Cl2,
Cl Anhydrous AlCl3

CH2C l CH3
B Cl2, uv Cl2,
Cl
Anhydrous AlCl3

CH3 CH2C l
C Cl2, Cl2, uv
Cl
Anhydrous AlCl3

CH2C l CH3
D Cl2, Cl2, uv
Cl
Anhydrous AlCl3

www.gradtutors.com
455
9
22 This question is about an organic compound X.

 When X was reacted with phosphorous pentachloride, fumes of hydrogen


chloride were evolved.

 When X was warmed with acidified aqueous potassium dichromate(VI), the


solution turned green.

Which of the following is a possible identity for X?

A CH3CH2CHO B CH3CH2CO2H
C CH3CH(OH)CH3 D CH3COCH3

23 Which sequence shows the correct order of increasing value of Ka?

A CH3CO2H, ClCH2CH2OH, CH3CH2OH


B ClCH2CO2H, BrCH2CO2H, ICH2CO2H
C CH3CHClCO2H, ClCH2CH2CO2H, CH3CH2CO2H
D Cl2CHCH2CO2H, ClCH2CHClCO2H, CH3CCl2CO2H

24 Ester Y, which is used in perfumes, has the molecular formula C6H12O2. It is found
that one of the products of acid hydrolysis of Y gives a yellow precipitate with hot
aqueous alkaline iodine.

What is the structural formula of Y?

A CH3CO2CH2CH(CH3)2 B CH3CH2CO2CH2CH2CH3
C (CH3)2CHCO2CH2CH3 D CH3CH2CH(CH3)CO2CH3

25 A compound W with molecular formula C5H8O3 has the following properties.

 It gives an orange precipitate with 2,4-dinitrophenylhydrazine


 It gives a yellow precipitate when warmed with alkaline aqueous iodine
 It does not react with Fehling’s solution
 It reacts with Na2CO3 to liberate a gas which gives a white precipitate with
limewater.

Which of the following is a possible identity for W?

A CHOCH2CH2CH2CH3
B CHOCH2CH2CH2CO2H
C CH3CH2COCH2CO2H
D CH3COCH2CH2CO2H

www.gradtutors.com
456
10
Section B

For the next five questions, one or more of the three numbered statements 1 to 3 may be
correct.

Decide whether each of the statements is or is not correct (you may find it helpful to put a
tick against the statements that you consider to be correct).

The response A to D should be selected on the basis of

A B C D
1, 2 and 3 are 1 and 2 only are 2 and 3 only are 1 only is correct.
correct. correct. correct.

No other combination of statements is used as a correct response.

26 The table below shows the lattice energies of two compounds.

Compound Lattice energy / kJ mol-1


NaF −915
MgO −3933

Which of the following statements help to explain the difference between these two
values?

1 In each of these compounds, the ions are isoelectronic.

2 The electrostatic force of attraction between doubly charged ions is about four
times that between singly charged ions.
3 The interionic distance in NaF is larger than that in MgO.

27 The reaction shown is reversible.


N2O4(g) 2NO2(g) ∆H = +61.7 kJ mol-1
Which of the following will decrease when pressure is increased while temperature is
kept constant?

1 The proportion of NO2(g) present at equilibrium.


2 The value of the equilibrium constant Kc.
3 The activation energies of both forward and reverse reactions.

www.gradtutors.com
457
11
The response A to D should be selected on the basis of

A B C D
1, 2 and 3 are 1 and 2 only are 2 and 3 only are 1 only is correct.
correct. correct. correct.

No other combination of statements is used as a correct response.

28 Which set of solutions of equal volume, when mixed, will not give an acidic buffer?

1 0.10 mol dm–3 CH3CO2H and 0.10 mol dm–3 NaOH


2 0.10 mol dm–3 CH3CO2H and 0.05 mol dm–3 Ca(OH)2
3 0.05 mol dm–3 HCl and 0.05 mol dm–3 CH3CO2Na

29 Compounds P and Q are important flavours in citrus fruits. These compounds are
commonly used in the food and perfume industries.

CH 3 CH3

CHO CH 2OH

H3C CH 2 H 3C CH 3
P Q

Which statements concerning P and Q are correct?

1 Q can be distinguished from P by using sodium metal.


2 P can be converted to Q using lithium aluminium hydride in dry ether.
3 Reaction of P with hot acidified KMnO4 gives two organic products.

www.gradtutors.com
458
12
The response A to D should be selected on the basis of

A B C D
1, 2 and 3 are 1 and 2 only are 2 and 3 only are 1 only is correct.
correct. correct. correct.

No other combination of statements is used as a correct response.

30 The diagram below shows the structure of salicyclic acid.

CO2H
OH

salicyclic acid

Which compounds give salicyclic acid on acid hydrolysis?

CN
Cl
1

CO 2CH 3
OCOCH 3
2

CO2CH3
ONa
3

End of Paper

www.gradtutors.com
459
13
Answers

1 C 11 D 21 B
2 B 12 A 22 C
3 B 13 A 23 D
4 A 14 B 24 C
5 B 15 B 25 D
6 D 16 C 26 C
7 B 17 A 27 D
8 D 18 A 28 A
9 A 19 C 29 D
10 C 20 D 30 C

7A
7B
8C
8D

www.gradtutors.com
460

Name: Index No.: CT Group: 12

PIONEER JUNIOR COLLEGE

2013 JC2 Preliminary Examination


HIGHER 1

CHEMISTRY 8872/02
Paper 2
18 September 2013

Candidates to answer Section A on the Question Paper

Additional Materials: Data Booklet


Writing Paper
Graph Paper 2 hours

READ THESE INSTRUCTIONS FIRST

Write your name, index number and CT group on all the work you hand in.
Write in dark blue or black pen.
You may use a pencil for any diagrams, graphs or rough working.

Section A
Answer all questions.

Section B
Answer any two questions on separate answer paper.

You are advised to show all working in calculations.


You may use a calculator.

The number of marks is given in brackets [ ] at the end of each question or part question.

At the end of the examination, fasten all your work securely together.

FOR EXAMINER’S USE


Section A Section B
1 /9 6 / 20
2 /7 7 / 20
3 /4 8 / 20
4 / 10 Penalty s.f. / units
5 / 10 Total / 80

This document consists of 14 printed pages.

www.gradtutors.com
461
2
Section A (40 marks)

Answer all questions in the spaces provided.

1 Calcium ethanedioate, CaC2O4, is a white needle-like crystalline solid. It can be


prepared by reacting calcium hydroxide, Ca(OH)2, and ethanedioic acid, H2C2O4.

(a) (i) Name the reaction between calcium hydroxide, Ca(OH)2, and ethanedioic
acid, H2C2O4.

………………………………………………………………………………………

(ii) Write an equation for the reaction.

………………………………………………………………………………………
[2]

(b) When a pure sample of solid anhydrous CaC2O4 was heated strongly at 400 C,
a white solid B and 0.028 g of carbon monoxide gas were obtained as the only
products.

(i) Given that 1 mole of CaC2O4 decomposes to give 1 mole of CO(g), identify
solid B and write an equation for the reaction.

………………………………………………………………………………………

………………………………………………………………………………………

(ii) Hence, determine the mass of B obtained in the reaction.

[4]

www.gradtutors.com
462
3
(c) In an experiment, 25.0 cm3 of 0.050 mol dm3 K2C2O4 was titrated against
0.022 mol dm3 acidified KMnO4. It was found that 22.70 cm3 of KMnO4 was
required for complete reaction. During the titration, effervescence was observed
and the gas evolved gave a white precipitate in limewater.

The reaction of manganate(VII) in acidic medium is given below.

MnO4 + 8H+ + 5e  Mn2+ +4H2O

(i) Using the information above, calculate the mole ratio of C2O42- to MnO4- in
the above reaction.

(ii) Hence, write an equation for the reaction between C2O42- and MnO4- in
acidic medium.

………………………………………………………………………………………
[3]

[Total: 9]

2 The diagram below shows a plot of first ionisation energy against atomic number for
the elements A to I. (The letters are not the chemical symbols for the elements
concerned.)
First ionisation energy / kJ mol–1

F H
C G
E
B D

Atomic number

www.gradtutors.com
463
4

(a) Give the full electronic configuration of the element D.

…………………………………………………………………………………………….
[1]

(b) Explain briefly why the first ionisation energy of G is less than F.

…………………………………………………………………………………………….

…………………………………………………………………………………………….

…………………………………………………………………………………………….
[2]

(c) (i) Draw a dot-and-cross diagram to show the electron arrangement in the
oxide of D.

(ii) The reaction of D with chlorine produces a chloride with a Mr of 133.5.


Draw a diagram to illustrate the shape of the chloride of D. Indicate clearly
the bond angle.

(iii) In terms of structure and bonding, explain why the oxide of D has a higher
melting point than the chloride of D.

………………………………………………………………………………………

………………………………………………………………………………………

………………………………………………………………………………………

………………………………………………………………………………………
[4]

[Total: 7]

www.gradtutors.com
464
5
3 Le Chatelier’s Principle predicts that the highest equilibrium yield of ammonia in the
Haber process,

N2(g) + 3H2(g) 2NH3(g) H = - 92 kJ mol-1

should occur at high pressure and at low temperature. However, in practice, these
conditions are not used.

(a) What are the typical values of pressure and temperature used industrially?
Explain why these conditions are used industrially.

…………………………………………………………………………………………….

…………………………………………………………………………………………….

…………………………………………………………………………………………….

…………………………………………………………………………………………….

…………………………………………………………………………………………….

…………………………………………………………………………………………….

…………………………………………………………………………………………….

…………………………………………………………………………………………….

…………………………………………………………………………………………….

…………………………………………………………………………………………….
[3]

(b) Why are the gases passed through a tower packed with lumps of iron?

…………………………………………………………………………………………….

…………………………………………………………………………………………….

…………………………………………………………………………………………….
[1]

[Total: 4]

www.gradtutors.com
465
6
4 (a) Consider the following reaction scheme.

H Br H OH

H C C H H C C H
I
H H

X Y

acidified K2Cr2O7
II
distil

B A

III

IV

H OH
CH3CH2OH C
H C C COOH
conc H2SO4, heat
H

Draw the structures of the compounds A, B and C in the boxes provided.


[3]

(b) State the type of reaction taking place in steps I and III.

Step I: ……………………………………………………………………………………

Step III: ………………………………………………………………………………….


[2]

www.gradtutors.com
466
7
(c) State the reagents and conditions required for steps I, III and IV.

Step I: ……………………………………………………………………………...........

Step III: ………………………………………………………………………………….

Step IV: ………………………………………………………………………………….


[3]

(d) Suggest a simple chemical test to distinguish compound X from compound Y.


You should state clearly the observations for each compound.

……. ………………………………………………………………………………………

……. ………………………………………………………………………………………

……. ………………………………………………………………………………………

……. ………………………………………………………………………………………

……. ……………………………………………………………………………………...
[2]

[Total: 10]

5 Compound K has the molecular formula C4H8O2.

K decolourises aqueous bromine.

(a) State the functional group in K that decolourises aqueous bromine.

…………………………………………………………………………………………….
[1]

(b) Draw the structural formula of all the possible isomers of K which have the
following features.
 K decolourises aqueous bromine.
 K reacts with sodium metal but not with aqueous sodium hydroxide.
 No oxygen atom is attached to sp2 hybridised carbon atom.
 No carbon atom has more than one oxygen atom joined to it.

[3]

www.gradtutors.com
467
8
(c) Compound K has another isomer, J, which does not decolourise aqueous
bromine but react with sodium hydroxide at room temperature.

(i) Draw the displayed formula of J.

(ii) Give the structure of the organic compound formed when J reacts with
sodium hydroxide at room temperature.

[2]

(d) An aqueous solution of an acid HA of concentration 0.100 mol dm-3 has a pH of


2.9.

(i) Define the term in italics.

………………………………………………………………………………………

(ii) Using the information, calculate the Ka of HA.

[4]

[Total: 10]

www.gradtutors.com
468
9
Section B (40 marks)

Answer two questions from this section on separate answer paper.

6 (a) 5.34 g of a salt, M2SO4 (where M is a metal), was dissolved in water. The
sulfate ion was precipitated by adding excess barium chloride solution. 4.66 g of
barium sulfate precipitate was obtained.

(i) Calculate the number of moles of sulfate ions precipitated.

(ii) Determine the relative formula mass of M2SO4. Hence, calculate the
relative atomic mass of metal M and state its identity.
[5]

(b) State and explain whether metal M or calcium would have a higher melting
point. [3]

(c) State another two physical properties of metal M. [2]

(d) Do you expect the chloride of M to dissolve in water? Briefly explain your
answer. [1]

(e) (i) When metal M reacts with water, effervescence is observed and an
alkaline solution is obtained. Write an equation of the reaction.

(ii) Explain what is meant by the term standard enthalpy change of


neutralisation.

(iii) 40 cm3 of 3.0 mol dm-3 hydrochloric acid was added to 60 cm3 of
1.4 mol dm–3 of the metal M hydroxide in a polystyrene cup. The maximum
temperature rise recorded was 11.6 oC.

Given that the specific heat capacity of the solution = 4.2 J g–1 K–1 and the
process is 100% efficient, calculate the enthalpy change of neutralisation
for the reaction.

(iv) The enthalpy change of neutralisation of ammonia and hydrochloric acid is


– 55.2 kJ mol-1.

Account for the difference in the values of the enthalpy change of


neutralisation.
[7]

www.gradtutors.com
469
10
(f) The enthalpy change of formation of ammonia gas is –46 kJ mol–1. Using the
following energy level diagram and other data from the Data Booklet, calculate
the bond energy of N–H bond in ammonia.

enthalpy / kJ mol-1
N(g) + 3H(g)

N(g) + 3/2 H2(g)

1/2 N2(g) + 3/2 H2(g)


0

NH3(g)

[2]

[Total: 20]

www.gradtutors.com
470
11
7 (a) Halogenoalkanes are useful intermediates in organic synthesis. They can
undergo hydrolysis with a base to produce alcohols.

The kinetics of the hydrolysis of 1-chloropropane with aqueous potassium


hydroxide may be investigated by determining the concentration of
1-chloropropane remaining at different time intervals. The reaction is
represented by the equation below.

CH3CH2CH2Cl + OH─ → CH3CH2CH2OH + Cl─

The following results were obtained from two experiments on such a hydrolysis.
The reaction was followed twice with different concentrations of potassium
hydroxide.
Time / minutes Experiment 1 Experiment 2
[KOH] = 0.10 mol dm-3 [KOH] = 0.15 mol dm-3

[1-chloropropane] / [1-chloropropane] /
mol dm-3 mol dm-3
0 0.0100 0.0100
40 0.0079 0.0070
80 0.0062 0.0049
120 0.0049 0.0034
160 0.0038 0.0024
200 0.0030 0.0017
240 0.0024 0.0012

(i) Using the same axes, plot graphs of [1-chloropropane] against time for the
two experiments, labelling each graph clearly.

(ii) Use your graph to deduce the order of reaction with respect to
1-chloropropane. Show your working clearly.

(iii) Use your graph to determine the initial rate of each reaction. Hence, use it
to deduce the order of reaction with respect to potassium hydroxide.

(iv) Construct a rate equation for the reaction and use it to calculate the rate
constant, giving its units.

(v) Explain, with an aid of the Maxwell-Boltzmann distribution curve, the effect
of increasing temperature on the rate of the reaction.
[10]

www.gradtutors.com
471
12
(b) Acrylic acid, CH2=CHCOOH, is a monobasic acid. 25.0 cm3 of the acid required
20.10 cm3 of 0.10 mol dm-3 NaOH(aq) for complete reaction, as shown by the
following titration curve.
pH

2.68

Volume of NaOH(aq) / cm3


0 20.10

(i) Use the titration curve to calculate the concentration of acrylic acid used.

(ii) Using the pH value of the acrylic acid shown in the graph, calculate the
concentration of the hydrogen ions in the solution of acrylic acid. Hence,
deduce whether acrylic acid is a weak or strong acid.

(iii) Suggest, with reason, a suitable indicator for the titration of acrylic acid
with sodium hydroxide.

(iv) Before the end-point is reached, the pH of the solution changes gradually
as shown in the graph. This is due to the presence of a mixture of acrylic
acid (CH2=CHCO2H) and its salt (CH2=CHCO2Na) in the solution.

By means of an ionic equation, show how this mixture of acrylic acid and
its salt removes the small amount of sodium hydroxide added as the
titration proceeds.
[6]

(c) The aldol reaction is a useful method for making new carbon–carbon bonds in
organic chemistry when carbonyl compounds react under suitable conditions.

In alkaline medium, two carbonyl compounds undergo aldol reaction as shown


below.

O O OH O
OH
C + R''' C R' C
R R' C R'' C R''
R
H H R'''
H
[R groups can be alkyl, aryl or H]

(i) Draw the structure of the aldol product formed between methanal and
ethanal.

www.gradtutors.com
472
13
(ii) Acrylic acid can be synthesised from the following reaction scheme.

O O
OH
+ aldol product
H H CH3 H Step I
methanal ethanal
Step II

O
H2C A
OH Step III
Acrylic acid

Give the structure of compound A formed and state the reagents and
conditions for Step II and III.
[4]

[Total: 20]

www.gradtutors.com
473
14
8 (a) The chlorides of elements sodium to phosphorus all dissolve or react with water.

Describe the reaction of chlorides of aluminium and phosphorus with water.


Suggest the pH values of the resulting solutions and explain your answer with
equations. [5]

(b) (i) Explain what is meant by first ionisation energy of an element, X.

(ii) State and explain the difference in first ionisation energy between

I aluminium and phosphorus

II magnesium and aluminium


[5]

(c) Compound P is found to contain 75.3% carbon, 10.6% hydrogen and 14.1%
oxygen by mass. The relative molecular mass of P is 112.

When P is oxidised with acidified potassium manganate(VII), a compound Q,


C4H6O3, is produced together with propanoic acid. Treatment of P with
phosphorus pentachloride produces no white fumes.

Q produces a pale yellow precipitate when warmed with alkaline solution of


iodine, accompanied by a solution which, when treated with dilute hydrochloric
acid, gave compound R.

R may also be obtained when NC – CH2 – CN is heated with a dilute acid. R


reacts with excess pentachloride to produce a compound S, together with white
fumes evolved.

Deduce the structure of compounds P, Q, R and S and explain the chemistry of


the reactions described above. Write equations where appropriate. [10]

[Total: 20]

End of Paper

www.gradtutors.com
474

Name: Index No.: CT Group: 12

PIONEER JUNIOR COLLEGE

2013 JC2 Preliminary Examination


HIGHER 1

CHEMISTRY 8872/02
Paper 2
18 September 2013

Candidates to answer Section A on the Question Paper

Additional Materials: Data Booklet


Writing Paper
Graph Paper 2 hours

READ THESE INSTRUCTIONS FIRST

Write your name, index number and CT group on all the work you hand in.
Write in dark blue or black pen.
You may use a pencil for any diagrams, graphs or rough working.

Section A
Answer all questions.

Section B
Answer any two questions on separate answer paper.

You are advised to show all working in calculations.


You may use a calculator.

The number of marks is given in brackets [ ] at the end of each question or part question.

At the end of the examination, fasten all your work securely together.

FOR EXAMINER’S USE


Section A Section B
1 /9 6 / 20
2 /7 7 / 20
3 /4 8 / 20
4 / 10 Penalty s.f. / units
5 / 10 Total / 80

This document consists of 14 printed pages.

www.gradtutors.com
475
2
Section A (40 marks)

Answer all questions in the spaces provided.

1 Calcium ethanedioate, CaC2O4, is a white needle-like crystalline solid. It can be


prepared by reacting calcium hydroxide, Ca(OH)2, and ethanedioic acid, H2C2O4.

(a) (i) Name the reaction between calcium hydroxide, Ca(OH)2, and ethanedioic
acid, H2C2O4.

Acid-base or neutralisation

(ii) Write an equation for the reaction.

Ca(OH)2 + H2C2O4  CaC2O4 + 2H2O

[2]

(b) When a pure sample of solid anhydrous CaC2O4 was heated strongly at 400 C,
a white solid B and 0.028 g of carbon monoxide gas were obtained as the only
products.

(i) Given that 1 mole of CaC2O4 decomposes to give 1 mole of CO(g), identify
solid B and write an equation for the reaction.

B is CaCO3.

CaC2O4  CaCO3 + CO

(ii) Hence, determine the mass of B obtained in the reaction.

Amount of B (CaCO3) = amount CO


0.028
=
28.0
= 0.0010 mol

Mass of B (CaCO3) obtained = 0.0010 x 100


= 0.10 g

[4]

www.gradtutors.com
476
3

(c) In an experiment, 25.0 cm3 of 0.050 mol dm3 K2C2O4 was titrated against
0.022 mol dm3 acidified KMnO4. It was found that 22.70 cm3 of KMnO4 was
required for complete reaction. During the titration, effervescence was observed
and the gas evolved gave a white precipitate in limewater.

The reaction of manganate(VII) in acidic medium is given below.

MnO4 + 8H+ + 5e  Mn2+ +4H2O

(i) Using the information above, calculate the mole ratio of C2O42- to MnO4- in
the above reaction.

25.0
Amount of K 2C2O4 x 0.050
1000 5
= = 2.5 or
Amount of KMnO 4 22.70 2
x 0.022
1000

(ii) Hence, write an equation for the reaction between C2O42- and MnO4- in
acidic medium.

5C2O42 + 2MnO4 + 16H+  10CO2 + 2Mn2+ + 8H2O

[3]

[Total: 9]

2 The diagram below shows a plot of first ionisation energy against atomic number for
the elements A to I. (The letters are not the chemical symbols for the elements
concerned.)
First ionisation energy / kJ mol–1

F H
C G
E
B D

Atomic number

www.gradtutors.com
477
4
(a) Give the full electronic configuration of the element D.

1s2 2s2 2p6 3s2 3p1


[1]

(b) Explain briefly why the first ionisation energy of G is less than F.

The first ionisation energy of both F and G involve the removal of an electron
from the 3p orbital. However, electron removed from G is from a 3p orbital
which contains 2 electrons. The paired electrons experience inter-electron
repulsion. Thus, less energy is required to remove the 3p electron in G.
[2]

(c) (i) Draw a dot-and-cross diagram to show the electron arrangement in the
oxide of D.

3+ 2-
xx x

2 x
x Al xx 3 Ox
xx

(ii) The reaction of D with chlorine produces a chloride with a Mr of 133.5.


Draw a diagram to illustrate the shape of the chloride of D. Indicate clearly
the bond angle.

Cl
120o
Al
Cl Cl

(iii) In terms of structure and bonding, explain why the oxide of D has a higher
melting point than the chloride of D.

Oxide of D has a giant ionic lattice structure whereas the chloride has
simple covalent structure.

Oxide of D has strong ionic bonds between its oppositely charged ions as
compared to the weak intermolecular van der Waals’ forces of attraction
present in the chloride of D. Hence, more energy is required to overcome
the stronger bonds in the oxide of D.

[4]

[Total: 7]

www.gradtutors.com
478
5
3 Le Chatelier’s Principle predicts that the highest equilibrium yield of ammonia in the
Haber process,

N2(g) + 3H2(g) 2NH3(g) H = - 92 kJ mol-1

should occur at high pressure and at low temperature. However, in practice, these
conditions are not used.

(a) What are the typical values of pressure and temperature used industrially?
Explain why these conditions are used industrially.

Pressure: 200 – 250 atm


Temperature: 450 – 500 °C

By Le Chatelier’s Principle, position of equilibrium of an exothermic reaction


shifts to the right at low temperature, giving a high yield. However, the rate of
reaction would be too slow at low temperature. Hence in practice, a moderate
temperature of 450 °C to 500 °C is adopted.

By Le Chatelier’s Principle, increasing pressure will shift equilibrium position to


the right, favouring the production of NH3. The rate of reaction is also faster
since concentration and the frequency of effective collisions increases.

However, with increasing pressure, the production cost increases. Therefore, a


compromise has to be reached between higher yield and rate at higher cost or
lower yield and rate at lower cost. In practice, a high pressure of about 200 -
250 atm is used.
[3]

(b) Why are the gases passed through a tower packed with lumps of iron?

Iron is used as a catalyst to allow equilibrium to be reached at a faster rate.

[1]

[Total: 4]

www.gradtutors.com
479
6

4 (a) Consider the following reaction scheme.

H Br
H OH
H C C H
H C C H
I
H
H

X Y

acidified K2Cr2O7
II
distil

B H OH A H O

H C C CN H C C H
III
H

IV

H OH C
CH3CH2OH H OH
H C C COOH
H C C COOCH 2CH 3
conc H2SO4, heat
H
H

Draw the structures of the compounds A, B and C in the boxes provided.


[3]

(b) State the type of reaction taking place in steps I and III.

Step I: (nucleophilic) substitution

Step III: (nucleophilic) addition


[2]

www.gradtutors.com
480
7
(c) State the reagents and conditions required for steps I, III and IV.

Step I: NaOH(aq), heat

Step III: HCN, trace NaCN / NaOH, 10 – 20 °C

Step IV: H2SO4(aq), heat


[3]

(d) Suggest a simple chemical test to distinguish compound X from compound Y.


You should state clearly the observations for each compound.

Reagent and conditions:


Add NaOH(aq) and heat. Cool the mixture. Acidify with HNO 3(aq). Add
AgNO3(aq).

Observations:
For X, cream precipitate of AgBr observed
For Y, no cream precipitate observed.

OR

Reagent and conditions: Na

Observations:
For X, no effervescence observed.
For Y, effervescence observed. Gas evolved (H2) which gives a ‘pop’ sound
with lighted splinter.

OR

Reagent and conditions: H2SO4(aq), K2Cr2O7(aq), heat

Observations:
For X, orange K2Cr2O7(aq) remains.
For Y,orange K2Cr2O7(aq) turns green.

OR

Reagent and conditions: PCl5, r.t.p / PCl3, heat. Place a glass rod dipped into
aqueous NH3 near the mouth of the test-tube.

Observations:
For X, no white fumes observed.
For Y, white fumes observed.
[2]

[Total: 10]

www.gradtutors.com
481
8

5 Compound K has the molecular formula C4H8O2.

K decolourises aqueous bromine.

(a) State the functional group in K that decolourises aqueous bromine.

alkene or carbon-carbon double bond


[1]

(b) Draw the structural formula of all the possible isomers of K which have the
following features.
 K decolourises aqueous bromine.
 K reacts with sodium metal but not with aqueous sodium hydroxide.
 No oxygen atom is attached to sp2 hybridised carbon atom.
 No carbon atom has more than one oxygen atom joined to it.

H H
H
C C C O
H H
H H H
C C C C
H
H H H
O O C
H H
H H O

H H
HO H
C H C
H H OH
H C C C
OH H
C C H C
H
H H OR OH

[3]
(c) Compound K has another isomer, J, which does not decolourise aqueous
bromine but react with sodium hydroxide at room temperature.

(i) Draw the displayed formula of J.


H

H H H O HH C H
O
H C C C C or H C C C
H H H O H O H
H H

www.gradtutors.com
482
9
(ii) Give the structure of the organic compound formed when J reacts with
sodium hydroxide at room temperature.
H

H H H O HH C H
O
H C C C C or H C C C
- + -
H H H O Na O Na+
H H

[2]

(d) An aqueous solution of an acid HA of concentration 0.100 mol dm-3 has a pH of


2.9.

(i) Define the term in italics.

pH is the negative logarithm to base 10 of the hydrogen ion concentration


in mol dm-3.
or
pH = – lg [H+] = – lg [H3O+]

(ii) Using the information, calculate the Ka of HA.

pH = 2.9 = -lg[H+]
[H+] = 1.258 x 10-3 mol dm-3

HA(aq) ⇌ H+(aq) + A-(aq)


Initial [ ] / 0.100 0 0
mol dm-3
Change in [ ] –1.258 x 10-3 +1.258 x 10- +1.258 x 10-3
/ mol dm-3 3

Equilibrium 0.100 - 1.258 x 10-3 1.258 x 10-3 1.258 x 10-3


[ ] / mol dm-3  0.100

Since HA is a weak acid, the extent of dissociation is very small.


Hence 1.258 x 10-3 is negligible compared to 0.10 mol dm-3.
 (0.100 – 1.258 x 10-3) ≈ 0.100

Ka of HA = (1.258 x 10-3)2/ 0.100


= 1.58 x 10-5 mol dm-3

Or

Ka of HA = (1.258 x 10-3)2/(0.100 - 1.258 x 10-3)


= 1.60 x 10-5 mol dm-3

[4]

[Total: 10]

www.gradtutors.com
483
10
Section B (40 marks)

Answer two questions from this section on separate answer paper.

6 (a) 5.34 g of a salt, M2SO4 (where M is a metal), was dissolved in water. The
sulfate ion was precipitated by adding excess barium chloride solution. 4.66 g of
barium sulfate precipitate was obtained.

(i) Calculate the number of moles of sulfate ions precipitated.

Ba2+(aq) + SO42-(aq) → BaSO4(s)


Amount of BaSO4 obtained = 4.66 / (137 + 32.1 + 4  16.0)
= 0.0200 mol
2-
BaSO4 ≡ SO4
Amount of sulfate ions = 0.0200 mol

(ii) Determine the relative formula mass of M2SO4. Hence, calculate the
relative atomic mass of metal M and state its identity.

M2SO4 ≡ SO42-
Amount of moles of M2SO4= 0.0200 mol

Mr of M2SO4 = 5.34 / 0.0200 = 267

2(Ar of M) + 32.1 + 4(16.0) = 267


Ar of M = 85.5

M is Rubidium (Rb).

[5]

(b) State and explain whether metal M or calcium would have a higher melting
point. [3]

Calcium would have a higher melting point than M due to smaller cationic radius
, hence higher charge density of cation and the stronger attraction for the
delocalised electrons, hence the stronger the metallic bond. There are also
more valence electrons contributed for metallic bonding in M, hence the
stronger the metallic bond.

(c) State another two physical properties of metal M. [2]

 Good conductor of electricity due to presence of mobile sea of


delocalised electrons.
 High thermal conductivity. When heat is supplied at one end of the
metal, kinetic energy of the atoms and electrons is increased. These
particles collide, transfer and conduct heat energy to surrounding atoms
towards the cooler region of the metal.
 Malleable and ductile. If stress is applied to metals, layers slide over
each other without breaking the metallic bond.

www.gradtutors.com
484
11
(d) Do you expect the chloride of M to dissolve in water? Briefly explain your
answer. [1]

Chloride of M is expected to be soluble in water as the ions can form


favourable ion-dipole interactions with water molecules.

(e) (i) When metal M reacts with water, effervescence is observed and an
alkaline solution is obtained. Write an equation of the reaction.

Rb + H2O → RbOH + ½H2

(ii) Explain what is meant by the term standard enthalpy change of


neutralisation.

Standard enthalpy change of neutralisation (Hn) is the energy evolved


when one mole of water is formed from the neutralisation between acid
and alkali under standard conditions.

(iii) 40 cm3 of 3.0 mol dm-3 hydrochloric acid was added to 60 cm3 of
1.4 mol dm–3 of the metal M hydroxide in a polystyrene cup. The maximum
temperature rise recorded was 11.6 oC.

Given that the specific heat capacity of the solution = 4.2 J g–1 K–1 and the
process is 100% efficient, calculate the enthalpy change of neutralisation
for the reaction.

Amount of HCl = 40/1000 x 3 = 0.1200 mol

Amount of MOH (base) = 60/1000 x 1.4 = 0.08400 mol

MOH is the limiting reagent.

Amount of H2O = 0.08400 mol

Heat evolved = mcT = 100 x 4.2 x 11.6 = 4872 J

Hn = -4872 / 0.084 = -58000 J mol-1 = -58.0 kJ mol-1

(iv) The enthalpy change of neutralisation of ammonia and hydrochloric acid is


– 55.2 kJ mol-1.

Account for the difference in the values of the enthalpy change of


neutralisation.

The enthalpy change of neutralisation involving HCl and NH3 is less


exothermic. NH3 is a weak base but RbOH is a strong base. Some of the
energy evolved from the neutralisation process is used to further
dissociate the weak base completely / to cause ionisation of the
undissociated weak base.
[7]

www.gradtutors.com
485
12

(f) The enthalpy change of formation of ammonia gas is –46 kJ mol–1. Using the
following energy level diagram and other data from the Data Booklet, calculate
the bond energy of N–H bond in ammonia.

enthalpy / kJ mol-1
N(g) + 3H(g)

3/2 BE(H-H)

N(g) + 3/2 H2(g) 3 BE(N-H)

1/2 BE( N N)

1/2 N2(g) + 3/2 H2(g)


0
Hf(NH3) NH3(g)

By Hess’ Law,
3BE(N-H) = 3/2BE(H-H) + 1/2BE(N≡N) – ∆Hθf(NH3)
= 3/2(436) + 1/2(994) – (-46) = 1197

BE(N-H) = 1197 / 3 = +399 kJ mol–1

[2]

[Total: 20]

www.gradtutors.com
486
13
7 (a) Halogenoalkanes are useful intermediates in organic synthesis. They can
undergo hydrolysis with a base to produce alcohols.

The kinetics of the hydrolysis of 1-chloropropane with aqueous potassium


hydroxide may be investigated by determining the concentration of
1-chloropropane remaining at different time intervals. The reaction is
represented by the equation below.

CH3CH2CH2Cl + OH─ → CH3CH2CH2OH + Cl─

The following results were obtained from two experiments on such a hydrolysis.
The reaction was followed twice with different concentrations of potassium
hydroxide.
Time / minutes Experiment 1 Experiment 2
[KOH] = 0.10 mol dm-3 [KOH] = 0.15 mol dm-3

[1-chloropropane] / [1-chloropropane] /
mol dm-3 mol dm-3
0 0.0100 0.0100
40 0.0079 0.0070
80 0.0062 0.0049
120 0.0049 0.0034
160 0.0038 0.0024
200 0.0030 0.0017
240 0.0024 0.0012

(i) Using the same axes, plot graphs of [1-chloropropane] against time for the
two experiments, labelling each graph clearly.

0.012

0.01

0.008
conc of 1-chloropropane

[KOH] = 0.10 mol dm-3


0.006
[KOH] = 0.15 mol dm-3

0.004

0.002

0
0 50 100 150 200 250 300
Time (minutes)

www.gradtutors.com
487
14
(ii) Use your graph to deduce the order of reaction with respect to
1-chloropropane. Show your working clearly.

From the graph, determine


The half-life for 2nd experiment is 78 min.
OR
The half-life for 1st experiment is 118 min.

As the half-life is a constant, the order of reaction with respect to


1-chloropropane is one.

(iii) Use your graph to determine the initial rate of each reaction. Hence, use it
to deduce the order of reaction with respect to potassium hydroxide.

Draw tangent to the two curves at t = 0 min.


Initial rate for [KOH] = 0.10 mol dm-3 is 5.25 x 10-5 mol dm-3 min-1
Initial rate when [KOH] is 0.15 mol dm-3 is 8.00 x 10-5 mol dm-3 min-1

[1-chloropropane] kept constant at t = 0 min.


When [KOH] is increased by 1.5 times, the rate increases by 1.5 times.
Thus, the order of reaction with respect to KOH is one.

(iv) Construct a rate equation for the reaction and use it to calculate the rate
constant, giving its units.

Rate = k[CH3CH2CH2Cl][KOH]

Substitute initial rate = 5.25 x 10-5 mol dm-3 min-1, [OH-] = 0.1mol dm-3
and [1-chloropropane] = 0.01mol dm-3 into the rate equation,

k = 0.0525 mol-1 dm3 min-1

(v) Explain, with an aid of the Maxwell-Boltzmann distribution curve, the effect
of increasing temperature on the rate of the reaction.
[10]
Maxwell-Boltzmann distribution curve

www.gradtutors.com
488
15
When temperature is increased, there is
 an increase in the average kinetic energy of the particles, resulting in,
 an increase in the number of reacting particles with energy  Ea,
 an increase in the frequency of collisions between reacting particles,
 resulting in an increase in effective collisions per unit time,
 an increase in rate of reaction.

Note: At higher temperature T2, the curve shifts right with a lower peak
since average kinetic energy of particles increase. The total number of
particles (total area under the curve) remains the same.

(b) Acrylic acid, CH2=CHCOOH, is a monobasic acid. 25.0 cm3 of the acid required
20.10 cm3 of 0.10 mol dm-3 NaOH(aq) for complete reaction, as shown by the
following titration curve.
pH

2.68
3
Volume of NaOH(aq) / cm
0 20.10

(i) Use the titration curve to calculate the concentration of acrylic acid used.

Amount of NaOH = 0.10 x 0.02010 = 2.01 x 10-3 mol

[acrylic acid] = 2.01 x 10-3 / 0.025 = 0.0804 mol dm-3

(ii) Using the pH value of the acrylic acid shown in the graph, calculate the
concentration of the hydrogen ions in the solution of acrylic acid. Hence,
deduce whether acrylic acid is a weak or strong acid.

[H+] = 10-2.68 = 2.09 x 10-3 mol dm-3


[H+] << [acid], acrylic acid is weak.

(iii) Suggest, with reason, a suitable indicator for the titration of acrylic acid
with sodium hydroxide.

Phenolphthalein

There is sharp / distinct colour change at end point from colourless to pale
pink.

OR

www.gradtutors.com
489
16

Working range of the indicator pH 8 to 10 coincides with the equivalence


point of titration.

(iv) Before the end-point is reached, the pH of the solution changes gradually
as shown in the graph. This is due to the presence of a mixture of acrylic
acid (CH2=CHCO2H) and its salt (CH2=CHCO2Na) in the solution.

By means of an ionic equation, show how this mixture of acrylic acid and
its salt removes the small amount of sodium hydroxide added as the
titration proceeds.

CH2=CHCO2H + OH- → CH2=CHCO2- + H2O


[6]

(c) The aldol reaction is a useful method for making new carbon–carbon bonds in
organic chemistry when carbonyl compounds react under suitable conditions.

In alkaline medium, two carbonyl compounds undergo aldol reaction as shown


below.
O O OH O
OH
C + R''' C R' C
R R' C R'' C R''
R
H H R'''
H
[R groups can be alkyl, aryl or H]

(i) Draw the structure of the aldol product formed between methanal and
ethanal.

HO O

(ii) Acrylic acid can be synthesised from the following reaction scheme.

O O
OH
+ aldol product
H H CH3 H Step I
methanal ethanal
Step II

O
H2C A
OH Step III
Acrylic acid

www.gradtutors.com
490
17
Give the structure of compound A formed and state the reagents and
conditions for Step II and III.

Step II: excess concentrated H2SO4, 170 °C

Step III: K2Cr2O7(aq), H2SO4(aq), heat

O
H2C
Structure of A: H

[4]

[Total: 20]

www.gradtutors.com
491
18
8 (a) The chlorides of elements sodium to phosphorus all dissolve or react with water.

Describe the reaction of chlorides of aluminium and phosphorus with water.


Suggest the pH values of the resulting solutions and explain your answer with
equations. [5]

AlCl3 dissolves in water and undergo hydrolysis

AlCl3(s) + 6H2O(l) → [Al(H2O)6]3+(aq) + 3Cl-(aq)

Al3+ has high charge density. It can polarise and weaken the O-H bond in water
molecules causing them to break, releasing H+. Thus, the solution has a pH of
2 – 3.

[Al(H2O)6]3+ + H2O ⇌ [Al(H2O)5OH]2+ + H3O+

PCl5 undergoes hydrolysis to from an acidic solution of H3PO4 and HCl. Thus,
the solution has a pH of 1 – 2.

PCl5 + 4H2O → 5 HCl (aq) + H3PO4 (aq)


OR
PCl3(l) + 3H2O(l) → H3PO3(aq) + 3HCl(aq)

(b) (i) Explain what is meant by first ionisation energy of an element, X.

First ionisation energy (1st I.E.) of an element is defined as the amount of


energy required, measured in kJ mol-1, to remove one mole of electrons
from one mole of gaseous X atoms to form one mole of gaseous X+ ions.

X(g)  X+(g) + e- ∆H = 1st I.E.

(ii) State and explain the difference in first ionisation energy between

I aluminium and phosphorus

 Nuclear charge increases as the number of protons increases.


 Electrons are added to the same quantum shell and shielding
effect remains approximately constant as the number of filled
inner shells remain constant.
 Overall effect is the increase of effective nuclear charge (Zeff =
Zactual - shielding effect)
 Increase in effective nuclear charge results in an increased
attraction between the nucleus and the outermost electron.
 More energy is required to remove the outermost electron.
 First IE of phosphorus is higher than aluminium

www.gradtutors.com
492
19

II magnesium and aluminium


Electronic configuration of
Mg : 1s2 2s2 2p6 3s2
Al : 1s2 2s2 2p6 3s2 3p1
 The outermost electron in Mg is located in the s orbital while the
outermost electron in Al is located in the p orbital.
 Within the same principal quantum shell, the p orbitals have a
higher energy compared to the s orbital (p electron is held less
strongly).
 More energy is required to remove the 3s electrons of Mg than
3p electron of Al.
 The first ionisation energy of Mg is higher than the first
ionisation energy of Al.
[5]

(c) Compound P is found to contain 75.3% carbon, 10.6% hydrogen and 14.1%
oxygen by mass. The relative molecular mass of P is 112.

When P is oxidised with acidified potassium manganate(VII), a compound Q,


C4H6O3, is produced together with propanoic acid. Treatment of P with
phosphorus pentachloride produces no white fumes.

Q produces a pale yellow precipitate when warmed with alkaline solution of


iodine, accompanied by a solution which, when treated with dilute hydrochloric
acid, gave compound R.

R may also be obtained when NC–CH2–CN is heated with a dilute acid. R


reacts with excess pentachloride to produce a compound S, together with white
fumes evolved.

Deduce the structure of compounds P, Q, R and S and explain the chemistry of


the reactions described above. Write equations where appropriate. [10]

C H O
% by 75.3 10.6 14.1
mass
amt 75.3/12.0 = 6.275 10.6 14.1/16 = 0.881
mole ratio 7 12 1

Mr of (C7H12O)n = 112  n = 1
P has a molecular formula: C7H12O.

P undergoes oxidation with acidified KMnO4 to produce Q and propanoic acid.


P is likely to contain C=C since 2 products are formed. (not likely to be ester as
there is only one oxygen).

P does not undergo (nucleophilic) substitution with PCl5. P does not contain
alcohol functional group.

www.gradtutors.com
493
20

Q undergoes oxidation with alkaline aqueous iodine. Since Q is a product of


C CH3
O
oxidation, it cannot contain alcohol group. Thus, it contains .

Upon acidification with HCl(aq), R is obtained. Solution contains a carboxylate


salt and R is carboxylic acid.

NC–CH2 –CN undergoes acid hydrolysis with dilute acid to form R.

R undergoes (nucleophilic) substitution with PCl5 to give S, which is an acid


chloride.

R is HO2CCH2CO2H

Q is HO2CCH2COCH3

P is CH3COCH2CH=CHCH2CH3

S is ClOCCH2COCl

Equations:
CH3COCH2CH=CHCH2CH3 + 4[O] → HO2CCH2COCH3 + CH3CH2CO2H

HO2CCH2COCH3 + 3I2 + 4OH- → HO2CCH2COO- + CHI3 + 3I- + 3H2O

HO2CCH2CO2- + H+ → HO2CCH2CO2H

HO2CCH2COOH + 2PCl5 → ClOCCH2COCl + 2HCl + 2POCl3

[Total: 20]

End of Paper

www.gradtutors.com
494
1

RAFFLES INSTITUTION
2013 YEAR 6 PRELIMINARY EXAMINATION

Higher 1

CHEMISTRY
8872/01
Paper 1 Multiple Choice 27 September 2013

50 minutes

Additional Materials: Multiple Choice Answer Sheet


Data Booklet

READ THESE INSTRUCTIONS FIRST

Do not open this question booklet until you are told to do so.

Write in soft pencil.


Do not use staples, paper clips, highlighters, glue or correction fluid.
Write your name, class and index number in the spaces provided on the Answer Sheet.

There are thirty questions on this paper. Answer all questions.

For each question there are four possible answers A, B, C and D.


Choose the one you consider to be correct and record your choice with a soft pencil on the
separate Answer Sheet.

Each correct answer will score one mark. A mark will not be deducted for a wrong answer.

Any rough working should be done in this booklet.

This document consists of 16 printed pages.

Raffles Institution 2013 8872/01/Preliminary Examination [Turn Over

www.gradtutors.com
495
2

Section A

For each question, there are four possible answers, A, B, C, and D. Choose the one you
consider to be correct and record your choice on the OMR Answer Sheet provided.

1 Use of the Data Booklet is relevant to this question.

Which of the following substances contains the greatest amount of particles?

A 50 g of solid Fe2O3
B 1.8 x 1023 molecules of solid C6H12O6
C 350 cm3 of 2.5 mol dm–3 of aqueous HNO3
D 5.0 dm3 of gaseous H2S at room temperature and pressure

2 Use of the Data Booklet is relevant to this question.

The mineral tellurite, TeO2 (Mr = 160.0) is often used in the manufacture of optic
fibres. It was found that 1.01 g of TeO2 in an ore sample required exactly 60 cm3 of
0.035 mol dm–3 acidified K2Cr2O7 for complete reaction. In this reaction, Cr2O72– is
converted into Cr3+.

What is the oxidation state of Te in the final product?

A +2 B +3 C +5 D +6

3 The successive ionisation energies (IE) of two elements, D and E, are given below:

IE/ kJ mol–1 1st 2nd 3rd 4th 5th 6th 7th 8th
D 550 1065 4138 5500 6910 8760 10230 11800
E 1140 2103 3470 4560 5760 8550 9940 18600

What is the likely formula of the compound that is formed when D reacts with E?

A DE2 B DE C D2E D D2E3

4 Which of the following pairs of compounds have the same shape?

A SO2 and CO2


B ICl4- and XeF4
C NCl3 and ClF3
D BrF5 and PCl5

Raffles Institution 2013 8872/01/Preliminary Examination [Turn Over

www.gradtutors.com
496
3

5 Which one e of the foollowing se


equences shows
s the bonds arraanged in order
o of
increasing polarity?

A O – H,
H O – F, O – O
B O – H,
H O – O, O – F
C O – O,
O O – F, O – H
D O – O,
O O – H, O – F

6 Which one of the follow


wing solids consists off atoms or molecules
m hheld only by van der
Waals’ forcces?

A CO2 B H2O C SiO2 D Cu

7 A student used the apparatus be elow to dettermine the standard eenthalpy ch


hange of
combustionn of propan--1-ol, CH3C
CH2CH2OH (M ( r = 60.0).

thermomeeter
can contaaining 200 g water

burner coontaining
propann-1-ol

The following results were


w obtain ed:

Masss of propan-1-ol burntt = 0.60 g


Masss of water heated = 20 00 g
Initial temperatture of wate
er = 21.0 °C
C

The specific heat capa er is 4.18 J g−1 K−1.


acity of wate
The standa
ard enthalpyy change off combustion of propan n-1-ol is –20021 kJ mol−1.

Assuming no
n heat loss
s, what wou
uld be the maximum
m temperature of the water?

A 24.2 °C
° B 29.1 °°C C 45.2 °C
° D 48.4 °C
°

Raffles IInstitution 20
013 8872/01//Preliminary Examination
n [Tu
urn Over

www.gradtutors.com
497
4

8 Iodine trichloide, ICl3, is made by reacting iodine with chlorine.


I2(s) + Cl2(g)  2ICl(s); ∆Ho = +14 kJ mol1
ICl(s) + Cl2(g)  ICl3(s); ∆Ho = 88 kJ mol1

By using the above data and the following energy cycle, what is the standard
enthalpy change of formation of solid iodine trichloride in kJ mol1?

ICl(s) + Cl2(g)

I2(s) + Cl2(g) ICl3(s)

A 60 B 74 C 81 D 162

9 Based on the thermochemical data below, what is the ratio of heat generated by
hydrogen to that of carbon if 1 g of each substance is burned?

Standard enthalpy change of formation of water = 285 kJ mol1


Standard enthalpy change of formation of carbon dioxide = 394 kJ mol1

A 0.4 B 0.7 C 3.4 D 4.3

10 J and L2 react according to the following equation:

2J(g) + L2(g) J2L(g) + L(g)

When 0.6 mole of J was mixed with 0.3 mole of L2 at 298 K in a 2 dm3 vessel, it was
found that one-third of J had been converted into products at equilibrium.

What is the numerical value of Kc for the reaction at 298 K?

A 0.625 B 0.556 C 0.313 D 0.125

Raffles Institution 2013 8872/01/Preliminary Examination [Turn Over

www.gradtutors.com
498
5

11 Consider th
he following
g equilibrium
m system:

Fe3O4(s)
( + CO(g)) CO2(g) + 3FeO
O(s) Ho > 0

Which of the following


g is correct?
?

Changes made Possition of equ


uilibrium
A Add solid
s Fe3O4 S
Shifts to the
e right
B Reduce the temp
perature S
Shifts to the
e left
C Reduce the conc
centration o
of CO(g) S
Shifts to the
e right
D Reduce the volume of the re
eaction ves
ssel S
Shifts to the
e left

12 The positio
on of equilib
brium lies to
o the right in
n each of the
ese reactionns.
N2H5+ + NH3 NH4+ + N2H4
N2H4 + HBr N2H5+ + Br –
Based on this
t information, what iss the order of acid stre
ength?

A NH3 > N2H4 > Brr –


B N2H5+ > HBr > NH4+
C HBr > N2H5+ > NH4+
D N2H5+ > N2H4 > NH
N 4+

13 Which of th g 1 dm3 acid


he following d solutions will form an acidic bufffer when added
a to
1 dm3 0.100 mol dm3 NaOH?

A 0 mol dm3 HI
0.200
B 0 mol dm3 CH
0.200 C 3CO2H
C 0 mol dm3 (COOH)
0.050 ( 2

D 0 mol dm3 H2SO4


0.100

Raffles IInstitution 20
013 8872/01//Preliminary Examination
n [Tu
urn Over

www.gradtutors.com
499
6

14 The reaction between acidified H2O2 and I has the following rate equation:
Rate = k [H2O2] [I]

Which graph would be obtained?

A [H +
[reactant]
] / mol dm -3 B
[H+]

0
time
time/s Rate

C [H2O2] D
Rate

time [I]

Raffles Institution 2013 8872/01/Preliminary Examination [Turn Over

www.gradtutors.com
500
7

15 The kinetics of the reaction between H+ and S2O32– can be investigated


experimentally by varying the volumes of HCl(aq) and Na2S2O3(aq) used and
determining the time taken, t, for the formation of sulfur to completely obscure the
cross as shown in the diagram.

S2O32–(aq) + 2H+(aq)  S(s) + SO2(g) + H2O(l)

HCl(aq) + Na2S2O3(aq)
cross

The table below shows the experimental results obtained.

Volume used / cm3


Experiment 1.0 mol dm3 0.040 mol dm3 t/s
H2O(l)
HCl(aq) Na2S2O3(aq)
1 10.0 5.0 25.0 170
2 15.0 5.0 20.0 170
3 15.0 10.0 15.0 85
4 20.0 20.0 0.0 x

What is the value of x in Experiment 4?

A 43 B 85
C 170 D 340

16 Which property of the first seven elements in Period 3 continuously increases


numerically?

A Ionic radius
B Boiling point
C First ionisation energy
D Highest oxidation number in oxide

Raffles Institution 2013 8872/01/Preliminary Examination [Turn Over

www.gradtutors.com
501
8

17 The oxides of the elements sodium to sulfur are separately added to water.
Which diagram best represents the pH of the solutions produced?

A B

pH pH

Na2O Al2O3 P4O10 Na2O Al2O3 P4O10


MgO SiO2 SO3 MgO SiO2 SO3
C D

pH pH

Na2O Al2O3 P4O10 Na2O Al2O3 P4O10


MgO SiO2 SO3 MgO SiO2 SO3

18 Which of the following pairs of molecules are not isomers?

A CH2 Cl CH3
Cl

B (CH3)2CHCH2OH CH3OCH(CH3)2

H3C CH2CH3 Cl
H3C
C C C C C
H Cl H CH2 CH3

D CH3OCOCH3 CH3COOCH3

Raffles Institution 2013 8872/01/Preliminary Examination [Turn Over

www.gradtutors.com
502
9

19 Tetracyanoethylene (TCNE) is used in the preparation of organic superconductors.

N N
C C
C C
C C
N N
TCNE

How many  and  bonds are there in TCNE?

 
A 13 5
B 10 8
C 9 9
D 4 14

Raffles Institution 2013 8872/01/Preliminary Examination [Turn Over

www.gradtutors.com
503
10

20 Indane is a hydrocarbon that reacts with chlorine under different conditions.

indane

Which of the following best describes the type of reaction, condition and a possible
product of the reaction between indane and chlorine?

Type of reaction Condition Product

Cl
A Substitution Ultra-violet light

Cl

B Substitution Heat

C Addition Fe

Cl

D Addition AlCl3

Cl

Raffles Institution 2013 8872/01/Preliminary Examination [Turn Over

www.gradtutors.com
504
11

21 Compound M reacts with excess hot acidified KMnO4 to give the following product.

CH3CCH2 CH2 CHCH2 C OH

CH3

What is the structure of compound M?

A B

C D

22 2-chlorocyclohexanol undergoes the following reaction. Which of the following are


appropriate reagents and conditions for steps 1 and 2?

OH OH
Cl step 1 step 2 CO2 H
Q

2-chlorocyclohexanol

Step 1 Step 2
K2Cr2O7, dilute H2SO4
A aqueous KOH
heat
ethanolic KOH KMnO4, dilute H2SO4
B
heat heat
HCN, trace KOH dilute H2SO4
C o
10 – 20 C heat
ethanolic KCN dilute HCl
D
heat heat

Raffles Institution 2013 8872/01/Preliminary Examination [Turn Over

www.gradtutors.com
505
12

23 A compound has the following properties:


 It is unaffected by hot acidified potassium dichromate (VI);
 It gives misty fumes on reacting with anhydrous thionyl chloride, SOCl2.

Which compound would give these results?

O H
A
CH3 C C O

O O
B
CH3 C C CH3

CH3 OH

C CH3 C C CH3

OH CH3

H O

D CH3 C C CH3

OH

24 The naturally-occurring molecule civetone is found in a gland of the African civet cat
and has been used in perfumery.

civetone

Which of the following will not react with civetone?

A Fehling’s reagent
B hydrogen bromide
C 2,4-dintrophenylhydrazine
D cold aqueous potassium manganate(VII)

Raffles Institution 2013 8872/01/Preliminary Examination [Turn Over

www.gradtutors.com
506
13

25 Ethanoic acid and 2-methylpropene react in an acid-catalysed reaction as shown


below.
O O CH3
H2 C
H2SO4
CH3 C OH C CH3 CH3 C O C CH3
H3C
CH3
What is the product formed in the following reaction?

O C2 H5
H2 SO4
(CH3 )2CH C OH CH3 CH C product

O H H

A (CH3)2CH C O C C

CH3 C2H5

O C2 H5

B (CH3)2 CH C O C

CH3

O CH3

C (CH3)2CH C O C

CH3

O C2 H5

D (CH3)2 CH C O C

C2H5

Raffles Institution 2013 8872/01/Preliminary Examination [Turn Over

www.gradtutors.com
507
14

Section B

For eacch of the fo ollowing que estions, one


e or more of
o the three
e numberedd statementts 1 to 3
may bee correct. Decide whether each off the statem ments is or is not correect (you ma
ay find it
helpful to put a tickk against the
e statemen ts which yo
ou consider to be correcct).

The ressponses A to
t D should be selected
d on the basis of

A B C D
1, 2, 3 1 and 2 onnly 2 and 3 only 1 only
are corrrect are correcct are
a correct is correct

No othe
er combinattion of state
ements is ussed as a correct respon
nse.

26 Ga, Ge, Ass and Se arre elementss in Period 4. Which of


o the follow
wing stateme
ents are
true?

1 Ga+ and
a Ge2+ are
e isoelectro
onic.
2 Ga3+ has a smaller ionic rad s3–.
dius than As
3 Both Se and Ge have two u
unpaired ele
ectrons at ground statee.

27 The following equilibrium exists b


between sulfur dioxide and sulfur ttrioxide:

2SO2(g) + O2(g) 2SO3(g) ∆H = –98 kkJ mol–1

Which of th
he following are correctt?

ard rate
Forwa Backwarrd rate
anges made
Cha e Kc
constant consttant
1 Ad
dd catalyst No change Incrrease Increa
ase
2 Add
d more SO2 No change Incrrease No cha
ange
Reduce
3 Inccrease Incrrease Decre
ease
tem
mperature

28 Which of th
he following
g statementss describing
g the characteristics off Period 3 chlorides
c
are correctt?

1 e hydrolysis
In the s of SiCl4, one of the
e products formed haas a giant covalent
c
structture.
2 NaCl and MgCl2 have veryy high melting points whereas
w AlC
Cl3, SiCl4 and
a PCl5
have low melting
g points.
3 An aqqueous solution of AllCl3 has a higher pH than an aqqueous sollution of
MgCll2.

Raffles IInstitution 20
013 8872/01//Preliminary Examination
n [Tu
urn Over

www.gradtutors.com
508
15

The responses A to D should be selected on the basis of

A B C D
1, 2, 3 1 and 2 only 2 and 3 only 1 only
are correct are correct are correct is correct

No other combination of statements is used as a correct response.

29 Organic reactions which involve an overall increase in the oxidation number of the
carbon atoms in the molecule are considered oxidation reactions.

Which of the following conversions are oxidation reactions?

CH3 COOH

H3C CH CH CH3 H3 C CH CH CH3


2
Br OH
H H

3 CH3 C OH CH3 C O

OH

Raffles Institution 2013 8872/01/Preliminary Examination [Turn Over

www.gradtutors.com
509
16

The responses A to D should be selected on the basis of

A B C D
1, 2, 3 1 and 2 only 2 and 3 only 1 only
are correct are correct are correct is correct

No other combination of statements is used as a correct response.

30 Compound R is heated under reflux in the presence of excess alkali. The distillate
forms a yellow precipitate with hot aqueous alkaline iodine.

water out
water in

compound R and
excess alkali
distillate

Which of the following could be R?

1 ethylbenzoate
2 3-iodohexane
3 hexanoic acid

END OF PAPER

Raffles Institution 2013 8872/01/Preliminary Examination [Turn Over

www.gradtutors.com
510

RAFFLES INSTITUTION
2013 Year 6 PRELIMINARY EXAMINATION

Higher 1

CANDIDATE NAME

CLASS INDEX NUMBER

CHEMISTRY 8872/02
Paper 2 18 September 2013
2 hours
Candidates answer Section A on the Question Paper

Additional Materials: Answer paper


Graph paper
Data Booklet

READ THESE INSTRUCTIONS FIRST

DO NOT open this booklet until you are told to do so.

Write your name, class and index number in the spaces provided above.
Write in dark blue or black pen on both sides of the paper.
You may use a soft pencil for any diagrams, graphs or rough working.
Do not use staples, paper clips, highlighters, glue or correction fluid.

Section A (40 marks)


Answer all the questions.

Section B (40 marks)


Answer any two questions on separate answer For Examiner’s Use
paper.
Paper 1 / 30
You are to begin each question on a fresh sheet of Paper 2
paper.
A1 / 11
At the end of the examination, fasten all your work A2 / 12
securely together.
A3 /7
The number of marks is given in brackets [ ] at the A4 / 10
end of each question or part question.
B5 / 20
You are reminded of the need for good English and B6 / 20
clear presentation in your answers.
B7 / 20
Total /110

This document consists of 16 printed pages.

www.gradtutors.com
511
2

Section A (40 marks)


For
Answer all questions in the spaces provided. Examiner’s
Use
A1 This question is about the chemistry of magnesium, aluminium and calcium.

(a) (i) Write the full electronic configurations of magnesium and aluminium.

Mg ………………………………………………………….

Al ………………………………………………………….

(ii) The first ionisation energy of aluminium is less than that of magnesium.
Explain.

…………………………………………………………………………………………………………………………………

…………………………………………………………………………………………………………………………………

…………………………………………………………………………………………………………………………………

…………………………………………………………………………………………………………………………………

[2]

(b) Describe and explain how the acid-base behaviour of aluminium oxide, Al2O3,
differs from that of magnesium oxide, MgO. Write equations for the reactions you
choose to illustrate your answer.

………………………………………………………………………………………………………………………………………..

………………………………………………………………………………………………………………………………………..

………………………………………………………………………………………………………………………………………..

………………………………………………………………………………………………………………………………………..

………………………………………………………………………………………………………………………………………..

………………………………………………………………………………………………………………………………………..

………………………………………………………………………………………………………………………………………..

………………………………………………………………………………………………………………………………………..

[4]

© Raffles Institution 8872 / 2013 H1 Preliminary Examination [Turn Over

www.gradtutors.com
512
3
For
Examiner’s
(c) Magnesium trisilicate, Mg2Si3Ox, can neutralise hydrochloric acid, producing a Use
magnesium salt, water and a crystalline solid with a giant covalent structure.

(i) Determine a value for x.

(ii) Using the value obtained in (c)(i), write an equation to show the reaction
between magnesium trisilicate and hydrochloric acid.

……………………………………………………………………………………………………………………..............

[2]

Iontophoresis is a technique which uses an electric current to deliver ions and drug
through the skin. Small amounts of inorganic salts, such as molten calcium chloride
and molten magnesium sulfate, can be used to aid drug delivery in iontophoresis.

(d) (i) Explain why molten calcium chloride and molten magnesium sulfate
can be used to aid drug delivery through the skin when an electric current is
applied.

……………………………………………………………………………………………………………………………….

…………………………………………………………………………………………………………………………………

(ii) With reference to the Data Booklet, state and explain which ion is larger,
Ca2+ or Mg2+.

…………………………………………………………………………………………………………………………………

…………………………………………………………………………………………………………………………………

…………………………………………………………………………………………………………………………………

…………………………………………………………………………………………………………………………………

[3]

[Total: 11]

© Raffles Institution 8872 / 2013 H1 Preliminary Examination [Turn Over

www.gradtutors.com
513
4
For
A2 But-2-yne and buta-1,3-diene are two unsaturated, gaseous hydrocarbons with the Examiner’s
Use
formula C4H6.
CH3C CCH3 CH2 CHCH CH2
but-2-yne buta-1,3-diene

The standard enthalpy change of formation for these two hydrocarbons are given
below:
Hf O [but-2-yne] = +165 kJ mol–1
Hf O [buta-1,3-diene] = +110 kJ mol–1

(a) Define the term standard enthalpy change of formation.

……………………………………………………………………………………………………………………………………

……………………………………………………………………………………………………………………………………

[1]

(b) The following conversion between these two isomeric hydrocarbons is proposed.

H1 O
CH3C CCH3 (g) CH2 CHCH CH2 (g)

(i) Label the values of standard enthalpy change of formation for the two
hydrocarbons clearly, on the energy level diagram below.

Energy / kJ mol–1

CH3C CCH3 (g)

CH2 CHCH CH2 (g)

4C (s) + 3H2 (g)


0

(ii) Label H1 O on the energy level diagram above and calculate its value.

© Raffles Institution 8872 / 2013 H1 Preliminary Examination [Turn Over

www.gradtutors.com
514
5
For
Examiner’s
(iii) If the structure of a molecule allows for the delocalisation of bonding Use
electrons, the molecule is likely to be more stable.

Based on this, state and explain the relative stability of buta-1,3-diene and
but-2-yne.

………………………………………………………………………………………………………………………………

………………………………………………………………………………………………………………………………

………………………………………………………………………………………………………………………………

(iv) Based on your answers above, state if the standard enthalpy change of
combustion of buta-1,3-diene is more or less exothermic than that of
but-2-yne.

………………………………………………………………………………………………………………………………

[6]

The standard enthalpy change of hydrogenation, H Ohydrogenation, is defined as the enthalpy


change when one mole of an unsaturated compound reacts completely with excess
hydrogen to form a saturated compound at 1 atm and 298 K.

(c) (i) Write an equation, with state symbols, for the hydrogenation of
but-2-yne.

(ii) The condition needed for this hydrogenation is similar to that needed for the
reaction between butene and hydrogen. State the condition needed for this
reaction in (c)(i).

……………………………………………………………………………………………………………………….

(iii) Use bond energies from the Data Booklet to calculate the standard enthalpy
change of hydrogenation of but-2-yne.

© Raffles Institution 8872 / 2013 H1 Preliminary Examination [Turn Over

www.gradtutors.com
515
6
For
Examiner’s
(iv) Based on your answers above, determine the standard enthalpy change of Use
formation of butane.

[5]

[Total: 12]

A3 Isoprene, the building block of Vitamin A, can undergo a wide range of reactions.

CH2 CHC CH2

CH3
isoprene
(a) Isoprene reacts with KMnO4 under different conditions. Other than decolorisation
of KMnO4, describe what you might observe in the following reactions. Give the
structural formulae of the organic products formed.

(i) Cold alkaline KMnO4

Structural formula

Observation: ………………………………………………………………………………………………………

© Raffles Institution 8872 / 2013 H1 Preliminary Examination [Turn Over

www.gradtutors.com
516
7
For
(ii) Hot acidified KMnO4 Examiner’s
Use

Structural formula

Observation: ………………………………………………………………………………………………………

[4]

(b) Iodine azide is a covalent inorganic azide that can undergo electrophilic addition
with alkenes in the following manner:
CH3

CH3CH CH2

N3 I
Propene Iodine azide

(i) Give the structural formula of the organic product formed from the reaction
between isoprene and IN3.

CH2 CHC CH2 2


CH3

Structural formula

© Raffles Institution 8872 / 2013 H1 Preliminary Examination [Turn Over

www.gradtutors.com
517
8
For
Examiner’s
(ii) Another covalent inorganic azide is chlorine azide, ClN3. Explain which Use
bond, Cl–N or I–N, has a higher bond energy.

………………………………………………………………………………………………………………………………

………………………………………………………………………………………………………………………………

………………………………………………………………………………………………………………………………

[3]

[Total: 7]

A4 Arsenic, As, is a toxic element present in the environment. It forms both inorganic and
organic compounds.

(a) Arsenic trioxide, As2O3, has a melting point of 312 C. It reacts with alkalis.

Describe the structure and bonding of As2O3.

……………………………………………………………………………………………………………………………………….

……………………………………………………………………………………………………………………………………….

[1]

(b) The oral LD50, also known as median lethal dose, of As in humans is estimated to
be 2 mg/kg. This indicates that 2 mg of As per kg of human–body–mass, if
administered, is fatal.

Calculate the mass of As2O3 needed to be taken orally to put the life of a man,
weighing 60 kg, into danger.

[2]

© Raffles Institution 8872 / 2013 H1 Preliminary Examination [Turn Over

www.gradtutors.com
518
9
For
Examiner’s
(c) Measurements of the level of arsenic in urine is a biological indicator of exposure Use
to arsenic. Urine samples are analysed by mass spectrometry to determine the
concentration of As. The graph below shows the relationship between the area
under the peaks in the mass spectra obtained from such samples, and the
concentration of As.

A man suspected of excessive exposure to As was admitted into the hospital, and
his urine sample was analysed 11 days after exposure.

(i) The peak area obtained from analysing the man’s urine sample is
2.0 × 105. Using the graph above, determine the concentration (in ppb) of
As in the urine sample 11 days after exposure.

Concentration of As in ppb = …………………………………………

(ii) Using the equation below, calculate the original concentration (in ppb) of As
in the urine sample on Day 0.
[As]n = [As]0 × [( ) ]
where, [As]n = concentration of As on nth day
[As]0 = concentration of As on Day 0
n = number of days after exposure

[2]

© Raffles Institution 8872 / 2013 H1 Preliminary Examination [Turn Over

www.gradtutors.com
519
10
For
Examiner’s
(d) Arsenite (AsO33–)
concentration can be found via iodometric titration with iodine Use
as shown in the equation below.

AsO33– + I2 + H2O → AsO43– + 2I + 2H+

(i) Deduce the type of reaction, based on changes in oxidation number.

……………………………………………………………………………………………………………………………….

……………………………………………………………………………………………………………………………….

(ii) With the aid of a sketch of the Boltzmann distribution, explain how an
increase in temperature would affect the rate of the above chemical reaction.

…………………………………………………………………………………………………………………………………

…………………………………………………………………………………………………………………………………

…………………………………………………………………………………………………………………………………

[5]

[Total: 10]

© Raffles Institution 8872 / 2013 H1 Preliminary Examination [Turn Over

www.gradtutors.com
520
11

Section B (40 marks)

Answer any two questions from this section on separate papers.

B5 This question concerns elements found in Period 3 of the Periodic Table.

(a) Pollution is the introduction of contaminants into a natural environment that causes
harm to the ecosystem. Sulfur hexafluoride, SF6, is the most potent green house gas.
In the presence of sparks and oxygen, SF6 decomposes to give toxic sulfur
oxyfluorides and sulfur tetrafluoride, SF4.

(i) Draw the structure of a molecule of sulfur tetrafluoride, SF4, showing the shape
clearly.

(ii) State and explain the shape around the central atom in SF4.

(iii) Draw a diagram to show the intermolecular forces between SF6 and water.
[5]

(b) Since the Industrial Revolution, emissions of sulfur dioxide and nitrogen oxides to the
atmosphere have increased. The increase in sulfur dioxide and nitrogen oxides
results in the production of acid rain which contains sulfuric(IV) acid and nitric(V)
acid.

(i) Assuming there is an average of 156 tonnes of rainfall annually and HNO3 is
present in 1.5 parts per million (ppm) of the annual rainfall, calculate the
concentration of HNO3, in mol dm3, present in the annual rainfall.
[Given 1 tonne = 1.0 × 106 g, Density of HNO3 = 1.0 g cm3]

(ii) A statue made of calcium carbonate is placed in the open. Write a balanced
equation between calcium carbonate and HNO3.

(iii) Using your answer to (b)(ii) and assuming 0.1% of the annual rainfall came into
contact with the statue, calculate the loss in mass of the statue in that year.
[4]

(c) In the manufacture of sulfuric acid, SO2 is converted into SO3 in the presence of a
vanadium(V) oxide catalyst.

SO2(g) + O2 SO3(g) reaction 1

(i) Explain why a catalyst is needed for reaction 1.

(ii) State what will happen to the yield of SO3 if the pressure of the system
increases.

(iii) At 298 K, Hf O (SO2) = 297 kJ mol1 and Hf O (SO3) = 396 kJ mol1.

Use these data to calculate H O for reaction 1.


[4]

© Raffles Institution 8872 / 2013 H1 Preliminary Examination [Turn Over

www.gradtutors.com
521
12

(d) Phosphorus forms many useful chlorides and oxychlorides.

Phosphorus pentachloride, PCl5, is one of the most important chlorides, others being
PCl3 and POCl3. PCl5 exists as white solid and is widely used as a chlorinating agent
of organic compounds.

In the solid state, PCl5 has an ionic lattice structure consisting of PClx+ and PCl6–
ions.

Deduce a likely identity and the shape of the cation, PClx+, and draw its dot-and-
cross diagram.
[2]

(e) Sulfur is a source of some important oxychlorides. Thionyl chloride, SOCl2, and
sulfuryl chloride, SO2Cl2, are both colourless liquids at room temperature.

The phosphorus chlorides and sulfur oxychlorides react with water to produce acidic
compounds.

(i) PCl5 reacts with water in two separate ways.


(1) 1 mole of PCl5 reacts with 1 mole of H2O to form a white solid, together
with white fumes.
(2) 1 mole of PCl5 reacts with excess H2O to form a very acidic solution.

Write equations, including state symbols, for these two reactions and explain
the observations.

State the observation when a few drops of Universal Indicator is added to the
resulting solution.

(ii) Both SOCl2 and SO2Cl2 react vigorously with water.

When 1 mole each of SOCl2 and SO2Cl2 is added separately to excess water,
2 moles of HCl and a sulfur–containing product are formed in each reaction.
The sulfur in the two compounds have different oxidation states.

Suggest the identities of the sulfur–containing products for each reaction and
state the oxidation number of sulfur in each case.

[5]

[Total: 20]

© Raffles Institution 8872 / 2013 H1 Preliminary Examination [Turn Over

www.gradtutors.com
522
13

B6 (a) The salt of ethylenediaminetetraacetic acid, commonly known as EDTA4–, and


tetraacetylethylenediamine, also known as TAED, are structurally similar to each
other.
O O O O
CCH2 CH2C
C C -
CH3 CH3 O O-
NCH2CH2N -
O NCH 2CH2N
CH3 CH3 O-
C C CCH2 CH2C
O O
O O
TAED EDTA4-

(i) Identify the functional group present in TAED.

EDTA4– has been used to inhibit the catalytic decomposition of hydrogen peroxide.
An example of such a catalyst is Mn2+. EDTA4– removes the Mn2+ catalyst by reacting
with it to form a stable ion with co-ordination number six.

(ii) EDTA4– forms dative bonds with Mn2+. Explain why EDTA4– is able to form
bonds with Mn2+.

(iii) Copy out the given formula of EDTA4– and circle, on your diagram, the six
atoms that form dative bonds with Mn2+.

(iv) Hence, state the formula of the ion formed between EDTA4– and Mn2+.
[4]

(b) In examining reactions of analogous organic compounds, the following reaction


scheme has been proposed.

HO2CCH2 CH2CO2H H2C CH CH CH2


step 1 step 2
B
HO2CCH2 CH2 CO2H H2C CH CH CH2

O O

CH C C
H2C CH CH2 step 3 step 4 CH3 CH3
D
CH3 CH3
C C
H2C CH CH CH2

O O

State the reagents and conditions needed for the above conversions, identifying B
and D formed.
[6]

© Raffles Institution 8872 / 2013 H1 Preliminary Examination [Turn Over

www.gradtutors.com
523
14

(c) Compound G, C5H9O2Cl, has the following characteristics.

 Effervescence is produced on adding sodium carbonate to G.


 It produces a yellow precipitate with hot aqueous alkaline iodine.
 It reacts with hot aqueous sodium hydroxide to give J.
 It does not decolourise hot acidified KMnO4.

Upon acidification, J forms compound K, C5H8O2 in the presence of a small amount


of hot concentrated sulfuric acid.

K has the following characteristics.

 It does not exhibit geometric isomerism.


 It does not decolourise aqueous bromine.

Deduce the structures of all the compounds, giving reasons for your answers.
[5]

(d) The reaction between propanal and water is a reversible reaction. The reaction is
described as being in dynamic equilibrium.

CH3CH2CHO(l) + H2O(l) CH3CH2CH(OH)2(l)

(i) State the meaning of the term dynamic equilibrium.

(ii) Write an expression for the equilibrium constant, Kc, for the reaction between
propanal and water as shown above.

(iii) At equilibrium, the concentrations of CH3CH2CHO(l), H2O(l) and


CH3CH2CH(OH)2(l) are 0.676 mol dm–3, 0.676 mol dm–3 and 0.324 mol dm–3
respectively. Calculate the equilibrium constant, Kc, at 25 oC, stating its units.

(iv) Propanone, an isomer of propanal, also reacts with water in a reversible


reaction as shown below.

CH3COCH3(l) + H2O(l) (CH3)2C(OH)2(l)

The numerical value for the equilibrium constant of this reaction was found to
be 1.40 x 10–3 at 25 oC. By using your answer to (c)(iii) and the above
information, compare the relative reactivities of propanal and propanone.
[5]

[Total: 20]

© Raffles Institution 8872 / 2013 H1 Preliminary Examination [Turn Over

www.gradtutors.com
524
15

B7 (a) Consider the reaction between hydrogen and nitrogen oxide at 800 K:

2NO(g) + 2H2(g)  N2(g) + 2H2O(g)

Experiments were carried out in an enclosed vessel to determine the effect of NO on


the initial rate of experiment. This was done by varying the initial concentration of NO
present in the system, while keeping the initial concentration of H2 constant.

[NO]2 / mol2 dm6 0.010 0.023 0.040 0.063 0.090


Initial rate /
6.5 14.6 26.0 40.6 58.5
x 104 mol dm3 s1

(i) Use the data provided above to plot the graph of initial rate against [NO]2.
Hence deduce the order of reaction with respect to NO.

(ii) Given that the rate of reaction is directly proportional to the concentration of H2,
write the rate equation for the reaction.

(iii) Given that the initial concentration of H2 for each experiment is


0.50 mol dm–3, calculate the rate constant for the reaction, stating its units.

(iv) Solid iron can be used to catalyse the reaction between NO and H2. Explain
this by means of sketching, on the same axes, a clearly labeled reaction
pathway diagram for the catalysed and uncatalysed reaction.
[7]

(b) Succinic acid, historically known as spirit of amber, plays an important biochemical
role in the citric acid cycle. Its structure is shown below.

HO2CCH2CH2CO2H
succinic acid

Succinic acid can be synthesized from ethene using the reaction scheme shown
below.

Step I Step II
CH2A=CH2 CH2ClCH
CH2ClCH2Cl
2Cl A

HO2CCH2CH2CO2H
dilute H2SO4, heat

(i) State the reagents and conditions used for steps I and II. Draw the structure of
A.

(ii) State the type of reaction taking place in step II, and write a balanced equation
for the reaction.

© Raffles Institution 8872 / 2013 H1 Preliminary Examination [Turn Over

www.gradtutors.com
525
16

(iii) Compound P is a structural isomer of succinic acid:

HO2 CCHCHO

CH2 OH
Compound P

Suggest a simple chemical test that could be used to distinguish compound P


from succinic acid. State the reagents and conditions used and the expected
observation for each compound.
[7]

(c) (i) Calculate the pH of the 0.0100 mol dm3 hydrochloric acid solution.

(ii) Calculate the pH of the 0.0100 mol dm3 2-methylpropanoic acid solution, given
that pKa = 4.88.

(iii) Explain the difference in the pH values calculated in (c)(i) and (c)(ii).

(iv) 3-hydroxybutanoic acid, CH3CH(OH)CH2COOH, has a pKa value of 4.70. By


considering their molecular structures, explain the difference in pKa values
between 3-hydroxybutanoic acid and 2-methylpropanoic acid.
[6]

[Total: 20]

END OF PAPER

© Raffles Institution 8872 / 2013 H1 Preliminary Examination [Turn Over

www.gradtutors.com
526

H1 2013 Preliminary Examination Paper 2 (Suggested Solutions)

Section A
A1(a)(i) Mg: 1s22s22p63s2 Al: 1s22s22p63s23p1

(ii) The valence electron to be removed from an Al atom is in the 3p subshell while that
from a Mg atom is in the 3s subshell. The 3p subshell is further from the nucleus [1/2]
than the 3s subshell. Valence electron in Al experiences less electrostatic attraction
[1/2] than the valence 3s electron in Mg. Thus, less energy is required to remove an
electron in the 3p subshell than to remove an electron in the 3s subshell. [2]

Marker’s comments:
The correct terms should be used! E.g. “subshell”

(b) MgO is an ionic oxide which is basic and reacts with acids to form salts and water.
MgO + 2H+  Mg2+ + H2O
Al2O3 is an ionic oxide with covalent character in its ionic bonding.
It has both acidic and basic properties.
Al2O3 + 2OH- + 3H2O  2Al(OH)4-
Al3+ has high charge density, hence high polarising power. It distorts the electron
cloud of O2- such that there is covalent character to the Al-O interaction. [4]

Marker’s comments:
Candidates must relate the acid-base properties of the oxides to the bonding.
Aluminium oxide reacts with both acids and bases as its bonding is predominantly
ionic with covalent character – a brief explanation of the covalent character in its
bonding is required.

(c)(i) x = 8 [1]

(ii) Mg2Si3O8 + 4HCl → 2MgCl2 + 3SiO2 + 2H2O [1] [2]

(d)(i) Calcium chloride and magnesium sulfate are ionic compounds which exist as ions in
the molten state. [1/2] These mobile ions can act as mobile charged carriers [1/2]
when an electric current is applied, and delivered into the skin via the pores.

Marker’s comments:
To explain why they are conductors, there must be mention of mobile charge
carriers.
Many students mistakenly talked about “delocalised electrons” – which are the
mobile conductors in metallic structures, not in ionic structures like calcium chloride
and magnesium sulfate.

(ii) Quote data from Data Booklet


The ionic radius of Ca2+ is greater than that of Mg2+. Ca2+ ions has 1 additional filled
principal quantum shell than Mg2+ ions. The valence electrons in Ca2+ ions are
increasingly further away from the nucleus and are less strongly attracted to the
nucleus than in Mg2+. [3]

Marker’s comments:
The correct terminology must be used!
The ionic sizes from the Data Booklet must be quoted in your answer.

www.gradtutors.com
527

A2(a) The standard enthalpy change of formation is the enthalpy change when 1 mole of a
pure compound in a specified state is formed from its constituent elements in their
standard states, at 298 K and 1 atm. [1]

(b)(i)
Energy / kJ mol–1

CH3C CCH3 (g)

H O1
CH2 CHCH CH2 (g)

H Of [but-2-yne] H Of [buta-1,3-diene]
(+165 kJ mol–1) (+110 kJ mol–1)

4C (s) + 3H2 (g)


0

(ii) Labelling of H O1 on diagram with correct direction of arrow [

By Hess’ Law,
165 + H O1 = 110
H O1 = –55 kJ mol–1

(iii) Buta-1,3-diene is more stable than but-2-yne since the p-orbitals of all the C atoms
in buta-1,3-diene are able to overlap, thus allowing the delocalisation of  electrons to
occur throughout all 4 C atoms. [2]

Marker’s comments
Candidates do not seem to understand how “delocalisation” occurs. Delocalisation of 
electrons occurs in CH2=CHCH=CH2 as the 4 p-orbitals used for -bonding (for 2 
bonds) are adjacent to each other. In other words, this delocalisation does not occur in
penta-1,4-diene CH2=CHCH2CH=CH2, as the 4 p-orbitals are not adjacent to each
other but separated by a –CH2 group.

(iv) The enthalpy change of combustion of buta-1,3-diene is less exothermic than that of
but-2-yne. [6]

(c)(i) CH3C CCH3 (g) 2H2 (g) CH3CH2CH2CH3 (g)

Marker’s comments:
Question states that “excess” hydrogen is used. Hence the CC bond is reduced to
C-C and not C=C.

www.gradtutors.com
528

(ii) Use of Ni/Pt/Pd catalyst

(iii) H Ohydrogenation [but-2-yne]


= [E(CC) + 2E(H–H)] – [E(C–C) + 4E(C–H)]
= [(840) + 2(436)] – [(350) + 4(410)]
= –278 kJ mol–1 [2]

Marker’s comments:
Many candidates missed out on the bond energy of H-H bond.

(iv) H Of [butane] = +165 – 278


= –113 kJ mol–1 [1] [5]

A3(a)
(i)

A black precipitate of MnO2 is observed.

Marker’s comments:
Many candidates do not seem to know the difference between using HOT KMnO4 and
COLD KMnO4. Cold KMnO4 causes mild oxidation and a diol is formed, where 2 –OH
groups add to a C=C bond.
Many did not read the question properly as well. The observation of “KMnO4 is
decolorised” has already been stated in the question, hence it should not be repeated
in the answer.

(ii)

Effervescence. colourless gas (CO2), which forms a white ppt with


Ca(OH)2(aq) is evolved. [4]

Marker’s comments:
Candidates did not know what are “organic” products – organic compounds contain
both C & H atoms. Hence CO2 and H2O are not organic products and should not be
drawn in the answer.
An observation should be a visible change – hence instead of just saying that CO2 is
formed in the observation, “effervescence” and a positive test for the gas should be
stated.

(b)(i)

(iii) Cl–N bond would have a higher bond energy than I–N since the Cl orbital involved in
bonding is smaller and thus less diffuse compared to that of I, resulting in a more
effective overlap. [3]

www.gradtutors.com
529

Marker’s comments:
In comparing bond strengths, it is important to use the correct terms: Cl orbital is less
“diffuse”; “orbital overlap is more effective”.

A4(a) Simple molecular structure with weak intermolecular van der Waals’ forces [1]

Marker’s comments:
It is stated in the question that As2O3 reacts with alkalis. This means that As2O3 is
acidic, and hence its bonding is covalent.
Since As is not a group IV element like Si, the structure is not giant covalent structure.

(b) Mass of As needed = 2 × 60 = 120 mg


Amount of As needed = (120 × 10-3)/ 74.9 = 1.602 × 10–3 mol
Amount of As2O3 needed = (1.602 × 10–3)/ 2 = 8.010 × 10–4 mol [1]
Mass of As2O3 needed = 8.010 × 10–4 × 197.8 = 0.158 g or 158 mg [1] [2]

(c)(i) 12 ppb [1]


(ii) Concentration of As in urine = 2
= 80.7 ppb [1]
2 [2]

(d)(i) Redox reaction [1]. AsO33– is oxidised as the oxidation number of As increases from +3
to +5 [1/2] and I2 is reduced as the oxidation number of I decreases from 0 to -1 [1/2].

Marker’s comments:
The question states to deduce the type of reaction “based on changes in oxidation
number”. Hence the changes in oxidation no. should be stated in answer.

(ii) The distribution of molecular energies at two different temperatures.

number of molecules total number of molecules with


with a given energy energy > Ea at T2 K
T1 K total number of molecules with
energy > Ea at T1 K
T2 K
T2 > T1

kinetic energy
0 Ea (activation energy)
Diagram
An increase in temperature from T1 to T2 K increases the average kinetic energy of the
reactant molecules .
As such at the higher temperature, significantly more reactant molecules have energy
greater than or equal to the activation energy of the reaction (as shown by the larger
shaded area in the above diagram). This results in an increase in the effective collision
frequency and hence an increase in the rate of the reaction.
[5]

www.gradtutors.com
530

Year 6 H1 2011 Prelim Paper 2 - Section B

B5(a)(i) F
F
S
F
F [1]

(ii) There are 4 bond pairs and 1 lone pair around the S atom.

To minimise repulsion, the 5 electron pairs are directed to the corners of a trigonal
bipyramid.

Bond pair–lone pair repulsion > bond pair–bond pair repulsion


Hence shape is distorted tetrahedral/ see-saw/ sawhorse. [2]
(iii)

[2] [5]

Mass of HNO3 = (156 × 10 )  1.5 = 234 g


(b)(i) 6

1 106
No. of moles of HNO3 (in 156 x 106 cm3 of water) = 234 = 3.714 mol
63
[HNO3] = 3.714 -5
x 1000 = 2.38 × 10 mol dm -3

156  106 [2]

(ii) CaCO3 + 2HNO3  Ca(NO3)2 + CO2 + H2O [1]

(iii) No. of moles of HNO3 that came into contact with statue
= 0.1 × 3.714 = 3.714 × 10-3
100
CaCO3 ≡ 2HNO3
Mass of statue lost = (3.714 × 10-3) × 100.1  0.5 = 0.186 g [1] [4]

www.gradtutors.com
531

(c)(i) A catalyst is required to speed up the reaction, since the activation energy required
for the reaction is high. [1] [1]

(ii) When pressure is increased, by Le Chatelier’s Principle, the system at equilibrium


favours the side with less gaseous particles to decrease pressure, i.e. position of
equilibrium shifts to the right. Hence yield of SO3 increases. [1] [1]

(iii) H O = -396 – (-297) = -99 kJ mol-1 [2] [4]

(d) PCl4+: Since it has 4 electron-pairs with no lone pairs, its shape is tetrahedral.
+
Cl
Cl P Cl
[1]
Cl
[1] [2]

(e)(i) When a few drops of water is added, partial hydrolysis occurs forming a white solid
POCl3 is formed with white HCl fumes.
[0.5]
PCl5 (s)+ H2O (l) POCl3 (s) + 2HCl (g) [1]
When large amount of water is added, PCl5 undergoes complete hydrolysis to form
a very acidic solution consisting of H3PO4 and HCl.
PCl5 (s)+ 4H2O (l) H3PO4(aq) + 5HCl(aq) [1]
When a few drops of UI is added, the colour is red. [0.5]

(ii) From SOCl2 : H2SO3 ; oxidation number of S = +4 [1]


From SO2Cl2 : H2SO4 ; oxidation number of S = +6 [1] [5]
[Total: 20]

B6(a)(i) Amide [1] [1]

(ii) EDTA4- contains lone pairs of electrons which can be donated into the low-lying empty
orbitals of Mn2+ [1] [1]

(iIi) O O
CCH2 CH2C
-O
-O NCH2CH2N O-
O-
CCH2 CH2C
O O [1] [1]

(iv) [Mn(EDTA)]2- [1] [1]

www.gradtutors.com
532

(b) HO2 CCH2 CH2CO2H LiAlH4 HOCH2CH2 CH2CH2OH


dry ether
HO2 CCH2 CH2CO2H HOCH2CH2 CH2CH2OH

excess conc H2SO4


170 oC

H2C CH CH CH2

H2 C CH CH CH2

H2C CH CH CH2
cold conc H2SO4 CH3CH(OH) CH(OH)CH3

H2 C CH CH CH2 H3O+, warm


CH3CH(OH) CH(OH)CH3

K2Cr2O7, dil H2SO4

heat
O O

C C
CH3 CH3
CH3 CH3
C C

O O
[6] [10]

www.gradtutors.com
533

(c)  G produces effervescence with Na2CO3.


 G is a carboxylic acid. [½]
 G undergoes nucleophilic substitution and oxidation with alkaline I2 .

G contains the structure CH3CH

Cl [½]

 G undergoes nucleophilic substitution with NaOH(aq) to form J.



CH3CH
J contains the structure
OH [½]

 G is resistant to oxidation.
 A does not contain alkenes, primary or secondary alcohol, or aldehyde. [½]

After acidification, J undergoes condensation reaction with hot conc H2SO4 to form K.
K does not undergo electrophilic addition with aqueous Br2. [2]
 K is an ester. [½] [max 2m]

O Cl O OH
G J K
-O
HO +
Na [1m x 3] [3] [5]

(d)(i) A system is said to have reached dynamic equilibrium when the rates of forward and
backward reactions are equal and non-zero [1] and there is no change in the
concentrations of the reactants and products. [1]

(ii)
Kc =
[1]

(iii)
Kc = = 0.709 mol–1 dm3

computation [½], units: [½] [1]

(iv) The value of Kc for the reaction between propanone and water is smaller than that for
the reaction between propanal and water.
CH3COCH3(l) + H2O(l) (CH3)2C(OH)2(l) ……….( )
CH3CH2CHO(l) + H2O(l) CH3CH2CH(OH)2(l) ………(2)

The position of equilibrium of (2) lies more to the right hand side, as compared to (1)
[1]. Equilibrium (2) favours the formation of the product to a larger extent. Thus,
CH3CH2CHO is more reactive than CH3COCH3 [1]. [2] [5]
[Total: 20]

www.gradtutors.com
534

B7(a)(i)

initial rate vs [NO]2


70.0
60.0
50.0
Axis Title
40.0
30.0
20.0
10.0
0.0
0.000 0.020 0.040 0.060 0.080 0.100
Axis Title

Appropriate scale etc [0.5]


Best fit line through origin [0.5]

When the graph of initial rate against [NO]2 is plotted, a straight line graph passing
through the origin is obtained.
Hence initial rate is proportional to [NO]2
 reaction is second order with respect to [NO]. [1] [3]

(ii) rate = k[H2][NO]2 [1] [1]

(iii) Subst. initial rate = 6.5  104 mol dm3 s1, [NO] = 0.0100 mol dm–3,
[H2] = 0.50 mol dm–3,
6.5  104 = k(0.50)(0.0100)2
k = 13 mol2 dm6 s1 [1] correct answer + units [1]

(iv) Energy Profiles for a Catalysed and Uncatalysed Reaction

potential energy uncatalysed reaction

E1 catalysed reaction

E2

reactants

products
reaction coordinate

E1 = activation energy of uncatalysed reaction


E2 = activation energy of catalysed reaction
[2]
[2] [7]

www.gradtutors.com
535

(b)(i) Step I – Cl2 in the dark [1]


Step II – KCN in ethanol, heat under reflux [1]
A:
CH2CH2
| |
CN CN [1]

(ii) Type of reaction in step II – nucleophilic substitution [1]


Equation: CH2(Cl)CH2Cl + 2CN-  CH2(CN)CH2CN [1]

(iii) Warm each compound with a solution containing AgNO3 in excess NH3(aq) (Tollens’
reagent). [1] Only P gives silver mirror. [1]

OR Warm each compound with complex copper(II) ions in alkaline solution


(Fehling’s solution). Only P gives reddish-brown precipitate.

OR Heat each compound with dilute H2SO4 and K2Cr2O7(aq). Only P turns the
orange solution green.

OR Add 2,4-DNPH to each compound. Only P gives an orange solid. [2] [7]

(c)(i) pH of HCl solution = lg 0.0100 = 2 [1] [1]


(ii) Ka = 10-4.88 = 1.318 x 10-5 mol dm-3
[H+ ]= Ka ×c= 1.318×10-5 ×0.0100= 3.630×10-4mol dm-3
[1] [1]
4
pH  log(3.36  10 )  3.44 [1] [1]

(iii) HCl solution has lower pH than 2-methylpropanoic acid of the same concentration, as
[0.5]
HCl is a strong acid which is fully dissociated to give H+. Hence [H+] = [HCl]
Whereas 2-methylpropanoic acid has a higher pH as it is a weak acid that is only
partially dissociated. Hence [H+] << [acid] [0.5]

(iv) 3-hydroxybutanoic acid has a lower pKa value than 2-methylpropanoic acid, indicating
that it is a stronger acid. [1]

The anion CH3CH(OH)CH2COO formed from the dissociation of


3-hydroxybutanoic acid is more stable [1/2] than the anion CH3CH(CH3)COO from
dissociation of 2-methylpropanoic acid.

This is due to the electron-withdrawing –OH group decreasing the electron density on
the carboxylate ion [1/2] , hence stabilising the anion to a greater extent. This
increases the extent of dissociation of CH3CH(OH)CH2COOH relative to
CH3CH(CH3)COOH. [2] [6]
[Total: 20]

www.gradtutors.com
536
1

RIVER VALLEY HIGH SCHOOL


YEAR 6 PRELIMINARY EXAMINATION
CANDIDATE
NAME

CLASS 6

CENTRE INDEX
S
NUMBER NUMBER

H1 CHEMISTRY 8872/01
Paper 1 Multiple Choice 26 September 2013
50 mins

Additional Materials: Multiple Choice Answer Sheet


Data Booklet

READ THESE INSTRUCTIONS FIRST

Do not use staples, paper clips, highlighters, glue or correction fluid.


Write your name, class, centre number and index number on the Answer Sheet in the spaces
provided.

There are thirty questions on this paper. Answer all questions. For each question there are
four possible answers A, B, C and D.
Choose the one you consider correct and record your choice in soft pencil on the separate
Answer Sheet.

Read the instructions on the Answer Sheet very carefully.

Each correct answer will score one mark. A mark will not be deducted for a wrong answer.
Any rough working should be done in this booklet.

________________________________________________________________________
This document consists of 15 printed pages.

River Valley High School 8872/01/PRELIM/13 [Turn over


Prelim Examination 2013
www.gradtutors.com Year 6 H1 Chemistry 8872 Paper 1
537
2
Section A
For each question there are four possible answers, A, B, C and D. Choose the one you
consider to be correct.

1. In an attempt to establish the formula of an oxide of nitrogen, a known volume of the


pure NxOy gas was mixed with hydrogen and passed over a catalyst at a suitable
temperature. 100% conversion of the oxide to ammonia and water was known to
have taken place.
2400 cm3 of nitrogen oxide measured at room temperature and pressure produced
7.20 g of water. The ammonia produced was neutralised by 200 cm3 of 1.0 mol dm–3
of HCl.
What is the formula of the nitrogen oxide?

A N2O B N2O2

C N2O4 D NO2

2. In an experiment, 50.0 cm3 of a 0.100 mol dm−3 solution of a metallic ion, Mn+,
reacted exactly with 25.0 cm3 of 0.100 mol dm−3 aqueous sodium sulfite.
The half-equation for the oxidation of sulfite ion is shown below.
SO32− (aq) + H2O (l)  SO42− (aq) + 2H+ (aq) + 2e−
If the final oxidation number of the metal in the salt was +3, what would be the
original oxidation number of the metal?

A +1 B +2 C +4 D +5

3. Titanium has the electronic structure, 1s2 2s2 2p6 3s2 3p6 3d2 4s2.
Which one of the following compounds is unlikely to exist?

A TiO B TiCl3

C K3TiF6 D K2TiO4

4. Hydrazine, N2H4, and hydrogen peroxide, H2O2, are both used as rocket propellants
because they can produce large volumes of hot gases from a small volume of liquid.
Which of the following statements about these two compounds is correct?

A The bond angle in N2H4 is smaller than that in H2O2.

B The boiling point of N2H4 is lower than that of H2O2.

C The N─H bond is shorter than the O─H bond.

D N2H4 is a weaker base than H2O2.

River Valley High School 8872/01/PRELIM/13 [Turn over


Prelim Examination 2013
www.gradtutors.com Year 6 H1 Chemistry 8872 Paper 1
538
3

5. Which type of bonds / attractions is responsible for the intermolecular forces in liquid
tetrachloromethane, CCl4?

A Covalent bonds

B Hydrogen bonds

C Instantaneous dipole-induced dipole interactions

D Permanent dipole-permanent dipole interactions

6. When disturbed, bombardier beetles eject a hot noxious chemical spray, from the tip
of their abdomen, with a popping sound.
The spray is produced by a reaction between two chemical
compounds, hydroquinone (C6H4(OH)2) and hydrogen peroxide stored in separate
reservoirs in the beetle's abdomen and mixed when needed in a third chamber with
water and catalytic enzymes. When threatened, the beetle squeezes some fluid
from the inner compartment into the outer compartment, where a reaction takes
place
C6H4(OH)2(aq) + H2O2(aq)  C6H4O2 (aq) + 2H2O(l) Hrxn = ?

To calculate the enthalpy change of reaction, an energy cycle is drawn.


The energy cycle and additional information are as shown below.

C6H4(OH)2(aq)  C6H4O2(aq) + H2(g) H1 = +177.4 kJ mol–1


O2(g) + 2H2O(l)  2H2O2(aq) H2 = +189.1 kJ mol–1
Enthalpy change of formation of water = 285.8 kJ mol–1

C6H4(OH)2(aq) + H2O2(aq)  C6H4O2 (aq) + 2H2O(l)

C6H4O2(aq) + H2(g) + 1/2O2(g) + H2O(l)

What is the value of the enthalpy change of reaction?

A 203 kJ mol1 B 489 kJ mol1

C 13.8 kJ mol1 D 300 kJ mol1

River Valley High School 8872/01/PRELIM/13 [Turn over


Prelim Examination 2013
www.gradtutors.com Year 6 H1 Chemistry 8872 Paper 1
539
4

7. The lattice energies of rubidium fluoride, RbF, and caesium chloride, CsCl, are
760 kJ mol1 and 650 kJ mol1 respectively.
Which value is likely to be the lattice energy of caesium fluoride, CsF, in kJ mol 1?

A 460 kJ mol1

B 550 kJ mol1

C 680 kJ mol1

D 800 kJ mol1

8. Hf of H2O is 285 kJ mol1


Hf of CO2 is 394 kJ mol1

What is the ratio of heat generated by hydrogen to that of carbon if the same mass
of each substance is burned completely?

A 0.4 B 0.7 C 3.4 D 4.3

River Valley High School 8872/01/PRELIM/13 [Turn over


Prelim Examination 2013
www.gradtutors.com Year 6 H1 Chemistry 8872 Paper 1
540
5

9. At a temperature T K, 0.60 mol dm3 of CO and 0.30 mol dm3 of O2 were


introduced into a 5 dm3 vessel and allowed to reach equilibrium.

2 CO (g) + O2 (g) 2CO2 (g) ∆H < 0

The graph below shows the changes in the concentration of CO and CO 2 in the
system with time. A change was made to the system at time, t1 and t2.

What were the changes made at time, t1 and t2?

t1 t2

A A catalyst was added Volume of the system is increased

B More CO2 was added An inert gas was added at constant


volume

C The temperature was decreased More O2 was added

D The temperature was increased Volume of the system is decreased

River Valley High School 8872/01/PRELIM/13 [Turn over


Prelim Examination 2013
www.gradtutors.com Year 6 H1 Chemistry 8872 Paper 1
541
6

10. The percentage of ammonia obtainable, if equilibrium was established during the
Haber process, is plotted against the operating pressure for two temperatures,
400 °C and 500 °C.

Which diagram correctly represents the two graphs?

A B

% NH3 at % NH3 at
equilibrium equilibrium

pressure/ 103 kPa pressure/ 103 kPa

C D

% NH3 at
% NH3 at
equilibrium
equilibrium

pressure/ 103 kPa


pressure/ 103 kPa

11. Water dissociates into H+ and OH– as shown.

H2O H+ + OH–

At 25 °C, the equilibrium [H+] is 10–7 mol dm–3; [H2O] = 55.6 mol dm–3.

What is the order of increasing numerical value of pH, pKa and pKw for this equilibrium
at this temperature?

smallest largest

A pH pKw pKa

B pH pKa pKw

C pKw pKa pH

D pKa pKw pH

River Valley High School 8872/01/PRELIM/13 [Turn over


Prelim Examination 2013
www.gradtutors.com Year 6 H1 Chemistry 8872 Paper 1
542
7

12. When magnesium carbonate powder is added to a solution of aqueous ethanoic


acid, effervescence is observed.
Which of the following will not change the rate of effervescence?

A Using magnesium carbonate pellet instead of magnesium carbonate powder.

B Using hydrochloric acid instead of ethanoic acid.

C Increasing the temperature.

D Increasing the pressure.

13. Two solutions were prepared by dissolving a chloride and an oxide of elements in the
third period of the Periodic Table in separate portions of water.
Both solutions prepared can be used to dissolve Al2O3 but only one can be used to
dissolve SiO2.
Which of the following could be the chloride and the oxide used?

A PCl5 and Na2O

B NaCl and SO3

C MgCl2 and MgO

D SiCl4 and P4O10

14. X, Y and Z are consecutive elements in Period 3 of the Periodic Table. Y has the
highest first ionisation energy and the lowest melting point among the three
elements.

Which of the following could be X, Y and Z?

A sodium, magnesium, aluminium

B magnesium, aluminium, silicon

C aluminium, silicon, phosphorus

D silicon, phosphorus, sulfur

River Valley High School 8872/01/PRELIM/13 [Turn over


Prelim Examination 2013
www.gradtutors.com Year 6 H1 Chemistry 8872 Paper 1
543
8

15. An enzyme, found in the stomach, operates at maximum efficiency when placed in
an aqueous solution buffered at pH 5.
Which combination of substances, when dissolved in 10 dm3 of water, would give
the necessary buffer solution?

A 1 mol HCl and 1 mol of CH3COOH

B 1 mol CH3COOH and 1 mol of CH3COONa

C 1 mol HCl and 1 mol of CH3COONa

D 1 mol of CH3COONH4

16. The reaction between X and Y is an overall second–order reaction.


X+YZ
The rate of this reaction is plotted against the product [X][Y] of the concentrations
of X and Y. Which graph would be obtained?

A B
Rate Rate

[X][Y] [X][Y]

C D
Rate Rate

[X][Y] [X][Y]

River Valley High School 8872/01/PRELIM/13 [Turn over


Prelim Examination 2013
www.gradtutors.com Year 6 H1 Chemistry 8872 Paper 1
544
9

17. Assuming that only mono-chlorination takes place, which of the following will be the
correct combination of the products obtained when excess 2,3-dimethylbutane is
reacted with chlorine in the presence of uv light?

A 2 possible products in the ratio 6:1


B 3 possible products in the ratio 3:3:1
C 3 possible products in the ratio 6:3:1
D 4 possible products in the ratio 6:3:3:2

18. Which of the following cannot be prepared by the apparatus set-up as shown?

Water
out Water
in
Heat

A 1,2-dibromopropene by using bromine and propene


B Ethyl ethanoate by using ethanol and concentrated sulfuric acid
C Bromoethane by using sodium bromide and concentrated sulfuric acid
D Propanone by using propanol, sodium dichromate(VI) and sulfuric acid

River Valley High School 8872/01/PRELIM/13 [Turn over


Prelim Examination 2013
www.gradtutors.com Year 6 H1 Chemistry 8872 Paper 1
545
10

19. Which of the following reagents will not show a reaction when the reagent is added
to compound X?

Compound X

A Acidified potassium manganate(VII) solution

B Alkaline silver nitrate and ammonia solution

C Alkaline copper(II) sulfate solution

D Na metal

What is the total number of isomers that can be formed from the product of the
20.
reaction of the following compound with excess concentrated H2SO4 at 170oC?

A 2
B 4
C 8
D 16

18
21. O is an isotope of oxygen.
When propyl ethanoate is hydrolysed with dilute hydrochloric acid in the presence of
H218O, a mixture of two products is formed. Which of the following pairs gives the
correct structures of the two products?
A CH3COOH and CH3CH2CH218OH
B CH3CO18OH and CH3CH2CH2OH
C CH3CH2COOH and CH3CH218OH
D CH3CH2CO18OH and CH3CH2OH

River Valley High School 8872/01/PRELIM/13 [Turn over


Prelim Examination 2013
www.gradtutors.com Year 6 H1 Chemistry 8872 Paper 1
546
11

22. Serotonin is a monoamine neurotransmitter that contributes to feelings of well-being


and happiness.

serotonin
How many sigma () and pi () bonds does serotonin have?
A 20 and 4
B 22 and 4
C 26 and 4
D 28 and 4

23. Which of the following will not be produced when 1-bromopropane is heated with
ethanolic sodium hydroxide?
A CH3CH=CH2
B CH3CH2CH2OH
C CH3CH2CH2ONa
D CH3CH2CH2OCH2CH3

River Valley High School 8872/01/PRELIM/13 [Turn over


Prelim Examination 2013
www.gradtutors.com Year 6 H1 Chemistry 8872 Paper 1
547
12

24. Which line on the graph shows the relationship between the number of carbon
atoms in an alkane and the amount of oxygen gas needed for complete combustion
of the alkane?

Amount of
oxygen gas

Number of carbon atoms in alkane

River Valley High School 8872/01/PRELIM/13 [Turn over


Prelim Examination 2013
www.gradtutors.com Year 6 H1 Chemistry 8872 Paper 1
548
13

25. Ethanoic acid is prepared industrially by the direct carbonylation of methanol using a
rhodium catalyst.

Which compound can be expected to produce the following product by this method?

CO 2H
HC CH 2CO 2H
CH 2CO 2H

OH CH 2OH

A HC CO 2H B HC CH 2CO 2H

CH 2OH CH 2CO 2H

OH OH
C HC CH 2CO 2H D HC CH 2CO 2H
CO 2H CH 2OH

River Valley High School 8872/01/PRELIM/13 [Turn over


Prelim Examination 2013
www.gradtutors.com Year 6 H1 Chemistry 8872 Paper 1
549
14
Section B

For each of the questions in this section, one or more of the three numbered statements 1 to
3 may be correct.
Decide whether each of the statements is or is not correct.
The responses A to D should be selected on the basis of

A B C D
1, 2 and 3 are 1 and 2 only are 2 and 3 only are 1 only is
correct correct correct correct

No other combination of statements is used as a correct response.

26. The electronic configurations of elements X, Y and Z are as follows:


X: [Ar] 3d104s24p1
Y: [Ar] 3d104s24p4
Z: [Ar] 3d104s1
Which of the following statement(s) is / are true?

1 X3+ shows the greatest deflection towards the negatively charged plate with
constant electric field compared to Y+ and Z2+.

2 The first ionization energy of X is lower than both Z and Y.

3 Each of the X2+, Y2+ and Z2+ ions contain only one unpaired electron.

Which of the following trends concerning Period 3 elements (sodium to sulphur)


27.
are true?
1 There is a change from metallic to non-metallic behaviour.
Their compounds show an increase in the maximum oxidation number of the
2
element across the period.

3 Their compounds become less ionic and more covalent across the period.

River Valley High School 8872/01/PRELIM/13 [Turn over


Prelim Examination 2013
www.gradtutors.com Year 6 H1 Chemistry 8872 Paper 1
550
15
The responses A to D should be selected on the basis of

A B C D
1, 2 and 3 are 1 and 2 only are 2 and 3 only are 1 only is
correct correct correct correct

No other combination of statements is used as a correct response.

28. Given the following reaction:


P(g) + Q(g) ⇌ X(g) + Y(g) ΔH = Negative
Forward reaction: (Rate)f = kf[P][Q]
Backward reaction: (Rate)b = kb[X][Y]
Which of the following is correct?

1 Increasing the pressure will not cause kf and kb values to change and the
equilibrium position remains the same.

2 Increasing the temperature will only increase the value of kb, which causes the
equilibrium position to shift to the left.

3 Decreasing the pressure will not cause (Rate)f and (Rate)b values to change
and the equilibrium position remains the same.

29. Deuterium, D ( 2 H ), is a heavy isotope of hydrogen. Which of the following reactions


will incorporate D in the final product?

1 CH3COOCH2CH3 + hot aqueous NaOD

2 CH2=CH2 + Br2 (in D2O)

3 CH3COCH3 + DCN

30. In which of the following pairs of organic compounds is the compound on the left
more volatile than the one on the right?

1 Propylamine and propan-1-ol


2 Cyclohexylamine and aminoethanoic acid
3 Phenylethanol and phenylethanoate

End of paper

River Valley High School 8872/01/PRELIM/13 [Turn over


Prelim Examination 2013
www.gradtutors.com Year 6 H1 Chemistry 8872 Paper 1
551
16
Suggested Answers

1 C 11 A 21 B
2 C 12 D 22 C
3 D 13 A 23 C
4 B 14 D 24 B
5 C 15 B 25 D
6 A 16 D 26 B
7 C 17 A 27 A
8 D 18 A 28 D
9 D 19 C 29 C
10 C 20 B 30 B

River Valley High School 8872/01/PRELIM/13 [Turn over


Prelim Examination 2013
www.gradtutors.com Year 6 H1 Chemistry 8872 Paper 1
552
1

Suggested Answers

1 C 11 A 21 B
2 C 12 D 22 C
3 D 13 A 23 C
4 B 14 D 24 B
5 C 15 B 25 D
6 A 16 D 26 B
7 C 17 A 27 A
8 D 18 A 28 D
9 D 19 C 29 C
10 C 20 B 30 B

River Valley High School 8872/01/PRELIM/13 [Turn over


Prelim Examination 2013
www.gradtutors.com Year 6 H1 Chemistry 8872 Paper 1
553

RIVER VALLEY HIGH SCHOOL


YEAR 6 PRELIMINARY EXAMINATION
CANDIDATE
NAME

CLASS 6

CENTRE INDEX
NUMBER S NUMBER

H1 CHEMISTRY 8872/02
Paper 2 24 September 2013
2 hours
Candidates answer Section A on the Question Paper.
Additional Materials: Data Booklet

READ THESE INSTRUCTIONS FIRST

Write your name, class, Centre number and index number on all the work you hand in.
Write in dark blue or black pen on both sides of the paper.
You may use a soft pencil for any diagrams, graphs or rough working.
Do not use staples, paper clips, highlighters, glue or correction fluid.

Section A
Answer all the questions.

Section B
Answer two questions on separate answer paper.

A Data Booklet is provided. Do not write anything on it.

The number of marks is given in brackets [ ] at the end of each question or part question.
At the end of the examination, fasten all your work securely together.

For Examiner’s Use

Section A

B5

B6

B7

Total
________________________________________________________________________________________________________
This document consists of 15 printed pages

River Valley High School 8872/02/PRELIM/13 [Turn over


Preliminary Examination 2013
www.gradtutors.com
554
2

BLANK PAGE

River Valley High School 8872/02/PRELIM/13


Preliminary Examination 2013
www.gradtutors.com
555
3

Section A [40 marks]

Answer all questions in this section.

1 (a) X is an element in the Periodic Table. The ionisation energies of the first
11 electrons in element X is plotted against the order of removal of
electrons as shown in the diagram below:
Log (Ionisation Energy)

0 1 2 3 4 5 6 7 8 9 10 11
Order of electrons removed from X
[5]

(i) Suggest why log (ionisation energy) is used as the vertical axis,
instead of ionisation energy?

………………………………………………………………………………

(ii) Account for the shape of the graph as observed.

………………………………………………………………………………

………………………………………………………………………………

………………………………………………………………………………

………………………………………………………………………………

………………………………………………………………………………

………………………………………………………………………………

River Valley High School 8872/02/PRELIM/13 [Turn over


Preliminary Examination 2013

www.gradtutors.com
556
4

(iii) Identify X. Hence, state the formula of the oxide of X. Explain how
you arrive at your answer.

………………………………………………………………………………

………………………………………………………………………………

………………………………………………………………………………

………………………………………………………………………………

(b) Give the oxidation state of sulfur in SO32.

……………………………………………………………………………………… [1]

(c) Write equations for the reactions of each of these chlorides with water.
What is the effect of adding chlorides to water? [6]

(i) sodium chloride (include state symbols in your equation)

………………………………………………………………………………

………………………………………………………………………………

(ii) silicon (IV) chloride

………………………………………………………………………………

………………………………………………………………………………

(iii) phosphorus (V) chloride

………………………………………………………………………………

………………………………………………………………………………

[Total: 12]

River Valley High School 8872/02/PRELIM/13


Preliminary Examination 2013
www.gradtutors.com
557
5

2 (a) Explain what is meant by the half-life of a reaction. [1]

……………………………………………………………………………………….

……………………………………………………………………………………….

(b) The radioactive decay of uranium-238 to radon-222 is a first-order process.


The half-life of uranium-238 is 1630 years. How long will it take for a sample [2]
of uranium-238 to decay to 20% of its original radioactivity?

(c) In aqueous solution, iodic acid reacts with potassium hydroxide as such:
HIO3 + KOH → KIO3 + H2O
The reaction is carried out three times using varying concentrations of the
reactants. The initial rate of the reactions were determined and the results
are shown in the table below:

Experiment [HIO3] [KOH] Initial Rate


/ mol dm3 / mol dm3 / mol dm3s-1

1 0.040 0.030 3.25 x 104

2 0.120 0.030 2.93 x 103

3 0.040 0.080 8.66 x 104

[3]

River Valley High School 8872/02/PRELIM/13 [Turn over


Preliminary Examination 2013

www.gradtutors.com
558
6

(i) Using the data


U d given
n in the table, deduc
ce and exxplain the order of
re
eaction with respect tto each rea
actant.


…………… …………………………
…………… …………… ……………
……….


…………………………
…………………………
…………… ……………
……….


…………………………
…………………………
…………… ……………
……….


…………………………
…………………………
…………… ……………
……….

(ii) H
Hence, write
e a rate eq
quation for the reactio
on.


…………………………
…………………………
…………… ……………
…………

[Total: 6]

3 (a)

Co
ompound X
wchart above shows tthe various
The flow s reactions
s of ethylbeenzene. [7]

River Valley High School 88


872/02/PRELIM
M/13
Preliminary Examination 20
013
www.gradtutors.com
559
7

(i) Suggest the displayed formula of compound A and compound B.

(ii) State the reagents and conditions for Steps I, II and III.

Step I:

Step II:

Step III:

(iii) State the type of reaction in step I.

………………………………………………………………………………..

(iv) Draw the compound formed when limited amount of Compound A


reacts with compound X in the presence of hot concentrated sulphuric
acid under reflux conditions.

CH2CH2CHO
(b)

Suggest how you would distinguish between and


CH=CH2

using suitable chemical tests. State clearly the reagents


and conditions used, equations as well as the observations for each of the
following compounds. [3]

River Valley High School 8872/02/PRELIM/13 [Turn over


Preliminary Examination 2013

www.gradtutors.com
560
8

………………………………………………………………………………………...

………………………………………………………………………………………..

………………………………………………………………………………………..

……………………………………………………………………………………….

………………………………………………………………………………………...

………………………………………………………………………………………..

………………………………………………………………………………………..

……………………………………………………………………………………….

[Total : 10]

River Valley High School 8872/02/PRELIM/13


Preliminary Examination 2013
www.gradtutors.com
561
9

Use of the Data Booklet is relevant to this question.

4 Compressed natural gas (CNG) is a fossil fuel substitute for petrol, diesel fuel and
propane. It is widely considered as a more environmentally friendly alternative to
conventional fuels. In a world where fossil fuels are being depleted, CNG is
rapidly gaining popularity as an energy source.
CNG is made by compressing natural gas, which is mainly composed of
methane, CH4, to less than 1% of the volume it occupies at standard atmospheric
pressure. It is stored and transported in cylindrical containers. The mass of a full
tank of CNG is approximately 18 kg. It is used in automobiles where combustion
of CNG in the internal engine releases energy that in turn drives the automobile.
However in recent years, car companies have been adopting the option of using
liquefied natural gas (LNG) in cylindrical containers of the same size as CNG
tanks as an alternative in favour of CNG.
CNG’s specific energy density is measured to be 42% that of (LNG).
Specific energy density is the amount of energy stored in a given system per unit
mass. It is measured in mega joules per unit mass (MJ kg-1) where 1 megajoule =
1  106 J. [6]

(a) (i) Define the term enthalpy change of combustion.

……………………………………………………………………………….

………………………………………………………………………………

(ii) Find the amount of energy released from one cylinder of CNG given
that Hc(CH4) = -891 kJ mol-1. Express your answer in terms of MJ.

(iii) Hence, find the specific energy density of CNG.

(iv) Calculate the specific energy density of LNG? Explain, with reference
to the difference in specific energy density values of CNG and LNG,
why car companies prefer the use of LNG over CNG in recent years.

River Valley High School 8872/02/PRELIM/13 [Turn over


Preliminary Examination 2013

www.gradtutors.com
562
10

(b) Besides methane, many other organic compounds are also used as an
energy source. An example is the usage of methanol in spirit lamps for the
heating of water as shown in the diagram.

The volume of water in the beaker was 250 ml. Its initial temperature was
25.1 ºC and the highest temperature reached after heating was 28.6 ºC.
The mass of the spirit lamp with methanol was originally measured to be
51.6 g. After heating, the mass was 51.4 g.
(1 ml = 1cm3)
The density of water is 1 g cm-3. [4]

(i) Calculate the enthalpy change of combustion for methanol in kJ mol-1.

(ii) The theoretical enthalpy change of combustion of methanol is


-715 kJ mol-1. Comment on the difference between the calculated and
theoretical values.

……………………………………………………………………………..

………………………………………………………………………………

River Valley High School 8872/02/PRELIM/13


Preliminary Examination 2013
www.gradtutors.com
563
11

(c) Methanol is an alcohol. Another alcohol that is frequently used as an energy


source is ethanol. Apart from combustion, ethanol can also undergo
oxidation to form ethanoic acid when reacted with hot acidified potassium
manganite (VII).
Write an overall balanced equation to represent the reaction. [2]

………………………………………………………………………………………..

……………………………………………………………………………………….

[Total: 12]

River Valley High School 8872/02/PRELIM/13 [Turn over


Preliminary Examination 2013

www.gradtutors.com
564
12

Section B [40 marks]

Answer two of the three questions in this section on separate answer paper.

5 There are many different organic compounds that are commonly used in our
daily life. For example, ethanoic acid is also common known as vinegar.
Halogen derivatives are organic compounds which contain at least a C-X bond
where X can be Cl, Br or I. Halogen derivatives undergoes many reactions to
form other compounds.

(a) In a laboratory experiment, a student reacted chloropropane,


bromopropane and iodopropane with Ag(NO3) in the presence of
aqueous NaOH under suitable conditions to produce silver halide
precipitate. [4]

(i) State the colour of the precipitates formed from the above
experiment.

(ii) It is also observed that the rate of the formation of the precipitates
differs.
With the help of data booklet, state and explain the rate of
formation of the three precipitates.

(b) Organic acids and organic alcohols are known to be acidic. The Ka
values of some organic acids and alcohols are shown below.

Type of organic compound Ka values /mol dm3

Propanoic acid 1.34 x 10-5


(CH3CH2COOH)

2-chloropropanoic acid 1.48 x 10-3


(CH3CHClCOOH)

Propanol (CH3CH2CH2OH) 1.00 x 10-16

Explain the above observations. [3]

(c) Many fruit flavours and aromas that are used in the food industry are
esters. An example of the esters is ethyl propanoate which is used as a
kiwi or strawberry flavours. [3]

(i) Write an equation to show the hydrolysis of ethyl propanoate in basic


conditions.

(ii) Briefly outline how you would produce ethyl propanoate in a school
laboratory.

(d) Ethanoic acid has many uses and can undergo many different
reactions. [10]

River Valley High School 8872/02/PRELIM/13


Preliminary Examination 2013
www.gradtutors.com
565
13

(i) CH3OH(l) + CH3CO2H(l) ⇌ CH3CO2CH3(l) + H2O(l)

1. Define “dynamic equilibrium”

2. Calculate the Kc for the above reaction at equilibrium given


that 3 dm3 of 2.00 mol dm3 of ethanoic acid and
3 dm3 of 2.00 mol dm3 methanol is used and the degree of
dissociation is 0.15

(ii) In a separate experiment, 25.0cm3 of 0.50 mol dm3ethanoic acid is


titrated with 0.35 mol dm3 sodium hydroxide.

1. Sketch a titration curve for the above reaction, indicating the


buffer region and volume at equivalence point.

2. Suggest a suitable indicator for this titration

3. “Ethanoic acid behaves like a strong acid in an acid-base


titration.” Explain, with the help of equation(s), this
statement.

[Total: 20]

River Valley High School 8872/02/PRELIM/13 [Turn over


Preliminary Examination 2013

www.gradtutors.com
566
14

6 (a) A reaction can occur between sodium thiosulfate and hydrochloric acid.
The reaction is represented by the equation below:
Na2S2O3(aq) + 2HCl(aq) → S(s) + SO2(g) + 2NaCl(aq)
The kinetics of this reaction was investigated. The initial concentration of
hydrochloric acid was 0.100 mol dm–3 which was in excess. Hence the
rate of reaction is assumed to be dependent only on the concentration of
Na2S2O3(aq). The reaction was monitored using a photometer, an
instrument which measures the intensity of the yellow sulfur formed at
regular time intervals. The measurements were then compared against
those obtained from samples containing known concentrations of sulfur.
The results obtained are summarised in this table:

Time/s 0 60 120 180 240 480 600

[S]/ mol dm–3 0 0.0025 0.0053 0.0068 0.0079 0.0098 0.0100


[7]

(i) Plot the graph of [S] against time on the graph paper provided Use
your graph to determine the order of the reaction with respect to
Na2S2O3(aq).
Show clearly how you arrived at your answer.

(ii) Given that the order of this reaction is 1 with respect to hydrochloric
acid, and the rate of the reaction is 2.25 x 10–4 mol dm–3 s–1 when
[Na2S2O3] = 0.0035 mol dm–3, give the rate equation for the reaction
and calculate the rate constant, stating its units.

(b) The reaction between sodium thiosulfate and hydrochloric acid was
carried out at standard conditions. With the aid of a Boltzmann distribution
curve, illustrate how an increase in temperature can increase the rate of
the reaction. [3]

(c) Sodium, sulfur and chlorine are elements in Period 3. With the aid of a
diagram, explain the variation in electrical conductivity of Period 3
elements. [8]

(d) When aluminium chloride is dissolved in water, the resultant solution


turns blue litmus paper red. Write equation(s) to show why this is so. [2]

[Total: 20]

River Valley High School 8872/02/PRELIM/13


Preliminary Examination 2013
www.gradtutors.com
567
15

7. (a) This question is about compound A, C14H18O3.


A reacts with 2,4-dinitrophenylhydrazine to give an orange precipitate but
no reaction with Fehling’s reagent.
However, silver mirror is observed when A is added to hot silver
diammine complex solution. B and C are produced from this reaction.
Acidification of B produce D with molecular formula C10H10O4 .
D reacts with Na metal. Effervescence is observed. 1 mole of hydrogen
gas is produced. B is produced again. D shows no reaction with acidified
KMnO4 while E decolourise it.
Pale yellow precipitate with an antiseptic smell is observed when C reacts
with a solution of aqueous iodine and sodium hydroxide.
Identify compounds, A to D by providing their structural formulae. Explain
the chemistry of the reactions described. [10]

(b) Phosphorous and carbon are non-metals found in the periodic table.
Many derivatives of carbon and phosphorous are found to be useful and
common in chemistry
Benzene is an organic chemical compound with the formula of C6H6.
Benzene is a colourless and flammable liquid with a sweet smell and
relatively high melting point. Many important chemicals are derived from
benzene where one or more hydrogen atoms is replaced with another
functional group. [4]

(i) Explain the formation of the shape of benzene in terms of  and 


bonds.

(ii) State the hybridisation for the carbon atoms in benzene.

(c) PCl3 and PCl5 are two common reagents used in organic synthesis.
PCl5 is soluble in polar solvents. Previously, PCl5 in solution was thought
to form a dimeric structure, P2Cl10. In more recent times, it has been
proven that PCl5 is soluble in a polar solvents in the “ionic” form of P2Cl10
as shown in the equation below:
2 PCl5  [PCl4+][PCl6−]
State and draw the shapes of the ions of PCl4+ and PCl6− and suggest a
reason why PCl5 is said to be soluble in its “ionic” form. [6]

(i) State and draw the shapes of the ions of PCl4+ and PCl6−

(ii) Suggest a reason why PCl5 is said to be soluble in its “ionic” form.

(iii) Phosphorous can exist as PCl3 and PCl5. However, nitrogen


cannot exist as NCl5. Explain this observation.

[Total: 20]

END OF PAPER
River Valley High School 8872/02/PRELIM/13 [Turn over
Preliminary Examination 2013

www.gradtutors.com
568

RIVER VALLEY HIGH SCHOOL


YEAR 6 PRELIMINARY EXAMINATION
CANDIDATE
NAME

CLASS 6

CENTRE INDEX
NUMBER S NUMBER

H1 CHEMISTRY 8872/02
Paper 2 24 September 2013
2 hours
Candidates answer Section A on the Question Paper.
Additional Materials: Data Booklet

READ THESE INSTRUCTIONS FIRST

Write your name, class, Centre number and index number on all the work you hand in.
Write in dark blue or black pen on both sides of the paper.
You may use a soft pencil for any diagrams, graphs or rough working.
Do not use staples, paper clips, highlighters, glue or correction fluid.

Section A
Answer all the questions.

Section B
Answer two questions on separate answer paper.

A Data Booklet is provided. Do not write anything on it.

The number of marks is given in brackets [ ] at the end of each question or part question.
At the end of the examination, fasten all your work securely together.

For Examiner’s Use

Section A

B5

B6

B7

Total
________________________________________________________________________________________________________
This document consists of 13 printed pages

River Valley High School 8872/02/PRELIM/13 [Turn over


Preliminary Examination 2013
www.gradtutors.com
569
2

BLANK PAGE

River Valley High School 8872/02/PRELIM/13


Preliminary Examination 2013
www.gradtutors.com
570
3

Section A [40 marks]

Answer all questions in this section.

1 (a) X is an element in the Periodic Table. The ionisation energies of the first 11
electrons in element X is plotted against the order of removal of electrons
as shown in the diagram below:
Log (Ionisation Energy)

0 1 2 3 4 5 6 7 8 9 10 11
Order of electrons removed from X

(i) Suggest why log (ionisation energy) used as the vertical axis,
instead of ionisation energy?

To reduce the scale on the vertical axis.

(ii) Account for the shape of the graph as observed.

There is a general increasing trend in successive ionization energies


as the electron is being removed from ions of increasingly positive
charge.
There is a sharp increase in ionization energy between the first and
second electron removal as well as between the 9th and 10th electron
removal as they involve the removal of electron from the inner
principal quantum shell.

(iii) Identify X. Hence, state the formula of the oxide of X. Explain how
you arrive at your answer.

X is sodium.
Na2O.
The sharp increase in the 2nd ionisation energy indicates that the 2nd
electron is removed from an inner shell. Hence, the outer shell of
an atom of X contains 1 electron. X is from Group I of the Periodic
Table. [5]

(b) Give the oxidation state of sulfur in SO32.

River Valley High School 8872/02/PRELIM/13 [Turn over


Preliminary Examination 2013

www.gradtutors.com
571
4

+4 [1]

(c) Write equations for the reactions of each of these chlorides with water.
What is the effect of adding chlorides to water?

(i) sodium chloride (include state symbols in your equation)

NaCl(s) → Na+(aq) + Cl(aq)


Dissolves to give a neutral solution (pH = 7)

(ii) silicon (IV) chloride

SiCl4 + 2H2O → SiO2 + 4HCl


Dissolves and undergoes hydrolysis to form a strongly acidic
solution

(iii) phosphorus (V) chloride

PCl5 + H2O → H3PO4 + 5HCl


Dissolves and undergoes hydrolysis to form a strongly acidic
solution [6]

[Total: 12]

2 (a) Explain what is meant by the half-life of a reaction.

Half-life is the time taken for the concentration of a reactant to be [1]


halved.

(b) The radioactive decay of uranium-238 to radon-222 is a first-order process.


The half-life of uranium-238 is 1630 years. How long will it take for a sample
of uranium-238 to decay to 20% of it’s original radioactivity?

Let number of half-lives be a. [2]


a
 1 20
  
 2 100
1 20
a ln  ln
2 100
a = 2.32
Time taken = 3.32  1630 = 3780 years (3 s.f.)
OR
20
ln  kt
100

River Valley High School 8872/02/PRELIM/13


Preliminary Examination 2013
www.gradtutors.com
572
5

ln 2
Since k  ,
t 12

ln 0.2 = -t (ln 2 / 1630)


t = 3780 years (3 s.f.)

(c) In aqueous solution, iodic acid reacts with potassium hydroxide as such:
HIO3 + KOH → KIO3 + H2O
The reaction is carried out three times using varying concentrations of the
reactants. The initial rate of the reactions were determined and the results
are shown in the table below:

Experiment [HIO3] [KOH] Initial Rate


/ mol dm3 / mol dm3 / mol dm3s-1

1 0.040 0.030 3.25 x 104

2 0.120 0.030 2.93 x 103

3 0.040 0.080 8.66 x 104

(i) Using the data given in the table, deduce and explain the order of
reaction with respect to each reactant.

Comparing experiments 1 and 2,


When [HIO3] increases by a factor of 3 with [KOH] constant, the rate
of reaction increases by a factor of 9.
Hence, order of reaction with respect to HIO3 is two.
Comparing experiments 1 and 3,
When [KOH] increases by a factor of 2.7 with [HIO3] constant, the rate
of reaction increases by a factor of 2.7.
Hence, order of reaction with respect to KOH is one.

(ii) Hence, write a rate equation for the reaction.

Rate = k[HIO3]2[KOH] [3]

[Total: 6]

River Valley High School 8872/02/PRELIM/13 [Turn over


Preliminary Examination 2013

www.gradtutors.com
573
6

3 (a)

Co
ompound X
wchart above shows tthe various
The flow s reactions
s of ethylbeenzene. [7]

(i) S
Suggest the
e displayed
d formula of
o compound A and ccompound B.

C
Compound
dA
Comp
pound B

(ii) S
State the re
eagents an
nd condition
ns for Step
ps I, II andd III.

Step I: FeCl3, Cl22, heat

Step II: Limted Cll2 u.v light

Step III: Cold dilutte alkaline KMnO4

River Valley High School 88


872/02/PRELIM
M/13
Preliminary Examination 20
013
www.gradtutors.com
574
7

(iii) State the type of reaction in step I.

(Electrophilic) substitution

(iv) Draw the compound formed when limited amount of Compound A


reacts with compound X in the presence of hot concentrated sulphuric
acid under reflux conditions.

CH2CH2CHO
(b)

Suggest how you would distinguish between and


CH=CH2

using suitable chemical tests. State clearly the reagents


and conditions used, equations as well as the observations for each of the
following compounds. [3]

Test 1
2,4 DNPH
CH2CH2CHO

Orange ppt seen in , no precipitate observed in


CH=CH2

Eqn
Br2(aq, Br2(CCl4), tollen’s reagent, fehling’s reagent, are accepted too.

[Total : 10]

River Valley High School 8872/02/PRELIM/13 [Turn over


Preliminary Examination 2013

www.gradtutors.com
575
8

Use of the Data Booklet is relevant to this question.

4 Compressed natural gas (CNG) is a fossil fuel substitute for petrol, diesel fuel and
propane. It is widely considered as a more environmentally friendly alternative to
conventional fuels. In a world where fossil fuels are being depleted, CNG is
rapidly gaining popularity as an energy source.
CNG is made by compressing natural gas, which is mainly composed of
methane, CH4, to less than 1% of the volume it occupies at standard atmospheric
pressure. It is stored and transported in cylindrical containers. The mass of a full
tank of CNG is approximately 18 kg. It is used in automobiles where combustion
of CNG in the internal engine releases energy that in turn drives the automobile.
However in recent years, car companies have been adopting the option of using
liquefied natural gas (LNG) in cylindrical containers of the same size as CNG
tanks as an alternative in favour of CNG.
CNG’s specific energy density is measured to be 42% that of (LNG).
Specific energy density is the amount of energy stored in a given system per unit
mass. It is measured in mega joules per unit mass (MJ kg-1) where
1 megajoule = 1  106 J.

(a) (i) Define the term enthalpy change of combustion.

The enthalpy change of combustion of a substance is the enthalpy


change when one mole of the substance is completely burnt in
oxygen [1]

(ii) Find the amount of energy released from one cylinder of CNG given
that Hc(CH4) = -891 kJ mol-1. Express your answer in terms of MJ.

Amount of methane = 18000 / 16 = 1125 mol


Amount of energy released = 1125 x 891 x 10-3 = 1002 MJ [2]

(iii) Hence, find the specific energy density of CNG.

Specific energy density of CNG = 1002 / 18 = 55.7 MJ kg-1 [1]

(iv) Calculate the specific energy density of LNG? Explain, with reference
to the difference in specific energy density values of CNG and LNG,
why car companies prefer the use of LNG over CNG in recent years.

Specific energy density of LNG = 55.7 / 42.0 x 100 = 133 MJ kg-1


The specific energy density of LNG is more than that of CNG which
implies more energy can be released per unit mass of LNG. Hence,
LNG is more space and energy efficient and is preferred over CNG. [2]

River Valley High School 8872/02/PRELIM/13


Preliminary Examination 2013
www.gradtutors.com
576
9

(b) Besides methane, many other organic compounds are also used as an
energy source. An example is the usage of methanol in spirit lamps for the
heating of water as shown in the diagram.

The volume of water in the beaker was 250 ml. Its initial temperature was
25.1 ºC and the highest temperature reached after heating was 28.6 ºC.
The mass of the spirit lamp with methanol was originally measured to be
51.6 g. After heating, the mass was 51.4 g.
(1 ml = 1cm3)
The density of water is 1 g cm-3.

(i) Calculate the enthalpy change of combustion for methanol in kJ mol-1.

Mass of methanol used in combustion = 51.6 – 51.4 = 0.2g


Amount of methanol = 0.2 / 32.0 = 0.00623 mol
Amount of energy absorbed by water = 250 x 4.18 x (28.6 – 25.1) =
3657.5 J = 3.66 kJ
Hence, Hc(CH3OH) = 3.66 / 0.00623 = -587 kJmol-1 [3]

(ii) The theoretical enthalpy change of combustion of methanol is


-715 kJ mol-1. Comment on the difference between the calculated and
theoretical values.

The enthalpy change of combustion calculated in b(ii) is less negative


than the theoretical value as heat was lost to the surroundings. [1]

(c) Methanol is an alcohol. Another alcohol that is frequently used as an energy


source is ethanol. Apart from combustion, ethanol can also undergo
oxidation to form ethanoic acid when reacted with hot acidified potassium
manganite (VII).
Write an overall balanced equation to represent the reaction.

CH3CH2OH + H2O → CH3COOH + 4H+ + 4e


Overall:
[2]

River Valley High School 8872/02/PRELIM/13 [Turn over


Preliminary Examination 2013

www.gradtutors.com
577
10

5CH3CH2OH + 4MnO4- + 12H+ → 5CH3COOH + 4Mn2+ + 11H2O

[Total: 12]

River Valley High School 8872/02/PRELIM/13


Preliminary Examination 2013
www.gradtutors.com
578
11

Section B [40 marks]

Answer two of the three questions in this section on separate answer paper.

5 There are many different organic compounds that are commonly used in our
daily life. For example, ethanoic acid is also common known as vinegar.
Halogen derivatives are organic compounds which contain at least a C-X
bond where X can be Cl, Br or I. Halogen derivatives undergoes many
reactions to form other compounds.

(a) In a laboratory experiment, a student reacted chloropropane,


bromopropane and iodopropane with Ag(NO3) in the presence of
aqueous NaOH under suitable conditions to produce silver halide
precipitate. [4]

(i) State the colour of the precipitates formed from the above
experiment.

AgCl --- White ppt, AgBr--- Cream ppt, AgI--- Yellow ppt

(ii) It is also observed that the rate of the formation of the


precipitates differs.
With the help of data booklet, state and explain the rate of
formation of the three precipitates.

The rate of formation for AgI is the fastest while formation of


AgCl is the slowest.
(The alkyl halides undergo nucleophilic substitution with H2O
which involves breaking of the CX bond and releasing the
halide ion.
The halide ion formed gives a precipitate with AgNO3(aq).)

The CX bond strength increases in the order E(CI) < E(CBr)
< E(CCl)
E(CI) = 240 kJ mol1
E(CBr) = 280 kJ mol1
E(CCl) = 340 kJ mol1

From the values, it is shown that C-Cl has the highest bond
energy while C-I has the lowest bond energy. Hence, the CI
bond breaks most easily to give AgI ppt most quickly, followed
by CBr bond to give AgBr ppt and then CCl bond to give AgCl
ppt most slowly.

River Valley High School 8872/02/PRELIM/13 [Turn over


Preliminary Examination 2013

www.gradtutors.com
579
12

(b) Organic acids and organic alcohols are known to be acidic. The Ka
values of some organic acids and alcohols are shown below.

Type of organic compound Ka values /mol dm3

Propanoic acid 1.34 x 10-5


(CH3CH2COOH)

2-chloropropanoic acid 1.48 x 10-3


(CH3CHClCOOH)

Propanol (CH3CH2CH2OH) 1.00 x 10-16

Explain the above observations. [3]

Acid Strength : Propanol <propanoic acid < 2-chloropropanoic acid

Propanoate anion is more stable than the CH3CH2CH2O- as the


negative charge is delocalised over the COO− group. On the other
hand, the electron-donating ethyl group intensifies the negative charge
on oxygen of CH3CH2CH2O- anion.

2-chloropropanoic acid is a stronger acid than propanoic acid as the


presence of electron-withdrawing Cl atom disperses the negative
charge on the COO− group, making the substituted carboxylate anions
more stable than CH3CH2COO−.

(c) Many fruit flavours and aromas that are used in the food industry are
esters. An example of the esters is ethyl propanoate which is used as
a kiwi or strawberry flavours. [3]

(i) Write an equation to show the hydrolysis of ethyl propanoate in basic


conditions.

CH3CH2COOCH2CH3 + NaOH  CH3CH2COONa+ + CH3CH2OH

(ii) Briefly outline how you would produce ethyl propanoate in a


school laboratory.

Add Ethanol and propanoic acid into a flask. Add a few drops of
concentrated sulfuric acid as catalyst heat/ heat under reflux.

(d) Ethanoic acid has many uses and can undergo many different
reactions. [10]

(i) CH3OH(l) + CH3CO2H(l) ⇌ CH3CO2CH3(l) + H2O(l)

1. Define “dynamic equilibrium”

River Valley High School 8872/02/PRELIM/13


Preliminary Examination 2013
www.gradtutors.com
580
13

Dynamic equilibrium refers to a reversible process at


equilibrium in which the rate of the forward reaction is equal to
the rate of the backward reaction.

2. Calculate the Kc for the above reaction at equilibrium


given that 3 dm3 of 2.00 mol dm3 of ethanoic acid and
3 dm3 of 2.00 mol dm3 methanol is used and the degree
of dissociation is 0.15

CH3OH(l) + CH3CO2H(l) ⇌ CH3CO2CH3(l) + H2O(l)


Initial conc
/mol dm3 0.33 0.33 0 0
∆C
/mol dm3 –0.0495 –0.15 +0.15 +0.15
Eqm conc
/mol dm3 0.281 0.281 0.2 0.2

Kc = [CH3CO2CH3][ H2O] / [CH3OH][ CH3CO2H]


= (0.20)2 / (0.281)2 = 0.506

(ii) In a separate experiment, 25.0cm3 of 0.50 mol dm3ethanoic acid


is titrated with 0.35 mol dm3 sodium hydroxide.

1. Sketch a titration curve for the above reaction, indicating


the buffer region and volume at equivalence point.

‐ Correct shape of graph


‐ Correct indication of buffer region(before
equivalence point)
‐ Volume at equivalence point ( 35.70 cm3)

2. Suggest a suitable indicator for this titration

Phenolphthalein, bromothymol blue, thymol blue

3. “Ethanoic acid behaves like a strong acid in an acid-base


titration.” Explain, with the help of equation(s), this
statement.

CH3COOH(aq)  CH3COO(aq) + H+(aq)


H+ + OH H2O

When OH is added, it will react with H+ present. Hence


the equilibrium position will shift to the right to replenish
River Valley High School 8872/02/PRELIM/13 [Turn over
Preliminary Examination 2013

www.gradtutors.com
581
14

the H+. At the end of the titration, the ethanoic acid has
fully dissociated. Hence ethanoic acid behaves like a
strong acid in the titration.

[Total: 20]

6 (a) A reaction can occur between sodium thiosulfate and hydrochloric acid.
The reaction is represented by the equation below:
Na2S2O3(aq) + 2HCl(aq) → S(s) + SO2(g) + 2NaCl(aq)
The kinetics of this reaction was investigated. The initial concentration of
hydrochloric acid was 0.100 mol dm–3 which is in excess. Hence the rate
of reaction is assumed to be dependent only on the concentration of
Na2S2O3(aq). The reaction was monitored using a photometer, an
instrument which measures the intensity of the yellow sulfur formed at
regular time intervals. The measurements were then compared against
those obtained from samples containing known concentrations of sulfur.
The results obtained are summarised in this table:

Time/s 0 60 120 180 240 480 600

[S]/ mol dm–3 0 0.0025 0.0053 0.0068 0.0079 0.0098 0.0100

(i) Plot the graph of [S] against time on the graph paper provided Use
your graph to determine the order of the reaction with respect to
Na2S2O3(aq).
Show clearly how you arrived at your answer.

River Valley High School 8872/02/PRELIM/13


Preliminary Examination 2013
www.gradtutors.com
582
15

0.011

0.01

0.009

0.008

0.007
[S]/ mol dm–3

0.006

0.005

0.004

0.003

0.002

0.001

0
0 100 200 300 400 500 600

1st t½ time/ s

= 60.0s

2nd t½
= 60.0s
Graph with correct plotting:
Half-life clearly indicated:
Correct axes, labels:
Since half-lives are constant at 60.0s,
Order of reaction with respect to Na2S2O3(aq) is one.

(ii) Given that the order of this reaction is 1 with respect to hydrochloric
acid, and the rate of the reaction is 2.25 x 10–4 mol dm–3 s–1 when
[Na2S2O3] = 0.0035 mol dm–3, give the rate equation for the reaction
and calculate the rate constant, stating its units. [7]

Rate = k[Na2S2O3][HCl]
Rate = 2.25 x 10–4 mol dm–3 s–1 and [Na2S2O3] = 0.00355 mol dm–3
Substitute into the rate equation: 2.25 x 10–4 = k x 0.1 x 0.00355.
Solving, k = 2.25 x 10–4/(0.1 x 0.00355) = 0.634 mol–1 dm3 s–1

(b) The reaction between sodium thiosulfate and hydrochloric acid was [3]
River Valley High School 8872/02/PRELIM/13 [Turn over
Preliminary Examination 2013

www.gradtutors.com
583
16

carried out at standard conditions. With the aid of a Boltzmann distribution


curve, illustrate how an increase in temperature can increase the rate of
the reaction.

 
 

Peak moves to the right:


Peak of higher T curve lower than peak of lower T curve:
Increase in number of particles with energy greater than or equal to Ea:
When temperature increases, the average kinetic energy of the reactant
particles increases . As a result, the frequency of collisions between
reactant particles increases and the rate of reaction increases .

(c) Sodium, sulfur and chlorine are elements in Period 3. With the aid of a
diagram, explain the variation in electrical conductivity of Period 3
elements. [8]

Diagram: Correct shape and correct axes (1m each)


Electrical conductivities are high for the metals Na, Mg and Al, and
increases from Na to Al. Na, Mg and Al have giant metallic structures and
the number of delocalised valence electrons increases from Na to Al.
Electrical conductivity drops sharply at Si. Si has a giant covalent
structure and is a semi-conductor.
Electrical conductivities drop to zero at P4 and remains at zero to Ar. P4 to

River Valley High School 8872/02/PRELIM/13


Preliminary Examination 2013
www.gradtutors.com
584
17

Cl2 have simple covalent structures and Ar has a monoatomic structure


and there are no mobile charge carriers to conduct electricity.

(d) When aluminium chloride is dissolved in water, the resultant solution


turns blue litmus paper red. Write equation(s) to show why this is so. [2]

AlCl3(s) + 6H2O(l)  [Al(H2O)6]3+(aq) + 3Cl (aq)


[Al(H2O)6]3+(aq) + H2O(l)  [Al(H2O)5(OH)]2+(aq) + H3O+(aq)

[Total: 20]

7. (a) This question is about compound A, C14H18O3.


A reacts with 2,4-dinitrophenylhydrazine to give an orange precipitate but
no reaction with Fehling’s reagent.
However, silver mirror is observed when A is added to hot silver
diammine complex solution. B and C are produced from this reaction.
Acidification of B produce D with molecular formula C10H10O4 .
D reacts with Na metal. Effervescence is observed. 1 mole of hydrogen
gas is produced. B is produced again. D shows no reaction with acidified
KMnO4 while E decolourise it.
Pale yellow precipitate with an antiseptic smell is observed when C reacts
with a solution of aqueous iodine and sodium hydroxide.

Identify compounds, A to D by providing their structural formulae. Explain


the chemistry of the reactions described. [10]
 A has a benzene ring as C:H ratio is 1:1
 A undergoes condensation with 2,4-DNPH  A is a carbonyl
compound/ketone/aldehyde.
 A undergoes oxidation with tollen’s reagent but not fehling’s reagent
 A is a benzaldehyde
 A undergoes alkaline hydrolysis to form B and C
 A contains an ester.
 D undergoes acid-metal reaction to produce 1 mole of H2 gas  D is
has 2 carboxylic group.
 E undergoes oxidation with acidified KMnO4  E is an alcohol.
 E undergoes oxidation with alkaline I2  E contains a CH(CH3)(OH)
group.

River Valley High School 8872/02/PRELIM/13 [Turn over


Preliminary Examination 2013

www.gradtutors.com
585
18

A: B:
Mark awarded for substitution at
different position.

C: D:

(b) Phosphorous and carbon are non-metals found in the periodic table.
Many derivatives of carbon and phosphorous are found to be useful and
common in chemistry
Benzene is an organic chemical compound with the formula of C6H6.
Benzene is a colourless and flammable liquid with a sweet smell and
relatively high melting point. Many important chemicals are derived from
benzene where one or more hydrogen atoms is replaced with another
functional group. [4]

(b) (i) Explain the formation of the shape of benzene in terms of  and 
bonds.
In benzene, three sp2 orbitals and one unhybridised 2p orbital is formed.
Two sp2 orbitals of the C atom overlap head-on with the sp2 orbitals of
two neighbouring C atoms, forming two C—C  bonds.
The sp2 orbital of the C atom overlaps head-on with the 1s orbital of one

River Valley High School 8872/02/PRELIM/13


Preliminary Examination 2013
www.gradtutors.com
586
19

H atom, forming a C—H  bond.


The unhybridised 2p orbitals of all the six C atoms overlap
sideways to form a delocalised  electron system/cloud above and
below the plane of C atoms.

(ii) State the hybridisation for the carbon atoms in benzene.

sp2

(c) PCl3 and PCl5 are two common reagents used in organic synthesis.
PCl5 is soluble in polar solvents. Previously, PCl5 in solution was
thought to form a dimeric structure, P2Cl10. In more recent times, it has
been proven that PCl5 is soluble in a polar solvents in the “ionic” form of
P2Cl10 as shown in the equation below:
2 PCl5  [PCl4+][PCl6−]
State and draw the shapes of the ions of PCl4+ and PCl6− and suggest a
reason why PCl5 is said to be soluble in its “ionic” form. [6]

(i) State and draw the shapes of the ions of PCl4+ and PCl6−

Shape of PCl4+ : tetrahedral

Shape of PCl6− : octahedral

(ii) Suggest a reason why PCl5 is said to be soluble in its “ionic” form.

The “ionic” form of P2Cl10 is soluble in polar solvents via ion-


dipole interactions.

(iii) Phosphorous can exist as PCl3 and PCl5. However, nitrogen


cannot exist as NCl5. Explain this observation.

Nitrogen is in period II. It is unable to expand its octet structure as


it does not have accessible d orbitals available.

[Total: 20 ]

END OF PAPER

River Valley High School 8872/02/PRELIM/13 [Turn over


Preliminary Examination 2013

www.gradtutors.com
587

 

Section A
For each question there are four possible answers, A, B, C and D. Choose the one you
consider to be correct.

140
1 In the discovery of an element, E, scientists found that it consist of 3 isotopes, E,
142 144
E and E in the atomic ratio of 2: 3: 1. Calculate the relative atomic mass of
element E to 1 decimal place.
A 140.5
B 141.7
C 142.0
D 143.1

2 To determine the formula of an unknown hydrocarbon, Y, a student exploded 20 cm3 of


Y in 150 cm3 of oxygen. At the end of the reaction, an expansion of 20 cm3 was
recorded, which contracted by 80 cm3 when cooled. The resultant residual gas was
then passed through an alkali which resulted in a further contraction to 10 cm3. What is
the formula of the unknown hydrocarbon Y?
A C4H8
B C4H10
C C5H8
D C5H10

3 In a redox reaction, 0.635 g of pure copper, Cu, required 20.00 cm3 of 1.00 mol dm-3
nitric acid, HNO3, for complete reaction. If Cu2+ ions was produced at the end of the
reaction, what is the oxidation number of nitrogen in the products formed?
A +1
B +2
C +3
D +4

4 An element G in Period 2 has a general valence electronic configuration of ns2 np3.


What is the formula of the flouride that is most likely to be formed by element G?
A GF2
B GF3
C GF4
D GF5

    [Turn over 
www.gradtutors.com
588

 

5 Use of the Data Booklet is relevant to this question.


The table below shows statements that 2 students made about the Ca2+ and Fe2+ ions.
student statement
Q Both ions were obtained by removing the electrons from the same
orbital.
R Both ions have more protons than neutrons and more protons than
electrons.

Which students are correct?


A Student Q only
B Student R only
C Both Student Q and R
D None of them

6 Iodic acid is a strong acid that exist as a white solid at room temperature. It has a
chemical formula of HIO3 and dissolves readily in water to give H+ and IO3- ions. Which
of the following statements about the ion IO3- is true?
A It is a non-planar ion.
B It has a bond angle of 120°.
C Only sigma bonds are formed between iodine and oxygen in the ion.
D The oxidation number of iodine in the ion is negative.

    [Turn over 
www.gradtutors.com
589

 

7 The following compounds are isomers with molecular formula C5H10O2. Which of them
is the least volatile?

8 Graphite is a common form of carbon that has many uses. One of them is as lubricants
in machine parts. Which of the following properties of graphite allow for it to be used as
lubricants?
A Each carbon atom in graphite is sp2 hybridised.
B Graphite has a very high melting point.
C The interactions between layers of graphite is van der Waals forces of attraction.
D Graphite has an extended -electron cloud above and below each plane of
carbon atoms.

9 Which of the following arranges the ionic salts in increasing magnitude of their lattice
energy?
A Al2O3 < MgO < MgBr2 < NaBr
B NaBr < MgO< MgBr2 < Al2O3
C MgO < NaBr < Al2O3 < MgBr2
D NaBr < MgBr2 < MgO < Al2O3

    [Turn over 
www.gradtutors.com
590

 

10 The energy cycle below shows the reaction pathways between Compounds J – M.

∆H1 ∆H2

∆H4
∆H3 ∆H5

 
What is the enthalphy change for the following reaction?
Compound J Compound M
A ∆H1 + ∆H2
B ∆H2 - ∆H3 - ∆H4
C ∆H3 + ∆H5
D ∆H4 - ∆H1 - ∆H5

11 To study the kinetics of a reaction between X, Y and Z, a student conducted a series of


experiments and the following results were obtained:
Experiment Vol of X / Vol of Y / Vol of Z / Vol of H2O / Relative
3 3 3 3
cm cm cm cm rate
1 20 15 10 55 1
2 40 15 10 35 4
3 40 25 15 20 6
4 40 15 20 25 8

What is the rate equation for the reaction?


A Rate = [X]2[Z]
B Rate = [X]2[Y]
C Rate = [X][Y][Z]
D Rate = [X]2[Y][Z]

    [Turn over 
www.gradtutors.com
591

 

12 A technician found the following information in a handbook about an equilibrium


reaction that the factory carries out on an industrial scale.

Temperature / °C Kc
250 1 x 10-5
500 1 x 10-4
Pressure / atm % Product
5 33
10 47

Based on the above information, what can be said about the equilibrium reaction that
the factory carries out?
The forward reaction is endothermic and there are more gaseous molecules in
A
the products than reactants.
The forward reaction is exothermic and there are less gaseous molecules in the
B
products than reactants.
The forward reaction is endothermic and there are less gaseous molecules in the
C
products than reactants.
The forward reaction is exothermic and there are more gaseous molecules in the
D
products than reactants.

13 Which of the following equilibrium equations have no units for their equilibrium
constant, Kc?
A

14 The equation for the dissociation of water is as follows:


∆H = + 57 kJmol-1
                          
Which of the following statements about the above equilibrium is correct?
A As temperature increases, the pH of water also increases.
B As the volume of water increases, the pH of water also increases.
C The summation of pH and pOH is 14 at all temperatures.
D [H+] will always equal to [OH-] at all temperatures.

    [Turn over 
www.gradtutors.com
592

 

15 Given that both NaOH and KOH are strong mineral bases, calculate the resultant pH
when 20 cm3 of 0.300 moldm-3 of NaOH is mixed with 30 cm3 of 0.200 moldm-3 of KOH.
A 11.8
B 12.1
C 13.4
D 13.7

16 Which of the following properties show a decreasing trend across the elements of
Period 3?
A pH of their chlorides in water
B Melting point of their oxides
C Electrical conductivity
D Ionic radius

17 An element in Period 3 has the following properties:


- It does not conduct electricity.
- Its first ionisation energy is larger than both the elements before and after it.
Which of the following statements about this element is correct?
A It has a high melting and boiling point.
B It forms chlorides that are mildly acidic.
C It forms oxides that are very acidic.
Its oxidation state in compounds usually follows the group number it belongs to in
D
the Periodic Table.

18 What is the IUPAC name of the skeletal structure below?

A 2-ethyl-5,5-dimethylpentane
B 2-ethyl-5-methylhexane
C 1,2,5-trimethylhexane
D 2,5-dimethylheptane

    [Turn over 
www.gradtutors.com
593

 

19 How many positional isomers are there for a dichlorobutane?


A 5
B 6
C 7
D 8

20 Ethene is shaken with an aqueous mixture of iodine and sodium bromide.


Which of the following statements about the above reaction is incorrect?
A There will at least one bromine atom found in all of the products formed.
B There will be a mixture of three different products formed.
C This is an addition reaction.
D The role of iodine in this reaction is an electrophile.

21 Which of the following properties of benzene is not associated with the ring of
delocalised electrons?
A Benzene tend to attract electrophiles rather than nucleophiles.
B Benzene tend to undergo substitution reactions rather than addition reactions
C Conversion of benzene to cyclohexane requires very harsh conditions.
D Benzene is a planar molecule with all the carbon atoms lying on the same plane.

    [Turn over 
www.gradtutors.com
594

 

22 The following are three different types of halogen derivatives:

A student then added ethanolic AgNO3 separately to the 3 halogen derivatives above.
Which of the following shows the correct order in which the precipitate appears, from
the fastest to the slowest?
A 1-bromohexane, iodobenzene, 1-chloro-2-phenylethane
B 1-bromohexane, 1-chloro-2-phenylethane, iodobenzene
C iodobenzene, 1-bromohexane, 1-chloro-2-phenylethane
D 1-chloro-2-phenylethane, 1-bromohexane, iodobenzene

23 Which of the following reagents and conditions, when added to ethanol, involve the
cleavage of the O-H bond in ethanol?
A CH3COOH, excess concentrated H2SO4 and reflux
B I2 dissolved in NaOH and warm
C PCl5 at room temperature
D K2Cr2O7, H2SO4(aq) and heat

    [Turn over 
www.gradtutors.com
595
10 
 

24 Which of the following reagents can be used to distinguish between the following 2
compounds?

A 2,4-dinitrophenylhydrazine
B Fehling’s solution
C Tollen’s reagent
D LiAlH4 in dry ether

25 Below are 3 compounds of carboxylic acids and its derivatives.

Which of the following arranges the pH of their solutions in increasing order when they
are placed in water?
A , ,
B , ,
C , ,
D , ,

    [Turn over 
www.gradtutors.com
596
11 
 

Section B
For each of the questions in this section, one or more of the thress numbered statements 1
to 3 may be correct.
Decide whether each of the statements is or is not correct (you may find it helpful to put a
tick against the statements that you consider to be correct).
The responses A to D should be selected on the basis of
A B C D
1, 2 and 3 1 and 2 only 2 and 3 only 1 only
are correct are correct are correct is correct

26 BeCl2 is able to react with PCl3 to form a stable product.


Which of the statements about this reaction are correct?
1 The mole ratio of BeCl2 : PCl3 for this reaction is 1 : 2.
The product in this reaction is able to form because beryllium has energetically
2
accessible orbitals to expand its octet configuration.
3 The bond angle about phosphorus does not change during this reaction.

27 A student investigated a reaction between compounds T, U and V and found that the
order of reaction is first order with respect to T, zero order with respect to U and
second order with respect to V. When a graph was drawn, a straight line was obtained.
Which of the following could be the axis of this graph?
1 Rate against [T]
2 Rate against [U]
3 Rate against [V]2

    [Turn over 
www.gradtutors.com
597
12 
 

The responses A to D should be selected on the basis of

A B C D
1, 2 and 3 1 and 2 only 2 and 3 only 1 only
are correct are correct are correct is correct

28 Vitamin C is a common compound found in many citrus fruits such as oranges and
pineapples. It has the following structure:

Which of the following statements about Vitamin C is incorrect?


1 It is able to exhibit geometric isomerism.
2 It will give orange crystals with Brady’s reagent.
3 1 mole of Vitamin C will react with excess sodium to give 2 moles of hydrogen
gas.

    [Turn over 
www.gradtutors.com
598
13 
 

The responses A to D should be selected on the basis of

A B C D
1, 2 and 3 1 and 2 only 2 and 3 only 1 only
are correct are correct are correct is correct

29 The following synthetic pathway was proposed by a student:

 
Which of the following steps are unable to give the desired product that the student
proposed in this synthesis?
1 Step I
2 Step II
3 Step III

    [Turn over 
www.gradtutors.com
599
14 
 

The responses A to D should be selected on the basis of

A B C D
1, 2 and 3 1 and 2 only 2 and 3 only 1 only
are correct are correct are correct is correct

30 Below is the synthesis pathway of Carvone, the main flavouring compound in


spearmint oil.

 
 
Which of the following types of reactions are shown in the synthetic pathway above?
1 Addition
2 Reduction
3 Substitution

- THE END -

    [Turn over 
www.gradtutors.com
600
15 
 

Answers:
1 B 6 A 11 A 16 A 21 D 26 D
2 C 7 D 12 C 17 C 22 B 27 A
3 D 8 C 13 B 18 D 23 A 28 B
4 B 9 D 14 D 19 B 24 C 29 B
5 A 10 B 15 C 20 A 25 C 30 A

    [Turn over 
www.gradtutors.com
601
2

Section A
For each question there are four possible answers, A, B, C and D. Choose the one you
consider to be correct.

140
1 In the discovery of an element, E, scientists found that it consist of 3 isotopes, E,
142 144
E and E in the atomic ratio of 2: 3: 1. Calculate the relative atomic mass of
element E to 1 decimal place.
A 140.5
B 141.7
C 142.0
D 143.1
    
    
Ar of E = = = 141.67 = 141.7 (1 d.p)


Ans: B

2 To determine the formula of an unknown hydrocarbon, Y, a student exploded 20 cm3 of


Y in 150 cm3 of oxygen. At the end of the reaction, an expansion of 20 cm3 was
recorded, which contracted by 80 cm3 when cooled. The resultant residual gas was
then passed through an alkali which resulted in a further contraction to 10 cm3. What is
the formula of the unknown hydrocarbon Y?
A C4H8
B C4H10
C C5H8
D C5H10
Equation for the general combustion of hydrocarbon:
CxHy + (x+y/4) O2 xCO2 + (y/2) H2O
Since there is a contraction of 80 cm3 when cooled after combustion,
80 cm3 = volume of water produced
Thus, ratio of hydrocarbon : water = 20 : 80 = 1 : 4 and (y/2) = 4 and y = 8
Total volume of gas before combustion = 170 cm3
Total volume of gas after combustion = 190 cm3 (expansion of 20 cm3)
Thus after contraction of water (80 cm3) = 190 – 80 =110 cm3
After passing through alkali, volume of gas reduced to 10 cm3
Thus, volume of CO2 produced = 110 – 10 = 100 cm3
Ratio of hydrocarbon : CO2 = 20 : 100 = 1: 5 and x = 5
Formula of hydrocarbon = C5H8 and Ans: C

[Turn over
www.gradtutors.com
602
3

3 In a redox reaction, 0.635 g of pure copper, Cu, required 20.00 cm3 of 1.00 mol dm-3
nitric acid, HNO3, for complete reaction. If Cu2+ ions was produced at the end of the
reaction, what is the oxidation number of nitrogen in the products formed?
A +1
B +2
C +3
D +4
No. of moles of Cu = 0.635 / 63.5 =0.01
No. of moles of HNO3 = (20/1000) x 1 = 0.02; Oxidation number of N = +5
Thus, Cu : HNO3 = 0.01 : 0.02 = 1 : 2
Since Cu2+ is produced from Cu, 2 electrons are produced per mole of Cu
These 2 electrons are taken in by 2 moles of HNO3 (based on mole ratio calculated
above)
Thus 1 mole of HNO3 taken in 1 mole of electron and its also reduced (since Cu is
oxidised)
Therefore the new oxidation number of HNO3 = 5 – 1 = +4
Ans: D

4 An element G in Period 2 has a general valence electronic configuration of ns2 np3.


What is the formula of the flouride that is most likely to be formed by element G?
A GF2
B GF3
C GF4
D GF5
With a general electronic configuration of ns2 and np3, it belongs to Group V and its
nitrogen.
Thus fluoride formed by N is NF3 since it can only accommodate 8 outermost shell
electrons as it belongs to period 2.
Ans: B

[Turn over
www.gradtutors.com
603
4

5 Use of the Data Booklet is relevant to this question.


The table below shows statements that 2 students made about the Ca2+ and Fe2+ ions.
student statement
Q Both ions were obtained by removing the electrons from the same
orbital.
R Both ions have more protons than neutrons and more protons than
electrons.

Which students are correct?


A Student Q only
B Student R only
C Both Student Q and R
D None of them
Ca: 1s2 2s2 2p6 3s2 3p6 4s2
Ca2+: 1s2 2s2 2p6 3s2 3p6 (electrons removed from 4s orbital)
(protons = 20, electrons = 20- 2 = 18, neutrons = 40.1 – 20 = 20.1)
Fe: 1s2 2s2 2p6 3s2 3p6 3d6 4s2
Fe2+: 1s2 2s2 2p6 3s2 3p6 3d6 (electrons removed from 4s orbital)
protons = 26, electrons = 26- 2 = 24, neutrons = 55.8 – 26 = 29.8)
Thus only student Q is correct; student R is wrong as both have more neutrons than
protons.
Ans: A

6 Iodic acid is a strong acid that exist as a white solid at room temperature. It has a
chemical formula of HIO3 and dissolves readily in water to give H+ and IO3- ions. Which
of the following statements about the ion IO3- is true?
A It is a non-planar ion.
B It has a bond angle of 120°.
C Only sigma bonds are formed between iodine and oxygen in the ion.
D The oxidation number of iodine in the ion is negative.

[Turn over
www.gradtutors.com
604
5

- Bond between iodine and oxygen are both sigma and pi (due to double bond formed)
- oxidation number of iodine (x) in ion: -6 + x = -1 Thus x = +5
Ans: A
7 The following compounds are isomers with molecular formula C5H10O2. Which of them
is the least volatile?

Intermolecular forces for


A: VDWs
B: VDWs
C: Hydrogen bonding
D: Hydrogen bonding
Thus, C and D would have a higher boiling point (bp) than A and B
However, between C and D, C has intramolecular hydrogen bonding due to proximity
of the 2 OH groups resulting in a less extensive intermolecular hydrogen bonding in C.
Thus, D would have the highest bp among the 4 options making it the least volatile.
Ans: D

8 Graphite is a common form of carbon that has many uses. One of them is as lubricants

[Turn over
www.gradtutors.com
605
6

in machine parts. Which of the following properties of graphite allow for it to be used as
lubricants?
A Each carbon atom in graphite is sp2 hybridised.
B Graphite has a very high melting point.
C The interactions between layers of graphite is van der Waals forces of attraction.
D Graphite has an extended π-electron cloud above and below each plane of
carbon atoms.
The property that allow for graphite to be a lubricant is due to the fact that it is soft and
the layers can slide pass each other and this is due to the fact that the interactions
between the layers in graphite is weak VDWs forces of attraction.
Ans: C

9 Which of the following arranges the ionic salts in increasing magnitude of their lattice
energy?
A Al2O3 < MgO < MgBr2 < NaBr
B NaBr < MgO< MgBr2 < Al2O3
C MgO < NaBr < Al2O3 < MgBr2
D NaBr < MgBr2 < MgO < Al2O3
The magnitude of lattice energy is dependant/proportionate to (q+)(q-)/(r+)+(r-)
From the formula, one can see that
- it is dependant on both charge of the ions and the ionic radius.
- charge would take higher priority over ionic radius since in the formula, charge
is multiplied but ionic radius is just the sum or addition.
- The magnitude also varies proportionately to charge but inversely proportionate
to ionic radius
Thus, Al2O3 will have the largest magnitude since the product of the charges is the
largest (3 x 2 = 6)
This is followed by MgO, whose product of the charges is the second largest (2 x 2 =4)
This is then followed by MgBr2 (2 x 1 = 2)
The smallest magnitude is by NaBr whose product of the charges is (1 x 1 = 1)
Ans: D

[Turn over
www.gradtutors.com
606
7

10 The energy cycle below shows the reaction pathways between Compounds J – M.

∆H1 ∆H2

∆H4
∆H3 ∆H5

What is the enthalphy change for the following reaction?


Compound J Compound M
A ∆H1 + ∆H2
B ∆H2 - ∆H3 - ∆H4
C ∆H3 + ∆H5
D ∆H4 - ∆H1 - ∆H5
By Hess Law, going starting with compound J and ending with compound M,
We can go from compound J to L (-∆H3 as going against the arrow), then from L to K (-
∆H4 as going against the arrow) and finally from K to M (+∆H4 as going with the arrow)
Thus, ∆H for cpd J to M = - ∆H3- ∆H4+ ∆H2 = ∆H2- ∆H3- ∆H4
Ans: B

11 To study the kinetics of a reaction between X, Y and Z, a student conducted a series of


experiments and the following results were obtained:
Experiment Vol of X / Vol of Y / Vol of Z / Vol of H2O / Relative
3 3 3 3
cm cm cm cm rate
1 20 15 10 55 1
2 40 15 10 35 4
3 40 25 15 20 6
4 40 15 20 25 8

What is the rate equation for the reaction?


A Rate = [X]2[Z]
B Rate = [X]2[Y]
C Rate = [X][Y][Z]
D Rate = [X]2[Y][Z]

[Turn over
www.gradtutors.com
607
8

Since its volume and the total volume used for all experiment is 100, volume can be
used to compare directly as volume is proportionate to concentration.
Comparing expt 1 & 2,
When [X] doubled, rate increase by 4 times. Hence order of reaction w.r.t. X is 2
Comparing expt 2 & 4,
When [Z] doubled, rate increase by 2 times. Hence order of reaction w.r.t. Z is 1
Comparing expt 2 & 3 and based on what was calculated previously,
rate3/rate4 = [k(40)2(15)y(10)] / [k(40)2(25)y(15)]
4/6 = (10/15)(15/25)y
2/3 = (2/3)(3/5)y
(3/5)y = 1
y=0
Thus rate equation: rate = k[X]2[Z]
Ans: A
12 A technician found the following information in a handbook about an equilibrium
reaction that the factory carries out on an industrial scale.

Temperature / °C Kc
250 1 x 10-5
500 1 x 10-4
Pressure / atm % Product
5 33
10 47

Based on the above information, what can be said about the equilibrium reaction that
the factory carries out?
The forward reaction is endothermic and there are more gaseous molecules in
A
the products than reactants.
The forward reaction is exothermic and there are less gaseous molecules in the
B
products than reactants.
The forward reaction is endothermic and there are less gaseous molecules in the
C
products than reactants.
The forward reaction is exothermic and there are more gaseous molecules in the
D
products than reactants.

[Turn over
www.gradtutors.com
608
9

When temperature increased, Kc increased. This means the formation of the product is
favoured when temperature increase. That would mean that the forward reaction must
be endothermic (to absorb the excess heat)
When pressure increase, % of product also increased. This means the product side
would have less no. of gaseous molecules in order to counter the increase in pressure.
Ans: C

13 Which of the following equilibrium equations have no units for their equilibrium
constant, Kc?
A H2O(g) + C(s) H2(g) + CO(g)

D
Only option B includes all the species reflected in the equation in the Kc and the
number of moles of reactants is the same as the products resulting in the units in the
product cancelling the units of the reactants and hence no units for Kc.
Ans: B

14 The equation for the dissociation of water is as follows:


∆H = + 57 kJmol-1

Which of the following statements about the above equilibrium is correct?


A As temperature increases, the pH of water also increases.
B As the volume of water increases, the pH of water also increases.
C The summation of pH and pOH is 14 at all temperatures.
D [H+] will always equal to [OH-] at all temperatures.
A: When temperature increase, the forward reaction would be favoured. This would
result in [H+] to also increase. This would result in pH to decrease and not increase.
B: [H+] would remain constant even when volume of water increase (as both no. of
moles of water and volume increase proportionately). Hence pH would be constant and
not increase.
C: pH + pOH is only 14 at 25°C
D: This one is true as the dissociation of water at any temperature would always yield
the same no. of moels of H+ and OH-.
Ans: D

[Turn over
www.gradtutors.com
609
10

15 Given that both NaOH and KOH are strong mineral bases, calculate the resultant pH
when 20 cm3 of 0.300 moldm-3 of NaOH is mixed with 30 cm3 of 0.200 moldm-3 of KOH.
A 11.8
B 12.1
C 13.4
D 13.7
No. of moles of OH- from NaOH = (20/1000)x 0.3 = 0.006
No. of moles of OH- from KOH = (30/1000) x 0.2 = 0.006
Total concentration of OH- = (0.006 x 2) / [(20+30)/1000] = [0.012/(50/1000)]
= 0.24 moldm-3
Thus, pOH = -log(0.24) = 0.620 and pH = 14 – 0.620 = 13.4
Ans: C

16 Which of the following properties show a decreasing trend across the elements of
Period 3?
A pH of their chlorides in water
B Melting point of their oxides
C Electrical conductivity
D Ionic radius
A: It decreases across all elements in Period 3.
B: Melting point actually increases first from Na2O to MgO before decreasing.
C: This one also increases from Na to Al before decreasing.
D: There is an increase from Si to P
Ans: A

17 An element in Period 3 has the following properties:


- It does not conduct electricity.
- Its first ionisation energy is larger than both the elements before and after it.
Which of the following statements about this element is correct?
A It has a high melting and boiling point.
B It forms chlorides that are mildly acidic.
C It forms oxides that are very acidic.
Its oxidation state in compounds usually follows the group number it belongs to in
D
the Periodic Table.

[Turn over
www.gradtutors.com
610
11

Based on the information given in the question, the element has to be phosphorus.
Hence, based on the property of P, the only one that is true is option C
Ans: C

18 What is the IUPAC name of the skeletal structure below?

A 2-ethyl-5,5-dimethylpentane
B 2-ethyl-5-methylhexane
C 1,2,5-trimethylhexane
D 2,5-dimethylheptane
The longest carbon chain is as highlighted in the question.
Since longest carbon chain is 7 carbon, the backbone of the molecule is heptane
The methyl group is on carbon 2 and 5
Hence IUPAC name is 2,5-dimethylheptane
Ans: D

19 How many positional isomers are there for a dichlorobutane?


A 5
B 6
C 7
D 8
The isomers are as follows:

[Turn over
www.gradtutors.com
611
12

Ans: B

20 Ethene is shaken with an aqueous mixture of iodine and sodium bromide.


Which of the following statements about the above reaction is incorrect?
A There will at least one bromine atom found in all of the products formed.
B There will be a mixture of three different products formed.
C This is an addition reaction.
D The role of iodine in this reaction is an electrophile.
This is an addition reaction between ethane and iodine.
Hence, iodine is the electrophile that will be attacked by the double bond
The OH- and Br- from NaBr will only be added to the intermediate to complete the
reaction together with iodide ion (I-) and hence a possibility of 3 different products
The possible products are:

Ans: A

21 Which of the following properties of benzene is not associated with the ring of
delocalised electrons?
A Benzene tend to attract electrophiles rather than nucleophiles.
B Benzene tend to undergo substitution reactions rather than addition reactions
C Conversion of benzene to cyclohexane requires very harsh conditions.
D Benzene is a planar molecule with all the carbon atoms lying on the same plane.

[Turn over
www.gradtutors.com
612
13

A: Due to the ring of delocalised electrons, they would attract electrophiles, which are
electron deficient species
B: A benzene reaction would always want to re-establish the ring of delocalised
electrons and hence undergo substitution rather than addition that would destroy the
ring of delocalised electrons
C: The delocalised electrons help to stabilize benzene and hence any reaction that
destroys the ring of delocalised electrons will need very harsh conditions
D: Althought true, benzene is planar has nothing to do with the ring of delocalised
electrons but due to the no. of electrons and lone pair each C atom have
Ans: D

22 The following are three different types of halogen derivatives:

A student then added ethanolic AgNO3 separately to the 3 halogen derivatives above.
Which of the following shows the correct order in which the precipitate appears, from
the fastest to the slowest?
A 1-bromohexane, iodobenzene, 1-chloro-2-phenylethane
B 1-bromohexane, 1-chloro-2-phenylethane, iodobenzene
C iodobenzene, 1-bromohexane, 1-chloro-2-phenylethane
D 1-chloro-2-phenylethane, 1-bromohexane, iodobenzene
The reaction mentioned in the questions for RX molecules involve the breaking of the
C-X (halogen) bond. The strongest is the iodine attached to benzene and hence it
would be the slowest to show the ppt.
Between the first and the 3rd molecule, the C-Br bond is weaker and hence would
break faster and hence show the ppt the fastest.
Hence the order from the fastest to slowest should be molecule 3, 1 and then 2.
Ans: B

[Turn over
www.gradtutors.com
613
14

23 Which of the following reagents and conditions, when added to ethanol, involve the
cleavage of the O-H bond in ethanol?
A CH3COOH, excess concentrated H2SO4 and reflux
B I2 dissolved in NaOH and warm
C PCl5 at room temperature
D K2Cr2O7, H2SO4(aq) and heat
A: It involved the breaking of the O-H bond in the alcohol
B: It breaks the C-CH3 bond in ethanol
C: It breaks the C-O bond in ethanol.
D: It involves the breaking of the bonds of the C with the OH group but does not break
the O-H bond.
Ans: A

24 Which of the following reagents can be used to distinguish between the following 2
compounds?

A 2,4-dinitrophenylhydrazine
B Fehling’s solution
C Tollen’s reagent
D LiAlH4 in dry ether
A: Will react with both as both are carbonyl compounds
B: Will not react with both of them as both are non-aliphatic aldehydes
C: Will only react with benzaldehyde but not the ketone
D: Will react with both and reduce both to alcohols.
Ans: C

[Turn over
www.gradtutors.com
614
15

25 Below are 3 compounds of carboxylic acids and its derivatives.

Which of the following arranges the pH of their solutions in increasing order when they
are placed in water?
A , ,
B , ,
C , ,
D , ,
The third one is an acid chloride and hence when dissolves in water to give HCl, a very
strong mineral acid and hence the most acidic with the lowest pH.
Between the first 2, the 2nd molecule has the Cl group nearer the COOH group. Being a
electron-withdrawing group, the Cl nearer the COOH group help to better disperse the
negative charge in the conjugate base in the 2nd molecule than the 1st
Hence the 2nd molecule is more acidic with a lower pH than the 1st molecule.
Hence in increasing order of pH, it should be the 3rd molecule, followed by the 2nd and
then the 1st.
Ans: C

[Turn over
www.gradtutors.com
615
16

Section B
For each of the questions in this section, one or more of the thress numbered statements 1
to 3 may be correct.
Decide whether each of the statements is or is not correct (you may find it helpful to put a
tick against the statements that you consider to be correct).
The responses A to D should be selected on the basis of
A B C D
1, 2 and 3 1 and 2 only 2 and 3 only 1 only
are correct are correct are correct is correct

26 BeCl2 is able to react with PCl3 to form a stable product.


Which of the statements about this reaction are correct?
1 The mole ratio of BeCl2 : PCl3 for this reaction is 1 : 2.
The product in this reaction is able to form because beryllium has energetically
2
accessible orbitals to expand its octet configuration.
3 The bond angle about phosphorus does not change during this reaction.
1: True as BeCl2 has only 4 outer most shell electrons around Be and hence have 2
empty orbitals to accept 4 more electrons to fulfil its octet rule. Since PCl3 only has one
lone pair, 2 moles of PCl3 is needed and hence the ratio of the reaction between BeCl2
and PCl3 is 1: 2.
2: Not true as Be is in period 2 and hence unable to expand its octet.
3. Not true as a new bond is formed between P and Be in the reaction, the bond angle
around P is bound to change.
Ans: D

27 A student investigated a reaction between compounds T, U and V and found that the
order of reaction is first order with respect to T, zero order with respect to U and
second order with respect to V. When a graph was drawn, a straight line was obtained.
Which of the following could be the axis of this graph?
1 Rate against [T]
2 Rate against [U]
3 Rate against [V]2

1: True as the rate is first order w.r.t T, you will get a straight line through the origin

[Turn over
www.gradtutors.com
616
17

when this graph is plotted.


2: True as the rate is zero order w.r.t U, you will get a straight vertical line when this
graph is plotted
3: True as the rate is second order w.r.t V, you will get a straight line through the origin
when this graph is plotted.
Ans: A
The responses A to D should be selected on the basis of

A B C D
1, 2 and 3 1 and 2 only 2 and 3 only 1 only
are correct are correct are correct is correct

28 Vitamin C is a common compound found in many citrus fruits such as oranges and
pineapples. It has the following structure:

Which of the following statements about Vitamin C is incorrect?


1 It is able to exhibit geometric isomerism.
2 It will give orange crystals with Brady’s reagent.
3 1 mole of Vitamin C will react with excess sodium to give 2 moles of hydrogen
gas.
1: Its incorrect (and hence an answer) as the molecule does not exibit geometric
isomerism as it does not have an aliphatic alkene or cyclic alkane with different groups.
2. Its incorrect (and hence an answer) as it does not contain any carbonyl group.
3.Its correct (and hence not an answer) as it will react with all 4 OH groups and hence
give 2 moles of H2 gas.
Ans: B

[Turn over
www.gradtutors.com
617
18

The responses A to D should be selected on the basis of

A B C D
1, 2 and 3 1 and 2 only 2 and 3 only 1 only
are correct are correct are correct is correct

29 The following synthetic pathway was proposed by a student:

Which of the following steps are unable to give the desired product that the student
proposed in this synthesis?
1 Step I
2 Step II
3 Step III
Step 1: Unable to as NaBH4 is unable to reduce the acid group, only LiAlH4 is able to.
Step 2: Unable to work as well as the AlCl3 is (aq) which will destroy it and hence not
perform its role as a catalysis to generate the Cl electrophile.
Step 3: Able to work as it’s a reagent to substitute the OH group with a Cl group.
Ans: B

[Turn over
www.gradtutors.com
618
19

The responses A to D should be selected on the basis of

A B C D
1, 2 and 3 1 and 2 only 2 and 3 only 1 only
are correct are correct are correct is correct

30 Below is the synthesis pathway of Carvone, the main flavouring compound in


spearmint oil.

Which of the following types of reactions are shown in the synthetic pathway above?
1 Addition
2 Reduction
3 Substitution
1: Step 2 is an addition reaction of the ketone group.
2: Step 3 is a reduction reaction of the CN group.
3: Step 1 is a substitution reaction being a free radical substitution.
Ans: A
- THE END -

[Turn over
www.gradtutors.com
619
20

Answers:
1 B 6 A 11 A 16 A 21 D 26 D
2 C 7 D 12 C 17 C 22 B 27 A
3 D 8 C 13 B 18 D 23 A 28 B
4 B 9 D 14 D 19 B 24 C 29 B
5 A 10 B 15 C 20 A 25 C 30 A

[Turn over
www.gradtutors.com
620
2
 

Section A (40 marks)

Answer all the questions in this section in the space provided. 

1 (a) The first eight ionisation energies of element X are shown in the graph below.

Ionisation energy / kJ

0
1 2 3 4 5 6 7 8 Electrons removed

(i) To which Group does element X belong to? Explain your answer.

X: _____________

_____________________________________________________________

_____________________________________________________________

W, X and Y are consecutive elements of increasing proton number in Periods 3.


These elements forms ions which are isoelectronic with element Ar.
(ii) Explain the term ‘isoelectronic’.

_____________________________________________________________

_____________________________________________________________

(iii) Write the formula of the ions of W, X and Y which are isoelectronic with
element Ar.

W: ________ X: ________ Y: ________ [5]

[Turn Over

 
www.gradtutors.com
621
3
 

1 (b) (i) Sketch the pH trend of the resultant solutions when the oxides of elements
W, X and Y are added to water in the axis below.

pH

Oxides of Elements
W X Y

(ii) Account for the trend above.

_____________________________________________________________

_____________________________________________________________

_____________________________________________________________

_____________________________________________________________ [4]

[Total: 9]

2 Nitrogen dioxide, NO2, dimerises to form dinitrogen tetroxide, N2O4, which is the more
stable form in the following equilibrium.
2 NO2(g) N2O4(g) ∆H = - 57.2 kJ mol-1

(a) (i) Draw the dot-and-cross diagram of NO2 and N2O4.


NO2 N2O4

[Turn Over

 
www.gradtutors.com
622
4
 

2 (a) (ii) Suggest a reason why the enthalpy change for the forward reaction is
exothermic.
_____________________________________________________________

_____________________________________________________________ [3]

(b) During an experiment, changes were made to the conditions in the reaction vessel.
At each time, there was only one change made to the condition in the reaction
vessel.
The change in the concentrations in the equilibrium mixture with time is shown in
the graph below.
Concentration/ mol dm-3

NO2

1.0

0.9

N2O4
0.6

0.3

T1 T2 T3 time/ s
(i) Write the Kc expression for the equilibrium.

[Turn Over

 
www.gradtutors.com
623
5
 

2 (b) (ii) Calculate the Kc at T1, stating the units.

[3]

(c) (i) What changes in condition were made at time T1 and T2?

T1: ________________________

T2: ________________________

(ii) Sketch and label clearly on the given graph on page 4, the changes in the
concentration of NO2 and N2O4, when the mixture was cooled at time T3.
Explain your answer.

_____________________________________________________________

_____________________________________________________________

_____________________________________________________________ [5]

(d) Sketch, on the axes below, how the rates of the forward and reverse reactions
change from the time NO2 was placed in the vessel to the time equilibrium was
reached at time T1. Label your graphs clearly.

rate

time
[1]
[Total: 12]
[Turn Over

 
www.gradtutors.com
624
6
 

3 (a) Hydrazine, N2H4, is a flammable liquid with an ammonia like odour. It has basic
chemical properties like ammonia.
(i) Suggest the equation for the reaction between hydrazine and water,
assuming N2H4 is a monoprotic base.

___________________________________________________________

The graph below shows the pH changes when 10.0 cm3 of hydrazine was titrated
with 0.05 mol dm-3 hydrochloric acid.

14

12 A
B
10

pH 8
C
6 D

4
E F G
2

0
10.0 20.0 30.0
Volume of HCl added / cm3

Indicator pKa acid form base form

Congo red 3-5 blue red

Phenol Red 6.8 - 8.4 yellow red

Thymolphthalein 9.3-10.5 colourless blue

(ii) From the table above, suggest a suitable indicator for the above titration.

Indicator:______________________

(iii) Suggest the colour change observed at the end-point.

___________________ to ______________________

[Turn Over

 
www.gradtutors.com
625
7
 

3 (a) (iv) When 7.5 cm3 of HCl is added to excess hydrazine, the resultant solution act
as a buffer. Write an equation to explain what happens when some solid
sodium hydroxide is added to this buffer solution.

_____________________________________________________________ [4]

(b) Hydrazine and its derivatives can undergo condensation reactions with carbonyl
compounds as shown.

A similar condensation reaction takes place between CH3CH2NH2 and propanone,


CH3COCH3.
Draw the structural formula of the product formed for the above reaction.

[1]
[Total: 5]

4 (a) Pentanoic acid could be synthesized from 2-bromo-hexane via two different
synthetic routes as shown below, where X, Y and Z are different compounds.
 

[Turn Over

 
www.gradtutors.com
626
8
 

4 (a) (i) Suggest the reagents and conditions for steps 2 – 4.

Step 2: ______________________________

Step 3: ______________________________

Step 4: ______________________________

(ii) State the type of reaction undergone in step 3.

Type of reaction: ________________________

(iii) Draw the full structural formulae of Compounds X, Y and Z.

X Y

[Turn Over

 
www.gradtutors.com
627
9
 

4 (a) (iv) Suggest a simple chemical test which you will carry out to distinguish
between X and Y.

Reagents and Conditions: _____________________________________

Observations: _______________________________________________

___________________________________________________________

___________________________________________________________ [9]

(b) The following table shows the pKa values of pentanoic acid, 3-hydroxypentanoic
acid and pentanol.
Compound pKa
pentanoic acid 4.87
3-hydroxypentanoic acid 3.89
pentanol 16

(i) Suggest a reason why the pKa value of pentanoic acid is less than that of
pentanol.

____________________________________________________________

____________________________________________________________

____________________________________________________________

(ii) Suggest a reason why 3-hydroxypentanoic acid is more acidic than


pentanoic acid.

____________________________________________________________

____________________________________________________________

____________________________________________________________  [4]

[Turn Over

 
www.gradtutors.com
628
10
 

4 (c) When concentrated sulfuric acid was added to 3-hydroxypentanoic acid, the
mixture was refluxed. Only one organic product was obtained.
Draw the structural formula of the product formed.

[1]
[Total: 14]

[Turn Over

 
www.gradtutors.com
629
11

Section B
Answer two questions from this section on separate answer paper.

5 Ethylamine, CH3CH2NH2, is a colourless gas with a strong odour. It can be synthesised


by reacting ethene with ammonia, as shown in the equation below:

(a) (i) State the hybridisation of the C atoms in ethene and in ethylamine.
(ii) The C-H bonds in ethene are shorter than the C-H bonds in ethylamine. With
reference to the hybridisation theory, explain the difference in bond length. [4]

(b) Using relevant data from the Data Booklet, calculate a value for the enthalpy
change for above reaction. [3]

(c) An experiment was carried out to study the kinetics of the synthesis of ethylamine
using ethene and ammonia. The rate equation was found to be
rate = k[CH2=CH2][NH3].
When CH2=CH2 and NH3, both at an initial concentration of 1.00 × 10-4 mol dm-3,
were mixed together at 20°C, [CH3CH2NH2] varies with time, as shown in the table
below.
Rate /
Time [CH3CH2NH2] [NH3] / mol [NH3]2 / [CH2=CH2] /
–3 –3 2 –6 –3
mol dm-3
/ min / mol dm dm mol dm mol dm
min-1
0 0 1.00 X 10-4 1.00 X 10-8 1.00 X 10-4 0.00
–5
5 5.00 × 10
10 6.90 × 10–5
15 7.70 × 10–5
20 8.20 × 10–5

(i) Explain the meaning of the term rate equation.


(ii) The rate at a particular time can be approximated using
∆[CH3 CH2NH2 ]
rate = .
∆t
Reproduce and complete the above table on your writing paper and hence
plot a suitable straight line graph to confirm that the reaction shows overall
second order kinetics.
[Turn Over

www.gradtutors.com
630
12

5 (c) (iii) Hence, determine the rate constant k, with units. [8]

(d) The yield of propylamine from the reaction between propane and ammonia is low.
Propylamine can be produced from ethene via another process. Suggest how you
can make propylamine from ethene in 3 steps, showing the reagents and conditions
necessary as well as all intermediates. [5]
[Total: 20]

[Turn Over

www.gradtutors.com
631
13

6 (a) A solid sample contains a mixture of sodium oxide and barium oxide only. The
percentage by mass of barium oxide in the sample was determined using a
titrimetric method.

The sample of unknown mass was reacted with 50 cm3 of 0.0100 mol dm-3 of dilute
sulfuric acid, forming a white precipitate, barium sulfate, and a colourless solution.
The resultant mixture was filtered. Upon drying, the residue weighed 0.0352 g.

The filtrate was titrated against 0.0200 mol dm-3 solution of NaOH. 17.30 cm3 of
NaOH was required to completely neutralise the excess sulfuric acid.
(i) Write two separate equations, with state symbols, for the reactions between
the two oxides and sulfuric acid.
(ii) Calculate the mass of barium oxide present in the sample.
(iii) Calculate the amount of sulfuric acid that reacted with the sample of solid
oxides.
(iv) Hence calculate the amount of sulfuric acid reacted with solid sodium oxide
only.
(v) Calculate the total mass of the sample of solid oxides and hence the
percentage by mass of barium oxide in the sample. [9]

(b) Solution P was made by dissolving 0.500 g of Na2O and 0.232 g of BaO in 500 cm3
of water. Solution P was then used to determine the concentration of a solution of
ethanoic acid via titration.
(i) Calculate the pH of solution P.
(ii) Suggest a suitable indicator for the titration. [4]

(c) Alcohol F, C6H14O, changes orange acidified potassium dichromate (VI) to green,
and gives a yellow precipitate with warm aqueous alkaline iodine. F reacts with
concentrated sulfuric acid at 180°C to give G as the only product.
On treating G with an excess of hot acidified potassium manganate (VII),
compound H, C5H10O2, is formed.
H gives effervescence with solid sodium hydrogencarbonate.
Use the information above to deduce the structural formulae for F, G and H,
explaining your reasoning. [7]
[Total: 20]
[Turn Over

www.gradtutors.com
632
14

7 (a) Grignard reagents are typically used as a reagent to form alcohols from carbonyl
compounds.
.

(i) Suggest the starting carbonyl compound needed for the formation of pentan-
3-ol using CH3CH2MgBr as the Grignard reagent.

(ii) One of the problems associated with the use of magnesium ribbon to produce
Grignard reagents is the slow initial step. This is especially so when the
magnesium ribbon exposed to air is used. Suggest a reason.

(iii) A few drops of Universal Indicator turns yellow in an aqueous solution of


MgBr2. Explain with the aid of relevant equations. [4]

(b) Benzyl halides are chemical warfare agents used as tear agents. Two such benzyl
halides are benzyl chloride and benzyl bromide.

The boiling points of benzyl chloride and benzyl bromide are given below.
Compound Boiling point (°C)
benzyl chloride 179
benzyl bromide 201

(i) State the reagent and conditions needed to convert methylbenzene into
benzyl chloride. State the type of reaction involved.
(ii) Suggest a simple test-tube reaction you could use to distinguish benzyl
chloride from its isomer, 2-chloromethylbenzene. You should state the
reagent used and the observation you would expect to make.
[Turn Over

www.gradtutors.com
633
15

7 (b) (iii) Account for the difference in the boiling points between benzyl chloride and
benzyl bromide.
(iv) State the reagents and conditions needed to convert benzyl chloride to
benzaldehyde, C6H5CHO, in two steps, drawing any intermediate product
formed. [9]

(c) The enthalpy change of solution of barium hydroxide, ∆Hsol, is defined according to
the following equation:

Ba(OH)2(s) + aq Ba2+(aq) + 2OH-(aq)

An energy cycle is constructed using the information in the table below.

Enthalpy change kJ mol-1

Enthalpy change of hydration of Ba2+, ∆Hhyd(Ba2+) −1505

Enthalpy change of hydration of OH-, ∆Hhyd(OH-) −664

∆H1 −2850

∆Hsol
Ba(OH)2(s) + aq Ba2+(aq) + 2OH-(aq)

∆H1 ∆Hhyd(Ba2+) 2 x ∆Hhyd(OH-)

Ba2+(g) + 2OH-(g)

(i) Identify ∆H1.


(ii) Using the information given and the energy cycle above, calculate the
enthalpy change of solution for Ba(OH)2. [3]

[Turn Over

www.gradtutors.com
634
16

7 (d) The standard enthalpy change of neutralisation was determined experimentally by


mixing known volumes of aqueous benzoic acid and
aqueous barium hydroxide. The process was known to be only 80% efficient. The
following results were obtained.
 Initial temperature = 25.0 C
 Final temperature = 29.8 C
 Volume of 1.50 mol dm–3 C6H5COOH (aq) used = 40 cm3
 Volume of 0.50 mol dm–3 Ba(OH)2 (aq) used = 40 cm3

(i) Use the above data to calculate the standard enthalpy change of
neutralisation of benzoic acid. Assume that the specific heat capacity of all
solutions = 4.2 J K-1 cm-3
(ii) Predict, with a reason, if the enthalpy change of neutralisation reaction will be
more or less exothermic when hydrochloric acid is used instead. [4]
[Total: 20]

--The End--

www.gradtutors.com
635
2
 

Section A (40 marks)

Answer all the questions in this section in the space provided. 

1 (a) The first eight ionisation energies of element X are shown in the graph below.

Ionisation energy / kJ

0
1 2 3 4 5 6 7 8 Electrons removed

(i) To which Group does element X belong to? Explain your answer.

X: Group V

There is a big jump from the 5th IE to the 6th IE, suggesting that there are 5

valence electron and the 6th electron to be removed lies in the inner shell.

W, X and Y are consecutive elements of increasing proton number in Periods 3.


These elements forms ions which are isoelectronic with element Ar.
(ii) Explain the term ‘isoelectronic’.

Isoelectronic refers to species having the same number of electrons.

(iii) Write the formula of the ions of W, X and Y which are isoelectronic with
element Ar.

W: W4–/ Si4- X: X3–/ P3- Y: Y2– / S2- [5]

[Turn Over

www.gradtutors.com
636
3
 

1 (b) (i) Sketch the pH trend of the resultant solutions when the oxides of elements
W, X and Y are added to water in the axis below.

pH

Oxides of Elements
W X Y

(ii) Account for the trend above.

(Oxide of W)SiO2 has a giant molecular structure and is insoluble in water.

Hence, the pH of the solution is neutral at 7. (Oxides of X)P4O10/P4O6 and

(Oxides of Y) SO2/SO3 are acidic oxides which will dissolve in water to give

acidic solutions. Hence their pH is at 2 - 3. [4]

[Total: 9]

2 Nitrogen dioxide, NO2, dimerises to form dinitrogen tetroxide, N2O4, which is the more
stable form in the following equilibrium.
2 NO2(g) N2O4(g) ∆H = - 57.2 kJ mol-1

(a) (i) Draw the dot-and-cross diagram of NO2 and N2O4.


NO2 N2O4

[Turn Over

www.gradtutors.com
637
4
 

2 (a) (ii) Suggest a reason why the enthalpy change for the forward reaction is
exothermic.
Enthalpy change is exothermic as an N–N bond is formed (bond formation)

which releases energy. [3]

(b) During an experiment, changes were made to the conditions in the reaction vessel.
At each time, there was only one change made to the condition in the reaction
vessel.
The change in the concentrations in the equilibrium mixture with time is shown in
the graph below.
Concentration/ mol dm-3

NO2

1.0

0.9

N2O4
0.6

0.3

T1 T2 T3 time/ s
(i) Write the Kc expression for the equilibrium.

Kc = [N2O4]/ [NO2]2 

Note: (no curved brackets)

[Turn Over

www.gradtutors.com
638
5
 

2 (b) (ii) Calculate the Kc at T1, stating the units.

Kc = 0.3/0.92 = 0.37 mol-1dm3 (ecf from (i))

[3]

(c) (i) What changes in condition were made at time T1 and T2?

T1: Decrease in volume of vessel/Increase in Pressure

T2: Increase in concentration of NO2

(ii) Sketch and label clearly on the given graph on page 4, the changes in the
concentration of NO2 and N2O4, when the mixture was cooled at time T3.
Explain your answer.

According to Le Chatelier’s Principle, the equilibrium position will shift to the

right to favour the exothermic reaction to release heat when temperature is

decreased. Hence the mixture will contain more N2O4 and less NO2. [5]

(d) Sketch, on the axes below, how the rates of the forward and reverse reactions
change from the time NO2 was placed in the vessel to the time equilibrium was
reached at time T1. Label your graphs clearly.

rate

 
time
t1  [1]
[Total: 12]
[Turn Over

www.gradtutors.com
639
6
 

3 (a) Hydrazine, N2H4, is a flammable liquid with an ammonia like odour. It has basic
chemical properties like ammonia.
(i) Suggest the equation for the reaction between hydrazine and water,
assuming N2H4 is a monoprotic base.

N2H4 + H2O N2H5+ + OH–

The graph below shows the pH changes when 10.0 cm3 of hydrazine was titrated
with 0.05 mol dm-3 hydrochloric acid.

14

12 A
B
10

pH 8
C
6 D

4
E F G
2

0
10.0 20.0 30.0
Volume of HCl added / cm3

Indicator pKa acid form base form

Congo red 3-5 blue red

Phenol Red 6.8 - 8.4 yellow red

Thymolphthalein 9.3-10.5 colourless blue

(ii) From the table above, suggest a suitable indicator for the above titration.

Indicator: Congo Red

(iii) Suggest the colour change observed at the end-point.

Red to Blue

[Turn Over

www.gradtutors.com
640
7
 

3 (a) (iv) When 7.5 cm3 of HCl is added to excess hydrazine, the resultant solution act
as a buffer. Write an equation to explain what happens when some solid
sodium hydroxide is added to this buffer solution.

OH- + N2H5+  N2H4 + H2O [4]

(b) Hydrazine and its derivatives can undergo condensation reactions with carbonyl
compounds as shown.

A similar condensation reaction takes place between CH3CH2NH2 and propanone,


CH3COCH3.
Draw the structural formula of the product formed for the above reaction.

[1]

[Total: 5]

4 (a) Pentanoic acid could be synthesized from 2-bromo-hexane via two different
synthetic routes as shown below, where X, Y and Z are different compounds.
 

www.gradtutors.com
641
8
 

[Turn Over
4 (a) (i) Suggest the reagents and conditions for steps 2 – 4.

Step 2: KMnO4(aq), H2SO4(aq), heat

Step 3: NaOH(aq), I2(aq), warm

Step 4: H2SO4(aq)

(ii) State the type of reaction undergone in step 3.

Type of reaction: Oxidation

(iii) Draw the full structural formulae of Compounds X, Y and Z.

X Y

[Turn Over

www.gradtutors.com
642
9
 

4 (a) (iv) Suggest a simple chemical test which you will carry out to distinguish
between X and Y.

Reagents and Conditions: _____________________________________

Observations: _______________________________________________

Reagents and Conditions: PCl5 (s), rtp


Observations: white fumes for Y, no white fumes for X.
OR [9]
Reagents and Conditions: I2(aq), NaOH(aq), warm
Observations: Yellow ppt for Y, no yellow ppt for X
OR
Reagents and Conditions: KMnO4, NaOH(aq)
Observations: Purple KMnO4 decolourised with formation of brown ppt for
X, purple KMnO4 remained purple for Y.
OR
Reagents and Conditions: K2Cr2O7, H2SO4(aq), heat
Observations: Orange K2Cr2O7 turned green for Y, orange K2Cr2O7
remained orange for X.
OR
Reagents and Conditions: Br2 (aq)
Observations: Orange Br2 decolurised for X, orange Br2 remained orange
for Y.
Reagents and Conditions: Br2 in CCl4 in the dark
Observations: Reddish brown Br2 decolurised for X, reddish brown Br2
remained reddish brown for Y.
OR
Reagents and Conditions: Br2 (aq)
Observations: Orange Br2 decolurised for X, orange Br2 remained orange
for Y.
OR

www.gradtutors.com
643
10
 

(b) The following table shows the pKa values of pentanoic acid, 3-hydroxypentanoic
acid and pentanol.
Compound pKa
pentanoic acid 4.87
3-hydroxypentanoic acid 3.89
pentanol 16

(i) Suggest a reason why the pKa value of pentanoic acid is less than that of
pentanol.

Pentanoic acid is more acidic as the carboxylate ion can disperse the
negative charge over the two oxygen atoms and stabilises the anion. For
pentanol, the electron donating R group on pentoxide ion intensifies the
negative charge and destabilises the anion, making it much less acidic.

(ii) Suggest a reason why 3-hydroxypentanoic acid is more acidic than


pentanoic acid.

3–hydroxypentanoic acid can form intra–molecular hydrogen bonds


between the –OH and the –COO– and this stabilises the anion.
OR
the electron withdrawing –OH group disperses the negative charge on the
[4]
anion to a greater extent, hence stabilizing the anion to a greater extent. 

(c) When concentrated sulfuric acid was added to 3-hydroxypentanoic acid, the
mixture was refluxed. Only one organic product was obtained.
Draw the structural formula of the product formed.

O O

O
    OR  
[1]
[Turn Over

www.gradtutors.com
644
11
 

Section B

Answer two questions from this section on separate answer paper.

5 Ethylamine, CH3CH2NH2, is a colourless gas with a strong odour. It can be synthesised


by reacting ethene with ammonia, as shown in the equation below:

(a) (i) State the hybridisation of the C atoms in ethene and in ethylamine.
ethene – sp2
ethylamine – sp3
(ii) The C-H bonds in ethene are shorter than the C-H bonds in ethylamine. With
reference to the hybridisation theory, explain the difference in bond length. [4]
3 2
sp hybrid orbital has more p character / sp hybrid orbital has more s
character. Hence the sp3-s overlap is less effective / sp2-s overlap is more
effective.
(b) Using relevant data from the Data Booklet, calculate a value for the enthalpy
change for above reaction. [3]
Energy required to break bonds = 610 + 4(410) + 3(390)
= 3420 kJ mol-1
Energy evolved from forming bonds = 350 + 5(410) + 305 + 2(390)
= 3485 kJ mol-1
∆H = 3420 – 3485 = -65.0 kJ mol-1

www.gradtutors.com
645
12
 

(c) An experiment was carried out to study the kinetics of the synthesis of ethylamine
using ethene and ammonia. The rate equation was found to be
rate = k[CH2=CH2][NH3].
When CH2=CH2 and NH3, both at an initial concentration of 1.00 × 10-4 mol dm-3,
were mixed together at 20°C, [CH3CH2NH2] varies with time, as shown in the table
below.
Rate /
2
Time [CH3CH2NH2] [NH3] / mol [NH3] / [CH2=CH2] /
/ min / mol dm–3 dm–3 mol2 dm–6 mol dm–3 mol dm-3
min-1

0 0 1.00 X 10-4 1.00 X 10-8 1.00 X 10-4 0.00

5 5.00 × 10–5 5.00 × 10–5 2.50 × 10–9 5.00 × 10–5 1.00× 10–5

10 6.90 × 10–5 3.10 × 10–5 9.61 × 10–10 3.10 × 10–5 3.80× 10–6

15 7.70 × 10–5 2.30 × 10–5 5.29 × 10–10 2.30 × 10–5 1.60× 10–6

20 8.20 × 10–5 1.80 × 10–5 3.24 × 10–10 1.80 × 10–5 1.00× 10–6

(i) Explain the meaning of the term rate equation.


A rate equation is determined experimentally and it shows the relationship
between the rate of reaction and the concentrations of each reactant raised to
a specific power.
(ii) The rate at a particular time can be approximated using
∆[CH3 CH2NH2 ]
rate = .
∆t
Reproduce and complete the above table on your writing paper and hence
plot a suitable straight line graph to confirm that the reaction shows overall
second order kinetics.
Since the reaction is overall second order, the following rate equations may
be possible:
Rate = k [NH3]2 or Rate = k [NH3][CH2=CH2]
As such, a graph of rate against [NH3]2 or a graph of rate against
[NH3][CH2=CH2] would give a straight line. Values for [NH3]2 and
[NH3][CH2=CH2] should be identical since [NH3] = [CH2=CH2] at every timing.

www.gradtutors.com
646
13
 

3.00E‐09

2.50E‐09

Rate /mol dm‐3 min‐1
2.00E‐09

1.50E‐09

1.00E‐09

5.00E‐10

0.00E+00
0.00E+00 2.00E‐04 4.00E‐04 6.00E‐04 8.00E‐04 1.00E‐03 1.20E‐03
[NH3]2 /mol2 dm‐6

5 (c) (iii) Hence, determine the rate constant k, with units. [8]
-6 -1 3 -1
k = gradient of graph = 2.00 × 10 mol dm min

(d) The yield of propylamine from the reaction between propane and ammonia is low.
Propylamine can be produced from ethene via another process. Suggest how you
can make propylamine from ethene in 3 steps, showing the reagents and conditions
necessary as well as all intermediates. [5]

[Total: 20]

www.gradtutors.com
647
14
 

6 (a) A solid sample contains a mixture of sodium oxide and barium oxide only. The
percentage by mass of barium oxide in the sample was determined using a
titrimetric method.

The sample of unknown mass was reacted with 50 cm3 of 0.0100 mol dm-3 of dilute
sulfuric acid, forming a white precipitate, barium sulfate, and a colourless solution.
The resultant mixture was filtered. Upon drying, the residue weighed 0.0352 g.

The filtrate was titrated against 0.0200 mol dm-3 solution of NaOH. 17.30 cm3 of
NaOH was required to completely neutralise the excess sulfuric acid.
(i) Write two separate equations, with state symbols, for the reactions between
the two oxides and sulfuric acid.
Na2O (s) + H2SO4 (aq)  Na2SO4 (aq) + H2O (l)
BaO (s) + H2SO4 (aq)  BaSO4 (s) + H2O (l)
(ii) Calculate the mass of barium oxide present in the sample.
amount of BaO = amount of BaSO4 = 0.0352/233.1 = 1.510 × 10-4 mol
mass of BaO = 1.510 × 10-4 × 153 = 0.0231 g
(iii) Calculate the amount of sulfuric acid that reacted with the sample of solid
oxides.
amount of excess H2SO4 = 0.5 × 0.0173 × 0.0200 = 1.73 × 10-4 mol
amount of H2SO4 reacted with the oxides = 0.05 × 0.01 – 1.73 × 10-4
= 3.27 × 10-4 mol
(iv) Hence calculate the amount of sulfuric acid reacted with solid sodium oxide
only.
amount of H2SO4 reacted with Na2O = 3.27 × 10-4 - 1.510 × 10-4
= 1.76 × 10-4 mol
(v) Calculate the total mass of the sample of solid oxides and hence the
percentage by mass of barium oxide in the sample. [9]
mass of Na2O in the sample = 1.76 × 10-4 × 62 = 0.0109 g
total mass of the sample = 0.0109 + 0.0231 = 0.0340 g
percentage by mass of BaO = 0.0231/0.0340 × 100% = 67.9 %

www.gradtutors.com
648
15
 

(b) Solution P was made by dissolving 0.500 g of Na2O and 0.232 g of BaO in 500 cm3
of water. Solution P was then used to determine the concentration of a solution of
ethanoic acid via titration.
(i) Calculate the pH of solution P.
Na2O + H2O  2 NaOH
amount of OH- from Na2O = 0.500/62 × 2 = 0.01613 mol
BaO + H2O  Ba(OH)2
amount of OH- from BaO = 0.232/153 × 2 = 0.003033 mol
total amount of OH- = 0.01916 mol
[OH-] = 0.01916/0.5 = 0.03833 mol dm-3
pOH = -lg [OH-] = 1.417
pH = 14 – pOH = 12.6
(ii) Suggest a suitable indicator for the titration. [4]
Phenolphthalein

(c) Alcohol F, C6H14O, changes orange acidified potassium dichromate (VI) to green,
and gives a yellow precipitate with warm aqueous alkaline iodine. F reacts with
concentrated sulfuric acid at 180°C to give G as the only product.
On treating G with an excess of hot acidified potassium manganate (VII),
compound H, C5H10O2, is formed.
H gives effervescence with solid sodium hydrogencarbonate.
Use the information above to deduce the structural formulae for F, G and H,
explaining your reasoning. [7]

Information Deductions
Alcohol F changes orange F undergoes oxidation. [1/2]
K2Cr2O7 to green. F contains 1° or 2° alcohol.[1/2]
F gave yellow precipitate with F has the structure CH3CH(OH)-.
warm aqueous alkaline iodine. [1/2]
Oxidation. [1/2]

F reacts with concentrated sulfuric Elimination [½]


acid at 180°C to give G as the only G is an alkene [1/2]
product. Only one neighbouring C of the C
atom with the –OH group on F has
H atom attached to it. [1/2]

On treating G with an excess of Oxidation

www.gradtutors.com
649
16
 

hot acidified KMnO4, compound H, H has one less C atom, C=C of G


C5H10O2, is formed. on terminal carbon. [1]
H gives effervescence with solid H is a carboxylic acid. [1/2]
sodium hydrogencarbonate. acid-carbonate / neutralisation
reaction [1/2]

[Total: 20]

7 (a) Grignard reagents are typically used as a reagent to form alcohols from carbonyl
compounds.
.

(i) Suggest the starting carbonyl compound needed for the formation of pentan-
3-ol using CH3CH2MgBr as the Grignard reagent.
Propanal

(ii) One of the problems associated with the use of magnesium ribbon to produce
Grignard reagents is the slow initial step. This is especially so when the
magnesium ribbon exposed to air is used. Suggest a reason.
MgO will be formed.

www.gradtutors.com
650
17
 

(iii) A few drops of Universal Indicator turns yellow in an aqueous solution of


MgBr2. Explain with the aid of relevant equations. [4]
2+
The high polarising power of Mg causes MgBr2 to undergo partial hydrolysis
+
and H ions are formed, accounting for the mild acidic solution.
MgBr2 (s) + aq  [Mg(H2O)6]2+ (aq) + 2Br (aq)

[Mg(H2O)6]2+ (aq) + H2O [Mg(H2O)5OH]+ (aq) + H3O+ (aq)

(b) Benzyl halides are chemical warfare agents used as tear agents. Two such benzyl
halides are benzyl chloride and benzyl bromide.

The boiling points of benzyl chloride and benzyl bromide are given below.
Compound Boiling point (°C)
benzyl chloride 179
benzyl bromide 201

(i) State the reagent and conditions needed to convert methylbenzene into
benzyl chloride. State the type of reaction involved.
limited Cl2(g), UV light
Substitution/ Free radical substitution
(ii) Suggest a simple test-tube reaction you could use to distinguish benzyl
chloride from its isomer, 2-chloromethylbenzene. You should state the
reagent used and the observation you would expect to make.
aqueous NaOH, heat. Cool, excess dil HNO3, AgNO3(aq)
OR
Heat with ethanolic AgNO3
Benzyl chloride will give a white ppt, 2-chloromethylbenzene will not give
white ppt.
(iii) Account for the difference in the boiling points between benzyl chloride and
benzyl bromide.
Benzyl bromide has more electrons and hence more polarisable, resulting in
a more extensive induced dipole-induced dipole interactions.
More energy required to overcome the stronger id-id in benzyl bromide.

www.gradtutors.com
651
18
 

(iv) State the reagents and conditions needed to convert benzyl chloride to
benzaldehyde, C6H5CHO, in two steps, drawing any intermediate product
formed. [9]
Step 1: NaOH (aq), heat
Step 2: K2Cr2O7 (aq), H2SO4 (aq), heat to distill

Intermediate:
(c) The enthalpy change of solution of barium hydroxide, ∆Hsol, is defined according to
the following equation:

Ba(OH)2(s) + aq Ba2+(aq) + 2OH-(aq)

An energy cycle is constructed using the information in the table below.

Enthalpy change kJ mol-1

Enthalpy change of hydration of Ba2+, ∆Hhyd(Ba2+) −1505

Enthalpy change of hydration of OH-, ∆Hhyd(OH-) −664

∆H1 −2850

∆Hsol
Ba(OH)2(s) + aq Ba2+(aq) + 2OH-(aq)

∆Hhyd(Ba2+) 2 x ∆Hhyd(OH-)
∆H1
Ba2+(g) + 2OH-(g)

(i) Identify ∆H1.


Lattice energy of Ba(OH)2
(ii) Using the information given and the energy cycle above, calculate the
enthalpy change of solution for Ba(OH)2. [3]
- 2+
∆H1 + ∆Hsol = 2∆Hhyd(OH ) + ∆Hhyd(Ba )
∆Hsol = 2 x (−664) + (−1505) + −2850
= +17 kJ mol-1

www.gradtutors.com
652
19
 

7 (d) The standard enthalpy change of neutralisation was determined experimentally by


mixing known volumes of aqueous benzoic acid and
aqueous barium hydroxide. The process was known to be only 80% efficient. The
following results were obtained.
 Initial temperature = 25.0 C
 Final temperature = 29.8 C
 Volume of 1.50 mol dm–3 C6H5COOH (aq) used = 40 cm3
 Volume of 0.50 mol dm–3 Ba(OH)2 (aq) used = 40 cm3

(i) Use the above data to calculate the standard enthalpy change of
neutralisation of benzoic acid. Assume that the specific heat capacity of all
solutions = 4.2 J K-1 cm-3
q = mc∆T
= (80)(4.2)(29.8-25)
= 1612.8J
Since Ba(OH)2 is the limiting reagent, no. of moles of Ba(OH)2 = 0.020 mol
nH2O= 0.040 mol
∆H = -1612.8 (100/80) / 0.040 = -50400J = -50.4 kJ mol-1
(ii) Predict, with a reason, if the enthalpy change of neutralisation reaction will be
more or less exothermic when hydrochloric acid is used instead. [4]
Enthalpy change for neutralisation reaction will be more exothermic.
Hydrochloric acid is a strong acid, hence it dissociates completely and no
energy is taken in to dissociate the acid.
[Total: 20]

www.gradtutors.com
653
20
 

www.gradtutors.com
654

SERANGOON JUNIOR COLLEGE


General Certificate of Education Advanced Level
Higher 1

Candidate Name

Class

CHEMISTRY 8872/01
Preliminary Examination 30 August 2013
Paper 1 Multiple Choice 1 hour

Additional Materials: Data Booklet


Optical Mark Sheet (OMS)

READ THESE INSTRUCTIONS FIRST

On the separate multiple choice OMS given, write your name, FIN/NRIC and class in the
spaces provided.
Shade correctly your class and FIN/NRIC number.
Eg. If your NRIC is S9306660Z, shade S9306660Z for the item “index number”.

There are thirty questions in this paper. Answer all questions.


For each question there are four possible answers A, B, C and D.
Choose the one you consider correct and record your choice using a soft pencil on the
separate OMS.
Each correct answer will score one mark.
A mark will not be deducted for a wrong answer.

You are advised to fill in the OMS as you go along; no additional time will be given for the
transfer of answers once the examination has ended.

Any rough working should be done in this question paper.

 
 

This document consists of 14 printed pages and 0 blank page


 
www.gradtutors.com
655

Section A
Answer all questions

1 The percentage by mass of water in a hydrated manganese(II) chloride salt is 36.4%.


What is the empirical formula of the hydrated salt?

A MnCl2.2H2O

B MnCl2.3H2O

C MnCl2.4H2O

D MnCl2.5H2O

2 In which one of the following reactions is there a reduction in the oxidation state of the
underlined element?

A NH3 + HCl  NH4Cl

B 2V3+ + H2O2  2VO2+ + 2H+

C H2O2 + 2I  + 2H+  2H2O + I2

D CaCO3  CaO + CO2

3 Which of the following particles has more electrons than protons, and more protons
than neutrons? [H = 11 H , D = 12 H , He = 24 He , O = 168 O ]

A He+

B D

C OD

D OH
 


 
www.gradtutors.com
656

4 Under the same conditions, which of the following ions would be deflected in a mass
spectrometer to the same extent as 126 C 2+ ?
6
(i) 3 Li +
12
(ii) 6 C+
12
(iii) 7 N 2+
13
(iv) 6 C 2+

A (i), (ii), (iii) and (iv)

B (i) and (iii) only

C (ii) only

D None of the above.

5 Which of the following statements best explains why the boiling point of
CH3CH2CH2NH2 is higher than that of (CH3)3N?

A CH3CH2CH2NH2 has a higher relative molecular mass than (CH3)3N.

B The CH3CH2CH2NH2 molecule has a larger surface area than the (CH3)3N
molecule.

C CH3CH2CH2NH2 is polar but (CH3)3N is not.

D There are hydrogen bonds between CH3CH2CH2NH2 molecules but not between
(CH3)3N molecules.

6 Although F–F bond is the shortest among the halogens, it has the weakest bond
strength. What could be the possible reason for the unusual phenomenon?

A Fluorine has the highest electronegativity in Group VII.


B Fluorine has the smallest atomic radius in Group VII.
C Due to lone pair-lone pair repulsion between the 2 fluorine atoms.
D Due to the non-polar nature of the F–F bond.


 
www.gradtutors.com
657

7 Orange dichromate, Cr2O72- and yellow chromate, CrO42- exist in equilibrium in aqueous
solution.

Cr2O72- + H2O 2CrO42- + 2H+

Which statement about this equilibrium is correct?

A Cr2O72- is reduced to CrO42-.


B The solution appears to be yellow in NaOH (aq).
C Kc value increases with the addition of NaOH (aq).
D Lowering of pH will increase the concentration of CrO42-.

8 Each of the following equilibria is subjected to two changes which are carried out
separately:
(i) The pressure is reduced at constant temperature;
(ii) The temperature is increased at constant pressure.

For which equilibrium will both of these changes result in an increase in the proportion
of products?

A H2 (g) + I2 (g) 2HI (g); ∆H = +53 kJ mol-1

B 4NH3 (g) + 5O2 (g) 4NO (g) + 6H2O (g); ∆H = -950 kJ mol-1

C N2 (g) + 3H2 (g) 2NH3 (g); ∆H = -92 kJ mol-1

D N2O4 (g) 2NO (g); ∆H = +57 kJ mol-1


 

9 Nitrogen dioxide decomposes on heating according to the following equation

2NO2 (g) 2NO (g) + O2 (g)

When 4 moles of nitrogen dioxide was put into a 1 dm3 container and heated, the
equilibrium mixture contained 0.8 moles of oxygen. What is the numerical value of the
equilibrium constant, Kc, at the temperature of the experiment?

1.6  0.8
A
2.4 2
1.6 2  0.8
B
2.4 2
0.8  0.8
C
2.4
0.8 2  0.8
D
42
 


 
www.gradtutors.com
658

10 Values for the ionic product of water, Kw, at two different temperatures are given below.

Temperature / oC Kw / mol2 dm-6


25 1.00 x 10-14
30 1.44 x 10-14

Which is correct for pure water at 30 oC?

A pH < 7

B [H+] < [OH-]

C [H+] = 1.44 x 10-7 mol dm-3

D Dissociation of water is an exothermic process


 

11 Lead is the final product formed by a series of changes in which the rate-determining
stage is the radioactive decay of uranium-238. This decay is a first-order reaction with
half-life of 4.5 x 109 years.

What would be the age of a rock sample originally lead-free, in which molar ratio of
uranium to lead is now 0.143?

A 1.5 x 109 years 


B 4.5 x 109 years
C 12.6 x 109 years
D 13.5 x 109years


 
www.gradtutors.com
659

12 A reaction involving reactant R is found out to be first order. Which of the following graphs
best describes the decay of reactant R?

A B

Rate Rate
of of
decay decay

time [R]

C D
[R] [R]

time time

 
13 The magnitude of enthalpy change of neutralisation of aqueous sodium hydroxide by
hydrochloric acid is 57.2kJ mol-1, but the magnitude of enthalpy change of neutralisation
of aqueous sodium hydroxide by aqueous ethanoic acid is 55.2 kJ mol-1.

Which statement best explains the numerical difference in these values?

A The process CH3CO2H (aq) → CH3CO2─ (aq) + H+ (aq) is exothermic

B The process CH3CO2H (aq) → CH3CO2─ (aq) + H+ (aq) is endothermic

C Aqueous ethanoic acid is a weak acid and has fewer hydrogen ions than
hydrochloric acid of the same concentration.

D A smaller volume of aqueous hydrochloric acid is required for the neutralisation


compared to that of ethanoic acid of the same concentration.


 
www.gradtutors.com
660

14 SiCl4 is known to hydrolyse readily in water but CCl4 is inert to water. Which statement
best explains this observation?

A CCl4 has a giant molecular structure, large amount of energy is required to


overcome the strong covalent bond to allow solvation.

B There are vacant orbitals to accommodate additional electrons in SiCl4 but not in
CCl4.

C CCl4 has weaker van der Waals’ forces of attraction which cannot displace the
stronger hydrogen bonding in water.

D Si4+ has high charge density to polarise the large chlorine electron cloud.

 
15 Use of the Data Booklet is relevant to this question.

The graph below shows the variation in the melting points for eight consecutive
elements in the Periodic Table, all with atomic number below 20.

What statement is correct?

A Element F conducts electricity at room temperature.


B Element D burns with a brilliant yellow flame.
C Element H does not react with air at room temperature.
D Element C is a gas which is chemically inert at room temperature.
 


 
www.gradtutors.com
661

16 The graph below shows the first thirteen ionisation energies for element J.

ionisation
energy

number of electrons removed

What can be deduced from the graph about element J?

A It is a d-block element.

B It is in Group IV of the Periodic Table.

C It has one electron in its outermost shell.

D It is in the third period (Na to Ar) of the Periodic Table.

 
17 How many different alkenes, including geometrical isomers, could be produced when
  
Br H
 
  C C CH3
  H H
 
CH3
                                             reacts with hot ethanolic NaOH?

A 2 B 3 C 4 D 5
 

18 2,2-dimethylbutane undergoes free radical substitution with chlorine under ultraviolet


light. How many different mono-substituted chlorinated products are possible?

A 2 B 3 C 4 D 5
 


 
www.gradtutors.com
662

19 Chlorofluorocarbons (CFCs) have been widely used in aerosol sprays, refrigerators and
in making foamed plastics, but are now known to destroy ozone in the upper
atmosphere.

Which of the following will not destroy ozone, and therefore can be used as a safer
replacement for CFCs?

A CH2FCH2CH2CH3
B CCl3CBr3
C CHBr3
D CHClFCClF2
 
 
20 The following is a method of synthesising tartaric acid, a compound found in wine.
X
H2C CH2 HOCH2CH2OH O C C O
H H

COOH COOH CN CN
Z
HO C C OH HO C C OH

H H H H
         

Which set of reagents and conditions can be used for the synthesis?

Stage X Stage Y Stage Z


A H2O (g), H3PO4 (aq) Cold HCN, NaOH (aq) Hot K2Cr2O7, H2SO4 (aq)

B Cold KMnO4, H2SO4 (aq) Ethanolic KCN, heat HCl (aq), heat

C H2O (g), H3PO4 (aq) Ethanolic KCN, heat Hot K2Cr2O7, H2SO4 (aq)

D Cold KMnO4, NaOH (aq) Cold HCN, NaOH (aq) HCl (aq), heat


 
www.gradtutors.com
663

21 Data about the reactions of compound K are given in the table.

Reaction Reagent Result


1 Hot acidified potassium Orange solution turns green
dichromate, followed by 2,4- and orange precipitate seen
DNPH after addition of 2,4-DNPH.
2 Warm alkaline aqueous Iodine Yellow precipitate observed.
3 Aqueous sodium carbonate Effervescence seen.
4 AgNO3 (aq), warm Cream solid formed.

Which of the following compounds is K?

Br
O

A
O

HO

OH

Br
B

OH

OH
Br
C

O OH

Br OH

O OH
 
 

   

10 
 
www.gradtutors.com
664

22 Which of the following reactions will not produce 1,3-dibenzoic acid?

CN

A + aqueous H2SO4, reflux

COOCH3

CH(OH)CH3

B
+ akaline aqueous iodine followed by acidification

COCH3

CHO

C
+ Tollens' reagent followed by acidification
CH2CHO

C CH2CH3
D
+ hot acidified KMnO4

COOCH3
 

23 Citric acid can be found in lemon juice and orange juice and it causes the sharp unique
taste.

CH2 COOH

HO C COOH

CH2 COOH
                                                 

Which of the following reacts completely with 1 mole of citric acid?

A 3 moles of PCl5

B 3 moles of NaHCO3

C 4 moles of H2SO4

D 4 moles of KOH

11 
 
www.gradtutors.com
665

24 Arrange the following molecules in terms of increasing pKa values.

Compound L CH3CH2CH2COOH

Compound M CH2(Br)CH2CH2COOH

Compound N CH3CH2CH(Br)COOH

Compound P CH3CH2CH2CH2COOH

A M<N<L<P
B M<N<P<L
C N<M<L<P
D N<P<L<M
 

25 An ester, C4H8O2, undergoes the following reactions:

 Reacts with hot sulfuric acid to give carboxylic acid Q and alcohol R.
 The product from the oxidation of alcohol R gives a brick red precipitate with hot
alkaline copper (II) complex.
 Carboxylic acid Q reacts with lithium aluminium hydride in dry ether to give R.

Which of the following could be the structure of the ester?

A HCOOCH(CH3)2
B CH3COOCH2CH3
C HCOOCH2CH2CH3
D CH3OCOCH2CH3
   

12 
 
www.gradtutors.com
666

Section B

For questions 26-30, one or more of the numbered statements 1 to 3 may be correct.
Decide whether each of the statements is or is not correct. The responses A to D should be
selected on the basis of:

A B C D

1, 2 and 3 1 and 2 only 2 and 3 only 1 only


are correct are correct are correct is correct

No other combination of statements is to be used as a correct response.

26 Which one of the following statements is true?

1 AlCl3 is covalent while AlF3 is ionic.

2 The melting point of Na is lower than that of Mg.

3 An aqueous solution of HBr conducts electricity because HBr is ionic.

27 Which of the following mixtures will result in an acidic buffer solution?

1 25 cm3 of CH3COO─Na+ and 25 cm3 of CH3COOH of same concentration

2 25 cm3 of CH3COO─Na+ and 12.5 cm3 of HCl of same concentration

3 25 cm3 of NaOH and 12.5 cm3 of CH3COOH of same concentration

28 Why is a solution of aluminium chloride more acidic than magnesium chloride?

1 The O-H bonds are weaker in [Al(H2O)6]3+ than in H2O.

2 Chloride ions react with water to form hydrochloric acid.

3 Aluminium atom has high charge density.

29 Which of the following statements are true about the chemical reactions of
cyclohexane, cyclohexene and methylbenzene?

1 One mole each of cyclohexene and methylbenzene will react with one mole and
three moles of hydrogen gas respectively when heated in the presence of a
catalyst under appropriate condition.

2 All three compounds decolourise bromine gas under uv light.

3 All three compounds decolourise hot acidified managate (VII).

13 
 
www.gradtutors.com
667

30 Which starting materials will give sodium benzoate on heating with aqueous sodium
hydroxide under reflux?

1 phenyl ethanoate

2 phenyl benzoate

3 ethylbenzene and sodium manganate (VII)


 

END

14 
 
www.gradtutors.com
668

SERANGOON JUNIOR COLLEGE


General Certificate of Education Advanced Level
Higher 1

Candidate Name

Class

CHEMISTRY 8872/01
Preliminary Examination 30 August 2013
Paper 1 Multiple Choice 1 hour

Additional Materials: Data Booklet


Optical Mark Sheet (OMS)

READ THESE INSTRUCTIONS FIRST

On the separate multiple choice OMS given, write your name, FIN/NRIC and class in the
spaces provided.
Shade correctly your class and FIN/NRIC number.
Eg. If your NRIC is S9306660Z, shade S9306660Z for the item “index number”.

There are forty questions in this paper. Answer all questions.


For each question there are four possible answers A, B, C and D.
Choose the one you consider correct and record your choice using a soft pencil on the
separate OMS.
Each correct answer will score one mark.
A mark will not be deducted for a wrong answer.

You are advised to fill in the OMS as you go along; no additional time will be given for the
transfer of answers once the examination has ended.

Any rough working should be done in this question paper.

 
 

This document consists of 20 printed pages and 0 blank page

www.gradtutors.com
669

Section A
Answer all questions

1 The percentage by mass of water in a hydrated manganese(II) chloride salt is


36.4%. What is the empirical formula of the hydrated salt?

A
MnCl2.2H2O
B
MnCl2.3H2O
C
MnCl2.4H2O
D
MnCl2.5H2O
Answer: C
Let the empirical formula of the hydrated salt be MnCl2.xH2O
Hence, % by mass of water in MnCl2.xH2O = (18.0x)(100)/(54.9 + 71 + 18.0x) = 36.4%
Solving x, x = 4

2 In which one of the following reactions is there a reduction in the oxidation state of the
underlined element?

A NH3 + HCl  NH4Cl

B 2V3+ + H2O2  2VO2+ + 2H+

C H2O2 + 2 I  + 2H+  2H2O + I2

D CaCO3  CaO + CO2


Answer: C

A NH3 + HCl  NH4Cl (remains at –3)


B 2V3+ + H2O2  2VO2+ + 2H+ (increases from +3 to +4)
C H2O2 + 2 I  + 2H+  2H2O + I2 (reduces from –1 to –2)
D CaCO3  CaO + CO2 (remains at – 4)

www.gradtutors.com
670

3 Which of the following particles has more electrons than protons, and more protons
than neutrons? [H = 11 H , D = 12 H , He = 24 He , O = 168 O ]

A He+

B D

C OD

D OH
Answer: D

A He+ (4 protons, 3 electrons, 2 neutrons)


B D (1 protons, 2 electrons, 1 neutrons)
C OD (8 + 1protons, 8 + 1 + 1 electrons, 8 + 1neutrons)
D OH (8 + 1 protons, 8 + 1 + 1 electrons, 8 neutrons)
 

4 Under the same conditions, which of the following ions would be deflected in a mass
spectrometer to the same extent as 126 C 2+ ?
(i) 6
3 Li +
(ii) 12
6 C+
12
(iii) 7 N 2+
(iv) 13
6 C 2+

(i), (ii), (iii) and (iv)


A
(i) and (iii) only
B
(ii) only
C
None of the above.
D
Answer: B

Angle of deflection α e/m


e/m ( 126 C 2+) = 2/12 = 1/6
e/m ( 36 Li +) = 1/6
e/m ( 126 C +) = 1/12
12
e/m ( 7 N 2+) = 2/12 =1/6
e/m ( 136C 2+) = 2/13

www.gradtutors.com
671

5 Which of the following statements best explains why the boiling point of
CH3CH2CH2NH2 is higher than that of (CH3)3N?

CH3CH2CH2NH2 has a higher relative molecular mass than (CH3)3N.


A
The CH3CH2CH2NH2 molecule has a larger surface area than the (CH3)3N
B molecule.

CH3CH2CH2NH2 is polar but (CH3)3N is not.


C
There are hydrogen bonds between CH3CH2CH2NH2 molecules but not between
D
(CH3)3N molecules.
Answer: D

6 Although F–F bond is the shortest among the halogens, it has the weakest bond
strength. What could be the possible reason for the unusual phenomenon?
A
Fluorine has the highest electronegativity in Group VII.
B
Fluorine has the smallest atomic radius in Group VII.
C
Due to lone pair-lone pair repulsion between the 2 fluorine atoms.
D
Due to the non-polar nature of the F–F bond.
Answer: C

7 Orange dichromate, Cr2O72- and yellow chromate, CrO42- exist in equilibrium in aqueous
solution.

Cr2O72- + H2O 2CrO42- + 2H+

Which statement about this equilibrium is correct?

A Cr2O72- is reduced to CrO42-.

B The solution appears to be yellow in NaOH (aq).

C Kc value increases with the addition of NaOH (aq).

D Lowering of pH will increase the concentration of CrO42-.

Answer: B
A: Both contains Cr in O.S. of +6
B: In NaOH, [H+] decreases, by LCP, position of equilibrium shift to the right resulting in
an increase in [CrO42-] causing the solution to appear yellow.
C: Value of Kc is temperature dependent only.
D: Lowering pH results in increase [H+]m by LCP, position of equilibrium shift left,
decreasing [CrO42-].

www.gradtutors.com
672

8 Each of the following equilibria is subjected to two changes which are carried out
separately:
(i) The pressure is reduced at constant temperature;
(ii) The temperature is increased at constant pressure.

For which equilibrium will both of these changes result in an increase in the proportion
of products?

A H2 (g) + I2 (g) 2HI (g); ∆H = +53 kJ mol-1

B 4NH3 (g) + 5O2 (g) 4NO (g) + 6H2O (g); ∆H = -950 kJ mol-1

C N2 (g) + 3H2 (g) 2NH3 (g); ∆H = -92 kJ mol-1

D N2O4 (g) 2NO (g); ∆H = +57 kJ mol-1

Answer: D (for both (i) and (ii) position of equilibrium shift right)

Option Reduce P on P.O.E. Increase T on P.O.E.


A No change Right
B Right Left
C Left Left
D Right Right

9 Nitrogen dioxide decomposes on heating according to the following equation

2NO2(g) 2NO(g) + O2(g)

When 4 moles of nitrogen dioxide was put into a 1 dm3 container and heated, the
equilibrium mixture contained 0.8 moles of oxygen. What is the numerical value of the
equilibrium constant, Kc, at the temperature of the experiment?

A 1.6  0.8
2.4 2

B 1.6 2  0.8
2.4 2

C 0.8  0.8
2.4

D 0 .8 2  0 .8
42

Answer: B

www.gradtutors.com
673

10 Values for the ionic product of water, Kw, at two different temperatures are given below.

temperature / oC Kw / mol2 dm-6


25 1.00 x 10-14
30 1.44 x 10-14

Which is correct for pure water at 30 oC?

A pH < 7

B [H+] < [OH-]

C [H+] = 1.44 x 10-7 mol dm-3

D dissociation of water is an exothermic process

Answer: A
A: [H+] = 1.44  1014 = 1.2 x 10-7 mol dm-3 ; pH = -lg (1.2 x 10-7) = 6.92
B: [H+] = [OH-]
C: [H+] = 1.44  1014 = 1.2 x 10-7 mol dm-3
D: Kw value increases with increased in temperature, hence dissociation of water is an
endothermic process.

11 Lead is the final product formed by a series of changes in which the rate-determining
stage is the radioactive decay of uranium-238. This decay is a first-order reaction with
half-life of 4.5 x 109 years.

What would be the age of a rock sample originally lead-free, in which molar ratio of
uranium to lead is now 0.143?

A 1.5 x 109 years 


B 4.5 x 109 years
C 12.6 x 109 years
D 13.5 x 109years
Answer D

100% U  50% U  25% U  12.5% U  6.25% U

0% Pb  50% Pb  75% Pb  87.5% Pb  93.75%

When 12.5% U : 87.5% Pb  molar ratio is 0.143.

It has went through 3 half lives


3 x 4.5 x 109 = 1.35 x 1010

www.gradtutors.com
674

12 A reaction involving reactant R is found out to be first order. Which of the following graphs
best describes the decay of reactant R?
A B

Rate Rate
of of
decay decay

time [R]

C D
[R] [R]

time time

Answer: B
A is completely wrong.
C is more for zero order.
B and D are close match.
B is a better graph as it shows the linear relationship of Rate = k[R]
D, however, can represent a first order or second order graph. If B is not present then D will
make a better choice, which is not a case for this question.

 
13 The magnitude of enthalpy change of neutralisation of aqueous sodium hydroxide by
hydrochloric acid is 57.2 kJ mol-1, but the magnitude of enthalpy change of
neutralisation of aqueous sodium hydroxide by aqueous ethanoic acid is 55.2 kJ mol-1.

Which statement best explains the numerical difference in these values?

A The process CH3CO2H (aq) → CH3CO2- (aq) + H+ (aq) is exothermic


B The process CH3CO2H (aq) → CH3CO2- (aq) + H+ (aq) is endothermic
C Aqueous ethanoic acid is a weak acid and has fewer hydrogen ions than
hydrochloric acid of the same concentration.
D A smaller volume of aqueous hydrochloric acid is required for the neutralisation
compared to that of ethanoic acid of the same concentration.
Answer: B
The process CH3CO2H (aq) → CH3CO2- (aq) + H+ (aq) is endothermic means energy need ti
be absorbed to cause the dissociation of the H+ from the acid resulting in a smaller magnitude
for enthalpy change of neutralisation.

C is not a wrong statement but it does not fully explained the lower magnitude of enthalpy
change of neutralisation

D does not take into account energetics, it is related to volumetric analysis.

www.gradtutors.com
675

14 SiCl4 is known to hydrolyse readily in water but CCl4 is inert to water. Which statement
best explains this observation?

A CCl4 has a giant molecular structure, large amount of energy is required to


overcome the strong covalent bond to allow solvation.

B There are vacant orbitals to accommodate additional electrons in SiCl4 but not in
CCl4.

C CCl4 has weaker van der Waals’ forces of attraction which cannot displace the
stronger hydrogen bonding in water.

D Si4+ has high charge density to polarise the large chlorine electron cloud.

Answer: B

A is wrong as CCl4 is not a giant molecular structure.

C is wrong as this statement explain physical property of hydration rather than relating to the
inertness of CCl4

D is wrong as SiCl4 is not ionic and thus concept of high density is not applicable.

 
15 Use of the Data Booklet is relevant to this question.

The graph below shows the variation in the melting points for eight consecutive
elements in the Periodic Table, all with atomic number below 20.

What statement is correct?

A Element F conducts electricity at room temperature.

www.gradtutors.com
676

B Element D burns with a brilliant yellow flame.


C Element H does not react with air at room temperature.
D Element C is a gas which is chemically inert at room temperature.
Ans: C

A is F, B is Ne, C is Na, D is Mg, E is Al, F is Si, G is P, H is S

S burns in oxygen to form SO2.

16 The graph below shows the first thirteen ionisation energies for element J.

ionisation
energy

number of electrons removed

What can be deduced from the graph about element J?

A It is a d-block element.

B It is in Group IV of the Periodic Table.

C It has one electron in its outermost shell.

D It is in the third period (Na to Ar) of the Periodic Table.

Answer: B

www.gradtutors.com
677

17 How many different alkenes, including geometrical isomers, could be produced when
Br H
C C CH3
H H

CH3
 reacts with hot ethanolic NaOH?

A 2 B 3 C 4 D 5

Answer: C

H CH3 H H
C C C C
H
CH3

Br H CH3 CH3
C C CH3
H H trans isomer cis isomer

CH3

CH2CH3 H
C C
H CH2CH3
CH3 CH3

trans isomer cis isomer

www.gradtutors.com
678

18 2,2-dimethylbutane undergoes free radical substitution with chlorine under ultraviolet


light. How many different mono-substituted chlorinated products are possible?

A 2 B 3 C 4 D 5

Answer: B

H H CH3
H C C C CH2Cl
H H CH3 H H CH3
H C C C CH3
H H CH3 H H CH3
H C C C CH3
H Cl CH3

H H CH3
Cl C C C CH3
H H CH3

19 Chlorofluorocarbons (CFCs) have been widely used in aerosol sprays, refrigerators and
in making foamed plastics, but are now known to destroy ozone in the upper
atmosphere.

Which of the following will not destroy ozone, and therefore can be used as a safer
replacement for CFCs?

A CH2FCH2CH2CH3

B
CCl3CBr3
C
CHBr3
D
CHClFCClF2
Answer: A
C-F bond is strong and it is difficult to break the C-F bond to form radicals.

www.gradtutors.com
679

20 The following is a method of synthesising tartaric acid, a compound found in wine.


X
H2C CH2 HOCH2CH2OH O C C O
H H

COOH COOH CN CN
Z
HO C C OH HO C C OH

H H H H
 
Which set of reagents and conditions can be used for the synthesis?

Stage X Stage Y Stage Z


A H2O (g), H3PO4 (aq) Cold HCN, NaOH (aq) Hot K2Cr2O7, H2SO4 (aq)

B Cold KMnO4, H2SO4 (aq) Ethanolic KCN, heat HCl (aq), heat

C H2O (g), H3PO4 (aq) Ethanolic KCN, heat Hot K2Cr2O7, H2SO4 (aq)

D Cold KMnO4, NaOH (aq) Cold HCN, NaOH (aq) HCl (aq), heat

Answer: D
 

21 Data about the reactions of compound K are given in the table.

Reaction Reagent Result


1 Hot acidified potassium Orange solution turns green
dichromate, followed by 2,4- and orange precipitate seen
DNPH after addition of 2,4-DNPH.
2 Warm alkaline aqueous Iodine Yellow precipitate observed.
3 Aqueous sodium carbonate Effervescence seen.
4 AgNO3 (aq), warm Cream solid formed.

Which of the following compounds is K?

Br
O

A
O

HO

www.gradtutors.com
680

OH

Br
B

OH

OH
Br
C

O OH

Br OH

O OH
 
Answer: C

Option A is incorrect as the compound will not reduce orange potassium dichromate to
green.
Option B is incorrect as there is a lack of a carboxylic acid functional group to react with
aqueous sodium carbonate.
Option D is incorrect as Br attached directly to benzene ould have no reaction with
AgNO3.
Option C is correct as
OH  Able to reduce orange
 Produce cream ppt of CH2Br dichromate to green Cr3+.
AgBr with silver Ketone would be formed and
nitrate. thus orange ppt with 2,4-
DNPH
 Has the special structure to
                                             
COOH
produce yellow ppt with
aqueous alkaline iodine.

 Able to produce CO2 when


reacted with aq sodium
carbonate.

www.gradtutors.com
681

22 Which of the following reactions will not produce 1,3-dibenzoic acid?

CN
+ aqueous H2SO4, reflux
A

COOCH3

CH(OH)CH3
+
B akaline aqueous iodine followed by acidification

COCH3

CHO
+ Tollens' reagent followed by acidification
C

CH2CHO

C CH2CH3
D
+ hot acidified KMnO4

COOCH3
Answer: C

Tollens’ reagent can react with both aliphatic and aromatic aldehyde. But compound in
choice C will give the following product:
COOH

CH2COOH
 

www.gradtutors.com
682

23 Citric acid can be found in lemon juice and orange juice and it causes the sharp unique
taste.

CH2 COOH

HO C COOH

CH2 COOH

Which of the following reacts completely with 1 mole of citric acid?

A 3 moles of PCl5

B 3 moles of NaHCO3

C 4 moles of H2SO4

D 4 moles of KOH

Answer : B

24 Arrange the following molecules in terms of increasing pKa values.

Compound L CH3CH2CH2COOH

Compound M CH2(Br)CH2CH2COOH

Compound N CH3CH2CH(Br)COOH

Compound P CH3CH2CH2CH2COOH

A M<N<L<P

B
M<N<P<L
C
N<M<L<P
D
N<P<L<M
Answer: C

Increasing pKa  Decreasing acidity

Factors that decrease the acidity:

 Electron donating group will intensify the negative charge on carboxylate ion,
destabilising it, thus lowering acidity
 The closer the electron donating to the COO-, the more destabilised the
carboxylate ion is, thus lowering acidity

www.gradtutors.com
683

Compound C is the least acidic as it has the electron donating group closer to the COO-
as compared to the 5 carbons arranged in a linear fashion.

25 An ester, C4H8O2, undergoes the following reactions:

 Reacts with hot sulfuric acid to give carboxylic acid Q and alcohol R.
 The product from the oxidation of alcohol R gives a brick red precipitate with hot
alkaline copper (II) complex.
 Carboxylic acid Q reacts with lithium aluminium hydride in dry ether to give R.

Which of the following could be the structure of the ester?

HCOOCH(CH3)2
A

B
CH3COOCH2CH3
C
HCOOCH2CH2CH3
D
CH3OCOCH2CH3
Answer: B

Ester in option B is made up of CH3COOH and CH3CH2OH.


CH3COOH can be reduced to CH3CH2OH.

The alcohol CH3CH2OH can be oxidised to form aldehyde (CH3CHO) which undergoes
oxidation with Fehling’s reagent to give brick-red ppt.

   

www.gradtutors.com
684

Section B

For questions 26-30, one or more of the numbered statements 1 to 3 may be correct.
Decide whether each of the statements is or is not correct. The responses A to D should be
selected on the basis of:

A B C D

1, 2 and 3 1 and 2 only 2 and 3 only 1 only


are correct are correct are correct is correct

No other combination of statements is to be used as a correct response.

26 Which one of the following statements is true?

1 AlCl3 is covalent while AlF3 is ionic.

2 The melting point of Na is lower than that of Mg.

3 An aqueous solution of HBr conducts electricity because HBr is ionic.


Answer: B

HBr (aq) conducts electricity because it dissociates in water to form H+ and Br- ions that
will conduct electricity. But HBr is a covalent compound.
 

27 Which of the following mixture will result in an acidic buffer solution?

1 25 cm3 of CH3COO-Na+ and 25 cm3 of CH3COOH of same concentration

2 25 cm3 of CH3COO-Na+ and 12.5 cm3 of HCl of same concentration

3 25 cm3 of NaOH and 12.5 cm3 of CH3COOH of same concentration

Answer: B (1 and 2)
Acidic buffer contains – weak acid and salt of conjugate base
1: CH3COOH (weak acid) and CH3COO- (conjugate base)
2: CH3COO- + HCl  CH3COOH + Cl-, since CH3COO- is in excess, the solution will
contain both CH3COO- and CH3COOH
3: NaOH is in excess, hence, the solution contains NaOH and CH3COO-

www.gradtutors.com
685

28 Why is a solution of aluminium chloride more acidic than magnesium chloride?

1 The O-H bonds are weaker in [Al(H2O)6]3+ than in H2O.

2 Chloride ions react with water to form hydrochloric acid.

3 Aluminium atom has high charge density.

Answer: D (1 only)

Statement 1 is correct: The O-H bond is weaken due to the high charge density of
Al3+

Statement 2 is wrong; the chloride ions do not react with water at all.

Statement 3 is wrong; it should be aluminium ion has high charge density.

29 Which of the following statements are true about the chemical reactions of
cyclohexane, cyclohexene and methylbenzene?

1 One mole each of cyclohexene and methylbenzene will react with one mole and
three moles of hydrogen gas respectively when heated in the presence of a
catalyst under appropriate condition.

2 All three compounds decolourise bromine gas under uv light.

3 All three compounds decolourise hot acidified managate (VII).

Answer: B (1 and 2)

30 Which starting materials will give sodium benzoate on heating with aqueous sodium
hydroxide under reflux?

1 phenyl ethanoate

2 phenyl benzoate

3 ethylbenzene and sodium manganate (VII)

Answer: C (2 and 3)

O
O C CH3
O-Na+ + CH3COO-Na+
 
 

www.gradtutors.com
686

O
O C O-Na+ + COO-Na+

[O] neutralisation
CH2CH3 COOH COO-Na+

END

www.gradtutors.com
687
1

SERANGOON JUNIOR COLLEGE


General Certificate of Education Advanced Level
Higher 1

CANDIDATE
NAME

CLASS INDEX NUMBER

CHEMISTRY 8872/02
Preliminary Examination 22 August 2013
Paper 2 2 hours

Additional Materials: Data Booklet, Cover Page for Section B


Writing Papers, Graph Paper

READ THESE INSTRUCTIONS FIRST

Write your name and class on all the work you hand in.
Write in dark blue or black pen on both sides of the paper.
You may use a soft pencil for any diagrams, graphs or rough work.

SECTION A:
Answer all questions in the space provided in the booklet.

SECTION B:
Answer any two questions on separate answer papers.

At the end of the examination, fasten all your work securely together.
The number of marks is given in brackets [ ] at the end of each question or part question.

For Examiner’s Use


MCQ
P1
Section 1 2 3 Total
A

B4

B5

B6

Total
/80

This document consist of 14 printed pages and 2 blank pages

SRJC 8872/02/Prelim/2013 [Turn Over


www.gradtutors.com
688
2

Section A
Answer all the questions in this section in the spaces provided.
For
1 (a) Arsenic acid, H3AsO4, is a colourless acid that is used as a wood preservative and as Examiner’s
a reagent in the synthesis of dyestuff. It is slowly formed when elemental arsenic, As, use

is treated with ozone in the presence of moisture. Oxygen gas is known to be a side
product for the reaction.

(i) Write a balanced chemical equation for the above mentioned treatment process.

.............................................................................................................................

(ii) Arsenic acid can oxidise iodide ions to iodine and itself is reduced to H3AsO3.
Write a half-equation for the reduction of arsenic acid to H3AsO3.
Hence write an overall equation for the reaction between iodide ions and arsenic
acid.

............................................................................................................................

............................................................................................................................

............................................................................................................................

(iii) The iodine, I2, that is liberated from the reaction between arsenic acid and iodide
ions can be estimated by titration against a standard thiosulfate, S2O32-, solution.

I2 + 2S2O32-  2I─ + S4O62-

When a sample of 25.0 cm3 of arsenic acid reacts with iodide ions, it was found
that 23.30 cm3 of 0.200 mol dm-3 solution of thiosulfate ions was required to
react completely with the iodine liberated.

Calculate the concentration of the sample of arsenic acid.

[5]
SRJC 8872/02/Prelim/2013 [Turn Over
www.gradtutors.com
689
3

For
(c) The following graph shows some data on consecutive elements from period 2 and Examiner’s
period 3. use

lg (2nd I.E.)
3.7

3.6

3.5

3.4

3.3

3.2

3.1

2.9
A B C D E F G

Elements

F is known to react with C to form an ionic compound.


G is a better conductor of electricity as compared to F.
The chloride of F gives a slightly acidic solution when dissolved in water.

(i) From the information given, identify F.


Hence, describe the type of bonding in F either in words or with a labelled
diagram.

(ii) Suggest if the compound formed from F and C is soluble in water. You must
support your answer with relevant explanations to gain full credit of this question.

............................................................................................................................

............................................................................................................................

............................................................................................................................
[4]
Total: [9]
   

SRJC 8872/02/Prelim/2013 [Turn Over


www.gradtutors.com
690
4

2 (a) For the reaction, For


Examiner’s
use
CH3CH2CO2H (l) + CH3OH (l) CH3CH2CO2CH3 (l) + H2O (l)

The value of the equilibrium constant is 5.0.

(i) Suggest one way to increase the yield of the above reaction.

............................................................................................................................

............................................................................................................................

(ii) Explain, in terms of chemical bonding and structure, why the product,
CH3CH2CO2CH3 has low solubility in water.

............................................................................................................................

............................................................................................................................

............................................................................................................................

............................................................................................................................

(iii) State the numerical value of the equilibrium constant for the reverse reaction.

............................................................................................................................

(iv) Suggest the reagent and condition for the reverse reaction.

............................................................................................................................

(v) In an experiment, 1 mol of CH3CH2CO2CH3 and 1 mol of water are mixed in


V cm3. Calculate the amount of acid present at equilibrium.

SRJC 8872/02/Prelim/2013 [Turn Over


www.gradtutors.com
691
5

(vi) Suggest a chemical test to distinguish between CH3CH2CO2CH3 and For


Examiner’s
CH3CH2CO2CH2CH3. use

............................................................................................................................

............................................................................................................................

............................................................................................................................

............................................................................................................................

............................................................................................................................
Total: [8]

SRJC 8872/02/Prelim/2013 [Turn Over


www.gradtutors.com
692
6

3 Glycolic acid (pKa = 3.83), C2H4O3, is a colourless and odourless crystalline solid that For
Examiner’s
is highly soluble in water. It is the smallest α–hydroxy acid (AHA) and is commonly use
used in skin care products and most often as a chemical peel performed by plastic
surgeons due to its ability to penetrate the skin effectively.

It can be prepared by the reaction of chloroethanoic acid (pKa = 2.85) with hot sodium
hydroxide followed by re-acidification.
(a) (i) Write a chemical equation for the net reaction that has occurred showing the
displayed formulae of all organic compounds.

(ii) Explain why chloroethanoic acid has a lower pKa as compared to glycolic acid.

..............................................................................................................................

..............................................................................................................................

..............................................................................................................................

..............................................................................................................................

..............................................................................................................................

..............................................................................................................................

(iii) Chloroethanoic acid is defined as a weak acid.


With the aid of an equation, explain the words in italics.

..............................................................................................................................

..............................................................................................................................

..............................................................................................................................

SRJC 8872/02/Prelim/2013 [Turn Over


www.gradtutors.com
693
7

(iv) Besides pKa, percentage dissociation is another way to measure the strength For
Examiner’s
of an acid. Percentage dissociation is the ratio of the concentration of hydrogen use
ions to the concentration of the acid expressed in percentage.

An analysis was carried out on a 25 cm3 sample of a skin care product


containing glycolic acid by titrating with 0.125 mol dm-3 of sodium hydroxide.
The graph obtained is shown below. With this graph, determine the percentage
dissociation of glycolic acid.

pH
12.4

8.19

3.83

3.00

0 12.5 25.0 37.5


Volume of NaOH / cm3

(v) Prove that the final pH is 12.4 after 37.5 cm3 of sodium hydroxide is added.

SRJC 8872/02/Prelim/2013 [Turn Over


www.gradtutors.com
694
8

(vi) On the graph in (a)(iv), indicate the region where there is a buffer solution. For
Examiner’s
State the species involved in this buffer system. use

(vii) Using an equation only, suggest how the pH remains fairly constant when
small amount of sodium hydroxide is added to the solution in (a)(vi).

.............................................................................................................................
[13]

(b) The reaction scheme shows reactions involving glycolic acid.


I II 2,4-DNPH
chloroethanoic acid glycolic acid C2H2O3 Compound H

III

N
PCl3 +
H /heat O LiAlH4
Compound J Compound I Compound K

HO OH

(i) State the reagents and conditions for reactions II and III.

Reaction II: ........................................................................................................


.
Reaction III: ........................................................................................................

(ii) Draw the structural formulae for compounds H, I, J and K.

H I J K

SRJC 8872/02/Prelim/2013 [Turn Over


www.gradtutors.com
695
9

(iii) With the aid of the Data Booklet, suggest what will happen to the rate of For
Examiner’s
reaction for reaction I when chloroethanoic acid is replaced with use
bromoethanoic acid.

.............................................................................................................................

.............................................................................................................................

.............................................................................................................................

.............................................................................................................................

[8]

(c) H1ɵ
2C (s) + 2H2 (g) + O2 (g) CH3COOH (l)

H2ɵ H3ɵ

2CO2 (g) + 2H2O (l)

Using the following data and the energy cycle above, calculate the standard enthalpy
change of combustion of ethanoic acid.

Hcɵ carbon = -393 kJ mol-1


Hcɵ hydrogen = -286 kJ mol-1
Hfɵ ethanoic acid = -487 kJ mol-1

[2]
Total: [23]

SRJC 8872/02/Prelim/2013 [Turn Over


www.gradtutors.com
696
10

Section B
Answer two of the three questions in this section on separate paper.

4 (a) Using sodium, phosphorous and sulfur, describe the reactions of their oxides with either sodium
hydroxide or hydrochloric acid or both.
[4]

(b) Propanone undergoes keto-enol tautomerism when reacted with iodine under an acidic medium
at room temperature. The overall equation for this reaction is represented below:
H+
CH3COCH3 + I2 CH3COCH2I + HI

To investigate how the rate of reactions depends on the concentration of each of the three
reactants, two experiments were carried out. In each experiment the concentrations of two
reactants were in excess and kept constant, whilst the concentration of the third reactant was
measured over time. It is known that the order of reaction with respect to hydrogen ion is first
order.

reaction 1 reaction 2
time / s [CH3COCH3] / mol dm-3 [I 2] / mol dm-3
0 1.50 1.50
30 0.96 1.28
60 0.62 1.05
90 0.38 0.83
120 0.25 0.60
150 0.15 0.38
180 0.09 0.15

(i) Plot a graph of these results, putting all of the data on the same axes. Label each curve
clearly.

(ii) Use your graph to determine the order of reaction for each of the two reactants. Justify
your answer in each case.

(iii) Use your answer from (b)(ii) to write a rate equation for the reaction.

(iv) Explain how the rate of reaction would change if chlorine is used instead of iodine.

(v) With an aid of a suitable diagram, explain the effect on the rate of reaction if it is carried
out in an ice bath.

(vi) Propanone can also react with iodine under basic medium resulting in a different organic
product. Write an equation for this reaction, indicating the type of reaction involved and
the observation.
[14]
(d) Propose a chemical test to differentiate CH3COCH3 from CH3COCH2I.

[2]
Total: [20]
   

SRJC 8872/02/Prelim/2013 [Turn Over


www.gradtutors.com
697
11

5 (a) The table below he sources and the en


w shows th nthalpy changes of com
mbustion of
o two comm
mon
fuelss used in ve
ehicles.

Fuel urces
Main Sou halpy changge of
Enth
mbustion / kJJ mol-1
com
Octan
ne Refined frrom crude oil
o ─ 54
460.6
Ethan
nol Fermentaation ─13
359.8

(i) Define standard


s en
nthalpy chan
nge of comb
bustion usin
ng ethanol aas an exam
mple.

(ii) The figu


ure below shows
s agram of a calorimeterr used by a student to determine the
a dia
enthalpyy change off combustio
on of ethano
ol.

therm
mometer

small metal can

wate
er

wick
k

spirit lamp conta


aining
ethannol

When 0.960 g of ethan nol was combusted


d, the eenergy evo
olved hea
ated
100 cm3 of water from 25.0 C
C to 72.6 C.

Using the
t informa cy, x, of this
ation proviided, determine the percentagge efficienc
experim
ment, leaving wer to three significant figures.
g your answ

(iii) el value (in kJ g-1) of a substance


The fue e is the hea
at energy rreleased wh
hen 1 g of the
substance is combuusted.

Calculatte the fuel value


v for ea ch of the tw
wo fuels.

why octane has an advantage


Hence, suggest a reason w a oover ethan
nol as fuel for
vehicless.

(iv) Name th
he isomer of
o octane wh
hich has the
e lowest boiiling point.
[7]

(b) Etha
anol has many uses in
n the organ ory. One of its commo n uses is to manufactture
nic laborato
este
ers.
2CH3CH
H2OH + HO
OOCCH2CO
OOH ⇌ CH3CH2OC
COCH2COO
OCH2CH3 + 2H2O

(i) Deduce the effect on the equ


uilibrium con
nstant if a catalyst
c t the reaction
is i ntroduced to
chambe
er.

SRJ
JC 8872/02/P
Prelim/2013 [T
Turn Over
www.gradtutors.com
698
12

(ii) Equal amounts of ethanol and propanedioic acid reacted together to reach equilibrium.

Concentration / mol dm-3

ester

ethanol

time / min
t t2
Upon establishing equilibrium at time t, temperature of the reaction is decreased.
Determine whether the forward reaction is exothermic or endothermic.

(iii) State the observed changes in concentration of ethanol and ester as a result of the
change that occurred at t2.

(iv) A student intern at the laboratory made the following claim:

For the esterification reaction above, the enthalpy change is known as the enthalpy
change of neutralisation.

However, the enthalpy change cannot be calculated accurately by using only the bond
energies in the data booklet.

Comment on the intern’s claim.

[6]
(c) An ester L, C8H13O2Cl undergoes reaction to give ethanol and acid M C6H9O2Cl.

Decolourisation occurs when 1 mole of ester L reacts with 1 mole of liquid bromine.

When acid M reacts with hot acidified potassium manganate (VII), compound N is
formed together with 2 moles of gas. Compound N is inert towards sodium carbonate
powder and Fehling’s reagent but produces orange crystals when reacted with
hydrazine.

1 mole of compound N can react with alkaline aqueous iodine to give 1 mole of
ethanedioate ions and 2 moles of yellow precipitate.

Deduce the structural formulae of compounds L, M and N.


[7]
Total: [20]
   

SRJC 8872/02/Prelim/2013 [Turn Over


www.gradtutors.com
699
13

6 (a) (i) The Period 3 elements vary in their melting points, electrical conductivities and pH of
aqueous solutions of their chlorides. Sketch a clearly labelled graph to illustrate the pH
of solutions of the highest chloride of the Period 3 elements (from Na to P).

(ii) Describe what happens when aluminium chloride, AlCl3, is dissolved in water, writing an
equation to illustrate this.

(iii) Draw the structure of the compound formed when ammonia gas is pumped into a
container with aluminium chloride.

Hence, suggest the change in bond angle with respect to the aluminium atom in the
structures before and after the reaction.

(iv) Boron trifluoride is also capable of similar reaction with ammonia. Explain why this is so.

(v) Boron trifluoride gas and aluminium chloride liquid are common catalysts used in
isomerism, esterification and condensation reactions. Comment on the different physical
states of the two catalysts.

[8]

(b) (i) The following reaction scheme shows some reactions involving benzene

Compound P
II
CH3
CH3Cl
I AlCl3 III
Compound Q

Compound Q forms phenylethene in a 4-step synthesis reaction.

(i) State the type of reaction that took place for the conversion of methylbenzene from
benzene.

(ii) State the type of reaction that took place in reaction I.

(iii) From the information given in the reaction scheme, draw the structures of compound P
and Q and propose the reagents and conditions required for reaction II and III.
Compounds P and Q are isomers.

[6]

SRJC 8872/02/Prelim/2013 [Turn Over


www.gradtutors.com
700
14

(c) The ore, bauxite, consists of various forms of hydrated aluminium oxide, Al2O3.nH2O. It
is the raw material from which aluminium is obtained. The ore is usually found as a
mixture of Al2O3 and Fe2O3.

Some data on the properties of Al2O3 and Fe2O3 is tabulated below.

Al2O3 Fe2O3
Melting point (oC) 2072 1560
Solubility in hot soluble Insoluble
concentrated sodium
hydroxide
Molar mass (g mol-1) 101.96 159.69

(i) Suggest with the aid of an equation why aluminium oxide is soluble in hot concentrated
sodium hydroxide whereas iron(III) oxide is not soluble.

(ii) Draw the dot-and-cross diagram for aluminium oxide.

(iii) With the aid of the Data Booklet, explain why aluminium oxide has a higher melting point
as compared to iron(III) oxide.

(iv) Suggest a reason why BeO would exhibit similar chemical properties as Al2O3.

[6]
Total: [20]
 
END

SRJC 8872/02/Prelim/2013 [Turn Over


www.gradtutors.com
701
15

BLANK PAGE

SRJC 8872/02/Prelim/2013 [Turn Over


www.gradtutors.com
702
16

BLANK PAGE

SRJC 8872/02/Prelim/2013 [Turn Over


www.gradtutors.com
703
1

SERANGOON JUNIOR COLLEGE


General Certificate of Education Advanced Level
Higher 1

CANDIDATE
NAME

CLASS INDEX NUMBER

CHEMISTRY 8872/02
Preliminary Examination 22 August 2013
Paper 2 2 hours

Additional Materials: Data Booklet


Writing Papers

READ THESE INSTRUCTIONS FIRST

Write your name and class on all the work you hand in.
Write in dark blue or black pen on both sides of the paper.
You may use a soft pencil for any diagrams, graphs or rough work.

SECTION A:
Answer all questions in the space provided in the booklet.

SECTION B:
Answer any two questions on separate answer papers.

At the end of the examination, fasten all your work securely together.
The number of marks is given in brackets [ ] at the end of each question or part question.

For Examiner’s Use


MCQ
P1
Section 1 2 3 Total
A

B4

B5

B6

Total
/80

This document consist of 17 printed pages and 1 blank page

SRJC 8872/02/Prelim/2013 [Turn Over


www.gradtutors.com
704
2

Section A
Answer all the questions in this section in the spaces provided.

1 (a) Arsenic acid, H3AsO4, is a colourless acid that is used as a wood preservative and For
Examiner’s
as a reagent in the synthesis of dyestuff. It is slowly formed when elemental use
arsenic, As, is treated with ozone in the presence of moisture. Oxygen gas is
known to be a side product for the reaction.

(i) Write a balanced chemical equation for the above mentioned treatment
process.
2As + 3H2O + 5O3  H3AsO4 + 5O2 [1]

(ii) Arsenic acid can oxidise iodide ions to iodine and itself is reduced to H3AsO3.
Write a half-equation for the reduction of arsenic acid to H3AsO3.
Hence write an overall equation for the reaction between iodide ions and
arsenic acid.
Reduction: 2H+ + H3AsO4 + 2e  H3AsO3 + H2O [1]
Oxidation: 2I-  I2 + 2e

Overall: 2H+ + H3AsO4 + 2I─  H3AsO3 + H2O + I2 [1]

(iii) The iodine, I2, that is liberated from the reaction between arsenic acid and
iodide ions can be estimated by titration against a
standard thiosulfate, S2O32-, solution.

I2 + 2S2O32-  2I─ + S4O62-

When a sample of 25.0 cm3 of arsenic acid reacts with iodide ions, it was
found that 23.30 cm3 of 0.200 mol dm-3 solution of thiosulfate ions was
required to react completely with the iodine liberated.

Calculate the concentration of the sample of arsenic acid.


Amt of thiosulfate reacted =
I2 ≡ 2S2O32-
Amt of I2 =
[1]
I2 ≡ H3AsO4
Amt of H3AsO4 in 25.0 cm3 = 0.00233 mol
[H3AsO4] = [1]
[5]

SRJC 8872/02/Prelim/2013 [Turn Over


www.gradtutors.com
705
3

(c) The following graph shows some data on consecutive elements from period 2 and
period 3.

lg (2nd I.E.)
3.7

3.6

3.5

3.4

3.3

3.2

3.1

2.9
A B C D E F G

Elements

F is known to react with C to form an ionic compound.


G is a better conductor of electricity as compared to F.
The chloride of F gives a slightly acidic solution when dissolved in water.

(i) From the information given, identify F.


Hence, describe the type of bonding in F either in words or with a labelled
diagram.
From the data given, 2nd I.E. of F is the lowest thus it must be a group II
element in period 3.

Therefore F is magnesium. [1]

F has giant metallic structure with electrostatic forces of attraction


between the cations and sea of delocalised electrons. [1] or diagram

e e
2+ e 2+ e 2+ e
e e e e
2+ 2+ 2+ e
e
e Diagram – [1]

(ii) Suggest if the compound formed from F and C is soluble in water. You must
support your answer with relevant explanations to gain full credit of this
question.
Since the product is an ionic compound
There is favourable ion-dipole interactions which results in the release of
energy that can break the giant ionic structure. [1]
Hence it is soluble in water. [1]
[4]
Total: [9]

SRJC 8872/02/Prelim/2013 [Turn Over


www.gradtutors.com
706
4

2 (a) For the reaction, For


Examiner’s
use

CH3CH2CO2H (l) + CH3OH (l) CH3CH2CO2CH3 (l) + H2O (l)

The value of the equilibrium constant is 5.0.

(i) Suggest one way to increase the yield of the above reaction.
Increase the amount of either reactants (acid or alcohol) or remove the
products (ester or water) as they are formed. [1]

(ii) Explain, in terms of chemical bonding and structure, why the product,
CH3CH2CO2CH3 has low solubility in water.
Weaker van der Waal’s forces between solute molecules are not strong
enough to displace the stronger hydrogen bonding between water
molecules for hydration to occur [1]

(iii) State the numerical value of the equilibrium constant for the reverse reaction.
0.2 [1]
(iv) Suggest the reagent and condition for the reverse reaction.
Reagent: dilute HCl or H2SO4 [1]
Condition: Heat
(v) In an experiment, 1 mol of CH3CH2CO2CH3 and 1 mol of water are mixed in
V cm3. Calculate the amount of acid present at equilibrium.

CH3CH2CO2CH3 + H2O CH3CH2CO2H + CH3OH

Initial amt/ 1 1 0 0
mol
Change/ -x -x x x
mol
Eqm/ mol 1-x 1-x x x

Let x be amount of acid at equilibrium


Kc’ = 0.2
x x
( )( )
V V  0.2 [1] for showing V in working
1 x 1 x
( )( )
V V

(x )
 0.2
(1  x )

x = 0.309 mol [1] award ecf if Kc is calculated wrongly

SRJC 8872/02/Prelim/2013 [Turn Over


www.gradtutors.com
707
5

(vi) Suggest a chemical test to distinguish between CH3CH2CO2CH3 and


CH3CH2CO2CH2CH3.
Chemical test: Add H2SO4 (aq), heat followed by KMnO4 to separate test tubes
of CH3CH2CO2CH3 and CH3CH2CO2CH2CH3. [1]

Observation:
CH3CH2CO2CH3: Decolourisation of purple KMnO4, effervescence of CO2
which forms white ppt when bubbled into Ca(OH)2 (aq).
CH3CH2CO2CH2CH3: Decolourisation of purple KMnO4, no effervescence
[1]

OR

Test: Add NaOH (aq) and heat to each compound separately, followed by I2
(aq), NaOH (aq), heat. alternative[1]

Observation:
CH3CH2CO2CH3: No decolourisation of brown I2 and yellow ppt CHI3
CH3CH2CO2CH2CH3: Decolourisation of brown I 2 and yellow ppt CHI3
alternative [1]
Total: [8]

SRJC 8872/02/Prelim/2013 [Turn Over


www.gradtutors.com
708
6

3 Glycolic acid (pKa = 3.83), C2H4O3, is a colourless and odourless crystalline solid that is For
Examiner’s
highly soluble in water. It is the smallest α–hydroxy acid (AHA) and is commonly used in skin use
care products and most often as a chemical peel performed by plastic surgeons due to its
ability to penetrate the skin effectively.

It can be prepared by the reaction of chloroethanoic acid (pKa = 2.85) with hot sodium
hydroxide followed by re-acidification.
(a) (i) Write a chemical equation for the net reaction that has occurred showing the displayed
formulae of all organic compounds.
H
Cl O O
O
H C C + NaOH H C C + NaCl
O H H O H
H [1]

(ii) Explain why chloroethanoic acid has a lower pKa as compared to glycolic acid.
In chloroethanoic acid there is presence of a stronger electron withdrawing Cl. This
means the negative charge on the carboxylate ion of chloroethanoic acid will disperse
to a greater extent thus stabilising the carboxylate ion of chloroethanoic acid. This
facilitate the release of H+ suggesting chloroethanoic acid to be a stronger acid to
glycolic acid.
[2]

(iii) Chloroethanoic acid is defined as a weak acid.


With the aid of an equation, explain the words in italics.

A weak acid is one which dissociates partially in solution to give protons (H+) [1]
ClCH2COOH ClCH2COO- + H+ [1]

SRJC 8872/02/Prelim/2013 [Turn Over


www.gradtutors.com
709
7

(iv) Beside pKa, percentage dissociation is another way to measure the strength of an For
Examiner’s
acid. Percentage dissociation is the ratio of the concentration of hydrogen ions to the use
concentration of the acid expressed in percentage.

An analysis was carried out on a 25 cm3 sample of a skin care product containing
glycolic acid by titrating with 0.125 mol dm-3 of sodium hydroxide. The graph obtained
is shown below. With this graph, determine the percentage dissociation of glycolic
acid.

pH
12.4

8.19

3.83

3.00

0 12.5 25.0 37.5


Volume of NaOH / cm3

From graph [H+] = 10-3.00 = 1.00 x 10-3 mol dm-3 [1]


Glycolic acid ≡ NaOH [1]
Amount of glycolic acid = Amount of NaOH = 25/1000 x 0.125 = 0.003125 mol
[Glycolic acid] = 0.003125 / (25/1000) = 0.125 mol dm-3 [1]
% dissociation = [1.00 x 10-3 / 0.125 ] x 100% = 0.8 % [1]

(v) Prove that the final pH is 12.4 after 37.5 cm3 of sodium hydroxide is added.

Vol of excess NaOH = 37.5 – 25 = 12.5 cm3


Amt of excess NaOH = (12.5/1000) x 0.125 = 1.56 x 10-3 mol [1]
[NaOH] = 1.56 x 10-3 ÷ [(37.5+25)/1000] = 0.02496 mol dm-3 [1]

pOH = -lg 0.02496 = 1.603


pH = 14- 1.603 = 12.4 (proven) [1]

SRJC 8872/02/Prelim/2013 [Turn Over


www.gradtutors.com
710
8

(vi) On the graph in (a)(iv), indicate the region where there is a buffer solution. State the For
Examiner’s
species involved in this buffer system. use

[1]

OH O OH O
H C C & H C C
H O H O-Na+
H
Species present: [1] for both

(vii) Using an equation only, suggest how the pH remains fairly constant when small
amount of sodium hydroxide is added to the solution in (a)(vi).
OH O OH O
H C C + OH - H C C + H2O
-
H O H O
H [1]

[13]

(b) The reaction scheme shows reactions involving glycolic acid.


I II 2,4-DNPH
chloroethanoic acid glycolic acid C2H2O3 Compound H

III

N
PCl3 +
H /heat O LiAlH4
Compound J Compound I Compound K

HO OH

(i) State the reagents and conditions for reactions II and III.

Reaction II: ___________________________

Reaction III: __________________________

Reaction II: K2Cr2O7, dilute H2SO4, distillation [1]

Reaction III: HCN, NaCN (or NaOH), 10oC to 20oC [1]

SRJC 8872/02/Prelim/2013 [Turn Over


www.gradtutors.com
711
9

(ii) Draw the structural formulae for compounds H, I, J and K. For


Examiner’s
use

H I J K

NO2 H
OH Cl H2N CH2 C CH2OH
HOOC C COOH ClOC C COCl OH

NO2 H H
H N
N
O
C C
H OH
J [1] K [1]
H [1] I [1]

(iii) With the aid of the Data Booklet, suggest what will happen to the rate of reaction for
reaction I when chloroethanoic acid is replaced with bromoethanoic acid.

Bond energy: C-Br (280) < C-Cl(340) [1] with relevant quote from Data Booklet
Energy required: C-Br < C-Cl or Ease of cleavage of bond: C-Br > C-Cl
Rate of reaction: bromoethanoic acid > chloroethanoic acid [1]
[8]
(c) H1ɵ
2C (s) + 2H2 (g) + O2 (g) CH3COOH (l)

H2ɵ H3ɵ

2CO2 (g) + 2H2O (l)

Using the following data and the energy cycle above, calculate the standard enthalpy
change of combustion of ethanoic acid.

Hcɵ carbon = -393 kJ mol-1


Hcɵ hydrogen = -286 kJ mol-1
Hfɵ ethanoic acid = -487 kJ mol-1

By Hess Law: H1ɵ + H3ɵ = H2ɵ [1]


-487 + Hcɵ ethanoic acid = 2(-393) + 2(-286)

SRJC 8872/02/Prelim/2013 [Turn Over


www.gradtutors.com
712
10

Hcɵ ethanoic acid = - 871 kJ mol-1 [1]


[2]
Total: [23]
Section B
Answer two of the three questions in this section on separate paper.

4 (a) Using sodium, phosphorous and sulfur, describe the reactions of their oxides with either sodium
hydroxide or hydrochloric acid or both.
[4]
The oxide changes from basic to acidic across the period. [1]

Sodium oxide is a basic oxide, it undergoes neutralisation with acids to form salt and water.
Na2O(s) + 2 HCl(aq) 2 NaCl(aq) + H2O(l) [1]

Phosphorous oxides and sulfur oxides undergo neutralisation with bases to form salt and
water
P4O6(s) + 12 NaOH(aq)  4 Na3PO3(aq) + 6 H2O(l) or
P4O10(s) + 12 NaOH(aq)  4 Na3PO4(aq) + 6 H2O(l) [1]

SO2(g) + 2 NaOH(aq)  Na2SO3 (aq) + H2O(l) or


SO3(g) + 2 NaOH(aq)  Na2SO4(aq) + H2O(l) [1]

(b) Propanone undergoes keto-enol tautomerism when reacted with iodine under an acidic medium
at room temperature. The overall equation for this reaction is represented below:
H+
CH3COCH3 + I2 CH3COCH2I + HI

To investigate how the rate of reactions depends on the concentration of each of the three
reactants, two experiments were carried out. In each experiment the concentrations of two
reactants were in excess and kept constant, whilst the concentration of the third reactant was
measured over time. It is known that the order of reaction with respect to hydrogen ion is first
order.

reaction 1 reaction 2
time / s [CH3COCH3] / mol dm-3 [I 2] / mol dm-3
0 1.50 1.50
30 0.96 1.28
60 0.62 1.05
90 0.38 0.83
120 0.25 0.60
150 0.15 0.38
180 0.09 0.15

SRJC 8872/02/Prelim/2013 [Turn Over


www.gradtutors.com
713
11

(i) Plot a graph of these results, putting all of the data on the same axes. Label each curve
clearly.

Graph of [reactants]/mol dm-3 against time/s

[reactants]/mol dm-3
1.60

1.40

1.20

1.00

0.80 I2
CH3COCH3
0.60

0.40

0.20

0.00
0 50 100 150 200

t1/2 = 46s t1/2 = 46s time/s

axes and scale [1]


Correct points plotted (randomly checked 3 points on each graph) [1]
Straight line for I2 and smooth curves for CH3COCH3 [1]

(ii) Use your graph to determine the order of reaction for each of the two reactants. Justify
your answer in each case.
I2 is a downward sloping straight line, hence rate is indepenedent of the [I2], therefore,
order of reaction wrt I2= 0 [1]

CH3COCH3 show curve lines with constant half-lives of 46s. [1] for clear annotation of
two half-lives on graph
Therefore, order of reaction wrt CH3COCH3 = 1 [1]

(iii) Use your answer from (b)(ii) to write a rate equation for the reaction.
Rate = k [CH3COCH3][H+] [1]

(iv) Explain how the rate of reaction would change if chlorine is used instead of iodine.
Rate remained constant, since rate is independent of the halogen used / order wrt
halogen is zero / halogen is not involved in the rate determining step. [1]

SRJC 8872/02/Prelim/2013 [Turn Over


www.gradtutors.com
714
12

(v) With an aid of a suitable diagram, explain the effect on the rate of reaction if it is carried
out in an ice bath.

Fraction of molecules
with energy, E
Ice bath

Room temperature
Key:

No. of particles with E Ea


at higher temperature
No. of particles with E Ea
at lower temperature

Ea Energy, E
0

Diagram [1]

When temperature of the reaction decreases,


 average kinetic energy of the reactant particles decreases [1]
 less reactant particles with energy ≥ Ea less effective collisions
Since rate of reaction is proportional to the frequency of effective collisions, rate of
reaction decreases [1]

(vi) Propanone can also react with iodine under basic medium resulting in a different organic
product. Write an equation for this reaction, indicating the type of reaction involved and
the observation.

CH3COCH3 + 3I2 + 4OH-  CH3COO- + 3I- + 3H2O + CHI3 [1]


Oxidation [1]
Decolourisation of brown iodine solution and pale yellow ppt of CHI3 formed. [1]

[14]
(d) Propose a chemical test to differentiate CH3COCH3 from CH3COCH2I.

Test: Add NaOH (aq), heat, followed by HNO3 (aq) then AgNO3 (aq) [1]
Observation:
CH3COCH2I will produce yellow ppt while no ppt is observed for CH3COCH3 [1]
[2]
Total: [20]

SRJC 8872/02/Prelim/2013 [Turn Over


www.gradtutors.com
715
13

5 (a) The table below shows the sources and the enthalpy changes of combustion of two
common fuels used in vehicles.

Fuel Main Sources Enthalpy change of


combustion / kJ mol-1
Octane Refined from crude oil ─ 5460.6
Ethanol Fermentation ─1359.8

(i) Define standard enthalpy change of combustion using ethanol as an example.

The standard enthalpy change of combustion of ethanol is the energy evolved when one
mole of ethanol is completely burnt in excess oxygen at 298K and 1 atm pressure. [1]

(ii) The figure below shows a diagram of a calorimeter used by a student to determine the
enthalpy change of combustion of ethanol.

thermometer

small metal can

water

wick

spirit lamp containing


ethanol

When 0.960 g of ethanol was combusted, the energy evolved heated


100 cm3 of water from 25.0 C to 72.6 C.

Using the information provided, determine the percentage efficiency, x, of this


experiment, leaving your answer to three significant figures.

Heat absorbed by water, Q’ = 100  4.18  (72.6  25.0)


= 19897 J
= 19.897 kJ [1]
nethanol burnt = 0.96 = 0.0209 mol
2  12.0 + 6  1.0 + 16.0

Q
Hc (ethanol) = 
0.0209
Q = 1359.8 x 103 (0.0209) = 28.420 x 103 [1]
Q’ = x Q
100
19.897 x 103 = x x 28.420 x 103
100
x = 70.0% [1]

SRJC 8872/02/Prelim/2013 [Turn Over


www.gradtutors.com
716
14

(iii) The fuel value (in kJ g-1) of a substance is the heat energy released when 1 g of the
substance is combusted.

Calculate the fuel value for each of the two fuels.

Hence, suggest a reason why octane has an advantage over ethanol as fuel for
vehicles.
Molar mass of octane = 114.0 g mol-1
Molar mass of ethanol = 46.0 g mol-1
Fuel value of octane = 5460.6 / 114 = 47.9 kJ g-1
Fuel value of ethanol = 1359.8 / 46 = 29.6 kJ g-1 [1] for both values

Advantage: Octane has a larger fuel value. Thus it releases more energy per gram
of fuel. For the vehicle to travel a specific distance, less amount of octane is
needed as compared to using ethanol. [1]

(iv) Name the isomer of octane which has the lowest boiling point.
2,2,3,3-tetramethylbutane [1]

[7]
(b) Ethanol has many uses in the organic laboratory. One of its common uses is to manufacture
esters.

2CH3CH2OH + HOOCCH2COOH ⇌ CH3CH2OCOCH2COOCH2CH3 + 2H2O

(i) Deduce the effect on the equilibrium constant if a catalyst is introduced to the reaction
chamber.

When a catalyst is added, the rate of the forward and backward reactions will increase
by the same extent.
Therefore, the Kc value remain unchanged. [1]

(ii) Equal amounts of ethanol and propanedioic acid reacted together to reach equilibrium.

Concentration / mol dm-3

ester

ethanol

time / min
t t2

SRJC 8872/02/Prelim/2013 [Turn Over


www.gradtutors.com
717
15

Upon establishing equilibrium at time t, temperature of the reaction is decreased.


Determine whether the forward reaction is exothermic or endothermic.

By Le Chatelier’s Principle, a decrease in temperature favours an exothermic reaction.


When temperature is decreased, [reactants/ethanol] increases and [product/ester]
decrease, indicating that the backward reaction is favoured. [1] Hence, the forward
reaction is endothermic. [1]

(iii) State the observed changes in concentration of ethanol and ester as a result of the
change that occurred at t2.

Concentration of ethanol will decrease and concentration of ester will increase until
a new equilibrium is reached. [1]

(iv) A student intern at the laboratory made the following claim:

For the esterification reaction above, the enthalpy change is known as the enthalpy
change of neutralisation.

However, the enthalpy change cannot be calculated accurately by using only the bond
energies in the data booklet.

Comment on the intern’s claim.

The first sentence of the intern’s claim is incorrect. Esterification reaction is not a
neutralisation as the water produced in this case is not due to reaction between acid
and base, it is a condensation reaction. [1]

The second sentence is correct. The bond energies in the data booklet cannot be
used directly to calculate enthalpy change as the reactants and products are not in
the gaseous phase or Hvaporisation need to be considered. [1]

[6]
(c) An ester L, C8H13O2Cl undergoes reaction to give ethanol and acid M C6H9O2Cl.

Decolourisation occurs when 1 mole of ester L reacts with 1 mole of liquid bromine.

When acid M reacts with hot acidified potassium manganate (VII), compound N is
formed together with 2 moles of gas. Compound N is inert towards sodium carbonate
powder and Fehling’s reagent but produces orange crystals when reacted with
hydrazine.

1 mole of compound N can react with alkaline aqueous iodine to give 1 mole of
ethanedioate ions and 2 moles of yellow precipitate.

Deduce the structural formulae of compounds L, M and N.


[7]
Ester L undergoes acidic hydrolysis [1] to form ethanol and acid M.

Ester L undergoes electrophilic addition with liquid bromine, therefore ester L contains
a C=C double bond. [1]

Acid M undergoes oxidation with hot acidified potassium permanganate to give


compound N and gas. Gas is CO2. Therefore, acid M has a terminal double bond or
the oxidised product of HOOC-COOH. [1]

Compound N does not undergo neutralisation with Na2CO3, absence of RCOOH. [1]
SRJC 8872/02/Prelim/2013 [Turn Over
www.gradtutors.com
718
16

Compound N does not undergo oxidation with Fehlings, absence of aldehyde. [1]

Compound N undergoes condensation with hydrazine, presence of ketone. [1]

1 mol of N undergoes nucleophilic substitution with NaOH to give alcohol [1]


followed by oxidation with alkaline iodine to give 2 mol of CHI3, therefore N has:
Cl
O
C CH3
and C CH3
H
[1] Max – 4m for statements

B:L H
CH3 CH C C C O CH2CH3

Cl CH3 O
[1]

C: H
M
CH3 CH C C C OH

Cl CH3 O
[1]

D:
N CH3 CH C O

Cl CH3
[1]

Total: [20]

SRJC 8872/02/Prelim/2013 [Turn Over


www.gradtutors.com
719
17

6 (a) (i) The Period 3 elements vary in their melting points, electrical conductivities and pH of
aqueous solutions of their chlorides. Sketch a clearly labelled graph to illustrate the pH
of solutions of the highest chloride of the Period 3 elements (from Na to P).
pH 14

0
NaCl MgCl2 AlCl3 SiCl4 PCl5
Correct axis & Correct shape – [1]

(ii) Describe what happens when aluminium chloride, AlCl3, is dissolved in water, write an
equation to illustrate this.
AlCl3 undergo hydrolysis in water, to form an acidic solution. [1]

Al (H2O)63+ + H2O [Al (H2O)5(OH)]2+ + H3O+ [1]

(iii) Draw the structure of the compound formed when ammonia gas is pumped into a
container with aluminium chloride.

Hence, suggest the change in bond angle with respect to the aluminium atom in the
structures before and after the reaction.
Cl
Cl Cl
Al

N
H H
H [1] Lewis structure must be drawn.

Bond angle around Al changes from 120o to 109.50 [1]

(iv) Boron trifluoride is also capable of similar reaction with ammonia. Explain why this is so.
Boron has empty/ vacant / low lying ‘p’ orbital to accommodate the lone pair of
electrons from nitrogen in the ammonia structure. [1]

(v) Boron trifluoride gas and aluminium chloride liquid are common catalysts used in
isomerism, esterification and condensation reactions. Comment on the different physical
states of the two catalysts.

Both BF3 and AlCl3 have simple molecular structures.

No of electron: AlCl3 > BF3 [1]


Extensiveness of VDW: AlCl3 > BF3
Energy required to overcome VDW: AlCl3 > BF3 [1]
Boiling point: AlCl3 > BF3

BF3 is gas, AlCl3 is liquid.


[8]
SRJC 8872/02/Prelim/2013 [Turn Over
www.gradtutors.com
720
18

(b) (i) The following reaction scheme shows some reactions involving benzene

Compound P
II
CH3
CH3Cl
I AlCl3 III
Compound Q

Compound Q forms phenylethene in a 4-step synthesis reaction.

(i) State the type of reaction that took place for the conversion of methylbenzene from
benzene.
Electrophilic substitution. [1]

(ii) State the type of reaction that took place in reaction I.


Reduction [1]

(iii) From the information given in the reaction scheme, draw the structures of
compound P and Q which are isomers and propose the reagents and conditions
required for reaction II and III. Compounds P and Q are isomers.
Reaction II: AlCl3, Cl2, rtp [1]
Reaction III: Cl2 , uv light [1]
CH3 CH2Cl
Cl

Compound P: [1] Compound Q: [1]

For your information


CH3
Cl

II
CH3 CH2Cl CH2CN
CH3Cl
I AlCl3 III

H H
C C CH2CH2OH CH2COOH
H

[6]

SRJC 8872/02/Prelim/2013 [Turn Over


www.gradtutors.com
721
19

(c) The ore, bauxite, consists of various forms of hydrated aluminium oxide, Al2O3.nH2O. It
is the raw material from which aluminium is obtained. The ore is usually found as a
mixture of Al2O3 and Fe2O3.

Some data on the properties of Al2O3 and Fe2O3 is tabulated below.

Al2O3 Fe2O3
Melting point (oC) 2072 1560
Solubility in hot soluble Insoluble
concentrated sodium
hydroxide
Molar mass (g mol-1) 101.96 159.69

(i) Suggest with the aid of an equation why aluminium oxide is soluble in hot concentrated
sodium hydroxide whereas iron (III) oxide is not soluble.

Al2O3 is amphoteric, able to react as an acidic oxide, with the base, NaOH.
Soluble salt formed.
Al2O3 + 2NaOH + 3H2O  2NaAl(OH)4 (aq) [1]

Fe2O3 metallic oxide and thus is basic, therefore it is unable to react with NaOH. [1]

(ii) Draw the dot-and-cross diagram for aluminium oxide.

[1]
(iii) With the aid of the Data Booklet, explain why aluminium oxide has a higher melting point
as compared to iron (III) oxide.

Both aluminium oxide and iron (III) oxide have giant ionic structure.

Hlatt   (q+q-)/(r++r-)
r+ of Al3+ (0.050) < r+ of Fe3+ (0.064) [1] with relevant quote from Data Booklet
Hlatt Al2O3 > Hlatt Fe2O3
More energy is required to overcome the electrostatic forces of attraction [1]
between the oppositely charged ions in Al2O3 resulting in higher melting point.

(packing can be considered as an alternative answer).

(iv) Suggest a reason why BeO would exhibit similar chemical properties as Al2O3.
Be2+ and Al3+ have similar charge density [1] which results in diagonal relationship.

[6]
Total: [20]

END

SRJC 8872/02/Prelim/2013 [Turn Over


www.gradtutors.com
722

TAMPINES JUNIOR COLLEGE

H JC2 Preliminary Examination

1
CANDIDATE
NAME

TUTOR
CIVICS GROUP 1 2 NAME

CHEMISTRY 8872/ 01
Paper 1 Multiple Choice Monday, 23 September 2013
50 minutes

Additional Materials: Multiple Choice Answer Sheet


Data Booklet

READ THESE INSTRUCTIONS FIRST

Write in soft pencil.


Do not use staples, paper clips, highlighters, glue or correction fluid.
Write your name and civics group on the Answer Sheet in the spaces provided.

There are thirty questions on this paper. Answer all questions. For each question there are four
possible answers A, B, C and D.
Choose the one you consider correct and record your choice in soft pencil on the separate Answer
Sheet.

Read the instructions on the Answer Sheet very carefully.

Each correct answer will score one mark. A mark will not be deducted for a wrong answer.
Any rough working should be done on this paper.

This document consists of 13 printed pages and 1 blank page.

TPJC_2013_8872_01 [Turn over


www.gradtutors.com
723
2

Section A

For each question there are four possible answers, A, B, C and D. Choose the one you
consider to be correct.

1 Use of the Data Booklet is relevant to this question.


How many molecules are present in 1 cm3 of oxygen gas under room conditions?

1× 24000
A
6.02 × 10 23
1× 6.02 × 10 23
B
24000
6.02 × 10 23 × 24000
C
1× 1000
D 1× 6.02 × 10 23 × 32

2 Which of the following ions would undergo the greatest deflection in an electric
field?

16
A O2+ B 16
O18O+ C 16
O18O2+ D 18
O2+

3 10 cm3 of 0.2 mol dm−3 K2XO4 will just react with 40 cm3 of 0.1 mol dm−3
iron(II) sulfate solution.

If Fe2+ is oxidized to Fe3+ by K2XO4, what is the final oxidation state of X?

A +2 B +3 C +4 D +5

4 The table gives the successive ionisation energies for an element X.

1st 2nd 3rd 4th 5th 6th


Ionisation energy/
950 1800 2700 4800 6000 12300
kJ mol−1

What could be the formula of the chloride of X?

A XCl B XCl2 C XCl3 D XCl4

TPJC_2013_8872_01
www.gradtutors.com
724
3

5 Which of the following consists of species which are all planar?

A CO32– , SO32– and benzene


B Al2Cl6 , SOCl2 and methanal
C NO3– , XeF4 and ethanal
D BCl3 , ICl4− and chlorobenzene

6 Dot-and-cross diagrams for carbon monoxide are shown.

Which circled pair of electrons represents a coordinate bond?

A B C D

7 The diagram shows the Boltzmann distribution of the speeds of the molecules of a
gas.
Point X represents the most probable speed.
If the gas is cooled, in which direction does X move?

8 Iodine-131 is a radioactive isotope with a half-life of 8 days. Given that radioactive


decay is a first-order reaction, what fraction of the isotope would remain after
80 days?

1 1 1 1
A B C D
20 160 28 210

TPJC_2013_8872_01 [Turn over


www.gradtutors.com
725
4

9 The table shows the charge and radius of each of six ions.

ion J+ L+ M2+ X− Y− Z2−


radius/ nm 0.14 0.18 0.15 0.14 0.18 0.15

The ionic solids JX, LY, and MZ have the same lattice type.

What is the order of their lattice energies starting from the most exothermic first?

A JX, MZ, LY B LY, MZ, JX C MZ, JX, LY D MZ, LY, JX

10 Dinitrogen oxide, N=N=O, burns in ethyne, C2H2, in the gaseous phase to


produce water vapour, carbon dioxide and nitrogen gases as the only products.

5N2O(g) + C2H2(g) → H2O(g) + 2CO2(g) + 5N2(g) ΔH = −1668 kJ mol−1

Assuming N=N bond energy in dinitrogen oxide is +418 kJ mol−1, what is the
nitrogen-oxygen bond energy in dinitrogen oxide in kJ mol−1?

A 382 B 594 C 686 D 1350

11 Which is the correct statement about the following reaction?


M(s)
P(g) + Q(g) R(g) ∆H < 0

A The solid M will lower the activation energy of both forward and backward
reactions.
B Both the rate constant and equilibrium constant will increase with increasing
temperature.
C Increasing temperature will lower the activation energy resulting only in a
greater fraction of R molecules with energy greater than activation energy.
D The activation energy of the forward reaction is equal to the activation energy
of the backward reaction.

12 Phosphorus is an element in the third period, Na to Ar, of the Periodic Table.


What is true for phosphorus and none of the other elements in this period?

A Phosphorus has the highest melting point of the elements in this period.
B Phosphorus is the only element in this period that forms two acidic oxides.
C Phosphorus is the only element in this period with exactly four atoms in its
molecule.
D Phosphorus is the only element in this period whose chlorides react with
water to form acidic solutions.

TPJC_2013_8872_01
www.gradtutors.com
726
5

13 The bar chart shows the melting points of a series of consecutive elements
arranged in order of increasing atomic number. The elements sodium to chlorine
form part of this series.
Which bar represents sodium?

14 The chloride of element Q is hydrolysed by water to form an acidic solution and its
oxide reacts with acid to form a salt.
What could be element Q?
A sodium
B aluminium
C silicon
D phosphorus

TPJC_2013_8872_01 [Turn over


www.gradtutors.com
727
6

15 The reaction between NO and F2 was studied by mixing the two gases:

2NO(g) + F2(g) 2NOF(g) ΔH < 0

At different times during the experiment, various changes were made to the
conditions inside the reaction vessel. The changes in concentrations of the three
compounds in the equilibrium mixture with time are given by the graph below:

Which of the following statements is correct?

A There was a decrease in volume of the reaction vessel at 10.0 min.


B There was an increase in volume of the reaction vessel at 7.5 min.
C There was an increase in temperature at 10.0 min.
D A catalyst was added at 7.5 min.

16 A solution of pH 1.3 was produced when 0.10 mol of an acid, U, was dissolved in
2 dm3 of water.

Given the above information, which of the following statements is true?

A The solution contains 0.050 mol of hydrogen ions.


B U is a weak acid with a large Ka value.
C U is a diprotic strong acid.
D U is a monoprotic strong acid.

TPJC_2013_8872_01
www.gradtutors.com
728
7

17 The graph shows the change in pH when ethanoic acid is gradually added to
10 cm3 of 0.10 mol dm−3 sodium hydroxide. An indicator is used to determine the
end-point for the titration.

Which of the following contains the correct concentration of ethanoic acid and
indicator used for the titration?

A 0.05 mol dm−3 ethanoic acid, phenolphthalein


B 0.05 mol dm−3 ethanoic acid, screened methyl orange
C 0.10 mol dm−3 ethanoic acid, phenolphthalein
D 0.10 mol dm−3 ethanoic acid, screened methyl orange

18 What is meant by the term dynamic equilibrium?

A An equilibrium that is constantly changing its position.

B An equilibrium where the forward and reverse reactions are taking place at
different rates.
C An equilibrium where the forward and reverse reactions are taking place at
the same rate.

D An equilibrium which has not yet settled to a constant state.

TPJC_2013_8872_01 [Turn over


www.gradtutors.com
729
8

19 Dibenzalacetone is a bright yellow solid with the following structure.

dibenzalacetone

How many geometric isomers does dibenzalacetone have?

A 2 B 3 C 4 D 5

20 A food chemist wants to create the odour of green apples for a product. An ester
with this odour has the formula C2H5CO2CH(CH3)2.

In which of the following will the substances react together to produce this ester?

A C2H5OH and (CH3)2CHCOOH


B CH3COOH and CH3CH(OH)CH2CH3
C C2H5COOH and C2H5CH2OH
D C2H5COOH and (CH3)2CHOH

21 Which is not a possible product formed when the following compound is heated
with excess ethanolic KOH?
Br

Br

Br

A C

B D

TPJC_2013_8872_01
www.gradtutors.com
730
9

22 2,4–D and MCPA are two common selective weed killers.

Which one of the following reagents can be used to distinguish between them?

A sodium metal
B phosphorus pentachloride
C 2,4-dinitrophenylhydrazine
D hot acidified potassium dichromate(VI)

23 Deuterium, D, is an isotope of hydrogen, 21 H .


Which of the following reactions yields a stable organic compound containing
deuterium?

CH3COOD
C

TPJC_2013_8872_01 [Turn over


www.gradtutors.com
731
10

24 An organic compound K has the following properties:


• 1 mole of K reacts with excess Na(s) to produce one mole of H2(g).
• 1 mole of K reacts with warm alkaline aqueous iodine to produce
one mole of CHI3(s).
Which compound could K be?

A CH3COCH(OH)CH(OH)CH3

B HOCH2CH(OH)CH2COOH
C CH3CO2CH(OH)CH2OH

D CH3CH(OH)CH2COOH

25 Chlorofluorocarbons (CFCs) are commonly used as aerosols, propellants and


refrigerants. However in the stratosphere, CFCs can damage the ozone layer
through a radical chain reaction.

In which sequence are the following compounds listed in increasing order of their
ability to destroy ozone?

H H F F Cl

A F C Cl < F C C F < Cl C C F

F Cl Cl Cl Cl
H F Cl H F

B F C Cl < Cl C C F < F C C F

F Cl Cl Cl Cl
F Cl H H F

C Cl C C F < F C Cl < F C C F
Cl Cl F Cl Cl
H F F Cl H

D F C C F < Cl C C F < F C Cl
Cl Cl Cl Cl F

TPJC_2013_8872_01
www.gradtutors.com
732
11

Section B

For each of the questions in this section, one or more of the three numbered statements
1 to 3 may be correct.

Decide whether each of the statements is or is not correct (you may find it helpful to put
a tick against the statements that you consider to be correct).

The responses A to D should be selected on the basis of

A B C D
1, 2 and 3 1 and 2 2 and 3 1 only
are only are only are is
correct correct correct correct

No other combination of statements is used as a correct response.

26 Which physical properties are due to hydrogen bonding between molecules?

1 Water has a higher boiling point than H2S.


2 Ice floats on water.
3 The H–O–H bond angle in water is approximately 104°.

27 In an experiment, 2 moles of SO2 and 3 moles of O2 were allowed to react and


reach equilibrium in a 1 dm3 vessel at two different temperatures.

2SO2(g) + O2(g) 2SO3(g)

The following results were obtained:

temperature/ °C equilibrium yield of SO3/ mol


200 1.2
300 0.8

What can you deduce about the reaction from the results?

1 It is an exothermic reaction.
2 The equilibrium constant at 200 °C is 0.94 mol 1 dm3.−

3 The backward reaction is favoured at higher temperature.

TPJC_2013_8872_01 [Turn over


www.gradtutors.com
733
12

The responses A to D should be selected on the basis of

A B C D
1, 2 and 3 1 and 2 2 and 3 1 only
are only are only are is
correct correct correct correct

No other combination of statements is used as a correct response.

28 Which graph shows the correct trend for the physical property stated?

1  Bond length
in HX

HCl HBr HI

2  
Boiling point
of HX

HCl HBr HI

3  
Boiling point
of X2

Cl2 Br2 I2
2

29 Which of the following compounds form a single organic product when they are
reacted with hot acidified potassium manganate(VII)?

1 CH3CH=CH2
2 (CH3)2C=C(CH3)2
3 HOCH2CHO
 
 
 
 
 
 
 

TPJC_2013_8872_01
www.gradtutors.com
734
13

The responses A to D should be selected on the basis of

A B C D
1, 2 and 3 1 and 2 2 and 3 1 only
are only are only are is
correct correct correct correct

No other combination of statements is used as a correct response.

30 Safranal is a component of the spice saffron.

safranal

Which of the following descriptions about safranal are correct?

1 It reacts with KCN in H2SO4(aq) at 10 to 20 °C.


2 It reacts with Fehling’s solution.
3 It has all atoms lying in the same plane.
 
 
 

TPJC_2013_8872_01 [Turn over


www.gradtutors.com
735
14

BLANK PAGE

TPJC_2013_8872_01
www.gradtutors.com
736

TAMPINES JUNIOR COLLEGE

H JC2 Preliminary Examination

1
CANDIDATE
NAME

TUTOR
CIVICS GROUP 1 2 NAME

CHEMISTRY 8872/ 01
Paper 1 Multiple Choice Monday, 23 September 2013
50 minutes

Additional Materials: Multiple Choice Answer Sheet


Data Booklet

READ THESE INSTRUCTIONS FIRST

Write in soft pencil.


Do not use staples, paper clips, highlighters, glue or correction fluid.
Write your name and civics group on the Answer Sheet in the spaces provided.

There are thirty questions on this paper. Answer all questions. For each question there are four
possible answers A, B, C and D.
Choose the one you consider correct and record your choice in soft pencil on the separate Answer
Sheet.

Read the instructions on the Answer Sheet very carefully.

Each correct answer will score one mark. A mark will not be deducted for a wrong answer.
Any rough working should be done on this paper.

This document consists of 13 printed pages and 1 blank page.

TPJC_2013_8872_01 [Turn over


www.gradtutors.com
737
2

Section A

For each question there are four possible answers, A, B, C and D. Choose the one you
consider to be correct.

1 Use of the Data Booklet is relevant to this question.


How many molecules are present in 1 cm3 of oxygen gas under room conditions?

1× 24000
A
6.02 × 10 23
1× 6.02 × 10 23
B
24000
6.02 × 10 23 × 24000
C
1× 1000
D 1× 6.02 × 10 23 × 32

2 Which of the following ions would undergo the greatest deflection in an electric
field?

16
A O2+ B 16
O18O+ C 16
O18O2+ D 18
O2+

3 10 cm3 of 0.2 mol dm−3 K2XO4 will just react with 40 cm3 of 0.1 mol dm−3
iron(II) sulfate solution.

If Fe2+ is oxidized to Fe3+ by K2XO4, what is the final oxidation state of X?

A +2 B +3 C +4 D +5

4 The table gives the successive ionisation energies for an element X.

1st 2nd 3rd 4th 5th 6th


Ionisation energy/
950 1800 2700 4800 6000 12300
kJ mol−1

What could be the formula of the chloride of X?

A XCl B XCl2 C XCl3 D XCl4

TPJC_2013_8872_01
www.gradtutors.com
738
3

5 Which of the following consists of species which are all planar?

A CO32– , SO32– and benzene


B Al2Cl6 , SOCl2 and methanal
C NO3– , XeF4 and ethanal
D BCl3 , ICl4− and chlorobenzene

6 Dot-and-cross diagrams for carbon monoxide are shown.

Which circled pair of electrons represents a coordinate bond?

A B C D

7 The diagram shows the Boltzmann distribution of the speeds of the molecules of a
gas.
Point X represents the most probable speed.
If the gas is cooled, in which direction does X move?

A
8 Iodine-131 is a radioactive isotope with a half-life of 8 days. Given that radioactive
decay is a first-order reaction, what fraction of the isotope would remain after
80 days?

1 1 1 1
A B C D
20 160 28 210

TPJC_2013_8872_01 [Turn over


www.gradtutors.com
739
4

9 The table shows the charge and radius of each of six ions.

ion J+ L+ M2+ X− Y− Z2−


radius/ nm 0.14 0.18 0.15 0.14 0.18 0.15

The ionic solids JX, LY, and MZ have the same lattice type.

What is the order of their lattice energies starting from the most exothermic first?

A JX, MZ, LY B LY, MZ, JX C MZ, JX, LY D MZ, LY, JX

10 Dinitrogen oxide, N=N=O, burns in ethyne, C2H2, in the gaseous phase to


produce water vapour, carbon dioxide and nitrogen gases as the only products.

5N2O(g) + C2H2(g) → H2O(g) + 2CO2(g) + 5N2(g) ΔH = −1668 kJ mol−1

Assuming N=N bond energy in dinitrogen oxide is +418 kJ mol−1, what is the
nitrogen-oxygen bond energy in dinitrogen oxide in kJ mol−1?

A 382 B 594 C 686 D 1350

11 Which is the correct statement about the following reaction?


M(s)
P(g) + Q(g) R(g) ∆H < 0

A The solid M will lower the activation energy of both forward and backward
reactions.
B Both the rate constant and equilibrium constant will increase with increasing
temperature.
C Increasing temperature will lower the activation energy resulting only in a
greater fraction of R molecules with energy greater than activation energy.
D The activation energy of the forward reaction is equal to the activation energy
of the backward reaction.

12 Phosphorus is an element in the third period, Na to Ar, of the Periodic Table.


What is true for phosphorus and none of the other elements in this period?

A Phosphorus has the highest melting point of the elements in this period.
B Phosphorus is the only element in this period that forms two acidic oxides.
C Phosphorus is the only element in this period with exactly four atoms in its
molecule.
D Phosphorus is the only element in this period whose chlorides react with
water to form acidic solutions.

TPJC_2013_8872_01
www.gradtutors.com
740
5

13 The bar chart shows the melting points of a series of consecutive elements
arranged in order of increasing atomic number. The elements sodium to chlorine
form part of this series.
Which bar represents sodium?

B
14 The chloride of element Q is hydrolysed by water to form an acidic solution and its
oxide reacts with acid to form a salt.
What could be element Q?
A sodium
B aluminium
C silicon
D phosphorus

TPJC_2013_8872_01 [Turn over


www.gradtutors.com
741
6

15 The reaction between NO and F2 was studied by mixing the two gases:

2NO(g) + F2(g) 2NOF(g) ΔH < 0

At different times during the experiment, various changes were made to the
conditions inside the reaction vessel. The changes in concentrations of the three
compounds in the equilibrium mixture with time are given by the graph below:

Which of the following statements is correct?

A There was a decrease in volume of the reaction vessel at 10.0 min.


B There was an increase in volume of the reaction vessel at 7.5 min.
C There was an increase in temperature at 10.0 min.
D A catalyst was added at 7.5 min.

16 A solution of pH 1.3 was produced when 0.10 mol of an acid, U, was dissolved in
2 dm3 of water.

Given the above information, which of the following statements is true?

A The solution contains 0.050 mol of hydrogen ions.


B U is a weak acid with a large Ka value.
C U is a diprotic strong acid.
D U is a monoprotic strong acid.

TPJC_2013_8872_01
www.gradtutors.com
742
7

17 The graph shows the change in pH when ethanoic acid is gradually added to
10 cm3 of 0.10 mol dm−3 sodium hydroxide. An indicator is used to determine the
end-point for the titration.

Which of the following contains the correct concentration of ethanoic acid and
indicator used for the titration?

A 0.05 mol dm−3 ethanoic acid, phenolphthalein


B 0.05 mol dm−3 ethanoic acid, screened methyl orange
C 0.10 mol dm−3 ethanoic acid, phenolphthalein
D 0.10 mol dm−3 ethanoic acid, screened methyl orange

18 What is meant by the term dynamic equilibrium?

A An equilibrium that is constantly changing its position.

B An equilibrium where the forward and reverse reactions are taking place at
different rates.
C An equilibrium where the forward and reverse reactions are taking place at
the same rate.

D An equilibrium which has not yet settled to a constant state.

TPJC_2013_8872_01 [Turn over


www.gradtutors.com
743
8

19 Dibenzalacetone is a bright yellow solid with the following structure.

dibenzalacetone

How many geometric isomers does dibenzalacetone have?

A 2 B 3 C 4 D 5

20 A food chemist wants to create the odour of green apples for a product. An ester
with this odour has the formula C2H5CO2CH(CH3)2.

In which of the following will the substances react together to produce this ester?

A C2H5OH and (CH3)2CHCOOH


B CH3COOH and CH3CH(OH)CH2CH3
C C2H5COOH and C2H5CH2OH
D C2H5COOH and (CH3)2CHOH

21 Which is not a possible product formed when the following compound is heated
with excess ethanolic KOH?
Br

Br

Br

A C

B D

TPJC_2013_8872_01
www.gradtutors.com
744
9

22 2,4–D and MCPA are two common selective weed killers.

Which one of the following reagents can be used to distinguish between them?

A sodium metal
B phosphorus pentachloride
C 2,4-dinitrophenylhydrazine
D hot acidified potassium dichromate(VI)

23 Deuterium, D, is an isotope of hydrogen, 21 H .


Which of the following reactions yields a stable organic compound containing
deuterium?

CH3COOD
C

TPJC_2013_8872_01 [Turn over


www.gradtutors.com
745
10

24 An organic compound K has the following properties:


• 1 mole of K reacts with excess Na(s) to produce one mole of H2(g).
• 1 mole of K reacts with warm alkaline aqueous iodine to produce
one mole of CHI3(s).
Which compound could K be?

A CH3COCH(OH)CH(OH)CH3

B HOCH2CH(OH)CH2COOH
C CH3CO2CH(OH)CH2OH

D CH3CH(OH)CH2COOH

25 Chlorofluorocarbons (CFCs) are commonly used as aerosols, propellants and


refrigerants. However in the stratosphere, CFCs can damage the ozone layer
through a radical chain reaction.

In which sequence are the following compounds listed in increasing order of their
ability to destroy ozone?

H H F F Cl

A F C Cl < F C C F < Cl C C F

F Cl Cl Cl Cl
H F Cl H F

B F C Cl < Cl C C F < F C C F

F Cl Cl Cl Cl
F Cl H H F

C Cl C C F < F C Cl < F C C F
Cl Cl F Cl Cl
H F F Cl H

D F C C F < Cl C C F < F C Cl
Cl Cl Cl Cl F

TPJC_2013_8872_01
www.gradtutors.com
746
11

Section B

For each of the questions in this section, one or more of the three numbered statements
1 to 3 may be correct.

Decide whether each of the statements is or is not correct (you may find it helpful to put
a tick against the statements that you consider to be correct).

The responses A to D should be selected on the basis of

A B C D
1, 2 and 3 1 and 2 2 and 3 1 only
are only are only are is
correct correct correct correct

No other combination of statements is used as a correct response.

26 Which physical properties are due to hydrogen bonding between molecules?

1 Water has a higher boiling point than H2S.


2 Ice floats on water.
3 The H–O–H bond angle in water is approximately 104°.

27 In an experiment, 2 moles of SO2 and 3 moles of O2 were allowed to react and


reach equilibrium in a 1 dm3 vessel at two different temperatures.

2SO2(g) + O2(g) 2SO3(g)

The following results were obtained:

temperature/ °C equilibrium yield of SO3/ mol


200 1.2
300 0.8

What can you deduce about the reaction from the results?

1 It is an exothermic reaction.
2 The equilibrium constant at 200 °C is 0.94 mol 1 dm3.−

3 The backward reaction is favoured at higher temperature.

TPJC_2013_8872_01 [Turn over


www.gradtutors.com
747
12

The responses A to D should be selected on the basis of

A B C D
1, 2 and 3 1 and 2 2 and 3 1 only
are only are only are is
correct correct correct correct

No other combination of statements is used as a correct response.

28 Which graph shows the correct trend for the physical property stated?

1  Bond length
in HX

HCl HBr HI

2 Boiling
  point
of HX

HCl HBr HI

3 Boiling
  point
of X2

Cl2 Br2 I2
2

29 Which of the following compounds form a single organic product when they are
reacted with hot acidified potassium manganate(VII)?

1 CH3CH=CH2
2 (CH3)2C=C(CH3)2
3 HOCH2CHO
   
 
 
 
 
 
 

TPJC_2013_8872_01
www.gradtutors.com
748
13

The responses A to D should be selected on the basis of

A B C D
1, 2 and 3 1 and 2 2 and 3 1 only
are only are only are is
correct correct correct correct

No other combination of statements is used as a correct response.

30 Safranal is a component of the spice saffron.

safranal

Which of the following descriptions about safranal are correct?

1 It reacts with KCN in H2SO4(aq) at 10 to 20 °C.


2 It reacts with Fehling’s solution.
3 It has all atoms lying in the same plane.
 
 
Answer Key

1 B 6 B 11 A 16 D 21 B 26 B
2 A 7 A 12 C 17 A 22 D 27 A
3 C 8 D 13 B 18 C 23 B 28 D
4 C 9 C 14 B 19 B 24 D 29 B
5 D 10 C 15 A 20 D 25 A 30 B

TPJC_2013_8872_01 [Turn over


www.gradtutors.com
749

TAMPINES JUNIOR COLLEGE

H JC2 Preliminary Examination

1
CANDIDATE
NAME

TUTOR
CIVICS GROUP 1 2 NAME

CHEMISTRY 8872/ 02
Paper 2 Thursday, 05 September 2013
2 hours
Candidates answer Section A on the Question Paper.

Additional Materials: Answer Paper


Data Booklet

READ THESE INSTRUCTIONS FIRST

Write your name and civics group on all the work you hand in.
Write in dark blue or black pen.
You may use a pencil for any diagrams, graphs or rough working.
Do not use staples, paper clips, highlighters, glue or correction fluid.

Section A
Answer all the questions.

Section B
Answer two questions on separate answer paper.

At the end of the examination, fasten all your work securely together.
The number of marks is given in brackets [ ] at the end of each question or part question.

For Examiner’s Use

Section A

B5

B6

B7

Total

This document consists of 14 printed pages.

TPJC_2013_8872_02 [Turn over


www.gradtutors.com
750
2

Section A For
Examiner’s
Use
Answer all questions in this section in the spaces provided.

1 Radiotherapy is the medical use of radiation generated from radioactive isotopes to


destroy or weaken malfunctioning cells. Examples of radioactive isotopes used for the
therapy are those of iodine, phosphorus and lutetium.
175 176
(a) Lutetium has two naturally occurring isotopes, Lu and Lu. Their natural
abundances are 97.4% and 2.6% respectively.

(i) Define the term relative atomic mass.

…………...…………..………………………………………………………….……

…………...…………..………………………………………………………….……

…………...…………..………………………………………………………….……

(ii) Calculate, to one decimal place, the relative atomic mass of lutetium.

[2]
176
(b) Lu has a half-life of 3.78 × 10 10 years. The half-life of a radioactive isotope is
the time taken for half of the atoms in a given mass to decay.
176
Calculate the percentage of a sample of Lu isotopes remaining after
1.134 × 1011 years.

[2]

Iodine-131 is used to treat the thyroid for cancers and phosphorus-32 is used to control
the excess of red blood cells produced in the bone marrow.

(c) Complete the table below for the 131I and 32P isotopes.

Isotope Number of protons Number of neutrons


131
I
32
P
[1]
TPJC_2013_8872_02
www.gradtutors.com
751
3

Radioactive isotopes are commonly incorporated into compounds to trace the path of For
Examiner’s
biochemical reactions. These compounds are known as radioactive tracers. Use

The structure of fluorodeoxyglucose (18F-FDG), a radioactive tracer widely used in


medical imaging, is shown below.

OH
O
HO
HO 18
F
OH
18
F-FDG

(d) (i) Apart from ether (–O–), circle and name the functional groups that are
present in the 18F-FDG shown above.

(ii) Calculate the percentage composition by mass of carbon in 18F-FDG.

(iii) Would you expect 18F-FDG to be soluble in water? Explain your answer.

…………...…………..………………………………………………………….……

…………...…………..………………………………………………………….……

TPJC_2013_8872_02 [Turn over


www.gradtutors.com
752
4

(iv) 18 For
F-FDG is heated under reflux with an excess of the following isotopically Examiner’s
labelled carboxylic acid in the presence of concentrated sulfuric acid. Use

18
O H
H3C C
O

Give the structural formula of the organic product formed and state the type
of reaction that has occurred. You may assume that the ether group is inert.

[7]

[Total: 12]

2 In an alkaline fuel cell, the chemical energy from the hydrogen fuel supplied to one
electrode is converted into electricity through a chemical reaction with the oxygen
supplied to the other electrode. These two electrodes are connected using potassium
hydroxide as an electrolyte. A simplified diagram of the fuel cell is shown below.

Load

H2 O2

KOH

product

electrodes

The two half-equations for this cell are

2H2O + 2e− H2 + 2OH−

O2 + 2H2O + 4e− 4OH−

TPJC_2013_8872_02
www.gradtutors.com
753
5

(a) (i) Combine these two half-equations to show the overall reaction occurring in For
Examiner’s
the cell. Use

(ii) Use oxidation numbers to show which species in your equation is reduced
and which is oxidised.

[4]

Porous graphite impregnated with suitable catalysts could be used as electrodes for an
alkaline fuel cell.

(b) (i) Describe the structure of, and the bonding in, the element graphite.
Draw a diagram to illustrate your answer.

…………...…………..………………………………………………………….……

…………...…………..………………………………………………………….……

…………...…………..………………………………………………………….……

…………...…………..………………………………………………………….……

…………...…………..………………………………………………………….……

…………...…………..………………………………………………………….……
TPJC_2013_8872_02 [Turn over
www.gradtutors.com
754
6

(ii) State a physical property of graphite that metals also possess. For
Examiner’s
Explain, in terms of the bonding present, why it possesses this property. Use

property …………………………………………………………………………..

explanation ………………………………………………………………….……

…………………………………………………………………………...….……..[4]

[Total: 8]

3 This question is about period three elements and their compounds.

(a) (i) Sketch on the axes provided, the trend in first ionisation energy across
period three.

first
ionisation
energy
−1
/ kJ mol

Na Mg Al Si P S Cl Ar

(ii) Explain the general trend in first ionisation energy of period three elements.

…………...…………..………………………………………………………….……

…………...…………..………………………………………………………….……

…………...…………..………………………………………………………….……

…………...…………..………………………………………………………….……

(iii) Explain the difference between the values of the first ionisation energies of
phosphorus and sulfur.

…………...…………..………………………………………………………….……

…………...…………..………………………………………………………….……

…………...…………..………………………….……………………..………….[3]

TPJC_2013_8872_02
www.gradtutors.com
755
7

(b) Sulfuryl chloride, SO2Cl2, decomposes as follows when heated to 100 °C. For
Examiner’s
Use
SO2Cl2(g) SO2(g) + Cl2(g)

(i) Calculate the equilibrium constant, Kc, at 100 °C, given the following values:
[SO2Cl2] = 14.6 g dm−3,
[SO2] = 3.33 g dm−3,
[Cl2] = 11.5 g dm−3.

(ii) Draw a dot-and-cross diagram for sulfuryl chloride.

(iii) Complete the electronic configuration of a chlorine atom.

1s2 ………………………………………………..

Hence describe the bonding in the Cl2 molecule in terms of orbital overlap.
Include a diagram in your answer.

[7]

[Total: 10]

TPJC_2013_8872_02 [Turn over


www.gradtutors.com
756
8

4 An airbag is a vehicle safety device which is designed to inflate rapidly during an For
Examiner’s
automobile collision. Its purpose is to cushion occupants during a crash and provide Use
protection to their bodies when they strike interior objects such as the steering wheel or
a window.

An airbag typically contains a mixture of sodium azide (NaN3), potassium


nitrate (KNO3) and silicon dioxide (SiO2). Within about 40 milliseconds of impact, all
these components react in three separate reactions as described in order below.

The first reaction is the decomposition of sodium azide to produce sodium metal and
nitrogen gas. The equation is as follows.

2NaN3 → 2Na + 3N2

The highly reactive sodium metal formed then reacts with potassium nitrate to produce
more nitrogen gas according to the following equation.

10Na + 2KNO3 → K2O + 5Na2O + N2

The third reaction involves the removal of K2O and Na2O by silicon dioxide to produce
the metal silicates, K2SiO3 and Na2SiO3.

The airbag is inflated by the nitrogen gas produced in the first and second reactions.

(a) (i) Calculate the amount, in moles, of sodium metal and of nitrogen gas formed
when 110 g of sodium azide in an airbag is decomposed.

(ii) Hence calculate the number of moles of nitrogen gas produced in the
second reaction.

(iii) Using your answers to (i) and (ii), calculate the total volume of nitrogen gas
contained in the airbag at room temperature and pressure.

(iv) State an assumption you have made in the calculations above.

…………...…………..………………………………………………………….……
TPJC_2013_8872_02
www.gradtutors.com
757
9

(v) Construct a balanced equation for the third reaction. For


Examiner’s
Use
………………..………………………………………………..…………….……[6]

Sodium azide is also used in the following synthesis of undecyl isocyanate from lauroyl
chloride.
O O
NaN3 warm
CH3(CH2)9CH2 C CH3(CH2)9CH2 C CH3(CH2)9CH2 N C O
Cl N3
lauroyl chloride lauroyl azide undecyl isocyanate

(b) (i) Suggest the identity of the gas evolved in the conversion of lauroyl azide to
undecyl isocyanate.

…………...…………..………………………………………………………….……

(ii) Lauroyl chloride can be obtained from the reaction between lauric acid,
CH3(CH2)9CH2CO2H, and thionyl chloride, SOCl2.

Give the structural formula of the organic product formed when lauric acid is
treated with lithium aluminium hydride.

[2]

(c) In aqueous solutions, azide ions partially ionise as shown by the following
equilibrium.
N3 + H2O

HN3 + OH −

(i) Is azide ion behaving as an acid or a base? Explain your answer using the
Brønsted-Lowry theory of acids and bases.

………………...………………………………………………………………………

………………...………………………………………………………………………

(ii) The structure of the azide ion is given below.


+
N N N N

State the shape of the ion around the central N atom.

………………...……………………………………………………………………[2]

[Total: 10]

TPJC_2013_8872_02 [Turn over


www.gradtutors.com
758
10

Section B

Answer two questions from this section on separate answer paper.

5 (a) (i) Describe what is observed during the combustion in oxygen of magnesium,
writing an equation for the reaction.

(ii) Write an equation, with state symbols, for the reaction of sodium oxide with
water. Hence calculate the pH of the resulting solution when 0.35 g of sodium
oxide is added to 1 dm3 of water.
[6]

(b) An important step in the contact process is the conversion of sulfur dioxide to sulfur
trioxide as shown below.

2SO2(g) + O2(g) 2SO3(g) ΔH = −196 kJ mol−1

Industrially, this reaction is carried out at 450 °C and at a pressure of 1-2 atm in the
presence of vanadium(V) oxide, V2O5.

(i) State Le Chatelier’s Principle.

(ii) Would the production of sulfur trioxide be favoured by a high or low


temperature? Explain your reasoning.

Hence comment on the significance of the operating temperature.

(iii) Suggest the role of vanadium(V) oxide in the above reaction.

(iv) State the shape around the central atom of sulfur dioxide and suggest a value
for its bond angle.
[7]

TPJC_2013_8872_02
www.gradtutors.com
759
11

(c) The following organic reactions (I, II and III) involve the use of concentrated sulfuric
acid.
I II H2O
(CH3)2C=CH(CH3) (CH3)2C(OSO3H)CH2CH3 C
warm
B

CH(CH3)2CH(OH)CH3
A

III
CH(CH3)2CH(OCOCH3)CH3

(i) State the conditions required for reactions I and II.

(ii) Suggest the structural formula of an organic impurity that might be found in the
product of reaction I.

(iii) Name the organic reagent used in reaction III.

(iv) Give the displayed formula of the organic compound C.

(v) Draw the structural isomer of B that exhibits cis-trans isomerism.

(vi) Give the structure of the organic product formed when A is heated under reflux
with a mixture of concentrated sulfuric acid and NaBr.
[7]

[Total: 20]

TPJC_2013_8872_02 [Turn over


www.gradtutors.com
760
12

6 (a) (i) Describe one physical property of the chlorides of sodium and phosphorus and
the reactions, if any, of these chlorides with water. Give equations where
appropriate.

(ii) Relate the structures of, and bonding in, these chlorides to their physical
property and reaction with water.
[5]

(b) The lattice energy of magnesium chloride can be determined using the energy cycle
given below.
Δ H1
Mg(s) + Cl2(g) MgCl2(s)
Δ H2 Δ H3

Mg(g) + 2Cl(g)

Δ H4 Δ H5
Δ H6
Mg2+(g) + 2Cl−(g)

(i) Explain what is meant by the term lattice energy of magnesium chloride.

(ii) Name the enthalpy change represented as ΔH1.

(iii) Use relevant bond energy and ionisation energy values from the Data Booklet
to determine the values of ΔH3 and ΔH4.

(iv) Hence calculate the lattice energy of magnesium chloride given the following
data.

ΔH1 = −642 kJ mol−1


ΔH2 = +148 kJ mol−1
ΔH5 = −698 kJ mol−1
[5]

(c) J is an aromatic compound with the molecular formula C8H10O. When J is heated with
acidified potassium dichromate(VI), the orange solution turned green and K is
produced. When warmed with alkaline aqueous iodine, both J and K gave pale yellow
precipitate. Treatment of K with HCN and a trace amount of NaCN produced L.
Heating L under reflux with H2SO4(aq) formed M which liberates carbon dioxide gas
from NaHCO3.

Draw a structural formula for each of the organic compounds J – M and write
equations where appropriate to show the reactions that are occurring. Clearly show
the deductions that you make from the information that you have been given: full
marks cannot be gained by only giving the structures required.
[10]

[Total: 20]

TPJC_2013_8872_02
www.gradtutors.com
761
13

7 (a) Consider the following oxides and chlorides of period three elements.

Al2O3, AlCl3, SiO2, SiCl4, P4O10, SO2

(i) Identify, with reason(s), one compound above which is insoluble in water.

(ii) White fumes were produced when compound X was added to water and a red
solution was formed on adding Universal Indicator to the resulting solution.
Identify X from the compounds above and account for the observations.
Include suitable equation(s) in your answer.

(iii) One of the oxides above is amphoteric in nature. Illustrate the property of this
oxide by the use of relevant equations.

(iv) Identify the element having the highest oxidation state in the compounds above
and write its oxidation number.
[6]

(b) The following experimental procedure was carried out to determine the mass of
calcium carbonate (a weak base) in a sample of chalk powder.

25.0 cm3 of 0.160 mol dm−3 HCl(aq) was added in excess to 0.12 g of chalk powder in
a 250 cm3 conical flask. The resulting solution in the conical flask was then titrated
with 0.100 mol dm−3 NaOH(aq). 21.40 cm3 of NaOH(aq) was required to reach the
end-point using phenolphthalein as indicator.

(i) Calculate the number of moles of HCl neutralised by NaOH.

(ii) Calculate the initial number of moles of HCl added to the chalk powder and
hence determine the number of moles of HCl that reacted with CaCO3 found in
the sample.

(iii) Construct a balanced equation, with state symbols, for the reaction between
HCl and CaCO3.

Hence find the mass of CaCO3 present in the sample using your answer to
(b)(ii).
[4]

TPJC_2013_8872_02 [Turn over


www.gradtutors.com
762
14

(c) Consider the following equilibrium.

H2(g) + I2(g) 2HI(g)

(i) Using suitable values from the Data Booklet, calculate the enthalpy change of
the reaction.

(ii) State and explain the effect of a catalyst on the equilibrium composition.

(iii) On the same axes, sketch the energy profile diagram for the reaction when it is
uncatalysed and catalysed respectively.
[6]

(d) Describe, with the aid of equations, the role of the H2CO3/HCO3− buffer system in
controlling the pH of blood.
[2]

(e) Describe a simple chemical test to distinguish the pair of compounds below.

I and Br

[2]

[Total: 20]

TPJC_2013_8872_02
www.gradtutors.com
763

TAMPINES JUNIOR COLLEGE

H JC2 Preliminary Examination

1
CANDIDATE
NAME

TUTOR
CIVICS GROUP 1 2 NAME

CHEMISTRY 8872/ 02
Paper 2 Thursday, 05 September 2013
2 hours
Candidates answer Section A on the Question Paper.

Additional Materials: Answer Paper


Data Booklet

READ THESE INSTRUCTIONS FIRST

Write your name and civics group on all the work you hand in.
Write in dark blue or black pen.
You may use a pencil for any diagrams, graphs or rough working.
Do not use staples, paper clips, highlighters, glue or correction fluid.

Section A
Answer all the questions.

Section B
Answer two questions on separate answer paper.

At the end of the examination, fasten all your work securely together.
The number of marks is given in brackets [ ] at the end of each question or part question.

For Examiner’s Use

Section A

B5

B6

B7

Total

This document consists of 18 printed pages.

TPJC_2013_8872_02 [Turn over


www.gradtutors.com
764
2

Section A For
Examiner’s
Use
Answer all questions in this section in the spaces provided.

1 Radiotherapy is the medical use of radiation generated from radioactive isotopes to


destroy or weaken malfunctioning cells. Examples of radioactive isotopes used for the
therapy are those of iodine, phosphorus and lutetium.
175 176
(a) Lutetium has two naturally occurring isotopes, Lu and Lu. Their natural
abundances are 97.4% and 2.6% respectively.

(i) Define the term relative atomic mass.


Relative atomic mass is defined as the ratio of the average mass of one
1
atom of an element to the mass of an atom of 12C isotope, expressed on [1]
12
the 12C scale.

(ii) Calculate, to one decimal place, the relative atomic mass of lutetium.
97.4 × 175 + 2.6 × 176
Ar of Lu = = 175.0 [1]
100
[2]

176
(b) Lu has a half-life of 3.78 × 10 10 years. The half-life of a radioactive isotope is
the time taken for half of the atoms in a given mass to decay.
176
Calculate the percentage of a sample of Lu isotopes remaining after
1.134 × 1011 years.
1.134 × 1011
number of half-lives = =3
3.78 × 1010
3 [2]
⎛ 1 ⎞
% of sample remaining = 100 × ⎜ ⎟ = 12.5 %
⎝ 2 ⎠
[2]

Iodine-131 is used to treat the thyroid for cancers and phosphorus-32 is used to control
the excess of red blood cells produced in the bone marrow.

(c) Complete the table below for the 131I and 32P isotopes.

Isotope Number of protons Number of neutrons


131
I 53 78
[1]
32
P 15 17
[1]

www.gradtutors.com
765
3

Radioactive isotopes are commonly incorporated into compounds to trace the path of
biochemical reactions. These compounds are known as radioactive tracers.

The structure of fluorodeoxyglucose (18F-FDG), a radioactive tracer widely used in


medical imaging, is shown below.
OH
O
HO
HO 18
F
OH
18
F-FDG

(d) (i) Apart from ether (–O–), circle and name the functional groups that are
present in the 18F-FDG shown above.
primary alcohol
OH

secondary O
alcohol HO [2]
HO 18
F secondary
secondary secondary OH alcohol
alcohol fluoroalkane

(ii) Calculate the percentage composition by mass of carbon in 18F-FDG.


Mr of 18F-FDG = 6 × 12.0 + 5 × 16.0 + 1× 18 + 11× 1.0 = 181
6 × 12.0 [2]
% composition by mass of C in 18F-FDG = × 100% = 39.8 %
181

(iii) Would you expect 18F-FDG to be soluble in water? Explain your answer.
Yes, 18F-FDG is expected to be soluble in water because it has many –OH
[1]
groups which are capable of forming hydrogen bonds with water.

(iv) 18
F-FDG is heated under reflux with an excess of the following isotopically
labelled carboxylic acid in the presence of concentrated sulfuric acid.

18
O H
H3C C
O

Give the structural formula of the organic product formed and state the type
of reaction that has occurred. You may assume that the ether group is inert.
[7]

www.gradtutors.com
766
4

O
O
O
O
O 18
[2]
F
O O

O
Reaction Type: Condensation (or addition-elimination)
[Total: 12]

2 In an alkaline fuel cell, the chemical energy from the hydrogen fuel supplied to one
electrode is converted into electricity through a chemical reaction with the oxygen
supplied to the other electrode. These two electrodes are connected using potassium
hydroxide as an electrolyte. A simplified diagram of the fuel cell is shown below.

Load

H2 O2

KOH

product

electrodes

The two half-equations for this cell are

2H2O + 2e− H2 + 2OH−

O2 + 2H2O + 4e− 4OH−

(a) (i) Combine these two half-equations to show the overall reaction occurring in
the cell.
Oxidation: H2 + 2OH− → 2H2O + 2e− (×2)
Reduction: O2 + 2H2O + 4e− → 4OH− (×1) [2]
Overall: 2H2 + O2 → 2H2O

(ii) Use oxidation numbers to show which species in your equation is reduced
and which is oxidised.
H2 is oxidised as the oxidation number of H increases from 0 in H2 to +1 in
H2O.
[2]
O2 is reduced as the oxidation number of O decreases from 0 in O2 to −2 in
H2O.
[4]

www.gradtutors.com
767
5

Porous graphite impregnated with suitable catalysts could be used as electrodes for an
alkaline fuel cell.

(b) (i) Describe the structure of, and the bonding in, the element graphite. Draw a
diagram to illustrate your answer.

[3]

Description:
• Graphite has a giant molecular layered structure
• Within each layer, each C atom uses three out of its four valence
electrons to form covalent bonds with three other C atoms in a trigonal
planar arrangement to form hexagonal rings
• The 4th valence electron is delocalized over the whole layer
• The layers are held together by weak instantaneous dipole – induced
dipoles attractions

(ii) State a physical property of graphite that metals also possess.


Explain, in terms of the bonding present, why it possesses this property.

Property Conducts electricity in the solid state.


[1]
explanation The delocalised electrons along the graphite layers can act as
mobile charge carriers to conduct electricity.
[4]

[Total: 8]

www.gradtutors.com
768
6

3 This question is about period three elements and their compounds.

(a) (i) Sketch on the axes provided, the trend in first ionisation energy across
period three.

first
ionisation
energy
−1
/ kJ mol

Na Mg Al Si P S Cl Ar

[1]

(ii) Explain the general trend in first ionisation energy of period three elements.
Across the period, nuclear charge increases while screening effect remains
relatively constant. Thus effective nuclear charge increases and the valence
[1]
electrons are more strongly attracted by the nucleus. Hence more energy is
required to remove an electron and ionisation energy increases.

(iii) Explain the difference between the values of the first ionisation energies of
phosphorus and sulfur.
The 3p electron to be removed from S is paired and experiences
inter-electronic repulsion whereas the 3p electron to be removed from P is
[1]
unpaired. Thus less energy is required to remove the 3p electron from S
and S has a lower ionisation energy than P.
[3]

(b) Sulfuryl chloride, SO2Cl2, decomposes as follows when heated to 100 °C.
www.gradtutors.com
769
7

SO2Cl2(g) SO2(g) + Cl2(g)

(i) Calculate the equilibrium constant, Kc, at 100 °C, given the following values:
[SO2Cl2] = 14.6 g dm−3,
[SO2] = 3.33 g dm−3,
[Cl2] = 11.5 g dm−3.

14.6 14.6
[SO2Cl2] = = = 0.108 mol dm −3
32.1 + 2 × 16.0 + 2 × 35.5 135.1
3.33 3.33
[SO2] = = = 0.0520 mol dm −3
32.1 + 2 × 16.0 64.1
[3]
11.5 11.5
[Cl2] = = = 0.162 mol dm −3
2 × 35.5 71
[SO2 ][Cl 2 ] (0.0520 )(0.162)
Kc = = = 0.0780 mol dm −3
[SO2Cl2 ] (0.108)
(ii) Draw a dot-and-cross diagram for sulfuryl chloride.

[1]

(iii) Complete the electronic configuration of a chlorine atom.

1s2 2s22p63s23p5 [1]

Hence describe the bonding in the Cl2 molecule in terms of orbital overlap.
Include a diagram in your answer.

In the Cl2 molecule, the two Cl atoms are covalently bonded via a sigma
bond which is formed from the head-on overlap of 3p orbitals.

+ [2]

3p orbital 3p orbital head-on overlap


of Cl atom of Cl atom to form sigma bond

[7]

[Total: 10]

4 An airbag is a vehicle safety device which is designed to inflate rapidly during an


www.gradtutors.com
770
8

automobile collision. Its purpose is to cushion occupants during a crash and provide
protection to their bodies when they strike interior objects such as the steering wheel or
a window.

An airbag typically contains a mixture of sodium azide (NaN3), potassium


nitrate (KNO3) and silicon dioxide (SiO2). Within about 40 milliseconds of impact, all
these components react in three separate reactions as described in order below.

The first reaction is the decomposition of sodium azide to produce sodium metal and
nitrogen gas. The equation is as follows.

2NaN3 → 2Na + 3N2

The highly reactive sodium metal formed then reacts with potassium nitrate to produce
more nitrogen gas according to the following equation.

10Na + 2KNO3 → K2O + 5Na2O + N2

The third reaction involves the removal of K2O and Na2O by silicon dioxide to produce
the metal silicates, K2SiO3 and Na2SiO3.

The airbag is inflated by the nitrogen gas produced in the first and second reactions.

(a) (i) Calculate the amount, in moles, of sodium metal and of nitrogen gas formed
when 110 g of sodium azide in an airbag is decomposed.
110 110
η (NaN3 ) = = = 1.69 mol
23.0 + 3 × 14.0 65.0
Na ≡ NaN3
η(Na) = η(NaN3 ) = 1.69 mol [2]
3N2 ≡ 2NaN3
3 3
η (N2 ) = × η (NaN3 ) = × 1.69 = 2.54 mol
2 2

(ii) Hence calculate the number of moles of nitrogen gas produced in the
second reaction.
10Na ≡ N2
1 1 [1]
η (N2 ) = × η (Na) = × 1.69 = 0.169 mol
10 10

(iii) Using your answers to (i) and (ii), calculate the total volume of nitrogen gas
contained in the airbag at room temperature and pressure.
Total volume of N2 = (2.54 + 0.169)× 24 = 65.0 dm3 [1]

(iv) State an assumption you have made in the calculations above.


Sodium azide was completely decomposed/ No loss of nitrogen gas from
[1]
the airbag

(v) Construct a balanced equation for the third reaction.


www.gradtutors.com
771
9

K2O + Na2O + 2SiO2 → K2SiO3 + Na2SiO3 [1]


[6]

Sodium azide is also used in the following synthesis of undecyl isocyanate from lauroyl
chloride.

O O
NaN3 warm
CH3(CH2)9CH2 C CH3(CH2)9CH2 C CH3(CH2)9CH2 N C O
Cl N3
lauroyl chloride lauroyl azide undecyl isocyanate

(b) (i) Suggest the identity of the gas evolved in the conversion of lauroyl azide to
undecyl isocyanate.
N2 or nitrogen [1]

(ii) Lauroyl chloride can be obtained from the reaction between lauric acid,
CH3(CH2)9CH2CO2H, and thionyl chloride, SOCl2.

Give the structural formula of the organic product formed when lauric acid is
treated with lithium aluminium hydride.
CH3(CH2)9CH2CH2OH [1]
[2]

(c) In aqueous solutions, azide ions partially ionise as shown by the following
equilibrium.
N3 + H2O

HN3 + OH −

(i) Is azide ion behaving as an acid or a base? Explain your answer using the
Brønsted-Lowry theory of acids and bases.
Azide ion is a base because it accepts a proton from H2O. [1]

(ii) The structure of the azide ion is given below.


+
N N N N

State the shape of the ion around the central N atom.


Linear [1]
[2]

[Total: 10]

www.gradtutors.com
772
10

Section B

Answer two questions from this section on separate answer paper.

5 (a) (i) Describe what is observed during the combustion in oxygen of magnesium,
writing an equation for the reaction.
Mg burns with blinding white light.
2Mg + O2 → 2MgO

(ii) Write an equation, with state symbols, for the reaction of sodium oxide with
water. Hence calculate the pH of the resulting solution when 0.35 g of sodium
oxide is added to 1 dm3 of water.
Na2O(s) + H2O(l) → 2NaOH(aq)
0.35 0.35
η (Na2O) = = = 5.65 × 10 −3 mol
2 × 23.0 + 16.0 62.0
2NaOH ≡ Na2O
η(NaOH) = 2 × η(Na2O) = 2 × 5.65 × 10 −3 = 0.0113 mol
[OH− ] = [NaOH] = 0.0113 mol dm −3
pOH = − lg [OH− ] = − lg (0.0113) = 1.95
pH = 14 − pOH = 14 − 1.95 = 12.05
[6]

(b) An important step in the contact process is the conversion of sulfur dioxide to sulfur
trioxide as shown below.

2SO2(g) + O2(g) 2SO3(g) ΔH = −196 kJ mol−1

Industrially, this reaction is carried out at 450 °C and at a pressure of 1-2 atm in the
presence of vanadium(V) oxide, V2O5.

(i) State Le Chatelier’s Principle.


Le Chatelier’s Principle states that when a system at equilibrium is subjected to
a change (disturbance), the system responds in such a way as to counteract
the imposed change and re-establish the equilibrium state.

(ii) Would the production of sulfur trioxide be favoured by a high or low


temperature? Explain your reasoning.

Hence comment on the significance of the operating temperature.


The production of sulfur trioxide would be favoured by a low temperature. By
Le Chatelier’s principle, a low temperature will favour the forward reaction
which is exothermic to increase the temperature. Thus the position of
equilibrium shifts to the right and more sulfur trioxide will be produced.

A moderate operating temperature of 450 °C was used instead of a lower


temperature so that the rate of reaction will not be too slow.

www.gradtutors.com
773
11

(iii) Suggest the role of vanadium(V) oxide in the above reaction.


Vanadium(V) oxide acts as a catalyst for the reaction.

(iv) State the shape around the central atom of sulfur dioxide and suggest a value
for its bond angle.
Shape: bent
Bond angle: any value between 110° and 119° inclusive
[7]

(c) The following organic reactions (I, II and III) involve the use of concentrated sulfuric
acid.
I II H2O
(CH3)2C=CH(CH3) (CH3)2C(OSO3H)CH2CH3 C
warm
B

CH(CH3)2CH(OH)CH3
A

III
CH(CH3)2CH(OCOCH3)CH3

(i) State the conditions required for reactions I and II.


reaction I: excess concentrated sulfuric acid, 170 °C
reaction II: cold

(ii) Suggest the structural formula of an organic impurity that might be found in the
product of reaction I.
H H H
H3C C C C H
CH3

(iii) Name the organic reagent used in reaction III.


ethanoic acid

(iv) Give the displayed formula of the organic compound C.


H
H C H
H H H
H C C C C H
H O H H
H

(v) Draw the structural isomer of B that exhibits cis-trans isomerism.


H H
H3C C C CH2CH3

www.gradtutors.com
774
12

(vi) Give the structure of the organic product formed when A is heated under reflux
with a mixture of concentrated sulfuric acid and NaBr.
H H
H3C C C CH3
CH3 Br
[7]

[Total: 20]

6 (a) (i) Describe one physical property of the chlorides of sodium and phosphorus and
the reactions, if any, of these chlorides with water. Give equations where
appropriate.
Physical Property of chlorides:
• NaCl has high melting and boiling points while PCl3 / PCl5 has low melting
and boiling points OR
• Both NaCl and PCl3 / PCl5 do not conduct electricity in the solid state.

Reactions of chlorides with water:


• NaCl(s) dissolves in water to form a neutral solution.
NaCl(s) + aq → NaCl(aq)

• PCl3 / PCl5 reacts with water to form white fumes (of HCl) and a strongly
acidic solution.
PCl3 + 3H2O → H3PO3 + 3HCl or PCl5 + 4H2O → H3PO4 + 5HCl

(ii) Relate the structures of, and bonding in, these chlorides to their physical
property and reaction with water.

Physical Property of chlorides:


• NaCl has high melting and boiling points while PCl3 / PCl5 has low melting and
boiling points.
This is because NaCl has a giant ionic structure with strong ionic bonds between
oppositely charged Na+ and Cl− ions whereas PCl3 / PCl5 has a simple molecular
structure with weak van der Waals’ forces between its molecules.
More energy is required to overcome the strong ionic bonds than the weak van der
Waals’ forces during melting and boiling.
OR
• Both NaCl and PCl3 / PCl5 do not conduct electricity in the solid state.
This is due to the absence of mobile ions and delocalised electrons in NaCl(s) and
PCl3 / PCl5 to act as mobile charge carriers to conduct electricity.

Reactions of chlorides with water:


• NaCl(s) has strong electrostatic forces of attraction between the Na+ and Cl− ions.
It dissolves in water to give Na+(aq) and Cl−(aq) ions.
• PCl3 / PCl5 has weak van der Waals’ forces between its molecules. It is
hydrolysed by water.
[5]

www.gradtutors.com
775
13

(b) The lattice energy of magnesium chloride can be determined using the energy cycle
given below.

Δ H1
Mg(s) + Cl2(g) MgCl2(s)
Δ H2 Δ H3

Mg(g) + 2Cl(g)

Δ H4 Δ H5
Δ H6
Mg2+(g) + 2Cl−(g)

(i) Explain what is meant by the term lattice energy of magnesium chloride.
It is the heat evolved when one mole of solid magnesium chloride is formed
from its constituent gaseous Mg2+ and Cl− ions under standard conditions of
298 K and 1 atm.

(ii) Name the enthalpy change represented as ΔH1.


Enthalpy change of formation of MgCl2(s)

(iii) Use relevant bond energy and ionisation energy values from the Data Booklet
to determine the values of ΔH3 and ΔH4.
ΔH3 = bond energy of chlorine = +244 kJ mol−1

ΔH4 = sum of 1st and 2nd ionisation energies of magnesium


= 736 + 1450 = +2186 kJ mol−1

(iv) Hence calculate the lattice energy of magnesium chloride given the following
data.

ΔH1 = −642 kJ mol−1


ΔH2 = +148 kJ mol−1
ΔH5 = −698 kJ mol−1
By Hess’ law,
Δ H1 = Δ H2 + Δ H3 + Δ H4 + Δ H5 + Δ H6

Lattice energy of magnesium chloride


= Δ H6
= ΔH1 – (ΔH2 + ΔH3 + ΔH4 + ΔH5)
= −642 – (148 + 244 + 2186 −698) = –2522 ≈ −2520 kJ mol−1
[5]

www.gradtutors.com
776
14

(c) J is an aromatic compound with the molecular formula C8H10O. When J is heated with
acidified potassium dichromate(VI), the orange solution turned green and K is
produced. When warmed with alkaline aqueous iodine, both J and K gave pale yellow
precipitate. Treatment of K with HCN and a trace amount of NaCN produced L.
Heating L under reflux with H2SO4(aq) formed M which liberates carbon dioxide gas
from NaHCO3.

Draw a structural formula for each of the organic compounds J – M and write
equations where appropriate to show the reactions that are occurring. Clearly show
the deductions that you make from the information that you have been given: full
marks cannot be gained by only giving the structures required.
• J is an aromatic compound with the molecular formula C8H10O
⇒ J contains a benzene ring

• J undergoes oxidation with alkaline aqueous iodine to give pale yellow ppt
⇒ J contains the –CH(OH)CH3 group
⇒ J has the structure shown below
⇒ J contains a secondary alcohol functional group

OH
H C CH3

Equation:
OH O O
H C CH3 C

_
+ 4 I2 + 6 OH + CHI3 + 5I + 5 H2O
J

• J undergoes oxidation with K2Cr2O7/H+ to produce K which undergoes oxidation


with alkaline aqueous iodine to give pale yellow ppt
⇒ K has a ketone functional group and contains the –COCH3 group
⇒ K has the structure shown below.

O CH3
C

www.gradtutors.com
777
15

Equations:
OH O CH3
H C CH3 C

+ [O] + H2O

J K

O O O
CH3
C C

_
+ 3 I2 + 4 OH + CHI3 + 3I + 3 H2O

• K undergoes nucleophilic addition with HCN to produce L


⇒ L is a cyanohydrin which has the structure shown below.

OH
H3C C CN

Equation:
O OH
CH3
C H3C C CN

+ HCN

K L

• L undergoes acid hydrolysis with H2SO4(aq) to produce M which undergoes


acid-base reaction with NaHCO3 to liberate carbon dioxide gas.
⇒ M contains a carboxylic acid functional group and has the structure shown
below.
OH
H 3C C COOH

M
www.gradtutors.com
778
16

Equations:
OH OH
H3C C CN H3C C COOH

+ +
+ H + 2H2O + NH4

L M

OH OH
+
H3C C COOH H3C C COO Na

+ NaHCO3 + CO2 + H2O

M
[10]

[Total: 20]

7 (a) Consider the following oxides and chlorides of period three elements.

Al2O3, AlCl3, SiO2, SiCl4, P4O10, SO2

(i) Identify, with reason(s), one compound above which is insoluble in water.
• Al2O3; it has high lattice energy which requires a large amount of energy to
overcome OR
• SiO2; it has a giant molecular structure with numerous strong covalent
bonds which require a lot of energy to break

(ii) White fumes were produced when compound X was added to water and a red
solution was formed on adding Universal Indicator to the resulting solution.
Identify X from the compounds above and account for the observations.
Include suitable equation(s) in your answer.
X is SiCl4.
• The white fumes observed on adding X to water were due to the HCl(g)
evolved.
SiCl4 + 2H2O → SiO2 + 4HCl
• The HCl(g) produced readily ionises in water to give H+(aq) ions, making
the solution strongly acidic. Thus the universal indicator turns red.
HCl(g) + aq → HCl(aq)

(iii) One of the oxides above is amphoteric in nature. Illustrate the property of this
oxide by the use of relevant equations.
• Al2O3 is the amphoteric oxide.
• Equations:
Al2O3 + 6H+ → 2Al3+ + 3H2O
Al2O3 + 2OH− + 3H2O → 2[Al(OH)4]−

www.gradtutors.com
779
17

(iv) Identify the element having the highest oxidation state in the compounds above
and write its oxidation number.
Phosphorus.
Oxidation number of P in P4O10 is +5.
[6]

(b) The following experimental procedure was carried out to determine the mass of
calcium carbonate (a weak base) in a sample of chalk powder.

25.0 cm3 of 0.160 mol dm−3 HCl(aq) was added in excess to 0.12 g of chalk powder in
a 250 cm3 conical flask. The resulting solution in the conical flask was then titrated
with 0.100 mol dm−3 NaOH(aq). 21.40 cm3 of NaOH(aq) was required to reach the
end-point using phenolphthalein as indicator.

(i) Calculate the number of moles of HCl neutralised by NaOH.


HCl ≡ NaOH
21.40
η (HCl) neutralised = η (NaOH) = × 0.100 = 0.00214 mol
1000

(ii) Calculate the initial number of moles of HCl added to the chalk powder and
hence determine the number of moles of HCl that reacted with CaCO3 found in
the sample.
25.0
Initial η (HCl) = × 0.160 = 0.00400 mol
1000
η (HCl) that reacted with CaCO3 = 0.00400 − 0.00214 = 0.00186 mol

(iii) Construct a balanced equation, with state symbols, for the reaction between
HCl and CaCO3.

Hence find the mass of CaCO3 present in the sample using your answer to
(b)(ii).
2HCl(aq) + CaCO3(s) → CaCl2(aq) + CO2(g) + H2O(l)
CaCO 3 ≡ 2HCl
1 1
η (CaCO3 ) = × η (HCl) = × 0.00186 = 9.30 × 10 −4 mol
2 2
( )
mass of CaCO3 = 9.30 × 10 −4 (40.1 + 12.0 + 3 × 16.0) = 0.0931 g
[4]

(c) Consider the following equilibrium.

H2(g) + I2(g) 2HI(g)

(i) Using suitable values from the Data Booklet, calculate the enthalpy change of
the reaction.
ΔH = ∑ BE(bonds broken) − ∑ BE(bonds formed)
= (1× BE(H − H) + 1× BE(I − I)) − (2 × BE(H − I))
= (436 + 151) − (2 × 299)
= −11.0 kJ mol −1

www.gradtutors.com
780
18

(ii) State and explain the effect of a catalyst on the equilibrium composition.
The equilibrium composition remains unchanged in the presence of a catalyst.
This is because both the rates of the forward and backward reactions will
increase to the same extent.

(iii) On the same axes, sketch the energy profile diagram for the reaction when it is
uncatalysed and catalysed respectively.

[6]

(d) Describe, with the aid of equations, the role of the H2CO3/HCO3− buffer system in
controlling the pH of blood.
When a small amount of acid is added, it is removed by HCO3−.
H+ + HCO3− → H2CO3
When a small amount of base is added, it is removed by H2CO3.
OH− + H2CO3 → HCO3− + H2O
Therefore, pH of blood remains fairly constant.
[2]

(e) Describe a simple chemical test to distinguish each pair of compounds below.

I and Br

To separate samples of the two compounds, add NaOH(aq) and heat. Then acidify
with excess HNO3(aq) followed by the addition of AgNO3(aq).

Compound Observations

I Yellow ppt is formed.

Br Cream ppt is formed.

[2]

[Total: 20]

www.gradtutors.com
781

Section A

For each question there are four possible answers, A, B, C, and D. Choose the one you consider
to be correct.

1 Which one of the following statements about 20.3 g of Co2(SO4)3 is incorrect?


[The molar mass of Co2(SO4)3 = 406 g mol–1]

A It contains 1.5 x 1023 ions.


B It contains 0.25 mol of ions.
C It contains 0.15 mol of Co3+ ions.
D It contains 47.3 % of oxygen by mass.

2 25.0 cm3 of a solution of hydrogen peroxide, H2O2, was oxidised by excess potassium
manganate(VII) solution under acidic condition. The volume of oxygen gas produced at
room temperature and pressure was 26 cm3.

What is the concentration of hydrogen peroxide in the solution?


[The molar volume of a gas at r.t.p. = 24 dm3 mol 1] −

A 2.89 x 10 2 mol dm 3
− − C 8.66 x 10 2 mol dm 3
− −

B 4.33 x 10 2 mol dm 3
− − D 1.30 x 10 1 mol dm 3
− −

3 Which one of the following species has the electronic configuration as shown below?

A N C O2–
B S D F+

4 The equations for four reactions are given below.

Cl2 + 2I– → 2Cl– + I2


2Fe3+ + 2I– → 2Fe2+ + I2
Cl2 + 2Fe2+ → 2Cl– + 2Fe3+

What is the correct order of strength of Cl–, Fe2+ and I– as reducing agents?

weakest ⎯⎯⎯⎯⎯→ strongest


– –
A Cl I Fe2+
B Cl– Fe2+ I–
C Fe2+ I– Cl–
D I– Cl– Fe2+

www.gradtutors.com
3 782

5 Which one of the following pairs does the first species have higher ionisation energy than
the second?

A Sb, Te B Kr, Ar C Mg+, Al+ D Se, Se+

6 Which one of the following sets of solid substances includes a giant metallic structure, a
giant molecular structure and a simple molecular structure?

A Al, C, SiO2
B Pb, Al2O3, I2
C Ni, SiO2, KI
D Be, Si, SO3

7 Refer to the molecule below.

a
d
b

c
e

Which one of the following shows the correct information with regard to the various
labeled bond angles?

smallest largest
⎯⎯⎯⎯⎯→
bond angle bond angle
A c d e
B d a c
C e b a
D b e d

8 Which one of the following pairs of compounds does the first member have a higher
boiling point than the second member?

A 2–hydroxybenzoic acid, 4–hydroxybenzoic acid


B ethanol, water
C trans–but–2–ene, cis–but–2–ene
D butane, 2–methylpropane

© VJC 2013 8872/01/13 [Turn Over


www.gradtutors.com
4 783

9 Which one of the following quantities has the same value as the standard enthalpy change
of formation of carbon monoxide?

A ½ ΔHf (CO2(g))
ο

B ½ ΔHc (graphite)
ο

C ΔHc (graphite) – ΔHc (CO(g))


ο ο

D ΔHf (CO2(g)) – ½ ΔHc (graphite)


ο ο

10 The enthalpy change of combustion of alkanes to produce carbon dioxide and water is an
important exothermic reaction.

Which line on the graph shows the relationship between the number of carbon atoms in
the alkane and the number of moles of oxygen gas needed for complete combustion of
the alkane?

A B

moles of C
oxygen gas

1 number of carbon atoms in alkane

11 The rate of the reaction 3X(g) + Y(g) → Z(g) is given by the rate equation, rate = k [X]2[Y].

Two experiments using the same initial amounts of X and Y were carried out at
temperature, T. The data is shown below.

volume of initial rate of


experiment
reaction vessel reaction
1 V R1
2 2V R2

Which one of the following shows the relationship between R1 and R2?

1 1
A R2 = R1 B R2 = R1 C R2 = R1 D R2 = 8R1
8 4

© VJC 2013 8872/01/13


www.gradtutors.com
5 784

12 The reaction below is zero order with respect to P and first order with respect to Q.

P(aq) + Q(aq) → R(aq) + W(aq)

Which one of the following statements is correct?

A P is a catalyst in this reaction.


B Q is in excess.
C The concentration of Q decreases exponentially with time.
D The overall order of reaction is 2.

13 Given that
A2(g) + 4C(g) 2AC2(g) Kc = 4.8 (numerical value)

it follows that, for the reaction,

AC2(g) ½ A2(g) + 2C(g) Kc = Y (numerical value)

The value of Y would be

A 1
4 .8
B 1
2 .4
C 1
4 .8

D 2.4

14 The equilibrium constant for the following reaction is less than 1.

HPO42 (aq) + H2BO3 (aq)


− −
H2PO4 (aq) + HBO32 (aq)
− −

Which one of one the following gives the correct relative strengths of the acids and bases
in the reaction?

acids bases
A H2PO4 − > H2BO3 − and HBO32 − > HPO42 −

B H2BO3 − > H2PO4 − and HBO32 − > HPO42 −

C H2PO4 − > H2BO3 − and HPO42 − > HBO32 −

D H2BO3 − > H2PO4 − and HPO42- > HBO32 −

© VJC 2013 8872/01/13 [Turn Over


www.gradtutors.com
6 785

15 What is the pH of the final solution formed by mixing equal volumes of two separate
portions of dilute sulfuric acid of pH 2.0 and pH 4.0?

A 2.3 B 2.6 C 3.0 D 3.6

16 The successive ionization energies, in kJ mol 1,of an element P are given below:

558 1820 2704 5200

Which group does P belong to?

A I
B II
C III
D IV

17 Element G is found in Period 3 of the Periodic Table.

The oxide and chloride of element G are separately mixed with water. The two resulting
solutions have the same effect on litmus paper.

What is the identity of element G?

A aluminium C phosphorus
B magnesium D sodium

18 Which one of the following options correctly show the correct number of σ and π bonds in
the following molecule?

no. of σ bonds no. of π bonds


A 13 3
B 13 5
C 19 3
D 19 5

© VJC 2013 8872/01/13


www.gradtutors.com
7 786

19 Bromine in tetrachloromethane is added separately to benzene, methylbenzene, hexane


and hexene in the dark.

Which one of the following pairs will show the same observation?

A hexane and benzene C hexene and benzene


B hexane and hexene D hexene and methylbenzene

20 Which one of the following will not liberate 2 mol of carbon dioxide when 1 mol of the
compound is treated with excess hot acidified potassium manganate(VII)?

A B C D

21 How many geometric isomers does compound G have?

A 2 B 4 C 6 D 8

22 Compound H was subjected to dehydration followed by reaction with acidified potassium


manganate(VII). One of the final products formed gives a pale yellow precipitate with
alkaline aqueous iodine.

What is the formula for compound H?

A CH3CH2CH2CH2CH2OH
B CH3CH(OH)CH2CH2CH3
C CH3CH2CH(OH)CH2CH3
D CH3CH(CH3)CH(OH)CH3

23 In the upper atmosphere, chlorofluoroalkanes (CFCs) are broken down to give chlorine but
not fluorine radicals.

What is the best explanation for this?

A Fluorine is more electronegative than chlorine.


B Chlorine is more stable than fluorine.
C The C-F bond is stronger than C-Cl bond.
D The C-F bond is longer than C-Cl bond.

© VJC 2013 8872/01/13 [Turn Over


www.gradtutors.com
8 787

24 Estrone is one the three estrogenic hormones present in nature and it is the most
predominant type of estrogen found in postmenopausal women.

Estrone

Which of the following gives the correct set of observations for estrone?

(i) It gives a pale yellow precipitate on warming with alkaline aqueous iodine.
(ii) It gives an orange precipitate with 2,4-dinitrophenylhydrazine.
(iii) One mole of estrone reacts with sodium to form one mole of hydrogen gas.

A (ii) C (ii), (iii)


B (i), (ii) D (i), (ii), (iii)

25 Indole is an organic compound commonly used in fragrances and the precursor to many
pharmaceuticals. One way to synthesize indole is shown below.

Which one of the following gives the correct reagents required for the steps indicated?

I II III

A alcoholic KOH aqueous Cl2 concentrated H2SO4


B alcoholic KOH Cl2 in CCl4 concentrated H2SO4
C aqueous KOH aqueous Cl2 concentrated H2SO4
D aqueous KOH Cl2 in CCl4 aqueous H2SO4

© VJC 2013 8872/01/13


www.gradtutors.com
9 788

Section B

For each of the questions in this section, one or more of the three numbered statements 1 to 3 may
be correct.

Decide whether each of the statements is or is not correct (you may find it helpful to put a tick
against the statements that you consider to be correct).

The responses A to D should be selected on the basis of

A B C D
1, 2 and 3 1 and 2 2 and 3 1 only
are correct only are correct only are correct is correct

No other combination of statements is used as a correct response.

26 Hydrogen sulfide reacts with water according to the following equilibrium equation.

H2S(aq) + 2H2O(l) 2H3O+(aq) + S2 (aq)


Which of the following would cause a shift in equilibrium position when added to the above
equilibrium mixture?

1 water
2 sodium carbonate
3 ammonium sulfate

27 Which of the following pairs of substances have the same type of bonding and structure?

1 Rb Kr
2 diamond SiC
3 PH3 SiCl3H

28 Which of the following enthalpy changes are always exothermic?

1 Enthalpy change of neutralisation


2 Lattice energy
3 Ionisation energy

© VJC 2013 8872/01/13 [Turn Over


www.gradtutors.com
10 789

29 Which of the following decolourises a solution of hot acidified potassium manganate(VII)?

1 2 3

30 Benzylpenicillin, commonly known as penicillin G, has the following structure.

Which of the following statements are correct?

1 Penicillin G decolourises acidified potassium manganate(VII).


2 Penicillin G reacts with 2,4-dinitrophenylhydrazine to give an orange precipitate.
3 1 mol of penicillin G reacts with 1 mol of NaOH at room temperature.

© VJC 2013 8872/01/13


www.gradtutors.com
790

2013 VJC H1 Chemistry Prelim Exam 8872/1

Suggested Answers

1 C 6 D 11 A 16 C 21 C 26 C

2 B 7 D 12 C 17 C 22 D 27 C

3 D 8 D 13 A 18 D 23 C 28 B

4 B 9 C 14 A 19 A 24 A 29 B

5 A 10 B 15 A 20 B 25 A 30 D

www.gradtutors.com
2 791

Section A

Answer all questions in this section in the spaces provided.

Compound X is a dark brown compound which consists of oxygen and metal M only.
Successive heating of compound X transformed it into two other compounds.

(a) Calculate the mass of oxygen that is combined with 1.00 g of metal M in each of
these three compounds.

In compound X, 1.00 g metal M combines with ……………….… g of oxygen.

In compound Y, 1.00 g metal M combines with ……………….… g of oxygen.

In compound Z, 1.00 g metal M combines with ……………….… g of oxygen.


[2]

[Turn over
© VJC 2013
www.gradtutors.com
8872/02/PRELIM/13
3 792

(b) The empirical formula of compound X is MO2.


By considering the ratios of the amount of oxygen present in the three compounds,
Determine the empirical formulae for compound Y and compound Z.

Compound Y ……………….…

Compound Z ……………….…
[3]

(c) Identify metal M by calculating its molar mass.

Metal M is ……………….… with a molar mass of ……………….… g mol 1. −

[2]
[Total: 7]

[Turn over
© VJC 2013
www.gradtutors.com
8872/02/PRELIM/13
4 793

2 This question is about halogens and its compounds.

(a) World War I is sometimes known as ‘The Chemists’ War’. Knowledge of Chemistry was
applied towards developing high explosives and new methods such as the large scale use
of poison gas.

The first successful use of chlorine as a poison gas was at Ypres, Belgium, on 22 April
1915. 170 tonnes (1 tonne = 1000 kg) of chlorine contained in 5730 cylinders was released
forming a grey-green cloud which drifted across French troops. Chlorine can damage the
eyes, nose, throat and lungs and is fatal at concentrations of 1000 ppm and above
(1 ppm = 1 mg dm 3). Early counter-measures to chlorine included instructing troops to

cover their mouths with gauze pads soaked in sodium hydrogen carbonate solution.
Eventually, more effective counter-measures to chlorine were developed and thus other
poison gases were introduced.

(i) Calculate the maximum amount of chlorine gas that could be released from one of the
cylinders that was used at Ypres on 22 April 1015.

(ii) Determine the concentration of chlorine gas, in mol dm 3, in 1000 ppm of the gas.

(iii) In an accident, the chlorine gas from one such cylinder was released into a factory
room of volume 25.0 m3.

Determine if the concentration of chlorine gas was fatal. Assume that the gas was
released at room temperature and pressure.

[4]

[Turn over
© VJC 2013
www.gradtutors.com
8872/02/PRELIM/13
5 794

(b) Strontium chloride is used to prepare strontium ethanedioate, SrC2O4, which is widely used
as a red colorant in fireworks. The equation for the reaction is as shown:
SrCl2(aq) + Na2C2O4(aq) → SrC2O4(s) + 2NaCl(aq)
(i) State the type of bonding present in SrC2O4.

…………………………………………………………………………………………………

(ii) Deduce the shape and bond angle in the ethanedioate part of SrC2O4 by drawing its
structural formula.

(iii) State the number of sigma and pi bonds present in the ethanedioate part of SrC2O4.

…………………………………………………………………………………………………
[4]

(c) (i) Halogens can react with aluminium to form halides such as AlF3 and AlBr3.
When heated, aluminium bromide remains as a solid until 371K whereas aluminium
fluoride remains as a solid until 1530K.

Account for the difference in their melting points in terms of structure and bonding.

…………………………………………………………………………………………………

…………………………………………………………………………………………………

…………………………………………………………………………………………………

…………………………………………………………………………………………………

…………………………………………………………………………………………………

…………………………………………………………………………………………………

(ii) In the vapour phase, an equilibrium is established between aluminium bromide and
its dimer as follows:
2AlBr3(g) Al2Br6(g)

With the help of a dot-and-cross diagram, explain how the dimer is formed.

[5]
[Total: 13]

[Turn over
© VJC 2013
www.gradtutors.com
8872/02/PRELIM/13
6 795

3 This question is about Period 3 elements and their compounds.

(a) The ionic radii of four consecutive elements in period 3 are given below.

element A B C D
ionic radius / nm 0.065 0.050 0.041 0.212

(i) Deduce which one of the elements, A, B, C or D corresponds to magnesium.


Explain your answer.

………………………………………………………………..………………………….

………………………………………………………………..………………………….

………………………………………………………………..………………………….

………………………………………………………………..………………………….

………………………………………………………………..………………………….

(ii) Period 3 elements react with chlorine to form chlorides. Account for the
difference in the pH values when chloride of B and chloride of D is separately
dissolved in water. Write equations to support your answer.

[6]

(b) Period 3 elements also form oxides upon heating with oxygen.

(i) Write equations to illustrate the reaction between sodium oxide and water as
well as phosphorous pentoxide and water. Hence, deduce the pH of the
resultant solution after mixing. Support your answers with relevant equations.

(ii) Write equations to illustrate the amphoteric behaviour of aluminium oxide,


using potassium hydroxide and sulfuric acid in your answer.

[4]
[Total: 10]

[Turn over
© VJC 2013
www.gradtutors.com
8872/02/PRELIM/13
7 796

4 (a) Organic halogen compounds have a wide variety of commercial uses. For example,
the polymer, polyvinyl chloride (PVC) is commonly used as insulation for electrical
wiring.
The monomer of PVC, CH2=CHCl, is industrially produced from CH2=CH2 via the
following route:

(i) Step I is carried out by reacting CH2=CH2 with hydrogen chloride and oxygen
over a copper(II) chloride catalyst. Write a balanced equation for this reaction.

……………………………………………………………………………………………

(ii) Suggest the reagents and conditions that can be used to carry out step II in
the laboratory.

……………………………………………………………………………………………
[2]

(b) (i) CH2=CHCl can be formed directly in a single step from CH2=CH2 in the
following way:

This reaction also results in the formation of a product, C2H2Cl2, which exists
as a pair of geometric isomers R and S. Give the name of this product.

…………………………………………

(ii) Some physical properties of the geometric isomers, R and S are shown below:

Isomer Melting point / oC Boiling point / oC


R –49.4 48.5
S –81.5 60.2

Draw the structure of R and S in the boxes below.

R S

(iii) CH3CH2CH3 can also react with Cl2 in the presence of uv light to form 2
monochlorinated products, CH2ClCH2CH3 and CH3CHClCH3. It has been
determined experimentally that the hydrogen atoms in alkanes are replaced by
chlorine atoms at different rates as shown below:

Relative rate of
Type of H atom Reaction
replacement
Primary RCH3 → RCH2Cl 1
Secondary R2CH2 → R2CHCl 3.5
Tertiary R3CH → R3CCl 5

–R represents an alkyl group.

[Turn over
© VJC 2013
www.gradtutors.com
8872/02/PRELIM/13
8 797

By considering both the number and the rate of replacement of each type of
hydrogen atom present in CH3CH2CH3, predict the ratio of CH2ClCH2CH3 and
CH3CHClCH3 formed. Show your working clearly.

[4]

(c) Chlorofluorocarbons (CFCs) are another class of organic halogen compounds. They
were formerly used in aerosol propellants but due to concerns about their effect on
the depletion of the ozone layer, they have been replaced by volatile hydrocarbons
such as CH3CH2CH3 and CH3CH2CH2CH3.

Two common CFCs once used in aerosol propellants were CCl3F and CH3CClF2.

(i) By comparing the strength of the relevant carbon–halogen bonds, suggest and
explain which compound, CCl3F or CH3CClF2, is likely to have a smaller effect
on ozone depletion.

……………………………………………………………………………………………

……………………………………………………………………………………………

……………………………………………………………………………………………

……………………………………………………………………………………………

……………………………………………………………………………………………

(ii) Other than aerosol propellants, give one use of CFCs.

..............................................................................................................................

……………………………………………………………………………………………

(iii) State one possible hazard of using hydrocarbons instead of CFCs in aerosol
propellants.

……………………………………………………………………………………………
[4]

[Total: 10]

[Turn over
© VJC 2013
www.gradtutors.com
8872/02/PRELIM/13
9 798

Section B
Answer two questions from this section on separate answer paper.

5 Hydrogen has large scale use in the chemical industry. Although hydrogen is the most
abundant element in the Universe, very small quantities of molecular hydrogen have been
found to occur naturally.

(a) One of the most efficient methods of producing hydrogen involves steam reforming
of natural gas, an equilibrium reaction in which methane gas is reacted with steam to
produce carbon monoxide and hydrogen.

CH4(g) + H2O(g) CO(g) + 3H2(g)

(i) Use appropriate bond energies given in the Data Booklet to calculate an
approximate value for the steam reforming of natural gas.
[Take the bond energy for the C ≡ O bond in CO to be 1077 kJ mol–1]

(ii) Use the energy cycle and data given below to calculate another approximate
value for the steam reforming of natural gas.

CH4(g) + H2O(g) CO(g) + 3H2(g)

CO2(g) + 3H2O(l)

ΔHco of CH4(g) = –890 kJ mol–1


ΔHvap of H2O(l) = +40.7 kJ mol–1
ΔHco of CO(g) = –283 kJ mol–1
ΔHfo of H2O(l) = –286 kJ mol–1
(iii) Suggest two reasons for the discrepancy between the values in (i) and (ii).
[6]

(b) The water–gas shift reaction is often used together with the steam reforming of
natural gas to produce more hydrogen. This involves the highly exothermic reaction
of carbon monoxide with steam to produce carbon dioxide and hydrogen.

CO(g) + H2O(g) CO2(g) + H2(g)

The equilibrium constant, Kc is found to be 0.64 at 1000 K.

(i) Write an expression for Kc of this reaction.

(ii) A mixture containing 0.80 mol of CO, 0.80 mol of H2O, 0.40 mol of CO2 and
0.40 mol of H2 was placed in a 2 dm3 flask and allowed to come to equilibrium
at 1000 K.

Calculate the amount, in mol, of each substance present in the equilibrium


mixture at 1000 K.

[5]

[Turn over
© VJC 2013
www.gradtutors.com
8872/02/PRELIM/13
10 799

(c) The graph below shows the variation in equilibrium yield of the products with
temperature and pressure for the water–gas shift reaction.
Equilibrium yield
of products

1000 °C

1400 °C

pressure

(i) State Le Chatelier’s Principle.

(ii) Hence comment on the accuracy of the graph with respect to both temperature
and pressure.
[5]

(d) In the presence of a catalyst mixture made up of alumina, copper and zinc oxide,
carbon monoxide and hydrogen reacts to form compound D instead.

Compound D is commonly obtained from biomass and consist only of the elements
C, H and O. When 0.55 g of D was completely burnt in air, the heat produced raised
the temperature of 100 g of water by 29.4 °C. The standard enthalpy change of
combustion of D is found to be –715 kJ mol–1.

(i) Calculate the amount of heat released in the experiment.


[Assume the heat capacity of all solutions to be 4.18 J K–1 cm–3]

(ii) Hence deduce the molecular formula of compound D.


[4]
[Total:20]

[Turn over
© VJC 2013
www.gradtutors.com
8872/02/PRELIM/13
11 800

6 “The bubble has burst for several tea businesses here” quoted from The Straits Times on 28
May 2013.

The Agri-Food and Veterinary Authority of Singapore (AVA) recalled 11 types of tapioca
starch balls - also known as "pearls" - made in Taiwan that are used in popular 'bubble teas',
after they were found to contain maleic acid. Consuming high levels of maleic acid can cause
kidney damage.

Maleic acid

(a) Maleic acid is commonly produced via the following 2-step synthetic route industrially.

(i) State the type of reactions for I and II.

(ii) Write a balanced equation for step II.

(iii) Suggest how you can distinguish the following pairs of compounds:
I – Benzene and maleic anhydride
II – Maleic anhydride and maleic acid

In each case, state the reagents and conditions as well as observations for each
compound.
[7]

(b) Draw the structure of the major product formed when maleic acid reacts with

(i) cold concentrated sulfuric acid, followed by hydrolysis


(ii) hydrogen gas with palladium on carbon
(iii) thionyl chloride
(iv) methanol with concentrated sulfuric acid
(v) cold alkaline potassium manganate(VII)
[6]

(c) (i) State the type of isomerism displayed by maleic acid and draw its isomer, fumaric
acid.

(ii) Based on the structures of maleic acid and fumaric acid, predict which acid has a
lower melting point.

(iii) A student added some water to a mixture of maleic acid and fumaric acid and
performed a simple filtration. He concluded that the residue obtained must be
fumaric acid.

Do you agree with the student’s statement? Justify your answer.


[7]
[Total: 20]

[Turn over
© VJC 2013
www.gradtutors.com
8872/02/PRELIM/13
12 801

7 (a) Benzenesulfonic acid (BSA) reacts with aqueous NaOH to produce phenol. The graph
below shows the reaction between aqueous sodium hydroxide and solutions of
benzenesulfonic acid of different concentration at temperature T oC.

[BSA ] = 0.4 mol dm–3

[BSA ] = 0.8 mol dm–3

(i) With reference to the graph above, deduce the order of reaction with respect to
sodium hydroxide and benzenesulfonic acid and hence write down the rate equation.

(ii) Suggest a method to monitor the concentration of NaOH in the above kinetic study.

(iii) Determine the time taken for the concentration of benzenesulfonic acid to reduce
from 0.4 mol dm–3 to half of its original concentration.

(iv) On the grid drawn, sketch and label a graph for the reaction of 0.10 mol dm–3 of
aqueous sodium hydroxide with 0.4 mol dm–3 of BSA at (T−10) oC.

(v) Account for the shape of the graph in part (iv).


[9]

[Turn over
© VJC 2013
www.gradtutors.com
8872/02/PRELIM/13
13 802

(b) Hypochlorous acid, HClO, is a weak acid that is formed when chlorine reacts with
water.

(i) With the aid of an equation, explain what is meant by weak acid.

(ii) Write an expression for the acid dissociation constant, Ka, for HClO.

(iii) The following equilibrium concentrations were determined for a solution of HClO,
calculate the value of Ka of HClO, stating the units.

Species Concentration / mol dm−3


HClO(aq) 0.0200
H+(aq) 2.50 x 10−5
ClO−(aq) 2.50 x 10−5
[3]

(c) A solution containing hypochlorous acid, HClO, and sodium chlorate(I), NaClO, can act
as a buffer solution. Explain, with the help of equations, how this solution can regulate
pH when relatively small amount of acid or base is added to the solution.
[3]

(d) Ethylbenzene can react with chlorine in two ways, depending on the conditions of the
reaction.
CH2CH3
CH2CH3 CH2CH2Cl

Cl2 Cl2

reaction I reaction II
CI
B
A

(i) State the condition needed for reaction I.

(ii) Describe a chemical test to distinguish between A and B. State the reagents,
conditions, and the observations for each compound. Write balanced equations
for any reactions that occur.
[5]

[Total: 20]

[Turn over
© VJC 2013
www.gradtutors.com
8872/02/PRELIM/13
1 803

2013 VJC H1 Chemistry Prelim Exam 8872/2


Suggested Answers

Section A
Answer all the questions in this section in the spaces provided.

Compound X is a dark brown compound which consists of oxygen and metal M only.
Successive heating of compound X transformed it into two other compounds.

(a) Calculate the mass of oxygen that is combined with 1.00 g of metal M in each of
these three compounds.

Compound X contains 13.38 g of oxygen for every 86.62 g of metal M; by


13.38
division, every 1.00 g of metal M will combine with g of oxygen =
86.62
0.1545 g
[same method for compound Y and Z; no working required for full credit]

In compound X, 1.00 g metal M combines with • 0.1545 g of oxygen.

In compound Y, 1.00 g metal M combines with • 0.1029 g of oxygen.

In compound Z, 1.00 g metal M combines with • 0.07721 g of oxygen.


[2]
(b) The empirical formula of compound X is MO2.
By considering the ratios of the amount of oxygen present in the three compounds,
determine the empirical formulae for compound Y and compound Z.

mass of oxygen in compound Y 0.1029 2


= =
mass of oxygen in compound X 0.1545 3
2
We have as much oxygen in compound Y compared to compound X;
3
therefore, the empirical formula of compound Y is M3O4 (formula is MO3/4 but
we multiply both factors by 4 to get rid of the fraction).

mass of oxygen in compound Z 0.07721 1


= =
mass of oxygen in compound X 0.1545 2
1
We have as much oxygen in compound Z compared to compound X;
2
therefore, the empirical formula of compound Y is MO.

Compound Y M3O4

Compound Z MO

[3]

©VJC 2013 www.gradtutors.com


8872/02/PRELIM/13
2 804

(c) Identify metal M by calculating its molar mass.

Let the molar mass of metal M be x g/mol and consider the empirical formula
of compound X.
no. of moles of metal M

no. of moles of oxygen
1.00
x 1
= =
0.1545 2
16.0
∴ x = 207.2

Metal M is lead with a molar mass of 207.2 g mol 1.


[2]

[Total: 7]

2 This question is about halogens and its compounds.

(a) World War I is sometimes known as ‘The Chemists’ War’. Knowledge of Chemistry was
applied towards developing high explosives and new methods such as the large scale use
of poison gas.

The first successful use of chlorine as a poison gas was at Ypres, Belgium, on 22 April
1915. 170 tonnes (1 tonne = 1000 kg) of chlorine contained in 5730 cylinders was released
forming a grey-green cloud which drifted across French troops. Chlorine can damage the
eyes, nose, throat and lungs and is fatal at concentrations of 1000 ppm and above
(1 ppm = 1 mg dm−3). Early counter-measures to chlorine included instructing troops to
cover their mouths with gauze pads soaked in sodium hydrogen carbonate solution.
Eventually, more effective counter-measures to chlorine were developed and thus other
poison gases were introduced.

(i) Calculate the maximum amount of chlorine gas that could be released from one of the
cylinders that was used at Ypres on 22 April 1015.

Amount of chlorine in 1 cylinder = / 5730 = 4.18 x 102 mol

(ii) Determine the concentration of chlorine gas, in mol dm−3, in 1000 ppm of the gas.

1 mg dm 3 = 1 x 10 3 g dm 3
− − −

Concentration of Cl2 in g dm 3 = 1000x 1x 10 3 = 1.00 g dm 3


− − −

Concentration of Cl2 in mol dm 3 = = 0.0141 mol dm 3


− −

(iii) In an accident, the chlorine gas from one such cylinder was released into a factory
room of volume 25.0 m3.
Determine if the concentration of chlorine gas was fatal. Assume that the gas was
released at room temperature and pressure.

1 m3 = 1 x103 dm3
Volume of factory room = 25 x 1 x103 dm3 = 25000 dm3
Concentration of Cl2 in the factory = = 0.0167 mol dm 3 −

Since 0.0167 mol dm 3 > 0.0141 mol dm 3, it is fatal


− −

[4]

©VJC 2013 www.gradtutors.com


8872/02/PRELIM/13
3 805

(b) Strontium chloride is used to prepare strontium ethanedioate, SrC2O4, which is widely used
as a red colorant in fireworks. The equation for the reaction is as shown:

SrCl2 (aq) + Na2C2O4(aq) → SrC2O4(s) + 2NaCl(aq)

(i) State the type of bonding present in SrC2O4.

Ionic

(ii) Deduce the shape and bond angle in the ethanedioate part of SrC2O4 by drawing its
structural formula.
.

Trigonal planar wrt C and bond angle = 120o

(iii) State the number of sigma and pi bonds present in the ethanedioate part of SrC2O4.

5 sigma, 2 pi
[4]

(c) (i) Halogens can react with aluminium to form halides such as AlF3 and AlBr3.
When heated, aluminium bromide remains as a solid until 371K whereas aluminium
fluoride remains as a solid until 1530K.
Account for the difference in their melting points in terms of structure and bonding.

AlF3 has a giant ionic lattice with strong ionic bonds between Al3+ and F- ions.
The strong ionic attractions require a lot of energy to break them apart hence
the high melting point.
AlBr3 has a simple covalent structure with the much weaker dispersion forces
between the molecules hence very little energy is needed to melt the solid.
[3]

(ii) In the vapour phase, an equilibrium is established between aluminium bromide and
its dimer as follows:
2AlBr3(g) Al2Br6(g)

With the help of a dot-and-cross diagram, explain how the dimer is formed.

The aluminium atom in AlBr3 has only 6 electrons surrounding it hence it is


electron-deficient. To complete the octet, the lone pair electrons from a
bromine atom in a neighbouring molecule would be donated via a dative bond,
resulting in a dimer.

[2]

[Total: 13]

©VJC 2013 www.gradtutors.com


8872/02/PRELIM/13
4 806

3 This question is about Period 3 elements and their compounds.

(a) The ionic radii of four consecutive elements in period 3 are given below.

element A B C D
ionic radius / nm 0.065 0.050 0.041 0.212

(i) Deduce which one of the elements, A, B, C or D corresponds to magnesium.


Explain your answer.

A is magnesium.
There is a large dip in the ionic radius from element C to D, implying that
ion formed from element D has one electron shell less. Ion formed from
D is P3 which has 18 electrons while ions formed from elements A, B and

C have 10 electrons.

(ii) Period 3 elements react with chlorine to form chlorides. Account for the
difference in the pH values when chloride of B and chloride of D is separately
dissolved in water. Write equations to support your answer.
B is Al while D is P.
Since Al3+ has a high charge density, it hydrolyses in water to form an
acidic solution at pH 3.
AlCl3 + 6H2O → [Al(H2O)6]3+ + 3Cl −

[Al(H2O)6]3+ + H2O [Al(H2O)5(OH)]2+ + H3O+


Since P has vacant d orbitals to accept lone pair of electrons from water,
it undergoes complete hydrolysis in water completely to form strong acid
of HCl and H3PO4, thus the solution formed is at pH 2.
PCl5 + 4 H2O → H3PO4 + 5 HCl
[6]

(b) Period 3 elements also form oxides upon heating with oxygen.

(i) Write equations to illustrate the reaction between sodium oxide and water as
well as phosphorous pentoxide and water. Hence, deduce the pH of the
resultant solution after mixing. Support your answers with relevant equations.

Na2O + H2O → 2NaOH


P4O10 + 6H2O → 4H3PO4
[Or P2O5 + 3H2O → 2H3PO4]
3NaOH + H3PO4 → Na3PO4 + 3H2O
PO43 from Na3PO4, a strong conjugate base of HPO42 , undergoes
− −

hydrolysis to give excess OH ions, making the pH of the final solution to


be greater than 7, at pH 9.
PO43 + H2O

HPO42 + OH
− −

(ii) Write equations to illustrate the amphoteric behaviour of aluminium oxide,


using potassium hydroxide and sulfuric acid in your answer.

Al2O3 + 2KOH + 3H2O → 2KAl(OH)4


Al2O3 + 3H2SO4 → Al2(SO4)3 + 3H2O
[4]

[Total: 10]

©VJC 2013 www.gradtutors.com


8872/02/PRELIM/13
5 807

4 (a) Organic halogen compounds have a wide variety of commercial uses. For example,
the polymer, polyvinyl chloride (PVC) is commonly used as insulation of electrical
wiring.

The monomer of PVC, CH2=CHCl, is industrially produced from CH2=CH2 via the
following route:

(i) Step I is carried out by reacting CH2=CH2 with hydrogen chloride and oxygen
over a copper(II) chloride catalyst. Write a balanced equation for this reaction.

CH2=CH2 + 2HCl + ½O2 → CH2ClCH2Cl + H2O

(ii) Suggest the reagents and conditions that can be used to carry out step II in
the laboratory.

Alcoholic NaOH, reflux


[2]

(b) (i) CH2=CHCl can be formed directly in a single step from CH2=CH2 in the
following way:

This reaction also results in the formation of a product, C2H2Cl2, which exists
as a pair of geometric isomers R and S. Give the name of this product.

1, 2–dichloroethene

(ii) Some physical properties of the geometric isomers, R and S are shown below:

Isomer Melting point / oC Boiling point / oC


R –49.4 48.5
S –81.5 60.2

Draw the structure of R and S in the boxes below.

R S
√ √

   
   
(trans isomer) (cis isomer)

(iii) CH3CH2CH3 can also react with Cl2 in the presence of uv light to form 2
monochlorinated products, CH2ClCH2CH3 and CH3CHClCH3. It has been
determined experimentally that the hydrogen atoms in alkanes are replaced by
chlorine atoms at different rates as shown below:

©VJC 2013 www.gradtutors.com


8872/02/PRELIM/13
6 808

Relative rate of
Type of H atom Reaction
replacement
Primary RCH3 → RCH2Cl 1
Secondary R2CH2 → R2CHCl 3.5
Tertiary R3CH → R3CCl 5
–R represents an alkyl group.

By considering both the number and the rate of replacement of each type of
hydrogen atom present in CH3CH2CH3, predict the ratio of CH2ClCH2CH3 and
CH3CHClCH3 formed. Show your working clearly.

Any one of six H atoms can be replaced to form CH2ClCH2CH3 while any
one of two H atoms can be replaced to form CH3CHClCH3.

Hence, the ratio of CH2ClCH2CH3 to CH3CHClCH3 = 3.5

= 0.857
[4]

(c) Chlorofluorocarbons (CFCs) are another class of organic halogen compounds. They
were formerly used in aerosol propellants but due to concerns about their effect on
the depletion of the ozone layer, they have been replaced by volatile hydrocarbons
such as CH3CH2CH3 and CH3CH2CH2CH3.

Two common CFCs once used in aerosol propellants were CCl3F and CH3CClF2.

(i) By comparing the strength of the relevant carbon-halogen bonds, suggest and
explain which compound, CCl3F or CH3CClF2, is likely to have a smaller effect
on ozone depletion.

Since the C–Cl bond is weaker than C–F bond, only C–Cl is broken by uv
light in the stratosphere to form chlorine radicals which react with the
ozone layer.

Thus, since CH3CClF2 has a lower chlorine content, it has a smaller effect
on ozone depletion.

(ii) Other than aerosol propellants, give one use of CFCs.

CFCs are used in refrigerants (OR solvents for cleaning electronic circuit
boards OR fire extinguishers).

(iii) State one possible hazard of using hydrocarbons instead of CFCs in aerosol
propellants.

Hydrocarbons are highly flammable.


[4]

[Total: 10]

©VJC 2013 www.gradtutors.com


8872/02/PRELIM/13
7 809

Section B
Answer two questions from this section on separate answer papers.

5 Hydrogen has large scale use in the chemical industry. Although hydrogen is the most
abundant element in the Universe, very small quantities of molecular hydrogen have been
found to occur naturally.

(a) One of the most efficient methods of producing hydrogen involves steam reforming
of natural gas, an equilibrium reaction in which methane gas is reacted with steam to
produce carbon monoxide and hydrogen.

CH4(g) + H2O(g) CO(g) + 3H2(g)

(i) Use appropriate bond energies given in the Data Booklet to calculate an
approximate value for the steam reforming of natural gas.
[Take the bond energy for the C ≡ O bond in CO to be 1077 kJ mol–1]

ΔHrxn = B.E.(Breaking) – B.E.(Forming)


= 4B.E.(C–H) + 2(O–H) – 1(C ≡ O) – 3(H–H)
= •4(410) + 2(460) – 1(1077) – 3(436)
= •+175 kJ mol–1

(ii) Use the energy cycle and data given below to calculate another approximate
value for the steam reforming of natural gas.

CH4(g) + H2O(g) CO(g) + 3H2(g)

CO2(g) + 3H2O(l)

ΔHco of CH4(g) = –890 kJ mol–1


ΔHvap of H2O(l) = +40.7 kJ mol–1
ΔHco of CO(g) = –283 kJ mol–1
ΔHfo of H2O(l) = –286 kJ mol–1

ΔHrxn
CH4(g) + H2O(g) CO(g) + 3H2(g)

–890 –40.7 –283 3(–286)

CO2(g) + 3H2O(l)

ΔHrxn = (–890) + (–40.7) – (–283) – 3(–286)


= +210 kJ mol–1

(iii) Suggest two reasons for the discrepancy between the values in (i) and (ii).

The bond energy values in the Data Booklet are average values.
The energy cycle did not account for the energy required to cool the
water from 373 K to 298 K OR
The energy cycle did not account for the energy required to heat the
water from 298 K to 373 K.
[6]

©VJC 2013 www.gradtutors.com


8872/02/PRELIM/13
8 810

(b) The water–gas shift reaction is often used together with the steam reforming of
natural gas to produce more hydrogen. This involves the highly exothermic reaction
of carbon monoxide with steam to produce carbon dioxide and hydrogen.

CO(g) + H2O(g) CO2(g) + H2(g)

The equilibrium constant, Kc is found to be 0.64 at 1000 K.

(i) Write an expression for Kc for this reaction.

[CO 2 (g)].[H2 (g)]


Kc =
[CO(g)].[H2O(g)]

(ii) A mixture containing 0.80 mol of CO, 0.80 mol of H2O, 0.40 mol of CO2 and
0.40 mol of H2 was placed in a 2 dm3 flask and allowed to come to equilibrium
at 1000 K.

Calculate the amount, in mol, of each substance present in the equilibrium


mixture at 1000 K.

Let the change in concentration of CO(g) be x.

CO(g) H2O(g) CO2(g) H2(g)


Initial conc./ mol 0.80/2 0.80/2 0.40/2 0.40/2
Change in conc./ mol –x –x +x +x
Eqm. conc./ mol dm–3 0.40–x 0.40–x 0.20+x 0.20+x

[CO 2 (g)].[H2 (g)]


Kc =
[CO(g)].[H2O(g)]
⇒ 0.64 = [0.20 + x]2
2

[0.40 − x]
⇒ x = 0.0667 mol dm–3

nCO(g) = nH2O(g) = 2(0.40 – 0.0667) = 0.667 mol

nCO2(g) = nH2(g) = 2(0.20 + 0.0667) = 0.534 mol

[5]

©VJC 2013 www.gradtutors.com


8872/02/PRELIM/13
9 811

(c) The graph below shows the variation in equilibrium yield of the products with
temperature and pressure for the water–gas shift reaction.
equilibrium yield
of products

1000 °C

1400 °C
pressure

(i) State Le Chatelier’s Principle.

Le Chatelier’s Principle states that if a system at equilibrium is subjected


to a change which disturbs the equilibrium, the position of equilibrium
will shift in a direction to reduce the change.

(ii) Hence comment on the accuracy of the graph with respect to both temperature
and pressure.

Le Chatelier’s Principle states that if temperature increases, equilibrium


position will shift to favour the endothermic side to absorb the excess
heat. Since the forward reaction is exothermic, equilibrium position will
shift to the left, resulting in a decrease in equilibrium yield. Hence the
graph is correct with respect to temperature.

Le Chatelier’s Principle states that if pressure increases, equilibrium


position will shift to side with less gaseous particles to reduce the
pressure. However, since the no. of moles of gaseous reactants is equal
to the no. of moles of gaseous products, equilibrium yield remain
constant with increasing pressure. Hence the graph is incorrect with
respect to pressure.
[5]

(d) In the presence of a catalyst mixture made up of alumina, copper and zinc oxide,
carbon monoxide and hydrogen reacts to form compound D instead.

Compound D is commonly obtained from biomass and consist only of the elements
C, H and O. When 0.55 g of D was completely burnt in air, the heat produced raised
the temperature of 100 g of water by 29.4 °C. The standard enthalpy change of
combustion of D is found to be –715 kJ mol–1.

(i) Calculate the amount of heat released in the experiment.


[Assume the heat capacity of all solutions to be 4.18 J K–1 cm–3]

Heat released = m x c x ΔT
= 100 x 4.18 x 29.4
= 12300 J [OR 12.3 kJ]

©VJC 2013 www.gradtutors.com


8872/02/PRELIM/13
10 812

(ii) Hence deduce the molecular formula of compound D.

heat released 12300


ΔHc = – ⇒ –715 x 1000 = –
nD nD

12300
nD = – = 0.0172 mol
715000

m 0.55
Molar mass of D = = = 32.0
nD 0.0172

Molecular formula of D = CH3OH [OR CH4O]


[4]

[Total: 20]

6 “The bubble has burst for several tea businesses here” quoted from The Straits Times on 28
May 2013.

The Agri-Food and Veterinary Authority of Singapore (AVA) recalled 11 types of tapioca starch
balls - also known as "pearls" - made in Taiwan that are used in popular 'bubble teas', after
they were found to contain maleic acid. Consuming high levels of maleic acid can cause
kidney damage.

Maleic acid

(a) Maleic acid is commonly produced via the following 2-step synthetic route industrially.

(i) State the type of reactions for I and II.

I – oxidation
II – hydrolysis

(ii) Write a balanced equation for step II.

(iii) Suggest how you can distinguish the following pairs of compounds:
I – Benzene and maleic anhydride
II – Maleic anhydride and maleic acid

©VJC 2013 www.gradtutors.com


8872/02/PRELIM/13
11 813

In each case, state the reagents and conditions as well as observations for each
compound.

I – Benzene and maleic anhydride


Add aqueous bromine (or Br2 in CCl4) at room temperature.
Reddish-brown solution turns colourless for maleic anhydride. No colour
change observed for benzene.

II – Maleic anhydride and maleic acid


Add solid sodium carbonate (or NaHCO3(s)) at room temperature.
Effervescence which produced a white ppt in limewater observed for maleic
acid. No effervescence observed for maleic anhydride.
[7]

(b) Draw the structure of the major product formed when maleic acid reacts with

(i) concentrated sulfuric acid, followed by hydrolysis

malic acid

(ii) hydrogen gas with palladium on carbon

succinic acid

(iii) thionyl chloride

maleic acid chloride

(iv) methanol with concentrated sulfuric acid

dimethyl maleate

(v) cold alkaline potassium manganate(VII)

[6]

(c) (i) State the type of isomerism displayed by maleic acid and draw its isomer, fumaric
acid.

©VJC 2013 www.gradtutors.com


8872/02/PRELIM/13
12 814

Geometric (or cis-trans) isomerism.

fumaric acid

(ii) Based on the structures of maleic acid and fumaric acid, predict which acid has a
lower melting point.

Maleic acid has a lower melting point.


Intramoleculecular hydrogen bonding can take place in maleic acid due to the
close proximity of the –OH and =O groups. As such, only weaker dipole-dipole
interactions (or less extensive intermolecular hydrogen bonding) are present
between maleic acid molecules unlike the stronger (or more extensive)
intermolecular hydrogen bonding between fumaric acid molecules. Less
energy is required to overcome the weaker intermolecular interactions, which
results in the lower melting point.

(iii) A student added some water to a mixture of maleic acid and fumaric acid and
performed a simple filtration. He concluded that the residue obtained must be
fumaric acid.

Do you agree with the student’s statement? Justify your answer.

Yes, I agree.
Fumaric acid is less soluble than maleic acid, since more energy is required to
overcome the stronger solute-solute interaction, (or more extensive) hydrogen
bonding in fumaric acid, as compared to the weaker dipole-dipole interactions
(or less extensive intermolecular hydrogen bonding) in maleic acid.

Note: Both fumaric acid and maleic acid form strong hydrogen bonds with
water (solute-solvent interactions). Strong hydrogen bonding exists between
water molecules too (solvent-solvent interactions).
[7]

[Total: 20]

©VJC 2013 www.gradtutors.com


8872/02/PRELIM/13
13 815

7 (a) Benzenesulfonic acid (BSA) reacts with aqueous NaOH to produce phenol. The graph
below shows the reaction between aqueous sodium hydroxide and solutions of
benzenesulfonic acid of different concentration at temperature T oC.

[BSA ] = 0.4 mol dm–3

[BSA ] = 0.8 mol dm–3

(i) With reference to the graph above, deduce the order of reaction with respect to
sodium hydroxide and benzenesulfonic acid and hence write down the rate equation.

Gradient of graph with [BSA ] = 0.4 mol dm-3 is half that of graph with [BSA ] =
0.8 mol dm-3 .
Hence, the reaction is first order with respect to BSA.
Both graphs are straight lines with a constant gradient, which implies that
change in concentration of NaOH has no impact on the rate. Hence, the
reaction is zero order with respect to NaOH.
Rate = k[BSA]

(ii) Suggest a method to monitor the concentration of NaOH in the above kinetic study.
.
Use a pH probe to monitor the pH of the mixture. Or titrate NaOH with a
standard solution of HCl at regular time intervals.

(iii) Determine the time taken for the concentration of benzenesulfonic acid to reduce
from 0.4 mol dm–3 to half of its original concentration.

Rate = k[BSA] = 0.04/8 = 0.0005 mol dm−3 s−1.


Therefore, k = 0.0005/0.4 = 0.0125
t1/2 = ln 2/k = 55.5 s

(iv) On the grid drawn, sketch and label a graph for the reaction of 0.10 mol dm–3 of
aqueous sodium hydroxide with 0.4 mol dm–3 of BSA at (T−10) oC.

©VJC 2013 www.gradtutors.com


8872/02/PRELIM/13
14 816

(v) Account for the shape of the graph in part (iv).


For every 10 oC decrease in temperature, the rate is decreased by 2 times. •As
the order of the reaction with respect to the concentration of NaOH does not
change, the gradient of the graph at (T−10) oC is halved of that for T oC.

(b) Hypochlorous acid, HClO, is a weak acid that is formed when chlorine reacts with
water.

(i) With the aid of an equation, explain what is meant by weak acid.

HClO + H2O H3O+ + ClO−

The acid undergoes partial dissociation to produce H+.

(ii) Write an expression for the acid dissociation constant, Ka, for HClO.

[ H 3O + ][ClO − ]
Ka =
[ HClO]

(iii) The following equilibrium concentrations were determined for a solution of HClO,
calculate the value of Ka of HClO, stating the units.

Species Concentration / mol dm−3


HClO(aq) 0.0200
H+(aq) 2.50 x 10−5
ClO−(aq) 2.50 x 10−5
(2.50 × 10 −5 ) 2
Ka = = 3.13 × 10 −8 moldm −3
0.02 [3]

(c) A solution containing hypochlorous acid, HClO, and sodium chlorate(I), NaClO, can act
as a buffer solution. Explain, with the help of equations, how this solution can regulate
pH when a relatively small amount of acid or base is added to the solution.

The solution contains a large reservoir of HClO and ClO−.


When a small amount of acid is added, it will be neutralised by ClO− as shown:
H+ + ClO− → HClO
A large reservoir of ClO− ensures sufficient removal of H+ to maintain pH.
When a small amount of alkali is added, it will be neutralised by HClO:
OH− + HClO → ClO− + H2O
A large reservoir of HClO ensures sufficient removal of OH− to maintain pH.

©VJC 2013 www.gradtutors.com


8872/02/PRELIM/13
15 817

[3]

(d) Ethylbenzene can react with chlorine in two ways, depending on the conditions of the
reaction.
CH2CH3
CH2CH3 CH2CH2Cl

Cl2 Cl2

reaction I reaction II
CI
B
A

(i) State the condition needed for reaction I.

Anhydrous FeCl3 or AlCl3

(ii) Describe a chemical test to distinguish between A and B. State the reagents,
conditions, and the observations for each compound. Write balanced equations
for any reactions that occur.

Boil each compound with NaOH(aq), cool and acidify with HNO3(aq) followed
by adding AgNO3(aq).
B forms a white ppt, whereas no ppt is observed for A.

CH2CH2Cl CH2CH2OH

+ OH− → Cl− +

Ag+ + Cl− → AgCl (white ppt)

[5]

[Total: 20]

©VJC 2013 www.gradtutors.com


8872/02/PRELIM/13
818

CANDIDATE’S NAME: ______________________________ CTG: _________________

YISHUN JUNIOR COLLEGE


JC2 PRELIMINARY EXAMINATION 2013
CHEMISTRY 8872/01
HIGHER 1 28 August 2013
Paper 1: Multiple-Choice Questions 1400 hrs – 1450 hrs
50 minutes

Additional materials:
Optical Mark Sheet, Data Booklet

YISHUN JUNIOR COLLEGE YISHUN JUNIOR COLLEGE YISHUN JUNIOR COLLEGE YISHUN JUNIOR COLLEGE YISHUN JUNIOR COLLEGE
YISHUN JUNIOR COLLEGE YISHUN JUNIOR COLLEGE YISHUN JUNIOR COLLEGE YISHUN JUNIOR COLLEGE YISHUN JUNIOR COLLEGE
YISHUN JUNIOR COLLEGE YISHUN JUNIOR COLLEGE YISHUN JUNIOR COLLEGE YISHUN JUNIOR COLLEGE YISHUN JUNIOR COLLEGE
YISHUN JUNIOR COLLEGE YISHUN JUNIOR COLLEGE YISHUN JUNIOR COLLEGE YISHUN JUNIOR COLLEGE YISHUN JUNIOR COLLEGE
YISHUN JUNIOR COLLEGE YISHUN JUNIOR COLLEGE YISHUN JUNIOR COLLEGE YISHUN JUNIOR COLLEGE YISHUN JUNIOR COLLEGE
YISHUN JUNIOR COLLEGE YISHUN JUNIOR COLLEGE YISHUN JUNIOR COLLEGE YISHUN JUNIOR COLLEGE YISHUN JUNIOR COLLEGE
YISHUN JUNIOR COLLEGE YISHUN JUNIOR COLLEGE YISHUN JUNIOR COLLEGE YISHUN JUNIOR COLLEGE YISHUN JUNIOR COLLEGE
YISHUN JUNIOR COLLEGE YISHUN JUNIOR COLLEGE YISHUN JUNIOR COLLEGE YISHUN JUNIOR COLLEGE YISHUN JUNIOR COLLEGE
YISHUN JUNIOR COLLEGE YISHUN JUNIOR COLLEGE YISHUN JUNIOR COLLEGE YISHUN JUNIOR COLLEGE YISHUN JUNIOR COLLEGE
YISHUN JUNIOR COLLEGE YISHUN JUNIOR COLLEGE YISHUN JUNIOR COLLEGE YISHUN JUNIOR COLLEGE YISHUN JUNIOR COLLEGE
YISHUN JUNIOR COLLEGE YISHUN JUNIOR COLLEGE YISHUN JUNIOR COLLEGE YISHUN JUNIOR COLLEGE YISHUN JUNIOR COLLEGE
YISHUN JUNIOR COLLEGE YISHUN JUNIOR COLLEGE YISHUN JUNIOR COLLEGE YISHUN JUNIOR COLLEGE YISHUN JUNIOR COLLEGE
YISHUN JUNIOR COLLEGE YISHUN JUNIOR COLLEGE YISHUN JUNIOR COLLEGE YISHUN JUNIOR COLLEGE YISHUN JUNIOR COLLEGE
YISHUN JUNIOR COLLEGE YISHUN JUNIOR COLLEGE YISHUN JUNIOR COLLEGE YISHUN JUNIOR COLLEGE YISHUN JUNIOR COLLEGE
YISHUN JUNIOR COLLEGE YISHUN JUNIOR COLLEGE YISHUN JUNIOR COLLEGE YISHUN JUNIOR COLLEGE YISHUN JUNIOR COLLEGE
YISHUN JUNIOR COLLEGE YISHUN JUNIOR COLLEGE YISHUN JUNIOR COLLEGE YISHUN JUNIOR COLLEGE YISHUN JUNIOR COLLEGE
YISHUN JUNIOR COLLEGE YISHUN JUNIOR COLLEGE YISHUN JUNIOR COLLEGE YISHUN JUNIOR COLLEGE YISHUN JUNIOR COLLEGE
INSTRUC TIONS TO CANDIDATES
YISHUN JUNIOR COLLEGE YISHUN JUNIOR COLLEGE YISHUN JUNIOR COLLEGE YISHUN JUNIOR COLLEGE YISHUN JUNIOR COLLEGE
YISHUN JUNIOR COLLEGE YISHUN JUNIOR COLLEGE YISHUN JUNIOR COLLEGE YISHUN JUNIOR COLLEGE YISHUN JUNIOR COLLEGE
Section A & B: Multiple Choice Questions

Write in soft pencil.

Do not use paper clips, highlighters, glue or correction fluid.

Write your name and CTG on the Answer Sheet. Shade your NRIC number in the spaces provided
in the Optical Mark Sheet.

There are thirty questions on this paper. Answer all questions. For each question, there are four
possible answers A, B, C, and D.
Choose the one you consider correct and record your choice in soft pencil on the separate Answer
Sheet.

Read the instructions on the Answer Sheet very carefully.

Each correct answer will score one mark. A mark will not be deducted for a wrong answer.
Any rough working should be done in this booklet.

This question paper consists of 12 printed pages.

www.gradtutors.com
819
2

Section A

For each question there are four possible answers, A, B, C or D. Choose the one you consider to
be correct.

1 Sodium azide, NaN3 is the gas-forming component in many car airbag systems. The
common synthesis method for NaN3 is called the "Wislicenus process”, which involves two
steps starting from ammonia.

In the first step, ammonia is converted to sodium amide:

2Na + 2NH3 → 2NaNH2 + H2

The sodium amide is subsequently combined with nitrous oxide:

2NaNH2 + N2O → NaN3 + NaOH + NH3

What is the percentage yield of NaN3 when 8.54 g of NaN3 was produced from 10.1 g of Na?

A 60.0% B 65.0% C 70.0% D 75.0%

2 The first ionisation energy of five successive elements in the periodic table has the trend
shown in the diagram.

first
ionisation
energy

proton number

What could be the second element of this sequence?

A N B O C F D Ne

8871 / YJC / 2013 / Preliminary Examination / Paper 1

www.gradtutors.com
820
3

3 Which formula represents a particle with the composition 1 proton, 2 neutrons and 0
electron?
3
(T represents tritium, 1H)

A H+ B He C T D T+

4 Caffeine as shown below is found in varying quantities in the seeds, leaves and fruit of some
plants, where it acts as a natural pesticide that paralyzes and kills certain insects feeding on
the plants, as well as enhances the reward memory of pollinators.

CH3
N N O

N N
CH3
C H3 O
How many lone pairs of electrons are present in this molecule?

A 2 B 4 C 6 D 8

5 Which of the following involves an increase in oxidation number of an element?


A AgNO3 + NaCl → AgCl + NaNO3
B 2NO2 → N2O4
C 4Ti3+ + O2 + 2H2O → 4TiO2+ + 4H+
D 2H3PO4 + 3Ba(OH)2 → Ba2(PO4)3 + 3H2O

6 Dot and cross diagrams of dinitrogen oxide are shown below.

Which pair of electrons represents a coordinate bond?

A B C D
 
X• X• X •
X• X• X •

X• X• X •

X• X• X •

•  •   •  •     •  •   •  •  
•  •  
•  •  

•  •  

•  •  
 
XX
•  •  
•  •  

•  •  
 
XX
 
XX

•  •  

N N O N N O N N O
 
XX

N N   O •  •  
•  •   •  •   •  •  

8871 / YJC / 2013 / Preliminary Examination / Paper 1


[Turn Over]
www.gradtutors.com
821
4

7 Consider the following four compounds

1 (CH3)3CH

2 CH3CH2CH2CH2OH

3 CH3CH2CH2CHO

4 CH3CH2CH2CH3

What is the order of increasing boiling point of the compounds?

A 1 → 2 → 3 → 4
B 2 → 3 → 1 → 4
C 1 → 4 → 3 → 2
D 4 → 2 → 3 → 1

8 Silicon carbide (carborundum) is a shiny, hard, chemically inert material with a very high
melting point. It can be used to sharpen knives and make crucibles.

Which type of structure explains these properties?

A a giant structure with covalent bonds between silicon and carbon atoms
B a giant structure containing metallic bonding
C a giant layer structure with covalent bonds between atoms and van der Waals’ forces
between the layers
D a simple molecular structure with covalent bonds between the atoms of silicon and
carbon

9 The heat liberated in the neutralisation given below is −114 kJmol 1. −

2KOH(aq) + H2SO4(aq) → K2SO4(aq) + 2H2O(l)

By using the information, what is the most likely value for the heat liberated in the following
neutralisation?

Ba(OH)2(aq) + 2HNO3 → Ba(NO3)2(aq) + 2H2O(l)

A −29 kJmol 1− B −57 kJmol 1 −

C −86 kJmol 1−
D −114 kJmol 1 −

8871 / YJC / 2013 / Preliminary Examination / Paper 1

www.gradtutors.com
822
5

10 The lattice energies of rubidium fluoride, RbF, and caesium chloride, CsCl, are −760 kJmol 1 −

and −650 kJmol 1 respectively.


What is the lattice energy caesium fluoride, CsF, likely to be?

A −620 kJmol 1 − B −720 kJmol 1 −

C −800 kJmol 1 −
D −900 kJmol 1 −

11 Four equilibria are shown.

P N2(g) + 3H2(g) 2NH3(g)


Q 2NO(g) N2(g) + O2(g)
R CH3CHCH2(g) + CO(g) + H2(g) CH3CH2CH2CHO(g)
S I2(g) + H2(g) 2HI(g)

For which pair of equilibria does the position of equilibrium remain unchanged with an
expansion to the volume of the reaction vessel?

A P and Q B Q and R
C Q and S D R and S
 
 
 
12 Two diatomic gases, X2 and Y2 react as follows:

X2(g) + Y2(g) 2XY(g)

A mixture containing 0.5 moles of each X2 and Y2 is heated in a closed container and the
reaction allowed to reach equilibrium. The graph shows how the number of moles of each
gas varies with time.

 
0.6

0.5
no. of
0.4
moles
0.3

0.2
 

0.1

time
What is the value of the equilibrium constant, Kc for this reaction?

A 3.0 B 9.0
C 12.0 D 15.0
 
 
8871 / YJC / 2013 / Preliminary Examination / Paper 1
[Turn Over]
www.gradtutors.com
823
6

13 The energy diagram represents a reaction that occurs in the absence, and in the presence, of
a catalyst.

E1 E4

E2 E3
energy

progress of reaction

Which of the following statements is correct?

A The enthalpy change of reaction is (E2 – E3).

B E4 is the activation energy for the reverse catalysed reaction.

C The forward reaction, with catalyst, is endothermic.

D The enthalpy change of reaction is reduced by using a catalyst.

 
 
 
14 Photochromic glass, used for sunglasses, darkens when exposed to bright light and becomes
more transparent again when the light is less bright. The depth of the colour of the glass is
related to the concentration of silver atoms.

The following reactions are involved.

reaction 1 Ag+ + Cl −
Ag + Cl

reaction 2 Cu+ + Cl → Cu2+ + Cl −

reaction 3 Cu2+ + Ag → Cu+ + Ag+

Which statement about these reactions is correct?

A Cu+ ion act as a catalyst.


B Cl ions act as an oxidising agent in reaction 1.

C Reaction 2 is the one in which light is absorbed.


D Ag atoms are reduced in reaction 3.
 

8871 / YJC / 2013 / Preliminary Examination / Paper 1

www.gradtutors.com
824
7

15 An element of the third period (Na to S) is heated in chlorine. The product is purified and
then added to water. The resulting solution is found to be neutral.

What of the following is likely to be the element?

A magnesium B sodium

C silicon D phosphorous
 
 
 
16 Which element is expected to show the greatest tendency to form some covalent
compounds?

A barium B calcium
C potassium D beryllium
 
 

17 Many sunburn ointments contain benzocaine which relieves the pain caused by sunburn. It
can be made in the laboratory by using the following reaction scheme.

CH3 COOH CO2CH2CH3 CO2CH2CH3


Step 1 Step 2 Step 3

NO2 NO2 NO2 NH2

Which of the following statements about this reaction scheme is incorrect?

A Step 1 is an oxidation.
B Step 2 is an esterification.
C Step 3 is a substitution.
D Step 3 is a reduction.
 
 
 
 
 
 
 
 
 
 
 
 
 

8871 / YJC / 2013 / Preliminary Examination / Paper 1


[Turn Over]
www.gradtutors.com
825
8

18 Which hydrocarbon, on treatment with hot acidified potassium manganate(VII), would give
propanoic acid only?

A CH3CH2CH CHCH3

B H CH3
C C
CH3 CH3

C CH3CH2CH CHCH2CH3

D C H3 CH3
C C
C H3 CH3

 
 
 
19 Iron (III) bromide is added to a solution containing equimolar quantities of ethylbenzene and
bromine. The mixture is immediately placed in the dark and left until no further change takes
place.

What are likely to be the main products?

A CH2CH2Br CH2CHBr2

and

B CH2CH2Br CH2CH2Br
Br
and

Br
C CH2CH2Br CH 2CH 2Br

and
Br
D CH2CH3 CH2CH3
Br
and

Br
 

8871 / YJC / 2013 / Preliminary Examination / Paper 1

www.gradtutors.com
826
9

20 Ozone depletion potential (ODP) is a measure of the effectiveness of chlorofluoroalkanes in


destroying stratospheric ozone.

In which sequence are compounds listed in increasing order of their ODPs?

A CH3CClF2 < CH3CCl2F < CCl2FCClF2

B CH3CClF2 < CCl2FCClF2 < CH3CCl2F

C CCl2FCClF2 < CH3CClF2 < CH3CCl2F

D CH3CCl2F < CCl2FCClF2 < CH3CClF2

21 Which statement about ethanal and butanone is correct?

A Both may be prepared by the reduction of carboxylic acid.


B Both react with ammonical silver to give silver mirror.
C Both react with 2,4-dinitrophenylhydrazine to give orange precipitate.
D Both decolourises purple potassium manganate(VII).

22 Propanal can be converted into a four-carbon acid in a two-step process.

Which compound could be the intermediate?

A CH3COOH B CH3CH(OH)CN

C CH3CH2CN D CH3CH2CH2OH

23 Which of the following reacts with its own oxidation product (an oxidation which involves
no loss of carbon) to give a sweet-smelling liquid?

A Butanal

B Butanoic acid

C Butanone

D Butan-1-ol

8871 / YJC / 2013 / Preliminary Examination / Paper 1


[Turn Over]
www.gradtutors.com
827
10

24 Which pairs of reactions could have the same common intermediate?

G CH3CO2CH(CH3)2 intermediate CH3CHBrCH3


H CH3CH=CH2 intermediate CH3CH(NH2)CH3
J CH3CH=CH2 intermediate CH3COCH3
K CH3CH2CH2Cl intermediate CH3COOH

A G and J C G and K
B H and J D H and K

25 Separation of ethylbenzene from a mixture of ethylbenzene and a carboxylic acid


could involve

A Treating the mixture with dilute aqueous acid.

B Treating the mixture with dilute aqueous alkali.

C Extracting the carboxylic acid with cyclohexanol.

D Extracting the ethylbenzene with diethyl ether (CH3CH2OCH2CH3).

8871 / YJC / 2013 / Preliminary Examination / Paper 1

www.gradtutors.com
828
11

Section B

For each of the questions in this section, one or more of the three numbered statements 1 to 3 may
be correct.

Decide whether each of the statements is or is not correct (you may find it helpful to put a tick
against the statements that you consider to be correct).

The responses A to D should be selected on the basis

A B C D
1, 2 and 3 1 and 2 only 2 and 3 only 1 only is
are correct are correct are correct correct

No other combination of statements is used as a correct response.

26 Which statements about a 28.0 g sample of 14N2 are correct?


1 The number of molecules is half the number of atoms in 32.0 g of 16O2.
2 The number of atoms is twice the number of atoms in 4.0 g of 4He.
3 The number of atoms is the same as the number of atoms in 24.0 g of 12C.

27 Which of the following pairs do the molecules have similar shapes?

1 BCl3 and AlCl3


2 NO2 and SO2
3 BeF2 and PF3

28 Boron nitride is formed by the direct combination between boron and nitrogen when
heated. It has a structure similar to graphite. It is likely to be

1 lubricant.
2 abrasive.
3 hard.

8871 / YJC / 2013 / Preliminary Examination / Paper 1


[Turn Over]
www.gradtutors.com
829
12

The responses A to D should be selected on the basis

A B C D
1, 2 and 3 1 and 2 only 2 and 3 only 1 only is
are correct are correct are correct correct

No other combination of statements is used as a correct response.

29 An attractant produced by butterflies and moths has the following structural formula:

CH3CO2(CH2)6CH=CH(CH2)3CH3

Based on the above structure, which of the following is true of the attractant?

1 It can be hydrolysed by cold aqueous sodium hydroxide.


2 It decolourises hot acidified potassium manganate(VII) ions.
3 It could exist as a pair of geometric isomers.

30 Brefeldin A is an antibiotic.

HO

O
HO O CH3

Brefeldin A

Which of the following reagents react with Brefeldin A?

1 anhydrous PCl5

2 hot acidified potassium dichromate(VII)

3 2,4-dinitrophenylhydrazine

8871 / YJC / 2013 / Preliminary Examination / Paper 1

www.gradtutors.com
830
13

BLANK PAGE

8871 / YJC / 2013 / Preliminary Examination / Paper 1


[Turn Over]
www.gradtutors.com
Parent’s Signature: 831

CANDIDATE’S NAME: ______________________________ CTG: _____________

YISHUN JUNIOR COLLEGE


JC2 PRELIMINARY EXAMINATION 2013
CHEMISTRY 8872/02
HIGHER 1 21 August 2013
Paper 2: Structured & Free Response Questions 0800 hrs – 1000 hrs
2 hours

Additional materials:
Writing papers, Data Booklet, graph paper
YISHUN JUNIOR COLLEGE YISHUN JUNIOR COLLEGE YISHUN JUNIOR COLLEGE YISHUN JUNIOR COLLEGE YISHUN JUNIOR COLLEGE
YISHUN JUNIOR COLLEGE YISHUN JUNIOR COLLEGE YISHUN JUNIOR COLLEGE YISHUN JUNIOR COLLEGE YISHUN JUNIOR COLLEGE
YISHUN JUNIOR COLLEGE YISHUN JUNIOR COLLEGE YISHUN JUNIOR COLLEGE YISHUN JUNIOR COLLEGE YISHUN JUNIOR COLLEGE
YISHUN JUNIOR COLLEGE YISHUN JUNIOR COLLEGE YISHUN JUNIOR COLLEGE YISHUN JUNIOR COLLEGE YISHUN JUNIOR COLLEGE
YISHUN JUNIOR COLLEGE YISHUN JUNIOR COLLEGE YISHUN JUNIOR COLLEGE YISHUN JUNIOR COLLEGE YISHUN JUNIOR COLLEGE
YISHUN JUNIOR COLLEGE YISHUN JUNIOR COLLEGE YISHUN JUNIOR COLLEGE YISHUN JUNIOR COLLEGE YISHUN JUNIOR COLLEGE
YISHUN JUNIOR COLLEGE YISHUN JUNIOR COLLEGE YISHUN JUNIOR COLLEGE YISHUN JUNIOR COLLEGE YISHUN JUNIOR COLLEGE
YISHUN JUNIOR COLLEGE YISHUN JUNIOR COLLEGE YISHUN JUNIOR COLLEGE YISHUN JUNIOR COLLEGE YISHUN JUNIOR COLLEGE
YISHUN JUNIOR COLLEGE YISHUN JUNIOR COLLEGE YISHUN JUNIOR COLLEGE YISHUN JUNIOR COLLEGE YISHUN JUNIOR COLLEGE
YISHUN JUNIOR COLLEGE YISHUN JUNIOR COLLEGE YISHUN JUNIOR COLLEGE YISHUN JUNIOR COLLEGE YISHUN JUNIOR COLLEGE
YISHUN JUNIOR COLLEGE YISHUN JUNIOR COLLEGE YISHUN JUNIOR COLLEGE YISHUN JUNIOR COLLEGE YISHUN JUNIOR COLLEGE
YISHUN JUNIOR COLLEGE YISHUN JUNIOR COLLEGE YISHUN JUNIOR COLLEGE YISHUN JUNIOR COLLEGE YISHUN JUNIOR COLLEGE
YISHUN JUNIOR COLLEGE YISHUN JUNIOR COLLEGE YISHUN JUNIOR COLLEGE YISHUN JUNIOR COLLEGE YISHUN JUNIOR COLLEGE

INSTRUCTIONS TO CANDIDATES For Examiner’s Use


Paper 1
Write your name and CTG in the spaces at the top of this page.
Write in dark blue or black pen. Total
/30
You may use a soft pencil for any diagrams, graphs or rough working.
Paper 2
Do not use paper clips, highlighters, glue or correction fluid.
Section A
/40
Section A B5 /20
Answer all the questions.
Section B
B6
/20
Answer two questions on separate answer paper. B7
/20

You are advised to show all working in calculations. Total


/80
You may use a calculator.
The number of marks is given in brackets [ ] at the end of each
question or part question. Overall
/110
At the end of the examination, fasten all your work securely together.

This question paper consists of 11 printed pages.

www.gradtutors.com
832
2
Section A

Answer all the questions in this section in the spaces provided.

1 Olympic torches have used a variety of fuels since the first Olympic games in 776 B.C. Propane
gas was the fuel chosen for the Beijing Olympics in 2008.

(a) (i) Define the enthalpy change of combustion of propane.

(ii) In an experiment, 100 cm3 of propane was burnt to raise the temperature of
250 cm3 of water from 25 °C to 31 °C. Given that the process is 70% efficient,
determine the enthalpy change of combustion of propane.

(iii) Using your answer from (a)(ii) and the following data, calculate the enthalpy change
of formation of propane.

enthalpy change of combustion of carbon = −394 kJ mol−1


enthalpy change of combustion of hydrogen = −286 kJ mol−1

[6]

8871 / YJC / 2013 / Preliminary Examination / Paper 2


www.gradtutors.com
833
3
(b) The Beijing torch is made of an alloy of magnesium. Magnesium has several naturally
occurring isotopes.

(i) Write the electronic arrangement of Mg:

(ii) A sample of magnesium had the following isotopic composition:


24
Mg, 78.60%; 25Mg, 10.11%; 26Mg, 11.29%

Calculate the relative atomic mass, Ar, of magnesium in the sample.

[2]
Antimony, Sb, proton number 51, is another element used in alloys.

Magnesium and antimony both react when heated separately in chlorine.

When a sample of antimony was heated in chlorine under suitable conditions, a chloride A was
formed. It consists of 53.4% Sb and 46.6% Cl. The relative molecular mass of the chloride is
228.5.

(c) Determine the empirical formula of chloride A and hence, deduce its molecular formula.

[2]

(d) The chloride A melts at 73.4 oC while magnesium chloride melts at 714 oC.

With reference to the bonding and structure in each of the compounds, explain the
difference in their melting points.

[2]
[Total:12]
8871 / YJC / 2013 / Preliminary Examination / Paper 2
www.gradtutors.com
834
4
2 Pyrethroids are insecticides which are powerful against insects but are usually harmless to
mammals. The pyrethoid permethrin contains a mixture of stereoisomers, one of which is
biopermethrin.

O
O
O

Cl Cl

biopermethrin

(a) (i) Give the molecular formula of biopermethrin.

(ii) Explain why the C=C bond in biopermethrin does not give rise to geometric isomers.

[2]

(b) Biopermethrin can undergo reactions to form alcohols. One such reaction is the hydrolysis
of the ester. Apart from the ester, identify the two functional groups in biopermethrin that will
produce alcohols and state the type of reaction each functional group undergoes. In
addition, give the reagents and conditions for each reaction.

Functional group:

Type of reaction:

Reagents and conditions:

Functional group:

Type of reaction:

Reagents and conditions:


[4]

8871 / YJC / 2013 / Preliminary Examination / Paper 2


www.gradtutors.com
835
5
(c) Having synthesised biopermethrin, chemists set out to prepare even better pesticides that
will be less harmful to the environment. Suggest a change in the structure of biopermethrin
that will reduce its reactivity. Explain your answer.

[1]

(d) The substance biocypermethrin was found to be more efficient than biopermethrin.

O CN

O
O

Cl Cl

biocypermethrin

The formation of biocypermethrin from biomethrin can be shown in the following sequence.

H X CN
Step 1 Step 2
C C C

H H H

(i) State the type of reaction for step 1.

(ii) State the reagents and conditions used for steps 1 and 2.

Step 1

Step 2
[3]
[Total:10]

8871 / YJC / 2013 / Preliminary Examination / Paper 2


www.gradtutors.com
836
6
3 (a) Complete these sketches for elements of the third period to show how each property
changes along the period.

Ionic radius Melting point

Na Mg Al Si P S Cl Ar Na Mg Al Si P S Cl Ar

[2]

(b) Briefly explain the shapes of each of your sketches.

Ionic radius

[2]

Melting point

[2]
[Total:6]

8871 / YJC / 2013 / Preliminary Examination / Paper 2


www.gradtutors.com
837
7
4 Carbon dioxide (CO2) is the primary greenhouse gas emitted through human activities and it
makes up about 0.04 % of the Earth’s atmosphere. Carbon dioxide is naturally present in the
atmosphere as part of the Earth's carbon cycle (the natural circulation of carbon among the
atmosphere, oceans, soil, plants, and animals).

In the carbon cycle, carbon dioxide is produced through animal respiration. In mammals (and
many other animals), the glucose and oxygen needed for respiration are carried round the body
to the cells in the bloodstream, and the bloodstream also removes the carbon dioxide and water
produced by the cells' respiration. The carbon dioxide produced by respiration is a waste product
and must be removed from the bloodstream, but the water produced by respiration is called
metabolic water and this can be used by the body. In addition, the typical daily food requirement
of a human can be considered to be the equivalent of 1.20 kg of glucose, C6H12O6.

Human activities are altering the carbon cycle—both by adding more CO2 to the atmosphere and
by influencing the ability of natural sinks, like forests, to remove CO2 from the atmosphere. While
CO2 emissions come from a variety of natural sources, human-related emissions are responsible
for the increase that has occurred in the atmosphere since the industrial revolution.

The main human activity that emits CO2 is the combustion of fossil fuels (coal, natural gas, and
oil) for energy and transportation, although certain industrial processes and land-use changes
also emit CO2. When fossil fuels are burnt to release energy, carbon dioxide and water are also
produced. The hydrocarbon octane, C8H18, can be used to represent the fuel burned in motor
cars and the density of octane is 0.700 g cm-3. A typical fuel-efficient motor car uses about
4.00 dm3 of fuel to travel 100 km.

(a) (i) Construct a balanced equation for the complete oxidation of glucose.

(ii) Use the equation in (a)(i) to calculate the amount of CO2 produced by one person in
one day.

(iii) In 2012, the world population is estimated to be 6.82 x 109. Calculate the total mass
of CO2 produced by this number of people in one day. Give your answers in tonnes.
[1 tonne = 1000 kg]

[5]

8871 / YJC / 2013 / Preliminary Examination / Paper 2


www.gradtutors.com
838
8
(b) (i) Construct a balanced equation for the complete combustion of octane.

(ii) Calculate the amount of octane present in 4.00 dm3 of octane.

(iii) Calculate the mass of CO2 produced when the fuel-efficient car is driven for a
distance of 100 km.

[5]

(c) Using your answer in (a)(iii), calculate how many kilometres the same fuel-efficient car
would have to travel in order to produce as much CO2 as is produced by the respiration of
6.82 x 109 people during one day.

[2]
[Total:12]

8871 / YJC / 2013 / Preliminary Examination / Paper 2


www.gradtutors.com
839
9
Section B

Answer two questions from this section on separate answer paper.

5 (a) Halogenoethanes react with ammonia to produce amines which are weak Bronsted bases.

CH3CH2X + NH3 → CH3CH2NH2 + HX where X = Cl, Br or I

(i) Arrange the halogenoethanes in increasing reactivity towards ammonia.


Explain your answer.

(ii) State the conditions required for this reaction.

(iii) Draw a dot-and-cross diagram to show the bonding in an ammonia molecule and
use the VSEPR (valence shell electron pair repulsion) theory to predict its shape
and bond angle. [7]

(b) A student carried out an investigation with aqueous solutions of ethylamine, CH3CH2NH2
and hydrochloric acid.

(i) Explain what is meant by a weak Bronsted base.

(ii) What is the conjugate acid formed from ethylamine, CH3CH2NH2?

(iii) The pKb value of ethylamine, CH3CH2NH2 is 3.36.


Calculate the pH of 0.010 mol dm-3 of ethylamine, CH3CH2NH2. [6]

(c) The student prepared two solutions.


• Solution B was made by mixing 25 cm3 of 0.10 mol dm-3 of dilute hydrochloric acid
with 50 cm3 of 0.10 mol dm-3 of ethylamine. Solution B is a buffer solution.
• Solution C was made by mixing 50 cm3 of 0.10 mol dm-3 of dilute hydrochloric acid
with 25 cm3 of 0.10 mol dm-3 of ethylamine. Solution C is not a buffer solution.

(i) Explain why solution B is a buffer solution while solution C is not.

(ii) Write balanced equations to show how solution B is able to maintain pH when small
amounts of acid and base are added to it. [5]

(d) The student measured the pH of water as 7.0 at 25 oC. She then warmed the water to
40 oC and measured the pH as 6.7.

What do these results tell you about the tendency of water to ionise as it gets warmer?
Explain your reasoning in terms of LeChatelier’s Principle.
. [2]
[Total:20]

8871 / YJC / 2013 / Preliminary Examination / Paper 2


www.gradtutors.com
840
10
6 (a) In order to investigate the kinetics of the hydrolysis of an ester RCOOCH3, the
concentration of the carboxylic acid, RCOOH produced was determined.

RCOOCH3(aq) + H2O(l) → RCOOH(aq) + CH3OH(aq)

When 0.200 mol dm-3 of the ester was hydrolysed in the presence of dilute sulfuric acid as
catalyst, the following results were obtained.

Time / s [RCOOH] / mol dm-3


0 0
25 0.048
50 0.085
75 0.112
100 0.133
125 0.149

(i) Explain the term order of reaction.

(ii) By using a suitable graph, explain why the experimental results indicate that the
reaction is first order with respect to the ester.

(iii) Why is it not possible to determine the order with respect to water in this
experiment?

(iv) Use the following data to deduce the order of reaction with respect to hydrochloric
acid.

[RCOOCH3] / mol dm-3 [HCl] / mol dm-3 Relative rate


0.10 0.10 1
0.15 0.20 3
0.20 0.10 2

(v) Write the rate equation for the reaction and state the units of the rate constant.

(vi) With the aid of a Boltzman distribution and reference to the collision theory, explain
how a rate of reaction would change when temperature is increased. [13]

(b) The above hydrolysis reaction is reversible. In an experiment, the equilibrium


concentrations of the various species at 25 oC are as follows.
[RCOOCH3 (aq)] = 0.188 mol dm-3
[RCOOH (aq)] = 0.125 mol dm-3
[CH3OH (aq)] = 0.376 mol dm-3

Solid RCOOH is added to the equilibrium mixture. When the equilibrium is re-established
at 25 oC, the concentration of RCOOH (aq) becomes 0.200 mol dm-3.

Calculate the concentration of CH3OH when equilibrium is re-established at 25 oC.


Assume that there is no change in the total volume of the mixture. [3]

(c) Suggest a method by which ethyl propanoate could be distinguished from propyl ethanoate
by a chemical test. Give reagents and conditions for the reactions and state what you
would observe. Include balanced equations for any reactions that occur. [4]
[Total:20]

8871 / YJC / 2013 / Preliminary Examination / Paper 2


www.gradtutors.com
841
11
7 (a) Using the oxides of magnesium, silicon and phosphorus as examples, describe the
reactions of the oxides of elements in the third period of the Periodic Table with water.
Include balanced equations for any reactions. [4]

(b) Sodium nitrite, NaNO2 is used as a preservative in meat products such as sausages and
ham. In an acidic solution, the nitrite ion is converted to nitrous acid, HNO2, which reacts
with manganate(VII) ion to form nitrate ion, NO3-.

(i) Write a half equation for the reaction of the nitrate ion, NO3-.

(ii) Give the oxidation states of N in this reaction and use them to explain why NO3- is
acting as a reducing agent.

(iii) Give a balanced equation for the redox reaction between nitrite ions and
manganate(VII) ions in acid solution.

(iv) A 1.00 g sample of a water-soluble solid containing sodium nitrite was dissolved in
dilute hydrochloric acid and titrated with 0.0100 mol dm−3 KMnO4 solution. The
titration required 12.5 cm3 of KMnO4 solution.

Calculate the percentage by mass of sodium nitrite in the sample. [6]

(c) Compound P, C9H8O is the main component of cinnamon oil.

An orange precipitate is observed when P is warmed with 2,4-dinitrophenylhydrazine. In


addition, P gives a red precipitate with Fehling’s solution and decolourises aqueous
bromine.

When P was refluxed with acidified potassium manganate(VI), compound Q, C7H6O2 and
carbon dioxide are formed. Q is soluble in both aqueous sodium hydroxide and sodium
carbonate.

Treatment of P with sodium boronhydride produces compound R which gives hydrogen


gas with sodium metal. When R reacts with hydrogen chloride, compound S is formed.

Identify and suggest the structures for P, Q, R and S. Show how you deduced these
structures by explaining the reactions that are occurring. [10]
[Total:20]

~ END OF PAPER ~

8871 / YJC / 2013 / Preliminary Examination / Paper 2


www.gradtutors.com
842

Yishun  Junior  College  H1  Chemistry  Preliminary  Examinations  2013    

Paper  1  Answers    

1 A 6 A 11 C 16 D 21 C 26 A
2 B 7 C 12 B 17 C 22 B 27 B

3 D 8 A 13 A 18 C 23 D 28 D

4 D 9 D 14 A 19 D 24 A 29 C

5 C 10 B 15 B 20 A 25 B 30 B
 

Paper  2  Answers    
1 (a) (i) Enthalpy change of combustion of propane is the energy
evolved when one mole of propane is completely burnt in
oxygen. [1]

!""
(ii) No. of moles of propane = ÷ 24 = 4.167×10!!
!"""
Heat absorbed by water = 250 x 4.18 x 6 = 6270 J [1]
!"#$
Heat evolved = ×100 = 8957  𝐽 [1]
!"
!"#$×!"!!
∆𝐻! = − = −2150  𝑘𝐽  𝑚𝑜𝑙 !! [1]
!.!"#×!"!! [3]

(iii) ∆𝐻! 𝑝𝑟𝑜𝑝𝑎𝑛𝑒 = −394×3 + −286×4 − −2150 [1]


= −173  𝑘𝐽  𝑚𝑜𝑙 !! [1] [2]

(b) (i) 1s2 2s2 2p6 3s2 [1]

(ii) !".!×!" ! !".!!×!" !(!!.!"×!") [1]


Ar = = 24.3
!""

(c) Sb Cl
Mass ratio 53.6 46.6
Mole ratio 0.438 1.31
Simplest ratio 1 3
Empirical formula: SbCl3 [1]

n[(122 + 3(35.5)] = 228.5


n=1
Molecular formula: SbCl3 [1] [2]

(d) SbCl3 has a simple covalent structure and so, less energy is needed
to overcome the weak van der Waals’ forces between its molecules.
[1]
MgCl2 has a giant ionic lattice structure and so, a lot of energy is
needed to overcome the strong ionic bonds. [1] [2]

[Total:12]

www.gradtutors.com
843

2 (a) (i) C21H20O3Cl2 [1]

(ii) One of the carbon atoms involved in the C=C double bond is [1]
  (chlorine atoms)
bonded to 2 similar groups

(b) Functional group: alkene


Type of reaction: addition OR oxidation
Reagents and conditions: conc. H2SO4, followed by H2O and warm
OR KMnO4, aq. NaOH and cold

Functional group: halogen derivative


Type of reaction: substitution
Reagents and conditions: aq NaOH, heat.

Each functional group and type of reaction [1]


Each reagents and conditions [1] [4]

(c) Replace the chlorine atoms with fluorine atoms as the C-F bond is
very strong and difficult to break.[1] [1]

(d) (i) Substitution [1]


 
(ii) Step 1 X2(g) in CCl4, UV light [1]
Step 2 NaCN in ethanol, heat under reflux [1] [2]
[Total:10]
Ionic radius
3 (a) [1] for
/nm
each
sketch

Proton
number
Na Mg Al Si P S Cl

www.gradtutors.com
844

(b) Ionic radius


Cations (Na+, Mg2+, Al3+, Si4+) are isoelectronic – 1s2 2s2 2p6, and
anions (P3−, S2−, Cl−) are isoelectronic – 1s2 2s2 2p6 3s2 3p6. Since
nuclear charge increases across the period, electrons are more
strongly attracted to the nucleus. [1] Hence, cationic radius
decreases from Na+ to Si4+, and anionic radius decreases from P3− to
Cl−.
Anions have one more quantum shell of electrons than the cations.
[1]. Hence, there is a sharp increase in ionic radius from Si4+ to P3−. [2]
Melting point
Na, Mg and Al have giant metallic structure. A lot of energy is
needed to overcome the strong metallic bonds. Hence, they have
high melting points.

Si has giant molecular structure. A lot of energy is needed to


overcome the strong covalent bonds throughout the whole structure.
Hence, melting point of Si is high.

P, S, Cl and Ar have simple molecular structure. A small amount of


energy is needed to overcome the weak intermolecular van der
Waals’ forces of attraction. Hence, they have low melting points.

All 3 structures and corresponding bonds correct – [2]


At least 1 structure and corresponding bond correct – [1] [2]

4 (a) (i) C6H12O6 + 6 O2 → 6 CO2 + 6 H2O [1]

(ii) !.!"×!"""
𝑛𝑜. 𝑜𝑓  𝑚𝑜𝑙𝑒𝑠  𝑜𝑓  𝑔𝑙𝑢𝑐𝑜𝑠𝑒 =   = 6.67  𝑚𝑜𝑙 [1]
!"#
[2]
𝑛𝑜. 𝑜𝑓  𝑚𝑜𝑙𝑒𝑠  𝑜𝑓  𝐶𝑂! = 6  ×6.67 = 40  𝑚𝑜𝑙 [1]

(iii) 𝑡𝑜𝑡𝑎𝑙  𝑛𝑜. 𝑜𝑓  𝑚𝑜𝑙𝑒𝑠  𝑜𝑓  𝐶𝑂!


= 40  ×6.82  ×10! = 2.73  ×10!!  𝑚𝑜𝑙 [1]
𝑚𝑎𝑠𝑠  𝑜𝑓  𝐶𝑂!
= 2.73×10!! ×44 = 1.20  ×10!" 𝑔 = 1.20×10! 𝑡𝑜𝑛𝑛𝑒𝑠 [1] [2]

(b) (i) 2 C8H18 + 25 O2 → 16 CO2 + 18 H2O [1]

(ii) 𝑚𝑎𝑠𝑠  𝑜𝑓  𝑜𝑐𝑡𝑎𝑛𝑒 = 0.7×4×1000 = 2800  𝑔 [1]


!"## [2]
𝑛𝑜. 𝑜𝑓  𝑚𝑜𝑙𝑒𝑠  𝑜𝑓  𝑜𝑐𝑡𝑎𝑛𝑒 =   = 24.6  𝑚𝑜𝑙𝑒𝑠 [1]
!!"

(iii) 𝑛𝑜. 𝑜𝑓  𝑚𝑜𝑙𝑒𝑠  𝑜𝑓  𝐶𝑂! = 8×24.6 = 197  𝑚𝑜𝑙𝑒𝑠 [1]


𝑚𝑎𝑠𝑠  𝑜𝑓  𝐶𝑂! = 197  ×44 = 8660  𝑔 [1] [2]

(c) !.!"  ×!"!"


𝑁𝑜. 𝑜𝑓  100  𝑘𝑚 =   = 1.39  ×10! [1]
!""# [2]
𝑁𝑜. 𝑜𝑓  𝑘𝑚 = 1.39  ×10 ×100 = 1.39×10!! 𝑘𝑚[1]
!

[Total:12]

www.gradtutors.com
845

5 (a) (i) Reactivity : CH3CH2Cl < CH3CH2Br < CH3CH2I [1]


As the size of the halogen atom increases (Cl<Br<I), the
effectiveness of the overlapping of the p orbitals of the carbon
and halogen atoms decreases. [1]
This leads to a decrease in the strength of the C-X bond (C-
Cl>C-Br>C-I). As the bonds get easier to break, the reactivity
towards ammonia increases. [1] [3]

(ii) Concentrated NH3, heat in a sealed tube [1]

(iii)
H x Nx x H [1]
H
To minimise repulsion between the 3 bond pairs and 1 lone
pair of electrons, the electron pairs arrange themselves
tetrahedrally. [1]
Shape: trigonal pyramidal; Bond angle: 107 o [1]   [3]
(b) (i) A weak Bronsted base is a proton acceptor [1] which
undergoes partial dissociation [1] in water. [2]

(ii) CH3CH2NH3+ [1]

(iii) 𝐾! = 10!!.!" = 4.37×10!!  𝑚𝑜𝑙𝑑𝑚 !! [1]


𝑂𝐻 ! =   4.37×10!! ×0.01 = 2.09×10!!  𝑚𝑜𝑙𝑑𝑚 !! [1]
𝑝𝑂𝐻 =   − lg 2.09×10!! = 2.68
𝑝𝐻 = 14 − 2.68 = 11.3 [1] [3]

(c) (i) In solution B, ethylamine is in excess while hydrochloric acid is


in excess in solution C. [1]
Solution B comprises of a weak base, ethylamine and its salt,
CH3CH2NH3+. Hence, it is a buffer. [1]
Solution C comprises of a strong acid, HCl and its salt,
CH3CH2NH3+Cl-. Hence, it is not a buffer. [1] [3]

(ii) When small amounts of acid is added,


CH3CH2NH2 + H+ → CH3CH2NH3+ [1]
When small amounts of base is added,
CH3CH2NH3+ + OH- → CH3CH2NH2 + H2O [1] [2]

(d) 2 H2O ⇌ H3O+ + OH-


When temperature increases, the pH decreases which indicates that
the concentration of H+ increases. This suggests that the equilibrium
had shifted to the right, favouring the forward reaction. [1]
Hence, the forward reaction is endothermic and an increase in
temperature favours the ionisation of water. [1] [2]
Total: 20

6 (a) (i) Order of reaction is defined as the power to which the


concentration of that reactant is raised in an experimentally
determined rate equation. [1]

www.gradtutors.com
846

(ii) Time / s [RCOOH] / mol dm-3 [ester] / mol dm-3


0 0 0.200
25 0.048 0.152
50 0.085 0.115
75 0.112 0.088
100 0.133 0.067
125 0.149 0.051

Plot [ester] against time. [2]


Show at least 2 half-lives on graph. [1]
Constant half-life (approx. 63 s) [1] [4]

(iii) The concentration of water is constant as water is the solvent


and in large excess (pseudo-zero order w.r.t. water). [1]

(iv) Let x be the order of reaction w.r.t. hydrochloric acid


Comparing experiments 1 & 2,
𝑟𝑎𝑡𝑒! 𝑘 0.10 [0.10]! 1
= =
𝑟𝑎𝑡𝑒! 𝑘 0.15 [0.20]! 3
𝑥=1
Order of reaction w.r.t. hydrochloric acid = 1 [1]

(v) 𝑅𝑎𝑡𝑒 = 𝑘 𝑒𝑠𝑡𝑒𝑟 𝐻𝐶𝑙 [1]


Units: mol-1 dm3 s-1 [1] [2]

(vi)

(Correct diagram – [1])


When temperature increases, the average energy of the
reacting particles is increased. [1]
Hence, the proportion of particles with energy equal to or
greater than the activation energy increases significantly as
seen in the shaded area in the diagram. [1]
In addition, when the temperature of the reactants is increased,
their average speeds increase and so, the frequency of
collisions between reacting particles increases. This increases
the number of effective collisions and consequently the rate of
reaction. [1] [4]

www.gradtutors.com
847

(b) !"##$ [!"! !"] !.!"#×!.!"#


𝐾! = = = 0.25  𝑚𝑜𝑙  𝑑𝑚 !! [1]
[!"##"$! ] !.!""
RCOOCH3 (aq) + H2O (l) ⇌ RCOOH (aq) + CH3OH (aq)
eqm conc 0.188 + x 0.200 0.376 - x
/mol dm-3

0.200(0.376 − 𝑥)
𝐾! = = 0.25
0.188 + 𝑥
𝑥 = 0.0627 [1]

Hence, [CH3OH] = 0.376 – 0.0627 = 0.313 mol dm-3 [1] [3]

(c) Test: Warm with aq. I2 and aq. NaOH. [1]

A yellow precipitate will be formed with ethyl propanoate while no


precipitate will be formed with propyl ethanoate. [1]

CH3CH2CO2CH2CH3 + OH− → CH3CH2CO2− + CH2CH2OH [1]


CH3CH2OH + 4 I2 + 6 OH− → CHI3 + HCO3− + 5 I− +5 H2O [1]
[4]
Total: 20

7 (a) MgO dissolves slightly in water to give a weakly alkaline solution.


MgO(s) + H2O(l) ⇌ Mg(OH)2(aq)

SiO2 does not dissolve in water.

P4O10 undergoes hydrolysis with water to give a strongly acidic


solution.
P4O10(s) + 6 H2O(l) → 4 H3PO4(aq) [4]

(b) (i) HNO2 + H2O → NO3- + 3 H+ + 2 e- [1]

(ii) Oxidation state of N increase from +3 in HNO2 to +5 in NO3-.


Hence, nitrogen undergoes oxidation and act as a reducing
agent. [2]

(iii) 5 HNO2 + 2 MnO4- + H+ → 5 NO3- + 2 Mn2+ + 3 H2O

(iv) 12.5
𝑛𝑜. 𝑜𝑓  𝑚𝑜𝑙𝑒𝑠  𝑜𝑓  𝐾𝑀𝑛𝑂! = 0.01× = 1.25×10!!   𝑚𝑜𝑙
1000
𝑛𝑜. 𝑜𝑓  𝑚𝑜𝑙𝑒𝑠  𝑜𝑓  𝑁𝑎𝑁𝑂! = 𝑛𝑜. 𝑜𝑓  𝑚𝑜𝑙𝑒𝑠  𝑜𝑓  𝐻𝑁𝑂!
!
= ×1.25×10!! = 3.125×10!! 𝑚𝑜𝑙 [1]
!
𝑚𝑎𝑠𝑠  𝑜𝑓  𝑁𝑎𝑁𝑂! = 3.125×10!! ×85 = 0.0266  𝑚𝑜𝑙
%  𝑚𝑎𝑠𝑠 = 0.0266×100 = 2.66  % [1]
[2]

   

www.gradtutors.com
848

(c) Information Deductions


Orange ppt formed P is a carbonyl compound and it
when P is formed with undergoes condensation.
2,4-DNPH
P gives a red ppt with P is an aldehyde.
Fehling’s solution
P decolourises aq. Br2 P is an alkene and it undergoes addition
with Br2.
Q and CO2 is formed P undergoes side chain oxidation to form
when P is refluxed Q, benzoic acid.
with MnO4-/H+ OR P is an alkene and undergoes
oxidative cleavage to form Q, a
carboxylic acid.
Q is soluble in sodium Q is a carboxylic acid and it undergoes
hydroxide and sodium neutralisation.
carbonate.
Treatment of P with The aldehyde group in P is reduced to a
NaBH4 gives R. primary alcohol in R.
R gives H2 with Na. R is an alcohol and it undergoes redox
reaction with Na.
R reacts with HCl to R contains alkene and alcohol groups
form S. and undergoes addition and substitution
reactions respectively. S is a halogen
derivative.

Each deduction [1] Maximum 6M

H H O

C C C H
P

CO2H
Q

H H

R
C C CH2OH

H H

C C CH2Cl
S
H Cl

Each structure [1] [10]


Total:20

www.gradtutors.com

Das könnte Ihnen auch gefallen